Usmle Step 1 Secrets In Color [4 ed.] 9780323396790, 2016037935

684 70 21MB

English Pages [727] Year 2017

Report DMCA / Copyright

DOWNLOAD FILE

Polecaj historie

Usmle Step 1 Secrets In Color [4 ed.]
 9780323396790, 2016037935

  • Commentary
  • eBook

Table of contents :
Front Cover
IFC
USMLE STEP 1 SECRETS IN COLOR
USMLE STEP 1 SECRETS IN COLOR
Copyright
LIST OF CONTRIBUTORS
PREFACE
HOW EARLY SHOULD YOU BEGIN STUDYING FOR THE USMLE?
HOW WILL THIS BOOK HELP YOU PREPARE FOR STEP 1?
NOW THAT YOU HAVE SELECTED YOUR RESOURCES, HOW SHOULD YOU GO ABOUT STUDYING FOR THE EXAM?
WHEN WILL I GET MY SCORE?
TEN THINGS STUDENTS WISH THEY HAD KNOWN BEFORE TAKING THE USMLE STEP 1
ONE FINAL NOTE
MORE SECRETS FOR SUCCESS ON THE USMLE STEP 1
CONTENTS
1 - CARDIOLOGY
BASIC CONCEPTS—ARRHYTHMIAS
OTHER RELATED QUESTION
OTHER RELATED QUESTION
2 - PULMONOLOGY
BASIC CONCEPTS—MECHANICS OF BREATHING
BASIC CONCEPTS—VENTILATION-PERFUSION MATCHING
BASIC CONCEPTS—GAS EXCHANGE
RELATED QUESTIONS
RELATED QUESTION
DIFFERENTIAL DIAGNOSIS
3 - NEPHROLOGY
BASIC CONCEPTS
SOME DIFFERENTIAL DIAGNOSIS CONCEPTS
4 - FLUID AND ELECTROLYTES
BASIC CONCEPTS—RENAL FILTRATION AND TRANSPORT PROCESSES
BASIC CONCEPTS—RENAL CONTROL OF ACID-BASE BALANCE
BASIC CONCEPTS—RENAL CONTROL OF EXTRACELLULAR FLUID BALANCE
RENAL CONTROL OF EXTRACELLULAR FLUID OSMOLARITY
PHARMACOLOGY OF DIURETICS
5 - ACID-BASE BALANCE
BASIC CONCEPTS
6 - GASTROENTEROLOGY
BASIC CONCEPTS
7 - HEPATOLOGY
BASIC CONCEPTS
SECRET TO DIAGNOSING COMMON CAUSES OF JAUNDICE
Secrets for Diagnosing Stages of Hepatitis B Infection
8 - ENDOCRINOLOGY
BASIC CONCEPTS
RELATED QUESTIONS
SOME DIFFERENTIAL DIAGNOSIS AND PHYSIOLOGY CONCEPTS
SOME DIFFERENTIAL DIAGNOSIS CONCEPTS
RELATED QUESTIONS
RELATED QUESTIONS
RELATED QUESTIONS
9 - MALE AND FEMALE REPRODUCTIVE SYSTEMS
BASIC CONCEPTS
RELATED QUESTIONS ON LABOR AND DELIVERY
RELATED QUESTION ON GYNECOLOGIC INFECTIONS
RELATED QUESTIONS
10 - ONCOLOGY
BASIC CONCEPTS—CANCER EPIDEMIOLOGY
BASIC CONCEPTS—CANCER CLASSIFICATION
11 - GENETIC AND METABOLIC DISEASE
BASIC CONCEPTS
RELATED QUESTION
RELATED QUESTIONS
RELATED QUESTIONS
RELATED QUESTIONS
RELATED QUESTION
RELATED QUESTION
RELATED QUESTION
RELATED QUESTIONS
12 - ANEMIAS
BASIC CONCEPTS
SECRETS FOR APPROACHING ANEMIAS ON THE USMLE STEP 1
13 - BLEEDING DISORDERS
BASIC CONCEPTS
14 - HEMATOLOGIC MALIGNANCIES
BASIC CONCEPTS
DIFFERENTIAL DIAGNOSIS
15 - IMMUNOLOGY
BASIC CONCEPTS
RELATED QUESTIONS
RELATED QUESTION
RELATED QUESTIONS: B-CELL DISORDERS
RELATED QUESTIONS: PHAGOCYTE DISORDERS
RELATED QUESTION
RELATED QUESTIONS: MECHANISMS OF TOLERANCE
16 - PSYCHIATRY
17 - NEUROLOGY
BASIC CONCEPTS
HOW TO APPROACH CASES
18 - OPHTHALMOLOGY
BASIC CONCEPTS
RELATED QUESTIONS
19 - RHEUMATOLOGY
BASIC CONCEPTS
A FEW MORE MUSCULAR DYSTROPHIES …
20 - VASCULITIDES
BASIC CONCEPTS
21 - BACTERIAL DISEASES
BASIC CONCEPTS PART I: BACTERIAL MORPHOLOGY AND VIRULENCE FACTORS
BASIC CONCEPTS PART II: ANTIBACTERIAL PHARMACOLOGY
RELATED QUESTIONS
22 - VIRAL, PARASITIC, AND FUNGAL DISEASES
BASIC CONCEPTS—PARASITOLOGY
BASIC CONCEPTS—MYCOLOGY
23 - PHARMACOLOGY AND TOXICOLOGY
BASIC CONCEPTS
SECRETS FOR UNDERSTANDING MAXIMAL EFFECT AND SUBSTRATE CONCENTRATION FOR THE USMLE STEP 1
AMPHETAMINES
LYSERGIC ACID DIETHYLAMIDE (LSD)
PHENCYCLIDINE (PCP)
TETRAHYDROCANNABINOL (THC)
24 - BEHAVIORAL SCIENCES
BASIC CONCEPTS
25 - BIOSTATISTICS
BASIC CONCEPTS
MEASURES OF SPREAD
STUDY DESIGNS
26 - CLINICAL ANATOMY
OTHER IMPORTANT CONCEPTS IN EMBRYOLOGY OF THE FACE AND NECK
27 - DERMATOLOGY
BASIC CONCEPTS
28 - PATHOLOGY
INDEX
A
B
C
D
E
F
G
H
I
J
K
L
M
N
O
P
Q
R
S
T
U
V
W
X
Y
Z
IBC

Citation preview

Any screen. Any time. Anywhere. Activate the eBook version rge. of this title at no additional charge.

Student Consult eBooks give you the power to browse and find content, view enhanced images, share notes and highlights—both online and offline.

Unlock your eBook today. 1 Visit studentconsult.inkling.com/redeem 2

Scratch off your code

Scan this QR code to redeem your eBook through your mobile device:

3 Type code into “Enter Code” box 4

Click “Redeem”

5

Log in or Sign up

6

Go to “My Library”

It’s that easy! Place Peel Off Sticker Here

For technical assistance: email [email protected] call 1-800-401-9962 (inside the US) call +1-314-447-8200 (outside the US) Use of the current edition of the electronic version of this book (eBook) is subject to the terms of the nontransferable, limited license granted on studentconsult.inkling.com. Access to the eBook is limited to the first individual who redeems the PIN, located on the inside cover of this book, at studentconsult.inkling.com and may not be transferred to another party by resale, lending, or other means.

USMLE STEP 1 SECRETS IN COLOR

This page intentionally left blank

     

USMLE STEP 1 SECRETS IN COLOR FOURTH EDITION THOMAS A. BROWN, MD

Medical Director/Owner Kathy’s Urgent Care of Wethersfield Wethersfield, Connecticut

SONALI J. BRACKEN

MD/PhD Candidate University of Connecticut School of Medicine Farmington, Connecticut

3251 Riverport Lane St. Louis, Missouri 63043

USMLE STEP 1 SECRETS IN COLOR, FOURTH EDITION Copyright © 2017, Elsevier Inc. All rights reserved. Previous editions copyrighted 2013, 2008, and 2004.

ISBN: 978-0-323-39679-0

No part of this publication may be reproduced or transmitted in any form or by any means, electronic or mechanical, including photocopying, recording, or any information storage and retrieval system, without permission in writing from the publisher. Details on how to seek permission, further information about the Publisher’s permissions policies and our arrangements with organizations such as the Copyright Clearance Center and the Copyright Licensing Agency, can be found at our website: www.elsevier.com/permissions. This book and the individual contributions contained in it are protected under copyright by the Publisher (other than as may be noted herein). Notices Knowledge and best practice in this field are constantly changing. As new research and experience broaden our understanding, changes in research methods, professional practices, or medical treatment may become necessary. Practitioners and researchers must always rely on their own experience and knowledge in evaluating and using any information, methods, compounds, or experiments described herein. In using such information or methods they should be mindful of their own safety and the safety of others, including parties for whom they have a professional responsibility. With respect to any drug or pharmaceutical products identified, readers are advised to check the most current information provided (i) on procedures featured or (ii) by the manufacturer of each product to be administered, to verify the recommended dose or formula, the method and duration of administration, and contraindications. It is the responsibility of practitioners, relying on their own experience and knowledge of their patients, to make diagnoses, to determine dosages and the best treatment for each individual patient, and to take all appropriate safety precautions. To the fullest extent of the law, neither the Publisher nor the authors, contributors, or editors, assume any liability for any injury and/or damage to persons or property as a matter of products liability, negligence or otherwise, or from any use or operation of any methods, products, instructions, or ideas contained in the material herein. Library of Congress Cataloging-in-Publication Data Names: Brown, Thomas A. (Thomas Andrew), 1972- editor. | Bracken, Sonali J.,  editor. Title: USMLE step 1 secrets in color / [edited by] Thomas A. Brown, MD,   Medical Director/Owner, Kathy’s Urgent Care of Wethersfield, Wethersfield,   Connecticut, Sonali J. Bracken, MD/PhD Candidate, University of   Connecticut School of Medicine, Farmington, Connecticut. Other titles: USMLE step 1 secrets Description: Fourth edition. | St. Louis, Missouri : Elsevier, [2017] |   Revison of: USMLE step 1 secrets / Thomas A. Brown, David D. Brown. c2013.   3rd ed. Identifiers: LCCN 2016037935 | ISBN 9780323396790 Subjects: LCSH: Medical sciences--Outlines, syllabi, etc. |   Physicians--Licenses--United States--Examinations--Study guides. Classification: LCC R834.5 .B765 2017 | DDC 610.76--dc23 LC record available at https://lccn.loc.gov/2016037935

Content Strategist: James Merritt Content Development Specialist: Nicole Dicicco Publishing Services Manager: Hemamalini Rajendrababu Project Manager: Divya Krishna Kumar Design Direction: Ryan Cook

Printed in the United States of America Last digit is the print number: 9 8 7 6 5 4 3 2 1

LIST OF CONTRIBUTORS

Matthew N. Anderson, MD Neurological Surgery Resident Brown University Rhode Island Hospital Providence, Rhode Island Melissa Argraves, MD Resident, Department of Pediatrics Children’s Hospital of Philadelphia Philadelphia, Pennsylvania Yetunde Asiedu, MD University of Connecticut School of Medicine Farmington, Connecticut Melina Benson University of Connecticut School of Medicine Farmington, Connecticut Giana C. Bistany, MD, MS University of Connecticut School of Medicine Obstetrics and Gynecology Farmington, Connecticut Martina S. Burn, MD Department of Obstetrics, Gynecology and Reproductive Sciences Yale School of Medicine New Haven, Connecticut Samantha Chirunomula University of Connecticut School of Medicine Farmington, Connecticut Sarah E. Conway NYU School of Medicine New York, New York Sarah Cryer University of Connecticut School of Medicine Farmington, Connecticut Robert D’Angelo, MD Harvard Medical School Beth Israel Deaconess Hospital Boston, Massachusetts Apeksha Dave University of Connecticut School of Medicine Farmington, Connecticut Stephanie Davis University of Connecticut School of Medicine Farmington, Connecticut

Joravar Dhaliwal University of Connecticut School of Medicine Farmington, Connecticut Andrew J. Duarte University of Connecticut School of Medicine Farmington, Connecticut Ryan P. Duggan University of Connecticut School of Medicine Farmington, Connecticut Thomas J.S. Durant, MPT, MD Yale School of Medicine Department of Laboratory Medicine New Haven, Connecticut Cory Dwyer University of Connecticut School of Medicine Farmington, Connecticut Brian P. Epling University of Connecticut School of Medicine Farmington, Connecticut Patrick A. Field, MD University of Connecticut School of Medicine Farmington, Connecticut Boston Medical Center Internal Medicine Boston, Massachusetts Rebecca Flugrad University of Connecticut School of Medicine Farmington, Connecticut Eric Han, MD Yale School of Medicine Resident, Department of Obstetrics, Gynecology and Reproductive Sciences New Haven, Connecticut Alex M. Hennessey, MD University of Connecticut School of Medicine Farmington, Connecticut Matthew Howe University of Texas Health Science Center at Houston Houston, Texas Liza Karamessinis, MD The Children’s Hospital of Philadelphia Philadelphia, Pennsylvania

Christopher Del Prete, MD Dartmouth-Hitchcock Medical Center Lebanon, New Hampshire

v

vi  LIST OF CONTRIBUTORS Shirin Karimi, MD University of Connecticut School of Medicine Farmington, Connecticut Resident in Internal Medicine at Cambridge Health Alliance Clinical Fellow in Medicine at Harvard Medical School Boston, Massachusetts Adam J.S. Kaye, MD University of Connecticut School of Medicine Farmington, Connecticut Andrew Kelsey, MD University of Connecticut School of Medicine John Dempsey Hospital Farmington, Connecticut Kaitlyn Ryan LaMarche, MD University of Connecticut School of Medicine Farmington, Connecticut Hien Le, MD University of Connecticut School of Medicine Farmington, Connecticut Aaron Lee, MD University of Connecticut School of Medicine Farmington, Connecticut Deirdre Lewis, MD University of Massachusetts Medical School-Baystate Springfield, Massachusetts Maritza Montanez, MD University of Connecticut School of Medicine Farmington, Connecticut Henry L. Nguyen, MD University of California, San Diego La Jolla, California Lena M. O’Keefe, MS University of Connecticut School of Medicine Farmington, Connecticut Meaghen Roy-O’Reilly University of Texas Health Science Center at Houston Houston, Texas

Nicole J. Rubin, MD Resident, Department of Obstetrics and Gynecology University of Connecticut School of Medicine Farmington, Connecticut Joseph M. Ryan University of Connecticut School of Medicine Farmington, Connecticut Neda Shahriari University of Connecticut School of Medicine Farmington, Connecticut Kelsey Sokol, MD University of Connecticut School of Medicine Farmington, Connecticut Eunice Song, MD University of Connecticut School of Medicine Farmington, Connecticut Bryan Stenson University of Connecticut School of Medicine Farmington, Connecticut Margaret Stevenson University of Connecticut School of Medicine Farmington, Connecticut Long Tu, MD University of Connecticut School of Medicine Farmington, Connecticut Hollis A. Viray, MD University of Connecticut School of Medicine Farmington, Connecticut Gillian Weston, MD University of Connecticut Farmington, Connecticut David S. Wong University of Connecticut School of Medicine Farmington, Connecticut Kyle T. Wright, MD, PhD Harvard Medical School Brigham and Women’s Hospital Boston, Massachusetts

PREFACE

Preparing for the United States Medical Licensing Examination (USMLE) Step 1 can be an intimidating and nerve-wracking experience. For one thing, this score actually counts! Although for many of you the most important goal in taking Step 1 is passing this exam, we know that is not enough. Earning a spot in a competitive field or residency program requires you to do more than just pass in order to compete with other high-caliber students. Let us pause right here and take a moment to introduce ourselves. We are the authors of USMLE Step 1 Secrets, 4th edition, and we have one aim in writing this book: We are here to help you earn the highest score you possibly can on this exam.

HOW EARLY SHOULD YOU BEGIN STUDYING FOR THE USMLE? Students frequently ask this question, but unfortunately there is no simple way to answer it. Students commonly allocate anywhere from 2 to 6 months to study for boards, but some take more time and a rare few may need less. The point is that each student should begin his or her preparations at a time that makes sense for that particular individual. When planning your own study schedule, consider how busy you estimate you will be in the months leading up to your exam (do not neglect your coursework!), how many hours per day you are willing to dedicate to productive study time, and how well you think you retain information in the short term versus the long term. Most medical students will have figured out which study styles work best for them long before they even begin to think about boards. Do not change your study habits dramatically for the USMLE if you have found methods that work well for you. 

HOW WILL THIS BOOK HELP YOU PREPARE FOR STEP 1? As you may have already figured out, there are hundreds of review books available to help you prepare for this exam. While the content in these books may overlap quite a bit, the way that material is presented can vary dramatically from resource to resource. The trick to selecting good review books is to purchase a few that mesh well with your learning style and the actual format of the USMLE. The more books you have in front of you, the greater the potential for confusion and the less productive you will feel. In other words, an overabundance of resources eventually will become an impediment to your studying. The most efficient test takers are the students who consolidate their study materials as time goes by. Start with a fresh copy of the newest edition of First Aid for the USMLE Step 1. This will be your primary resource for the USMLE Step 1. Our book is designed to supplement the information that you learn in First Aid and help you place it into clinical context through a mix of basic-concept and case-based questions. The detailed explanations we provide to our questions will offer you insight into the way that the USMLE will expect you to think through questions on test day. In addition, our book will provide you with dozens of valuable study tips (including tips from third- and fourth-year medical students who have earned competitive scores on the USMLE Step 1) to facilitate your studying. We begin each chapter with an insider’s guide that will provide you with our best study strategies for that particular subject. In addition, each chapter includes a number of “Step 1 Secrets” that will point out the highest yield topics to focus on for boards. It is our mission to offer you the type of valuable information that you can really use to boost your score on test day, and you will find it exclusively in the fourth edition of USMLE Step 1 Secrets. 

NOW THAT YOU HAVE SELECTED YOUR RESOURCES, HOW SHOULD YOU GO ABOUT STUDYING FOR THE EXAM?





• Set up a study schedule as early as possible. Determine when you will begin studying, how much time you will dedicate to the exam each week or month, and when you would like to cover specific subject areas in your review process. Keep in mind that you will need the last few weeks before boards to review all of the content that you have studied. • Make a flexible study schedule, especially early in your preparations. Give yourself some free time every day to enjoy other activities and relax your mind. This will increase the productivity of your study time. • Purchase a copy of First Aid for the USMLE Step 1 as soon as possible, and casually review it when studying for your medical school exams, especially during your second year. There is no need to place your emphasis on studying for your board exam before you are ready, but at least familiarizing yourself with First Aid in advance will make you feel much more comfortable when beginning your USMLE studying. • Annotate your copy of First Aid with notes from USMLE Step 1 Secrets and other high-yield resources. All of your notes will therefore be in one place in the weeks leading up to your exam date, and you will have a much easier time getting through all of the material during your final review phase. vii

viii  PREFACE





• Begin using question bank software months before your exam date. Most students use Kaplan Qbank, USMLE World, USMLE Consult, or USMLERx. If you have the time and budget to do so, we recommend purchasing more than one product from the aforementioned list. You can use one program casually (tutor mode) and the other program more intensely (random questions, timed mode) to simulate actual exam conditions. No matter which mode you use, you will benefit from reading all of the answer explanations at the end. Consider marking questions with great learning points or excellent diagrams so that you can easily find them again. Keep in mind that you can download question bank applications for your smartphone. • The night before your exam, try to put your books away and get a good night’s rest. Half of the battle will be keeping your focus through an intense, 8-hour exam day. If you feel the need to study the day before or morning of your exam as a “warm up” or to relieve some anxiety, we recommend going through the high-yield review sections at the end of First Aid or a few of your own notes. You may also consider answering a couple of practice questions, but be wary of looking at the answers at this time in case you get them wrong. Avoid cramming any information (new or old) right before your exam to prevent an anxiety attack. • Most important, try not to worry too much about your score on Step 1. While your board score will be an important factor in your residency application, it is not the only factor. (On the other hand, keep in mind that a good Step 1 score will not make up for poor grades in school.) You would not have gotten into medical school if you were not competent enough to pass this exam. All you need to do is put in the time and effort. 

WHEN WILL I GET MY SCORE? Naturally, this is one of the most frequently asked questions among eager examinees who have completed the USMLE Step 1. Scores are typically made available on the NBME website 3 to 5 weeks after your exam date (lag time is determined by the number of students who have taken the exam during your window). On the morning that your score will be released, you will receive an email from NBME alerting you that your score will be made available that afternoon. Your score report will contain your numerical score and a brief outline of your performance in a broad array of areas. The information provided will be quite similar to the score report you receive if you elect to take a practice NBME exam through the NBME website. 

TEN THINGS STUDENTS WISH THEY HAD KNOWN BEFORE TAKING THE USMLE STEP 1









1. Questions on the USMLE Step 1 are often slightly longer than those found in most question bank programs. Most students finish in time, but keeping on pace will be very important to your success on this exam. 2. Before the start of your exam, you will be given a small whiteboard on which to scribble formulas and perform calculations during your exam. You may take a few minutes before you actually begin your exam to jot down some notes. Determine what you will write on your whiteboard during the final week of your review so as not to waste time during your exam. 3. Anatomy throws many students for a loop on Step 1 because they are often unsure how to prepare for this subject. Be sure to read our “Insider’s Guide to Clinical Anatomy for the USMLE Step 1” in Chapter 26 of this book. 4. You should expect to have a small percentage of questions on topics that you have never before seen or studied. You may also get four to five questions on the same topic. If you do not know the answer, take your best guess and move on. Do not let yourself become flustered or frustrated because you may otherwise miss some easy questions. 5. You are allowed 45 minutes of break time during your exam, but you can gain extra break time by skipping the tutorial (15 minutes; you can watch a similar tutorial on the NBME website before your exam date) or finishing a block before the allotted time expires. Most students find an hour of break time to be adequate, but you should spend some time before your exam planning out how you will allot your time. Do not forget that you are expected to include lunch in your break time. 6. Bring snacks. You will be facing a long day. We suggest that you eat a small lunch and a few snacks in between blocks rather than one big lunch (some students will otherwise become lethargic during the afternoon). Be wary of selecting high-sugar snacks (the last thing you need while taking the USMLE is a sugar crash!). 7. While it is no secret that you should dress in comfortable clothing while sitting for your exam, students often do not know that they should wear as little jewelry and clothing with as few pockets as possible. To prevent the use of prohibited items, most testing centers will scan you with a metal wand and ask you to turn out your pockets each time you re-enter the examination room following a break. Not only is this a frustrating process, but also it is a waste of your break time. You will get through this inspection much more quickly with less jewelry and fewer pockets. Also remember to bring your ID and locker key with you every time you leave the test center. 8. All NBME forms are different! Do not be fooled by students who tell you that their questions were identical to those in the USMLE World, Kaplan Qbank, USMLE Consult, or USMLERx. There is no guarantee that your experience will be the same as theirs. The more questions you do, the better prepared you will be. We recommend that you reserve at least 1000 practice questions to answer in conditions that closely simulate the exam (full blocks, random assortment, and timed mode). 9. When scheduling your exam date, keep in mind that having more time to study will not necessarily improve your performance. Every individual has a peak performance window, and trying to study past this window may hurt your

PREFACE  ix



score. For those of you who have more flexibility than your friends when planning your exam date, be careful about delaying boards for too long. It is possible that you will find it increasingly more difficult to concentrate on studying once your friends have moved past this stage. It is also not advisable to delay your exam too long after you have completed the second year of medical school because you may spend more time relearning the basics. 10. You should arrive at your testing center 30 minutes before your start time. If you speed through the registration process, you may be allowed to begin your examination early. This may be a good option for some students, particularly those who would otherwise spend the time building anxiety. Another good option for anxiety relief is to sign up for a practice examination at your testing site a few weeks before your actual exam date. In order to do this, you must request a permit from the NBME website. 

ONE FINAL NOTE Although it will be a challenging task, studying for the USMLE Step 1 will also be a rewarding experience that will prepare you for a successful transition into your clinical years. Students often say that they feel incredibly accomplished (and intelligent!) after sitting for this examination. We guarantee that your score on the USMLE will greatly reflect the work you put into studying and your attitude about the experience, so aim high and keep your chin up. Before you know it, the USMLE Step 1 will be behind you, and you will be well on your way to a wonderful career in medicine. Wishing you the best of luck,

Thomas A. Brown and Sonali J. Bracken

MORE SECRETS FOR SUCCESS ON THE USMLE STEP 1 Start reviewing early. Take frequent breaks in the weeks leading up to the exam. Bring something really delicious for lunch on test day. JAS, Medical Class of 2016 Stay curious and do not accept facts without reason. Learning why things are the way they are will help with long-term retention of the material. NA, Medical Class of 2017 Try to schedule your test so that you have at minimum a full week of vacation before starting your third year. One extra week of studying will not make or break your score, and you will greatly benefit from having the time to unwind before jumping into your clinical rotations. NR, Medical Class of 2016 Don’t forget to take care of yourself as you study for the big day. Exercise, good nutrition, and enough sleep will ensure adequate brain power when you need it most! MF, Medical Class of 2017 I found it helpful to try to redirect my nervous energy while studying. The phrase “This is a challenge! This is an opportunity! I’m so excited!” became my mantra in the weeks leading up to my exam. It will feel pretty silly at first, but two seconds of internal monologue each morning can really change the way you approach the final stretch! KW, Medical Class of 2017 Consider organizing a regular study group to review core material if these have worked well for you in the past. End your study sessions with questions and images to review as a group. JKE, Medical Class of 2017 Take as many practice tests as you can, and review the answers thoroughly. I found the NBMEs, UWorld Self-assessment exams, and the free 132 questions on the USMLE website to be the most helpful. Take your practice exams at the same time of your actual exam to better train your mind. JW, Medical Class of 2017 Balance your study resources well between books, video/audio lectures, and questions. At different times in the day your mind will me more receptive to one form over another, and this will help you maximize your study time. DM, Medical Class of 2016 There are so many Step 1 study guides on the market, and it is tempting to feel like you should buy them all. Do not get bogged down with too many study materials. Pick a few reputable sources that you like and know them well. HS, Medical Class of 2017 People always wonder which Qbanks to buy and how to use them. This decision should be made according to your individual schedule and pace. It may be helpful to purchase two—one of which you may use casually on your phone or tablet whenever you have down time (e.g., in between classes, in the car, before bed, etc.) and the other of which you only use in timed blocks to simulate the actual exam. PB, Medical Class of 2016

x

MORE SECRETS FOR SUCCESS ON THE USMLE STEP 1  xi Spend twice as much time on your weak areas as you do on your strengths. Turn as many weaknesses into strengths as you can before test day. LB, Medical Class of 2017 Step 1 is a marathon, not a sprint. Be ready to put in some serious work during the 3 to 6 months before taking the exam, and do not attempt to cram. However, if you embrace the challenge and stick to your study plan, you will feel a real sense of accomplishment at the end of the race. TJB, Medical Class of 2017 Do questions, questions, and more questions. Read every explanation no matter whether you got the question right or wrong. ZS, Medical Class of 2016 The worst thing that you can do is compare yourself or your study habits to those of your friends. Stick to what works for you, and do not give much regard to what your classmates are saying or doing to prepare. Paying too much attention to others will simply facilitate anxiety. KJS, Medical Class of 2017 Be sure to allot some time each day for relaxation and health. Exercising, eating well, and socializing with your loved ones will keep your mind sharp and your mood positive. KB, Medical Class of 2016 Don’t ignore your classes just because you are studying for Step 1. The most efficient students will figure out ways to combine studying for their classes with studying for Step 1 throughout the second year of medical school. LH, Medical Class of 2017 Create a weekly study schedule that works for you and stick to it. AML, Medical Class of 2017 Seek patterns in the material that you study. For example, I paid particular attention to the names and side effects of the most common drug classes. This will help you to easily rule out one or two answer choices per question on test day. JBS, Medical Class of 2016 Step 1 is as much of an emotional battle as it is a mental challenge. Expect to go through highs and lows during your preparation time, but be sure to surround yourself with supportive people and let words of confidence sink in every day. SJB, Medical Class of 2017

This page intentionally left blank

     

CONTENTS

CHAPTER 1

CARDIOLOGY  001

CHAPTER 2

PULMONOLOGY  023

Corey Dwyer, Patrick A. Field, MD, Thomas A. Brown, MD, and Sonali J. Bracken

Adam J.S. Kaye, MD, Brian P. Epling, Thomas A. Brown, MD, and Sonali J. Bracken

CHAPTER 3

NEPHROLOGY  048

CHAPTER 4

FLUID AND ELECTROLYTES 

CHAPTER 5

ACID-BASE BALANCE  089

CHAPTER 6

GASTROENTEROLOGY  106

CHAPTER 7

HEPATOLOGY  133

CHAPTER 8

ENDOCRINOLOGY  156

CHAPTER 9

 ALE AND FEMALE REPRODUCTIVE M SYSTEMS  195

Sonali J. Bracken and Thomas A. Brown, MD

CHAPTER 11

Ryan P. Duggan, Thomas A. Brown, MD, and Sonali J. Bracken

Liza Karamessinis, MD, Thomas A. Brown, MD, and Sonali J. Bracken

Apeksha Dave, Thomas A. Brown, MD, and Sonali J. Bracken

Margaret Stevenson, Thomas A. Brown, MD, and Sonali J. Bracken

Henry L. Nguyen, MD, Sarah Cryer, Andrew J. Duarte, Thomas A. Brown, MD, and Sonali J. Bracken

CHAPTER 14

HEMATOLOGIC MALIGNANCIES  328

CHAPTER 15

IMMUNOLOGY  345

CHAPTER 16

PSYCHIATRY  376

CHAPTER 17

NEUROLOGY  400

CHAPTER 18

OPHTHALMOLOGY  437

CHAPTER 19

RHEUMATOLOGY  445

CHAPTER 20

VASCULITIDES  474

CHAPTER 21

BACTERIAL DISEASES  483

CHAPTER 22

V IRAL, PARASITIC, AND FUNGAL DISEASES  520

ONCOLOGY  224

Shirin Karimi, MD, Hien Le, MD, Thomas A. Brown, MD, and Sonali J. Bracken

ANEMIAS  284

Nicole J. Rubin, MD, Hollis A. Viray, MD, Thomas A. Brown, MD, and Sonali J. Bracken

Stephanie Davis, Kaitlyn Ryan LaMarche, MD, Meaghen Roy-O’Reilly, Thomas A. Brown, MD, and Sonali J. Bracken

Stephanie Davis, Meaghen Roy-O’Reilly, Kaitlyn Ryan LaMarche, MD, Thomas A. Brown, MD, and Sonali J. Bracken

Eunice Song, MD, Neda Shahriari, Thomas A. Brown, MD, and Sonali J. Bracken

Yetunde Aseiedu, MD, Joseph M. Ryan, Thomas A. Brown, MD, and Sonali J. Bracken

Thomas A. Brown, MD, Sonali J. Bracken, Matthew Howe, and Lena M. O’Keefe, MS

Sarah E. Conway, Kelsey Sokol, MD, Thomas A. Brown, MD, and Sonali J. Bracken

Martina S. Burn, MD, David S. Wong, Thomas A. Brown, MD, and Sonali J. Bracken

Aaron Lee, MD, Robert D’Angelo, MD, Thomas A. Brown, MD, and Sonali J. Bracken

Deirdre Lewis, MD, Bryan Stenson, Thomas A. Brown, MD, and Sonali J. Bracken

Christopher Del Prete, MD, Maritza Montanez, MD, Thomas A. Brown, MD, and Sonali J. Bracken

GENETIC AND METABOLIC ­DISEASE  253

Samantha Chirunomula, Thomas A. Brown, MD, and Sonali J. Bracken

CHAPTER 12

BLEEDING DISORDERS  314

075

Giana C. Bistany, MD, MS, Thomas A. Brown, MD, and Sonali J. Bracken

CHAPTER 10

CHAPTER 13

CHAPTER 23

PHARMACOLOGY AND TOXICOLOGY  535

Alex M. Hennessey, MD, Thomas A. Brown, MD, and Sonali J. Bracken

xiii

xiv  CONTENTS CHAPTER 24

BEHAVIORAL SCIENCES  559

CHAPTER 25

BIOSTATISTICS  570

CHAPTER 26

Eric Han, MD, Thomas A. Brown, MD, and Sonali J. Bracken

Long Tu, MD, Joravar Dhaliwal, Thomas A. Brown, MD, and Sonali J. Bracken

CLINICAL ANATOMY  587

Matthew N. Anderson, MD, Melina Benson, Thomas A. Brown, MD, and Sonali J. Bracken

CHAPTER 27

DERMATOLOGY  622

CHAPTER 28

PATHOLOGY  639

Melissa Argraves, MD, Gillian Weston, MD, Andrew Kelsey, MD, Thomas A. Brown, MD, and Sonali J. Bracken

Kyle T. Wright, MD, PhD, Thomas J.S. Durant, MPT, MD, Rebecca Flugrad, Thomas A. Brown, MD, and Sonali J. Bracken

INDEX 675

Corey Dwyer, Patrick A. Field, MD, Thomas A. Brown, MD, and Sonali J. Bracken

CHAPTER 1

CARDIOLOGY

Insider’s Guide to Cardiology for the USMLE Step 1 Cardiology is a widely tested subject on the USMLE Step 1, so it is important to achieve a good understanding of both the physiology and pathology of the heart. Become well versed in pressure-volume loops, the Wiggers diagram, and murmurs (you will likely get a few audio questions on the USMLE that will require you to identify valvular defects based on the qualities of the murmurs detected through a movable virtual stethoscope), action potentials of atrial and ventricular myocytes versus pacemaker cells, and common pathologic conditions of the heart (e.g., rheumatic fever, congestive heart failure, cardiomyopathies, endocarditis). Cardiac pharmacology is also a high-yield subject. The majority of cardiology questions on the USMLE will require you to apply concepts rather than facts, so working through the cases in this chapter will be of tremendous value in your preparation for this subject.

BASIC CONCEPTS—HEMODYNAMICS 1. What are the mathematical determinants of the arterial blood pressure?   The mean arterial pressure (MAP) is determined by the amount of blood the heart pumps into the arterial system in a given time (the cardiac output [CO]) and the resistance the arteries exert (total peripheral resistance [TPR]). Mathematically, this is expressed as MAP = CO × TPR. Consequently, all drugs that lower blood pressure (BP) work by affecting either the CO or TPR (or both).     Note: The primary determinant of systolic blood pressure (SBP) is CO, whereas the primary determinant of diastolic blood pressure (DBP) is TPR. Because approximately one-third of the cardiac cycle is spent in systole and two-thirds in diastole, the MAP can be calculated as MAP = 1/3 SBP + 2/3 DBP. 2. What are the primary determinants of cardiac output?   Cardiac output is the amount of blood pumped by the ventricles per unit time. It is determined by the volume of blood ejected during each ventricular contraction (stroke volume [SV]) and how frequently the heart beats (heart rate [HR]), expressed as CO = HR × SV. The HR can be affected by a variety of factors but is principally under the control of the autonomic nervous system. Beta-blockers can reduce CO by decreasing HR and SV.     Note: In addition to their negative inotropic effect, the more cardioselective (nondihydropyridine) calcium channel blockers (CCBs; verapamil, diltiazem) can also reduce HR by slowing impulse transmission through the atrioventricular (AV) node. They achieve part of their antihypertensive effect through this mechanism. 3. What are the three main factors that affect stroke volume?   The determinants of SV are preload, contractility, and afterload. 4. What is preload, and how does it affect stroke volume?   Preload is the degree of tension (load) on the ventricular muscle when it begins to contract. The primary determinant of preload is end-diastolic volume.     The most widely accepted theory explaining the relationship of preload and SV is the Frank-Starling mechanism, which describes that increased preload results in increased SV. One proposed mechanism is that increased end-diastolic volumes result in stretching of ventricular muscle fibers, increasing overlap of actin and myosin within sarcomeres, which allows for a stronger ventricular contraction and larger SV. This mechanism allows the heart to maintain its ejection fraction in the face of increased preload. It is important to note that the Frank-Starling law only holds true in a physiologic range of end-diastolic volumes. Eventually, increasing end-diastolic volumes will result in a decrease in SV. This occurs in congestive heart failure, which is graphically represented in Fig. 1.1.     A general rule is that preload is affected by venous return, and afterload is affected by the arterial system. Preload increases with increased blood volume, sympathetic activation causing venoconstriction, passive leg raising, and decreased intrathoracic pressure (e.g., inspiration). Preload decreases with venodilators (nitrates), decreased blood volume (diuretics), Valsalva maneuver (expiration against a closed glottis), and compression of venous supply (later stages of pregnancy).     Note: Certain maneuvers can be performed to alter the preload or afterload, which will change the intensity of certain murmurs (Fig. 1.2).     Note: Another theory to explain the Frank-Starling relationship proposes that cardiac troponin becomes increasingly sensitive to cytosolic calcium at greater sarcomere lengths, thereby resulting in increased calcium binding and increased force of muscle contraction. 1

2  Cardiology

Ventricular output (L/min)

15 Right ventricle 10

Left ventricle

5

0

-4

0

+4 +8 +12 Atrial pressure (mm Hg)

+16

Figure 1.1.  Increased ventricular output as a function of enddiastolic volume (reflected by atrial pressure). (From Guyton AC, Hall JE. Textbook of Medical Physiology. 11th ed. Philadelphia: WB Saunders; 2006:112.)

Firing frequency

B (exercise) At rest A (stand up rapidly)

Mean arterial pressure (mm Hg)

Figure 1.2.  Control of blood pressure by the baroreceptor reflex. (From Brown TA. Rapid Review Physiology. Philadelphia: Mosby; 2007:144.)

5. What is contractility, and how does it affect stroke volume?   Contractility is a measure of how forcefully the ventricle contracts at a given preload. Naturally, a more forceful contraction will eject a greater fraction of blood from the ventricle, thereby increasing the SV. Contractility is principally influenced by the activities of the sympathetic nervous system (β1-adrenergic receptors) and parasympathetic nervous system (muscarinic [M2] cholinergic receptors) on ventricular myocytes. By antagonizing this sympathetic input to the myocardium, beta-blockers exert part of their antihypertensive effects by reducing contractility, which reduces SV, CO, and oxygen demand. Contractility is also increased by raising concentrations of intracellular calcium (which is indirectly achieved by digitalis and decreased concentrations of extracellular sodium). This mechanism will be explained in further detail in the discussion regarding digitalis. In addition to beta-blockade, contractility is decreased by systolic dysfunction, hypoxia, calcium channel blockade, and acidosis (K+ loss from cells secondary to H+/K+ exchange results in a more negative transmembrane potential that decreases myocyte excitability). 6. What is afterload, and how does it affect stroke volume?   Afterload is the ventricular wall stress that occurs during systolic ejection. Afterload is commonly simplified to the resistance that the ventricles need to overcome to eject blood from the heart (aortic pressure). The aortic pressure is undoubtedly a major factor in determining afterload, but the ventricular wall thickness and chamber radius also contribute. The law of Laplace states that wall stress, or tension (σ), is proportional to pressure (P) and radius (r) and inversely proportional to wall thickness (h). σ ∝



P−r

h [1.1]

    Afterload is most commonly described for the left ventricle, but the same principles can be applied to the right ventricle as in pulmonary hypertension (Table 1.1). 7. What are the primary determinants of peripheral resistance?   The resistance across a blood vessel is a factor of viscosity of the blood (η), vessel length (l), and the vessel radius (r). R = 8ηl/πr4 [1.2]

   

As a review, the total resistance across a network of vessels can be related to the concept of circuits:

In series: Rtotal = R1 + R2 + R3 … In parallel: 1/Rtotal = 1/R1 + 1/R2 + 1/R3 …     The vasculature of the body is composed of many combinations of networks in series and parallel. For the purposes of boards, calculating the total resistance within a network will be simplified to only a few vessels.

Cardiology  3

Table 1.1.   Hemodynamic Alterations Associated with Pathologic States CONDITION

EFFECT ON P, r, OR h

EFFECT ON AFTERLOAD (σ)

Hypertension (systemic HTN, vasoconstric- ↑ P tors)

↑ Afterload

Hypotension (↓ blood volume, vasodilators) ↓ P

↓ Afterload

Left ventricular hypertrophy (LVH) (diastolic ↓ r, ↑ h heart failure)

↓ Afterload However, a common cause of LVH is HTN.

Dilated cardiomyopathy (systolic heart failure)

↑ r, ↓ h

↑ Afterload

Aortic stenosis

↑ P, because of additional transvalvular pressure gradient

↑ Afterload

Pulmonary HTN

↑ P in the right ventricle

↑ Afterload on the right ventricle

h, wall thickness; HTN, hypertension; P, pressure; r, radius; σ, tension; ↓, decreased; ↑, increased.

    The total peripheral resistance (TPR) is the sum of all individual resistances. The TPR is principally mediated by arteriolar diameter, which is modified by arteriolar vasoconstriction and dilation. As seen in Equation 1.2, resistance to blood flow through a vessel is inversely proportional to the fourth power of the radius. Consequently, small changes in arteriolar diameter (and thus radius) can have profound effects on blood flow.     The sympathetic nervous system promotes arteriolar vasoconstriction by stimulating α1-adrenergic receptors, which increases calcium influx (via calcium channels) into arteriolar smooth muscle and stimulates their contraction. Therefore, α1-adrenergic receptors and arteriolar calcium channels are targets for antihypertensive drugs.     In all organs except for the lungs, arteriolar vasodilation is promoted by tissue hypoxia and accumulation of metabolic wastes, such as adenosine, that accumulate when oxygen demand increases (e.g., during exercise). This vasodilation allows supply to meet demand.     Note: In general, there is no direct parasympathetic innervation of the vasculature. However, vasodilation of arterioles can be caused by exogenous cholinomimetic administration. These drugs act on uninnervated muscarinic receptors (M3 receptors) on endothelial cells and stimulate release of nitric oxide. Nitric oxide diffuses to the adjacent smooth muscle, resulting in vasodilation and decreased peripheral resistance. 8. What is the mechanism by which the sympathetic nervous system responds to a reduction in blood pressure?   The baroreceptor reflex is a feedback mechanism that allows the autonomic nervous system to rapidly respond to changes in BP. The purpose is to maintain BP during sudden changes in pressure such as when standing up rapidly. Baroreceptors are stretch receptors located in arteries that adjust the frequency of their signal proportional to the stretch of the vessel. The two main locations of the baroreceptors are at the carotid sinus and the aortic arch, which send signals to the solitary nucleus of the medulla via the glossopharyngeal and vagus nerves, respectively. Increased signals to the solitary nucleus inhibit sympathetic outflow and excite parasympathetic outflow. The carotid sinus baroreceptors are more sensitive and therefore have a stronger effect on changes in BP. The aortic arch baroreceptors are less sensitive and only respond to increases in BP. The two reflex-loop scenarios can be simplified with the following flow charts: ↓ BP → ↓ stretch of carotid sinus baroreceptor → ↓ firing of afferent nerve → ↓ parasympathetic efferent firing and ↑ sympathetic efferent firing → ↑ BP via ↑ contractility, ↑ HR, and ↑ vasoconstriction.     ↑ BP → ↑ stretch of carotid sinus and aortic arch baroreceptor → ↑ firing of afferent nerve → ↑ parasympathetic efferent firing and ↓ sympathetic efferent firing → ↓ BP via ↓ contractility, ↓ HR, and ↓ vasoconstriction (Fig. 1.3).     Note: The aortic arch and carotid sinuses also have chemoreceptors, which should not be confused with the baroreceptors. Chemoreceptors work to maintain Po2, Pco2, and pH. 9. How do the α1-receptor antagonists work?   The α1-receptor antagonists include the “zosins” (prazosin, terazosin, doxazosin) and antagonize peripheral vasoconstriction stimulated by the sympathetic nervous system (which is mediated by α1-receptors). α1-Receptors are located on vascular smooth muscle and coupled to Gq proteins. Antagonists cause decreased release of inositol triphosphate (IP3) and subsequently prevent the release of calcium from intracellular stores, resulting in smooth muscle relaxation and arteriolar vasodilation.     α1-Receptors are also responsible for contraction of the pupillary dilator muscle and intestinal/bladder sphincters. Thus, α1-receptor antagonists can lead to miosis and bladder/bowel movement. 10. How do the α1-receptor antagonists cause orthostatic hypotension?   Upon standing from a supine or sitting position, transient hypotension and lightheadedness (from cerebral hypoperfusion) might occur as a result of venous pooling in the lower extremities, which decreases venous return and MAP. This

4  Cardiology

Medulla oblongata Nerves to brain Carotid artery baroreceptors Carotid arteries delivering blood to the brain

Direction of action potentials

Aortic baroreceptor Aorta

Figure 1.3.  Baroreceptor reflex diagram.

response is ordinarily compensated for by the baroreflex, which promotes peripheral venoconstriction and increased HR. However, if the α1-receptors are blocked in the peripheral venules, this reflex will be less effective at restoring the BP. Nevertheless, a reflex tachycardia, which is mediated by beta-receptors, will be maintained. This increase in pulse rate can be used in diagnosing orthostatic hypotension.     Note: Reflex tachycardia occurs to maintain CO. Recall that CO = HR × SV. Thus, if SV is reduced because of decreased venous return to the heart, HR must increase to maintain CO. 1. What hemodynamic changes occur during exercise? 1   Exercise requires more oxygen to be delivered to skeletal muscle to meet its increased metabolic demand. This delivery is accomplished mainly by an increase in CO secondary to increases in both SV and HR. Contraction of the lower limb muscles pushes blood toward the right atrium and increases venous return. The MAP is only modestly increased during exercise despite the large increase in CO because SVR significantly decreases because of widespread vasodilation in skeletal muscle. Local cellular metabolites (e.g., adenosine, hydrogen ion, CO2) are largely responsible for this vasodilation. 12. Which clinical scenarios would shift the carbon monoxide and venous return curves to the points labeled 1 to 4 in Fig. 1.4? 1.  Exercise: Lower limb muscles push blood toward the right atrium and increase venous return. Sympathetic activity increases CO by increasing HR, SV, and contractility. 2.  Arteriovenous fistulas: Increased venous return from an arteriovenous fistula will shift the venous return curve to the right. CO does increase but only because of the increased preload (Frank-Starling mechanism); this increase is therefore not due to a change in contractility (inotropy). If these arteriovenous anastomoses were much larger, the operating point of the heart would be shifted to (1) because it would cause a large decrease in SVR and stimulate the activity of the sympathetic nervous system, increasing inotropy; these large anastomoses are sometimes referred to as AV shunts. 3.  Compensated heart failure: Patients with this condition have elevated right atrial pressures due to an increased volume status caused by the activity of the renin-angiotensin-aldosterone system (RAAS). Their cardiac function is decreased (decreased inotropy), but they can maintain a normal CO at rest with the increased volume (Frank-Starling mechanism). 4.  Ventricular fibrillation: Ventricular fibrillation causes equalization of all pressures. Right atrial pressure increases to become equal to the mean systolic filling pressure. CO in ventricular fibrillation simply becomes equal to zero. 

Cardiology  5

(+) inotropy

CO or Venous Return

1

CO

2

Normal (CO = VR)

3 (–) inotropy

4 Right Atrial Pressure or LVEDV Figure 1.4.  Cardiac output (CO) and venous return curves. LVEDV, left ventricular end-diastolic volume.

BASIC CONCEPTS—EXCITATION-CONTRACTION COUPLING 1. What is the source of cytosolic calcium during ventricular systole?   During the plateau phase (phase 2) of the ventricular myocyte action potential, voltage-gated calcium channels allow calcium influx from the extracellular fluid into the cytosol, stimulating further calcium release from the sarcoplasmic reticulum, a phenomenon referred to as calcium-induced calcium release. In fact, the majority of the cytosolic calcium comes from the sarcoplasmic reticulum, not the extracellular fluid. This mechanism of calcium release is in contrast to release from skeletal muscle, where depolarization of the cell membrane triggers sarcoplasmic calcium release without entry of extracellular calcium into the cytosol. 2. What is the function of calcium in cardiac muscle contraction?   Cytosolic calcium binds to troponin C, resulting in a conformational change that removes tropomyosin from myosin-binding sites on actin to allow for the sliding filament mechanism of contraction. The force of contraction is proportional to the intracellular Ca2+ level. Note that unlike skeletal muscle, cardiac muscle is dependent on extracellular calcium influx for contraction to occur.     The cardioselective CCBs (verapamil, diltiazem) reduce contractility by antagonizing extracellular calcium entry and the subsequent calcium-induced calcium release that occurs in heart muscle. This action is another mechanism by which CCBs work to lower BP.     Note: The nondihydropyridine CCBs also decrease the HR by suppressing AV node conduction. 3. What is the mechanism by which β-adrenergic stimulation increases cardiac contractility?   Norepinephrine acts on β-adrenergic receptors to activate adenylate cyclase, resulting in an increase in cyclic adenosine monophosphate (cAMP). This promotes cAMP-dependent phosphorylation of a number of proteins via protein kinase A (PKA). Phosphorylation of L-type calcium channels results in increased calcium entry into the myocyte. In addition, β-adrenergic stimulation results in phosphorylation and inhibition of a protein called phospholamban, which normally serves as an inhibitor of the sarco/endoplasmic reticulum calcium adenosine triphosphatase, or ATPase (SERCA). Inhibiting phospholamban increases the calcium reuptake into the sarcoplasmic reticulum, which allows for myocyte relaxation that enables the heart to beat faster. Such rapid ventricular relaxation at elevated HRs is important to ensure adequate ventricular filling during the decreased period of diastole. Overall, the goal of sympathetic stimulation is to increase CO by increasing both HR and SV. 

BASIC CONCEPTS—ARRHYTHMIAS 1. What is the relationship between the various phases of the ventricular myocyte action potential and the different ion fluxes across the cell membrane?   In phase 0 of the action potential, the sharp rise in membrane voltage is due to sodium influx. Phase 1 involves a brief repolarization that is due to the transient outward flow of potassium that follows sodium channel inactivation. In phase 2,

6  Cardiology

Voltage (mV)

the action potential plateaus are due to a balance between calcium influx and potassium efflux. During phase 3, there is rapid repolarization due to unopposed potassium efflux. Phase 4 is the resting potential, which is maintained predominantly through the opening of potassium channels. Intracellular concentrations of K+ are maintained at high levels in cardiac myocytes because of the action of membrane-bound Na+-K+-ATPase. Opening of potassium channels during phase 4 leads to potassium efflux (down its concentration gradient). Since the cell is permeable only to potassium at this time, negatively charged counterions for K+ are unable to diffuse outward with potassium. As potassium leaves the cell, anions left behind cause the cell to become increasingly negative in charge. Therefore the effluxed potassium ions are attracted back toward the interior of the cell to maintain resting potential. Because phase 4 is dominated by potassium permeability, it therefore has a value close to the potassium reversal potential (−85 mV) (Fig. 1.5).

Phase 1 Phase 2 + (Ca2+ influx, K efflux)

Phase 0 (Na+ influx)

Phase 3 (K+ efflux) Phase 4 (Resting potential) Time (msec)

Figure 1.5.  Phases of the ventricular myocyte action potential. (From Brown TA, Brown D. USMLE Step 1 Secrets. Philadelphia: Hanley & Belfus; 2004:77.)

    Note: The antiarrhythmic agents all work by affecting one or more components of the action potential. Class I antiarrhythmics block sodium channels and antagonize phase 0. Class III antiarrhythmics work by blocking potassium channels, which prolongs phase 3 depolarization. Some class IA and all class III antiarrhythmics increase action potential duration as well as the QT interval. Toxicity of these agents can lead to torsades de pointes, which is associated with long QT syndrome. 2. What is the relationship between the various phases of the nodal cell action potential and the ion fluxes across the cell membrane?   Nodal cells possess a unique sodium channel, the If sodium channel, which allows them to spontaneously discharge without stimulation. The If sodium channel is open during the resting phase (phase 4), allowing for sodium influx that causes a gradual rise in membrane potential. Eventually the membrane potential reaches a threshold that triggers influx of calcium through voltage-gated calcium channels (this upstroke is termed phase 0). Nodal cells do not contain a plateau phase; thus phase 2 is absent. Phase 3, or the downstroke, involves inactivation of the calcium channels and opening of potassium channels, causing potassium efflux and repolarization of the membrane (Fig. 1.6).

Membrane potential (mV)

+20

Threshold for discharge

0

Sinus nodal fiber

−40 Ventricular muscle fiber

“Resting potential”

−80 0

1

2 Seconds

3

Figure 1.6.  Rhythmic discharge of a sinus nodal fiber. The sinus nodal action potential is also compared with that of ventricular muscle fiber. (From Guyton AC, Hall JE. Textbook of Medical Physiology. 11th ed. Philadelphia: WB Saunders; 2006:117.)

    Note: The CCBs verapamil and diltiazem (which are nondihydropyridine CCBs) reduce HR by antagonizing these slow calcium channels on the SA node. These drugs are considered class IV antiarrhythmics. 3. Through what mechanism does sympathetic stimulation increase heart rate?   The release of norepinephrine from sympathetic neurons causes activation of β1-adrenergic receptors in nodal tissue. These receptors stimulate production of cAMP, resulting in an increase in If and a positive chronotropic effect on the heart. In essence, sympathetic stimulation increases the cellular influx of sodium ions and decreases the efflux

Cardiology  7

of potassium ions, thus increasing the slope of the resting potential in the nodal cells. When the slope of phase 4 is increased, the threshold for the upstroke is reached faster, resulting in a faster HR. Beta-blockers reduce HR by antagonizing this effect.     Note: The beta-blockers are considered class II antiarrhythmics. 4. What are the classes of antiarrhythmics, and how do their mechanisms of action and potential side effects vary?   See Table 1.2 for this information. Table 1.2.   Antiarrhythmic Drugs PROTOTYPE AGENT(S)

POTENTIAL SIDE EFFECTS

DRUG CLASS

MECHANISM OF ACTION

IA

Inhibits Na+ and K+ channels, prolongs Quinidine, procainamide QRS complex and QT interval, prolongs effective refractory period (ERP)

IB

Inhibits Na+ channels, shortens repolariza- Lidocaine tion, ↓ QT interval

IC

Inhibits Na+ channels, prolongs QRS complex

Flecainide

II

↑ PR interval, ↓ automaticity (↓ slope of phase 4 depolarization in nodal cells)

Propranolol

III

Inhibits K+ channels

Amiodarone

Pulmonary fibrosis, corneal deposits, gray man syndrome, hepatotoxicity, thyroid dysfunction

IV

Inhibits calcium channels, ↑ PR interval, ↓ automaticity

Verapamil, diltiazem

Flushing

Lupus-like syndrome (procainamide), torsades de pointes

* There is considerable overlap regarding the mechanisms of action of these antiarrhythmics. For the sake of simplicity, only the primary mechanism of action is considered in this classification.

CASE 1.1 A 60-year-old Caucasian man presents for his third visit in 2 months with a blood pressure of approximately 155/95 mm Hg on each occasion. Physical examination is unremarkable. A 3-month trial of diet and exercise modifications fails to reduce his blood pressure.   

1. What are the types of hypertension, and which does this patient most likely have?   The types of hypertension are essential (primary, idiopathic) hypertension and secondary hypertension. Essential hypertension is thought to account for approximately 80% to 90% of cases of hypertension. When approaching a patient with hypertension, it is important to consider causes of secondary hypertension. Potential causes of secondary hypertension include renal artery stenosis; primary hyperaldosteronism; Cushing syndrome; pheochromocytoma; coarctation of the aorta; chronic renal disease; excessive alcohol use; pregnancy; increased intracranial pressure; and various medications, such as monoamine oxidase inhibitors (rarely used these days), oral decongestants, nonsteroidal antiinflammatory drugs, and oral contraceptives. If causes of secondary hypertension are ruled out, then essential hypertension is diagnosed by exclusion. 2. How is hypertension defined, and what are the potential complications?   The Eighth Joint National Committee on Detection, Evaluation, and Treatment of High Blood Pressure (JNC 8) guidelines for defining when to treat hypertension are presented in Table 1.3.     Evidence suggests that the risk for complications in hypertensive disease is a continuum, increasing as BP rises. It is also largely influenced by comorbid conditions. Major complications of hypertension include accelerated atherosclerosis, premature cardiovascular disease, diastolic (and to a lesser extent, systolic) heart failure, stroke, intracerebral hemorrhage, chronic renal insufficiency, end-stage renal disease, retinopathy, and acute hypertensive crisis. 3. What medications are the first-line treatments against primary hypertension?   The JNC 8 recommendations of how to treat primary hypertension based on race and coexisting conditions are listed in Table 1.4.     Note: Beta-blockers should not be used as monotherapy for primary hypertension. Beta-blockers decrease BP, but recent studies have shown that they do not adequately prevent stroke compared with other options. Beta-blockers are effective in treating heart failure, atrial fibrillation, angina, and many other conditions.

8  Cardiology Table 1.3.   JNC 8* Guidelines CHARACTERISTIC

BLOOD PRESSURE INDICATION TO TREAT

GOAL BLOOD PRESSURE

≤60 years old Diabetes, any age Chronic kidney disease, any age

Systolic: ≥140 or Diastolic: ≥90

Systolic: 20 mEq/L

5.3

Severe (serum − HCO3 often 2.5 mg/dL and a nontender palpable gallbladder.   

1. What diagnosis is suggested by this presentation, and what are the key pieces of evidence?   Pancreatic carcinoma is the suggested diagnosis. Pain is present in over 70% of cases of pancreatic carcinoma and often radiates to the back. Weight loss is suggestive of underlying neoplasm but may also be related to depressive symptoms often seen in pancreatic cancer. The scleral icterus suggests obstruction of the biliary ductal system. Courvoisier sign describes the finding of a nontender palpable gallbladder and also indicates obstruction of the biliary ductal system. Courvoisier sign is more commonly seen in obstruction by a neoplasm, whereas scleral icterus results from obstruction by any mechanism. 2. Where would a pancreatic mass most likely be located in order to cause biliary obstruction?   A pancreatic mass would most likely be in the pancreatic head. Two-thirds of pancreatic cancers are located in the head of the pancreas, where they can easily obstruct the biliary ductal system. One-third of pancreatic cancers occur in the body or tail, which leads to a more insidious presentation and delayed diagnosis (see question 4). 3. What are important acquired and hereditary risk factors for the development of pancreatic cancer?   Acquired risk factors include age greater than 50 years, male gender, Jewish background, African-American race, cigarette smoking (note that this patient was a smoker), industrial chemical exposure, and diabetes mellitus.     Hereditary risk factors include a family history of pancreatic cancer (7–8% of pancreatic cancer patients have a first-degree relative with pancreatic cancer, versus 0.6% of control subjects) and a familial form of chronic pancreatitis. The following familial cancer syndromes also confer an increased risk of developing pancreatic cancer: • Peutz-Jeghers syndrome • Ataxia-telangiectasia • Hereditary nonpolyposis colorectal cancer (HNPCC) • Familial breast cancer (BRCA2-positive) 4. What is the prognosis for pancreatic adenocarcinoma?   Prognosis is poor. Tumors located in the body or tail have an even poorer prognosis than those located in the head because they often do not produce signs and symptoms until they have invaded adjacent structures. Surgical resection (Whipple procedure) offers a median survival time of 18 months and a 5-year survival rate of approximately 20%. When tumors are not resectable and patients are treated with chemotherapy alone, the 5-year survival rate is less than 5%. Only 10% to 15% of all tumors are resectable at presentation.

Case 10.3 continued: CT of the abdomen reveals a mass at the pancreatic head. The splenic vein appears occluded by clot, and the tumor is compressing the superior mesenteric artery. No metastases are seen. The tumor is deemed unresectable, and he is offered palliative care.   

5. What serum cancer marker is likely to be elevated in this patient?   Serum cancer marker CA 19-9 is likely to be elevated. However, CA 19-9 lacks sufficient sensitivity (50–75%) and specificity (approximately 85%) to be used in the screening of asymptomatic individuals. However, there may be some utility in monitoring postresection levels in order to assess disease status.

234  Oncology     Please note that for screening purposes, the ideal screening test should have high sensitivity/low specificity during initial screening of the population, followed by testing the positive patients with a second test with low sensitivity/high specificity in order to identify the false positives as disease free. 6. What is the most common histologic type of pancreatic cancer?   The most common type of pancreatic cancer is pancreatic adenocarcinoma. In this presentation, neoplastic cells arise from ductal cells of the pancreas. Neoplastic cells will form ductules and may even secrete mucin.     Other types of pancreatic cancers include neuroendocrine tumors and cystic tumors. These tumors are less aggressive than adenocarcinoma. 7. Which genes are most frequently mutated in pancreatic cancer?   The KRAS gene, an oncogene, and the p16 gene, a tumor suppressor gene, are most frequently mutated in pancreatic cancer. The KRAS gene normally is involved in cellular signaling pathways through guanosine triphosphatase (GTPase). In pancreatic cancer (along with colon cancer and lung cancer), it is activated by a point mutation. These mutations cause a deactivation of the protein product’s GTPase activity. As a result, the protein is constitutively active. Ras activates multiple other intracellular signaling pathways.     The p16 gene is a tumor suppressor gene whose gene product is CDK inhibitor 2A. The p16 gene is inactivated in 95% of pancreatic cancers. It also plays a role in melanoma.

Case 10.3 continued: The patient is evaluated 1 month later for right calf pain. A random glucose test (performed via finger stick) reveals a plasma glucose concentration of 180 mg/dL. The posterior right calf is tender and the right calf diameter is enlarged relative to the left calf. A fasting glucose drawn 2 days later is 124 mg/dL.   

8. What two processes have developed in this patient?   Glucose intolerance and deep vein thrombosis (DVT) have developed. Glucose intolerance and often frank diabetes tends to occur with pancreatic cancer and is presumably due to the destruction of insulin-secreting beta cells by the tumor.     This patient has likely also developed a DVT of the calf. Patients with cancer are at an increased risk for clotting due to Virchow’s triad (hypercoagulability, endothelial damage, blood stasis) and frequently develop both deep and peripheral venous thromboses.

Clinical Pearl When migratory peripheral venous thromboses are noted in a patient with pancreatic cancer, it is called Trousseau syndrome or Trousseau sign. Deep venous thrombosis (DVT) and pulmonary embolism are the most common thrombotic conditions in patients with cancer. Trousseau syndrome can be an early sign of pancreatic or gastric adenocarcinoma, as early as months to years before the tumor would otherwise be detected. Thus, if this syndrome is observed, pancreatic and gastric adenocarcinoma should be included in the differential diagnosis and appropriately investigated. Of interest, Dr. Armand Trousseau first described the finding of migratory venous thromboses in himself; he was subsequently found to have pancreatic cancer.

SUMMARY BOX: PANCREATIC CANCER • Presentation: Epigastric pain radiating to the back, weight loss, scleral icterus, Courvoisier sign (nontender palpable gallbladder) • Epidemiology • General risk factors include age >50 years, male gender, Jewish background, African-American race, cigarette smoking, diabetes mellitus • Hereditary risk factors include a family history of pancreatic cancer and a familial form of chronic pancreatitis. • Diagnosis • History and physical exam (see Presentation) • Abdominal computed tomography (CT) scan or endoscopic ultrasound • Labs • Hyperbilirubinemia and abnormal γ-glutamyl transpeptidase and alkaline phosphatase levels • Elevated CA 19-9 levels • Pathophysiology • Most common genetic mutations: KRAS (oncogene), p16 (tumor suppressor gene) • Complications • Glucose intolerance, frank diabetes • Hypercoagulability • Migratory thrombophlebitis (Trousseau sign) • Treatment: Surgical resection (Whipple procedure), chemotherapy, radiation therapy • Prognosis: Poor

Oncology  235

Case 10.4 A 63-year-old man with COPD secondary to smoking is evaluated for a 1-week history of cough. In the past 2 to 3 days he notes that his sputum is blood tinged.   

1. What is the most common cause of hemoptysis?   Acute bronchitis is the most common cause of hemoptysis. Other causes of hemoptysis include lung cancer, pulmonary infections such as tuberculosis, and sarcoidosis. Given this man’s smoking history, lung cancer is a concern.

Case 10.4 continued: Chest x-ray film reveals a right hilar pulmonary nodule and right hilar lymphadenopathy. CT scan reveals a 2.2-cm intraluminal bronchial polypoid mass, multiple enlarged hilar lymph nodes, and no evidence of distant metastases.   

2. Aside from hemoptysis, how else might lung cancer clinically manifest?   Change in character of a “smoker’s cough,” persistent upper respiratory infections or “postobstructive” pneumonias, pleural or pericardial effusions, Horner syndrome, superior vena cava syndrome, asymptomatic pulmonary nodule found on routine chest x-ray, hoarseness, signs and symptoms of metastatic disease, or paraneoplastic syndrome might occur. Paraneoplastic syndromes include Cushing syndrome through adrenocorticotropic hormone (ACTH) secretion in small cell lung cancer, syndrome of inappropriate antidiuretic hormone (SIADH) through antidiuretic hormone (ADH) secretion in small cell lung cancer, and hypercalcemia through parathyroid hormone–related protein secretion in squamous cell lung cancer. 3. What percentage of lung cancer occurs in patients with a significant smoking (or exposure) history?   Approximately 90% of lung cancer occurs in patients with a significant smoking history. There is also a direct association between the frequency of lung cancer and these smoking characteristics: • Amount of daily smoking • Duration of smoking • Tendency to inhale     While cigarette smoking is an enormous risk factor for developing lung cancer, fortunately “only” 10% to 15% of people with high-risk smoking activity develop lung cancer. Of note, approximately 10% of lung cancers occur in nonsmokers or those with a very limited smoking history. 4. What are the four most common histologic types of lung cancers?   Squamous cell carcinoma, adenocarcinoma, small cell carcinoma, and large cell carcinoma comprise more than 90% of cases of primary lung cancer. For purposes of staging and treatment, these types of lung cancers are separated into two broad categories: • Small cell lung cancer (SCLC) • Non–small cell lung cancer (NSCLC), which is more common than SCLC     SCLC is considered separately from NSCLC because it has a different natural history and is therefore treated differently.     The remaining 10% of lung cancers that are not accounted for by the four major categories mentioned here are bronchoalveolar carcinoma (this is often considered to be a type of adenocarcinoma and is associated with nondestructive growth of the tumor along the alveolar architecture); adenosquamous carcinoma; carcinoma with pleomorphic, spindle, or sarcomatous elements; carcinoid tumors; and carcinomas of salivary gland type. Malignant mesothelioma is another type of lung cancer that is popularly tested on boards. This malignancy of the pleura occurs 25 to 40 years after exposure to asbestos and is associated with the formation of psammoma bodies (concentric calcium deposits).     Note: All the above-mentioned types are carcinomas (i.e., of epidermal origin). Other types are more rare (lympho­ proliferative disorders, mesothelial tumors, soft tissue tumors, etc.) and are not discussed. 5.  How do small cell lung cancer and non–small cell lung cancer behave differently in terms of aggressiveness, spread, treatment, and prognosis?   SCLC: Early hematogenous spread is typical. These cancers are rarely resectable or responsive to surgical therapy and are very aggressive, with a median untreated survival time of 6 to 18 weeks. Chemotherapy and radiation therapy are commonly administered.     NSCLC: This form tends to spread more slowly than SCLC and may even be cured if diagnosed in the early stages when local resection may be possible. 6. Which two types are most strongly associated with cigarette smoking?   Squamous cell carcinoma and small cell carcinoma are most strongly associated with cigarette smoking. These are located centrally within the lung tissue. Adenocarcinoma and large cell carcinoma are peripherally located and can arise in nonsmokers.

236  Oncology 7. Describe how the following symptoms or symptom complexes might be produced by local tumor invasion or regional metastases: 1. Superior vena cava (SVC) syndrome SVC syndrome (neck vein distention and facial swelling) is produced by compression of the superior vena cava by an enlarging tumor. SVC syndrome is generally associated with SCLC and presents with puffiness and purple discoloration of the face, arms, and shoulder regions. Fatal complications of SVC syndrome include retinal hemorrhage and stroke. Treatment for SVC syndrome includes radiation therapy and stents to bypass sites of obstruction. 2. Horner syndrome Horner syndrome (ptosis, miosis, anhydrosis) can be caused by invasion of the cervical sympathetic nerves and ganglia. This is most often associated with squamous cell carcinoma (in which the finding will be ipsilateral) and is referred to as Pancoast tumor. 3. Diaphragmatic paralysis Tumor invasion of the phrenic nerve can cause diaphragmatic paralysis. 4. Hoarseness Tumor invasion of the recurrent laryngeal nerve on the left can cause hoarseness. 5. Tamponade, congestive heart failure (CHF) Malignant pericardial effusion can produce these effects.

Case 10.4 continued: Sputum cytologic examination is performed. Results are shown in Fig. 10.7.   

Figure 10.7.  The many well-differentiated foci of eosinophilicstaining neoplastic cells produce keratin in layers (keratin pearls). (From Forbes C, Jackson W. Color Atlas and Text of Clinical Medicine. 3rd ed. St. Louis: Mosby; 2003:211, Fig. 4-184.)

8. What is the likely diagnosis?   Squamous cell carcinoma is the likely diagnosis. This can always be differentiated from other types of lung cancers by the presence of keratin in layers (keratin pearls). Small cell (oat cell) carcinoma is also easy to recognize from the presence of small, blue neuroendocrine cells (Fig. 10.8). Adenocarcinoma is easily classified by the presence of mucinous glands.

Figure 10.8.  Small cell carcinoma, oat cell type. The characteristic features of oat cell carcinoma, including nuclear molding, hyperchromatic granular chromatin, and high nucleocytoplasmic ratios, are seen. Bronchial brushing (Papanicolaou). (From Bibbo M, Wilbur D. Comprehensive Cytopathology. 3rd ed. Philadelphia: Saunders; 2009.)

STEP 1 SECRET USMLE test makers commonly include gross and histologic images of lung tumors on the examination.

Oncology  237

Case 10.4 continued: A polypoid lesion is surgically resected. Adjuvant chemotherapy is started. The patient dies of complications of metastatic disease 6 months later.   

9. To which distant sites does lung cancer commonly metastasize?   Lung cancer commonly metastasizes to the brain, bone, liver, and adrenal glands.

SUMMARY BOX: LUNG CANCER • Presentation: Change in cough, nonpurulent “pneumonia” in an adult, persistent upper respiratory infection, hemoptysis, hoarseness, superior vena cava (SVC) syndrome, Horner syndrome, paraneoplastic syndrome • Epidemiology • Lung cancer is the leading cause of cancer death. Lung cancer has a 16% incidence in men and a 13% incidence in women with a 33% mortality rate in men and 23% in women. • Percent occurring in smokers: 90% (Squamous cell carcinoma [SCC] and oat cell are most associated with smoking.) • Percent occurring in nonsmokers: 10% • Classification • By histology: squamous, small cell, adenocarcinoma • By size: small cell lung cancer (SCLC) and non–small cell lung cancer (NSCLC) • Diagnosis • History: Change in cough, hoarseness, hemoptysis • Exam: Wheezing, signs of bronchial obstruction • Imaging: Chest x-ray, computed tomography (CT) scan • Pathophysiology • Squamous cell carcinoma and small cell carcinoma are located centrally within the lung tissue and commonly result from cigarette smoke exposure. • Adenocarcinoma and large cell carcinoma are peripherally located and can arise in nonsmokers. • Complications: Metastasis (typically to brain, bone, liver, and adrenal glands) • Treatment: Surgical resection, chemotherapy

Case 10.5 A 50-year-old African-American with a history of well-controlled hypertension on hydrochlorothiazide is evaluated for his annual physical. Review of systems is entirely unrevealing. Exam is likewise entirely unrevealing as is his prostate exam. Screening labs reveal a prostate-specific antigen (PSA) of 5.4 ng/mL (normal 10.0 ng/mL will be found to have prostate cancer. Because PSA indicates volume of prostatic tissue, it can be elevated in prostate cancer (because of an increased volume of malignant prostatic cells) or benign prostatic hypertrophy (BPH) (because of an increased volume of benign prostatic cells). It can also be elevated in cases of prostatitis. Minor elevations in PSA values (4.1–10.0 ng/mL) can be associated with both BPH and prostate cancer. Always examine the values of free PSA. BPH is associated with elevations in free PSA values, but prostate cancer is not.

238  Oncology 3. What is a 50-year-old man’s lifetime risk of developing prostate cancer and of dying from prostate cancer?   A 50-year-old man has a 40% chance of developing prostate cancer. However, he has only a 10% chance of developing clinical disease, and he has only a 3% risk of dying from prostate cancer. Because such a small percentage of men with prostate cancer die from prostate cancer, the use of PSA as a screening tool remains controversial. USPSTF recommends against PSA-based screening for prostate cancer.

Case 10.5 continued: The patient undergoes a transrectal ultrasound–guided prostate biopsy. A hyperechoic area in the peripheral zone of the prostate is visualized and needle biopsied.   

4. What is the most commonly found histologic type of prostate cancer?   Adenocarcinomas are the most commonly found histologic type of prostate cancer. This type arises from the glandular acini; however, the regions seen on biopsy are often heterogeneous (i.e., multiple patterns with varying degrees of differentiation can be seen). The Gleason grading system was developed to account for multiple patterns within a single biopsy. A score of 1 (most differentiated) to 5 (no glandular differentiation) is applied to the dominant histologic pattern. A second score of 1 to 5 is applied to the second most abundant histologic pattern. Adding the two scores together gives the Gleason grade (2–10). This is the best marker, along with TNM staging, for predicting prognosis. 5. Why might a patient with prostate cancer present with urinary obstruction? With hematospermia?   Advanced local disease often presents with urinary obstruction as the tumor mass encroaches on the prostatic urethra. If the tumor invades the seminal vesicles, it can cause hematospermia or a decrease in ejaculate volume. 6. Where are the most common sites of metastasis? Which signs and symptoms might indicate metastases in this patient?   Bone and pelvic lymph nodes are the most common sites of metastasis. When affecting bone, prostate cancer most often metastasizes to the axial skeleton including vertebral bodies. As a result, patients with advanced disease may present with lower back or pelvic pain. Osteoblastic metastases are virtually diagnostic for prostate cancer. An increase in serum alkaline phosphatase is also commonly observed. When affecting pelvic lymph nodes, metastases can cause unilateral lymphedema.

SUMMARY BOX: PROSTATE CANCER • Presentations: Most cases of prostate cancer are asymptomatic and detected by abnormal findings on digital rectal examination or an elevated prostate-specific antigen (PSA) level. Prostate cancer spreads by local extension and metastases to bone (especially axial skeleton) and pelvic lymph nodes. • Epidemiology: Common in men older than 50 years old • Diagnosis • Increased PSA and alkaline phosphatase and a needle core biopsy to determine Gleason score • Pathogenesis • 70% of prostate cancer arises from the peripheral zone • Prognosis • Only a small number of men who develop prostate cancer will die as a result of it. • Gleason score: Prognosis of prostate cancer is determined by Gleason grade (2–10) and tumor staging (TNM system).

Case 10.6 A 45-year-old woman who is not closely followed by the medical community is evaluated for postcoital bleeding. Her vital signs are normal.   

1. What is the differential diagnosis for this lesion? • Carcinoma of the cervix: The most common presenting symptom for cervical cancer is irregular vaginal bleeding, particularly postcoital bleeding. • Endocervical polyp: These inflammatory polyps usually occur in the endocervical canal and may extrude from the external os and become visible. They occur in 2% to 5% of adult women and can also present with irregular vaginal bleeding.

Oncology  239

2. Since postcoital bleeding in a woman who does not see a doctor regularly is a red flag for malignancy, what risk factors for cervical cancer do you want to ask her about? • Multiple sexual partners • Early age of onset of sexual activity • Tobacco use • Immunosuppression   Note: In utero diethylstilbestrol (DES) exposure of a fetus is a risk factor for the development of a rare cancer type called cervical or vaginal clear cell carcinoma. 3.  Why is a sexual history particularly important in this patient?   Nearly all incidences of cervical cancer result from prior infection with human papillomavirus (HPV), which is transmitted sexually. HPV infection is very common and can be detected in more than 50% of sexually active women between the ages of 16 and 21. However, only a small percentage of these women develop cervical cancer because HPV infection is usually eradicated by the immune system. Cervical cancer is the result of persistent HPV infection.

Case 10.6 continued: Her history reveals that she has been sexually active since the age of 21 and has had two sexual partners. She has never been pregnant. She had a Chlamydia infection at age 29 that resolved with antibiotics. She does not smoke or take oral contraceptives. She has no family history of gynecologic cancer. Speculum examination reveals a fungating lesion near the external os of the cervix in the 3 o’clock position.   

4. What is the transformation zone of the cervix, and how is it relevant to the development of cervical cancer?   The transformation zone is the area that is most susceptible to neoplastic change. The cervix is a 3- to 4-cm cylindrical, fibrous organ that contains an exocervix, or exterior, composed of squamous epithelium and an endocervix made up of columnar epithelium. The interface of these two types of epithelia is called the squamocolumnar junction, and its location depends on the hormonal status of the patient. Until menarche, the squamocolumnar junction is on the surface of the cervix, meaning that the columnar cells extend out onto the exterior of the cervix. With age, columnar cells on the surface of the cervix begin to change into squamous cells in a metaplastic process. This results in a new squamocolumnar junction that appears to migrate toward the endocervical canal. As this process is occurring, the site of the original squamocolumnar junction remains visible. The area between the original squamocolumnar junction and new squamocolumnar junction is known as the transformation zone, and the cells within this zone are most susceptible to neoplastic changes. For this reason, squamous cell carcinoma of the cervix is most typically found in the transformation zone. Thus, this is the area that Papanicolaou (Pap) smears attempt to screen (Fig. 10.9). Note that Pap smears screen for but cannot diagnose cancer; a tissue biopsy is required for this. 5. Why is it important that this patient has not seen a physician in years?   Since the advent of the Pap smear, the incidence of invasive cervical cancer and related mortality risk has fallen by approximately 80%. Had this patient been seen regularly by a gynecologist, it is likely that she would have had an abnormal Pap smear before developing cervical cancer (the time from development of carcinoma in situ to invasion of the basement membrane is 10 to 20 years or more). 6. What are the current guidelines for cervical cancer screening?   The USPSTF recommends that women have their first Pap smear at age 21. Women ages 21 to age 29 should be screened with a Pap smear every 3 years. Women ages 30 through 65 have the option of being screened every 5 years with a Pap smear and HPV co-testing or every 3 years with a Pap smear alone. HPV testing detects high-risk HPV types. Women who have had a hysterectomy to remove the uterus and cervix do not need to have cervical cancer screening unless the hysterectomy was done to treat a precancerous lesion or cervical cancer. 7. Which types of human papillomavirus are considered high risk for the development of cervical cancer? What types are associated with genital warts?   High-risk HPV types are HPV 16 and 18. Other high-risk types that are not as prevalent include HPV 45, 31, and 33. Lowrisk HPV types associated with condyloma acuminata (genital warts) are HPV 6 and 11. These subtypes do not increase risk of cervical cancer. The HPV vaccine protects against HPV 16, 18, 6, and 11.     Note: The other genital “wart,” condyloma latum, is due to secondary syphilis. 8. What are the current recommendations regarding human papillomavirus vaccination?   The CDC recommends HPV vaccinations for males and females ages 11 to 26. The two types of vaccinations are Gardasil and Cervarix. Both vaccines protect against the high-risk serotypes (HPV 16 and 18). Gardasil also has additional protection against the low-risk strains HPV 6 and HPV 11. 9. Why are Pap smears effective in preventing the development of cervical cancer?   A Pap smear is obtained by direct scraping of the cells in the transformation zone. If abnormal cells are seen, then the patient returns for colposcopy (visual examination of the vaginal and cervical mucosa using a lighted colposcope) and biopsy to determine whether or not the patient has cervical intraepithelial neoplasia (CIN). CIN refers to abnormal growth of cervical squamous cells (cervical dysplasia) and is categorized into grades CIN I, CIN II, and CIN III (Table 10.3).

240  Oncology Uterus Endocervix Columnar epithelium Squamocolumnar junction at exocervix Squamous cells

AT BIRTH

Endocervix Exocervix Squamocolumnar junction Ectropion with exposed columnar epithelium IN THE YOUNG ADULT

Exocervix with "restored" squamocolumnar junction at original site "Transformation zone" with regrowth of squamous epithelium IN THE ADULT

Figure 10.9.  Schematic of the development of the cervical transformation zone. (From Kumar V, Abbas AK, Fausto N, Mitchell RN. Robbins Basic Pathology. 8th ed. Philadelphia: Elsevier; 2007.)

Table 10.3.   Cervical Intraepithelial Neoplasia (CIN) Grades HISTOLOGY GRADE

CORRESPONDING CYTOLOGY

DESCRIPTION

CIN 1

LSIL (Low-grade intraepithelial lesion)

Mild dysplasia Confined to the basal 1/3 of the epithelium

CIN 2

HSIL (High-grade intraepithelial lesion)

Moderate dysplasia Confined to basal 2/3 of the epithelium

CIN 3

HSIL

Severe dysplasia Spans more than 2/3 of the epithelium but does not invade the basement membrane May sometimes be referred to as cervical carcinoma in situ

Oncology  241

    Even if this patient does have CIN, there are methods of treatment (excisional cone biopsy, loop electrosurgical excision procedure) that can excise the lesion and prevent the development of invasive cervical carcinoma.     The abnormal cells that we look for to detect cervical dysplasia are called koilocytes. Koilocytes are squamous epithelial cells that have undergone transformation secondary to papillomavirus infection. You should know how to recognize these cells for boards. They typically present with large, hyperchromatic nuclei with perinuclear halos (Fig. 10.10).

Figure 10.10.  The cytologic appearance of cervical intraepithelial neoplasia as seen on the Papanicolaou smear. Normal cytoplasmic staining in superficial cells may be either red or blue. Image shows low-grade squamous intraepithelial lesion—koilocytes. (From Kumar V, Abbas AK, Fausto N, Aster J. Robbins and Cotran Pathologic Basis of Disease. 8th ed. Philadelphia: Saunders; 2010. Courtesy of Dr. Edmund S. Cibas, Brigham and Women’s Hospital, Boston, MA.)

0. Which HPV-associated oncogenes are responsible for causing cervical cancer? 1   Oncogenes E6 and E7 are responsible for causing cervical cancer. Protein products encoded by these genes interfere with functioning of the tumor suppressor proteins p53 and Rb. Specifically, the E6 protein binds p53 and increases its rate of proteolysis, in effect reducing levels of p53. The E7 protein prevents transcription of the Rb gene by binding and displacing bound transcription factors that are necessary for Rb transcription. This induces CIN, which progresses over time from CIN 1 to CIN 3.

Case 10.6 continued: The patient returns 1 week later for colposcopy and biopsy of the lesion. Pathologic examination shows squamous cell carcinoma, the most common histologic type of cervical cancer.   

STEP 1 SECRET Recall that if you are given an image of squamous cell carcinoma of the cervix, you will be able to spot keratin pearls (see Fig. 10.7).

1. Which symptoms are typically seen in a patient with cervical dysplasia or cervical carcinoma? 1   Many patients are asymptomatic, but some will present with vaginal bleeding (often postcoital), malodorous discharge, or dyspareunia. Advanced tumors can invade through the anterior uterine wall into the bladder and block the ureters. Hydronephrosis and post–renal failure are the most common causes of death in patients with cervical carcinoma.

SUMMARY BOX: CERVICAL CANCER • Presentation: Abnormal vaginal bleeding or postcoital spotting in a middle-aged female. Malodorous discharge and dyspareunia may also be present. • Risk factors: More common in individuals with a history of multiple sexual partners, early age of onset of sexual activity, tobacco use, immunosuppression, and lack of regular checkups • Pathophysiology: Caused by human papillomavirus (HPV); HPV 16 and HPV 18 are high-risk types. • HPV-associated viral oncogenes E6 and E7 interfere with the normal function of tumor suppressor proteins p53 and Rb. • Diagnosis: Presence of koilocytes on Pap smear followed by colposcopy and biopsy for confirmation of cervical intraepithelial neoplasia (CIN) • Complications: Hydronephrosis and post–renal failure (most common causes of death) • Treatment: Lesion excision

242  Oncology

Case 10.7 A 64-year-old man with a recent diagnosis of a right-sided glioblastoma multiforme is evaluated for sudden onset vision changes. Exam reveals a fixed and dilated right pupil.   

1. Which cranial nerve is compressed that is leading to his right-sided fixed and dilated pupil?   Cranial nerve III (oculomotor nerve) is being compressed. This nerve has a somatic component and an autonomic component. Somatic: Supplies four of the six extraocular muscles (superior rectus, inferior rectus, medial rectus, inferior oblique) and the levator palpebrae muscle Autonomic: Parasympathetic innervation of the constrictor pupillae and ciliary muscles     Compression of cranial nerve III typically involves the autonomic as well as the somatic limb. Removal of parasympathetic tone causes fixed dilation of the pupil unresponsive to light or accommodation. A fixed and dilated pupil could be an early sign of uncal herniation secondary to increased intracranial pressure from the glioblastoma. 2. What are the three most common types of primary brain neoplasms in adults, and what are their respective cells of origin? • Meningioma (most common) • Glioblastoma • Other astrocytomas   Meningiomas arise from arachnoidal cells in the meninges (arachnoidal fibroblasts). They are benign in 90% of cases and are usually operable. However, malignant meningiomas can invade adjacent brain tissue. Gliomas include glioblastomas and astrocytomas and arise from the supportive cells (glial cells, astrocytes) of the central nervous system (CNS). The tumor cells invade the surrounding parenchyma and are often associated with areas of necrosis.     Note: You should keep in mind that the most common brain tumors are actually metastatic tumors (most commonly from lung and breast). Metastatic brain cancer typically presents with multiple tumors that are well circumscribed at the gray-white junction. 3. The World Health Organization system for grading astrocytomas is important for understanding the histologic patterns of this common brain tumor. How does glioblastoma multiforme fit into this grading system? WHO grade I: Pilocytic tumors. Most are benign and are cured surgically. They are more common in children than adults. WHO grade II: Diffuse astrocytomas. In these tumors, invading cells can be found in the brain parenchyma in areas distant from the expanding mass, and gray/white matter boundaries may be eliminated. Other features include low cellularity, low nuclear pleomorphism, no endothelial proliferation, and no necrosis. WHO grade III: Anaplastic astrocytomas. These tumors are similar to grade II tumors except that there is much greater mitotic activity histologically. There is increased cellularity, nuclear pleomorphism, and mitotic activity. WHO grade IV: Glioblastoma multiforme. These high-grade tumors are distinguished from anaplastic astrocytomas by the presence of endothelial proliferation or necrosis. Gross specimens show discoloration and cystic changes that result from hemorrhage and necrosis. Glioblastoma multiforme tumors infiltrate the brain quickly and extensively, contributing to their poor prognosis. They are usually found in the cerebral hemispheres and can cross the corpus callosum. 4.  What does the histologic pattern of “pseudopalisading” represent?   In glioblastoma multiforme (Fig. 10.11), tumor cells can be seen to crowd around areas of necrosis, forming the appearance of palisades of cells rimming acellular/necrotic areas.

Figure 10.11.  Glioblastoma with small, anaplastic tumor cells, vascular proliferation, and areas of necrosis with pseudopalisading of tumor cells. (From Goetz CG. Textbook of Clinical Neurology. 3rd ed. Philadelphia: Saunders; 2008.)

Oncology  243

5. Different histologic stains can be used to highlight different types of central nervous system cells. What is the protein expressed in the cytoplasm of astrocytes that pathologists direct antibodies against in order to visualize cells of astrocytic origin?   Glial fibrillary acidic protein (GFAP) is expressed in the cytoplasm of astrocytes.

SUMMARY BOX: BRAIN CANCERS • Epidemiology: Most commonly arise due to metastasis • Complications: Cranial nerve compression, increased intracranial pressure, uncal herniation

Case 10.8 A 24-year-old man is evaluated for a small lump in his neck. He feels well but is concerned because his father, who died from a stroke at age 39 for unclear reasons, required neck surgery in the past. Examination reveals a 1-cm nodule in the right upper lobe of the thyroid. Laboratory workup reveals normal thyroid-stimulating hormone (TSH), moderately elevated calcium, and moderately reduced phosphate. Fine-needle aspiration of the nodule reveals cells suggestive of medullary thyroid carcinoma.   

1. What is the cell of origin in the development of medullary thyroid carcinoma?   The parafollicular cells or C cells are the cells of origin. This is a neuroendocrine tumor that secretes calcitonin, a hormone that lowers serum calcium levels by inhibiting intestinal calcium absorption and increasing calcium excretion into the urine. As a result, measurement of calcitonin can be used in the diagnosis of this tumor and in postoperative follow-up if it is resected. 2. What are the major histologic categories of thyroid cancer?   See Table 10.4. 3.  What test should be ordered to evaluate the hypercalcemia and hypophosphatemia?   Parathyroid hormone (PTH) levels should be evaluated. Keep in mind that someone with medullary thyroid cancer would Table 10.4.   Types of Thyroid Cancers be expected to have decreased to normal serum calcium levels due to increased calcitonin, suggesting that a more complex process RISK is occurring in this patient. PTH levels are often included in the workup of hypercalcemia and are especially HISTOLOGIC INCIDENCE FACTORS

Papillary 75–80%

Follicular 10–20%

Medullary 5%

APPEARANCE

PROGNOSIS

CHARACTERISTICS

Well differentiated Papillae lined by cells with “Orphan Annie eyes” with nuclear grooves Psammoma bodies Cervical lymph node metastasis is common

Good

Well differentiated Papillae lined by cells with clear “Orphan Annie” eyes with nuclear grooves Psammoma bodies Cervical lymph node metastasis is common Good prognosis Associated with a history of childhood exposure to radiation

Older age Well differentiated Diet low in iodine Malignant proliferation of follicles that invade the surrounding fibrous capsule Distinguished from follicular adenoma by invasion of the fibrous capsule

Good

Well differentiated Malignant proliferations of follicles that invade the surrounding fibrous capsule Must distinguish from follicular adenoma, which does not invade the fibrous capsule

Multiple Neuroendocrine tumor endocrine Parafollicular or C cell hyperneoplasia plasia (MEN) familial Malignant cells in an amyloid carcinomas stroma

Good

Neuroendocrine tumor Parafollicular or C cell hyperplasia that secretes calcitonin Pathology shows malignant cells in an amyloid stroma Associated with MEN familial carcinomas

History of childhood head and neck radiation

Continued

244  Oncology Table 10.4.   Types of Thyroid Cancers—cont’d RISK INCIDENCE FACTORS

Anaplastic 75% tubular architecture

Small and pedunculated Make up 90% of adenomas found in the colon

Villous adenomas >50% villous architecture

Large and sessile Make up 1% of adenomas found in the colon

Tubulovillous adenomas 25–50% villous architecture

Varying morphology Make up 5–10% of adenomas found in the colon

7. Which genes are typically altered in the development of colorectal adenocarcinoma?   The APC, KRAS, and p53 genes are altered. There may be as many as nine genes altered for adenocarcinoma to develop, but the first step is thought to be mutation of the APC gene on chromosome 5q. Because APC is a tumor suppressor gene, a mutation must occur in both alleles in order to inactivate it. The second mutation is the socalled second hit. After the second hit on APC, the sequence progresses with an activating mutation of the protooncogene KRAS. Because KRAS is a proto-oncogene, only one allele need be activated for tumor progression (i.e., only one mutation needs to occur). The final step appears to involve loss of function in the p53 tumor suppressor gene (Fig. 10.12).

NORMAL COLON

MUCOSA AT RISK

ADENOMAS

CARCINOMA

Mucosa Submucosa Muscularis propria Germline (inherited) Methylation or somatic (acquired) abnormalities mutations of cancer Inactivation of suppressor genes normal alleles ("first hit") ("second hit") APC at 5q21

APC β-catenin

Protooncogene Homozygous loss of mutations additional cancer suppressor genes Overexpression of COX-2 K-RAS at 12p12

p53 at 17p13 LOH at 18q21 (SMAD 2 and 4)

Additional mutations Gross chromosomal alterations

Telomerase, Many genes

Figure 10.12.  Schematic of the morphologic and molecular changes in the adenoma-carcinoma sequence. COX-2, cyclooxygenase-2; LOH, loss of heterozygosity. (From Kumar V, Abbas AK, Fausto N, Aster JC. Robbins and Cotran Pathologic Basis of Disease. 8th ed. Philadelphia: Saunders; 2010.)

STEP 1 SECRET Genetic mutations involved in cancer are not often tested on boards, although you may randomly get one question on this. You should determine how much time you would like to dedicate to this topic depending on the limitations of your own study schedule. At the minimum, you may want to know which genes are oncogenes and which are tumor suppressor genes. We would also suggest that you briefly look at which tumors are associated with which genes. Test makers will occasionally throw in a difficult question regarding the product of an individual gene, but this is not common. It is unimportant to know the chromosomes on which these genes are located, so do not dwell on this topic.

Oncology  247

8. Without taking into account familial cancer syndromes, what is the importance of family history in the development of colorectal cancer?   A patient who has a first-degree family member who was diagnosed with colon cancer before 45 years of age has a relative risk 3.8 times that of the general population. This relative risk decreases to 2.2 if the family member was diagnosed at 45 to 59 years of age and 1.8 if the family member was diagnosed after 60 years of age. 9.  What is the association of colon cancer with familial cancer syndromes? With inflammatory bowel disease?   Hereditary nonpolyposis colon cancer (HNPCC): Mutations in DNA mismatch repair genes result in “microsatellite instability” and an 80% lifetime risk of developing colon cancer. Microsatellite nucleotide sequences are repeating sequences of noncoding DNA that are maintained during cell division. The integrity of the sequence is referred to as its stability. HNPCC tumors are predominantly right-sided. These patients are also at an increased risk of tumors of endometrial origin (ovarian, stomach, small bowel cancers).     Familial adenomatous polyposis (FAP): Germline mutation of APC tumor suppressor leads to thousands of adenomatous polyps at a young age (i.e., the first hit is inherited). This confers a 100% lifetime risk of developing colon cancer. Other syndromes associated with FAP include Gardner syndrome (FAP, osteomas, and fibromatosis) and Turcot syndrome (FAP and CNS tumors). 0. What is the link between inflammatory bowel disease and colon cancer? 1   Though the link has been more firmly established between ulcerative colitis and colon cancer, it is now evident that both ulcerative colitis and Crohn’s disease confer an increased risk of colon cancer when compared to the general population. This risk begins to rise 7 to 10 years after disease onset and increases with duration of disease. Cancers tend to occur at a younger age than in the general population. There also has been an observed increased risk in small intestinal adenocarcinomas in patients with Crohn’s disease.

Case 10.9 continued: Biopsies reveal adenocarcinoma. As part of a surgical workup, a CT scan of chest, abdomen, and pelvis is performed and reveals metastases to the liver.   

11. Why are colon cancers thought to metastasize to the liver? What are other common sites of metastasis?   Venous drainage of the colon and upper rectum is through the portal vein. Therefore, metastases are often found in the liver of patients with colon cancer. Lymph nodes, lung, and peritoneal metastases are also seen.

SUMMARY BOX: COLON CANCER • Presentation • Left-sided colon carcinomas: decreased stool caliber, left lower quadrant pain, constipation, bowel obstruction, and blood-streaked stool • Right-sided carcinomas: Occult bleeding and iron-deficiency anemia • Epidemiology: Adenocarcinoma is the most common type; rectosigmoid colon is the most common location. • Pathophysiology: Mutation type is dependent on the subtype of colon cancer, but APC, KRAS, and p53 are commonly involved genes. • Diagnosis • Screening colonoscopy started at the age of 50. • Biopsy is required if polyps are found. • Complications: Metastases to the liver

Case 10.10 A 47-year-old man presents to his oncologist with persistent headaches for the past month. Six months prior, he had been treated with cisplatin and etoposide for limited-stage SCLC. He was told that his cancer had gone into remission and was asymptomatic until 1 month ago when he began developing headaches. His wife, who has accompanied him, says that he seems more confused lately. His vital signs are normal. On examination, he has no focal neurologic findings but does have trouble following some directions and seems confused.   

1. What is the most potentially serious explanation for this patient’s headaches?   Metastatic disease to the brain is a potential explanation. The patient’s cancer initially had been classified as limitedstage SCLC, yet it is assumed that micrometastases are present whenever SCLC is diagnosed. Although chemotherapy is

248  Oncology quite successful in limited-stage disease (50–70% complete response), remissions tend to last only 6 to 8 months. When cancer recurs, the median survival time is 3 to 4 months. 2. Which cancers commonly metastasize to brain?   (Listed in order from most to least common) • Lung cancer • Melanoma • Renal cell carcinoma • Breast cancer • Colorectal cancer

Case 10.10 continued: The CT scan shows no brain metastases. The patient’s laboratory tests reveal a serum Na level of 124 mmol/L and a urine Na level of 40 mEq/L. The patient is admitted to the hospital and is noted to have a blood pressure of 124/75 mm Hg that does not change significantly when taken lying down versus standing up. He has no edema. He says he drinks one to two glasses of water per day and stopped drinking alcohol 6 months ago when he was diagnosed with lung cancer.   

3. What basic electrolyte disturbance does this patient have?   This patient has hyponatremia. 4. What is the likely cause of this patient’s electrolyte disturbance?   Syndrome of inappropriate secretion of antidiuretic hormone (SIADH) secondary to SCLC is the likely cause. The patient has a euvolemic hyponatremia based on his lack of orthostatic hypotension. In the context of euvolemia, the kidneys’ ability to excrete free water must be evaluated. The fact that this patient has a high urine sodium indicates that he is reabsorbing free water in the face of excess intravascular free water. 5. What are the causes of SIADH?   ADH is normally secreted by the posterior pituitary gland, and SIADH may occur with CNS lesions (e.g., head trauma, stroke, subarachnoid hemorrhage, hydrocephalus). SIADH may also occur with lung lesions (tuberculosis, bacterial pneumonia, aspergillosis, etc.), but the pathophysiologic mechanism for this is unknown. It is also seen in many malignancies as a paraneoplastic syndrome, or it may be the result of drug effects. 6. To what does the term paraneoplastic syndrome refer?   Paraneoplastic syndrome refers to a constellation of symptoms attributable to the ectopic secretion of peptides or antibodies by a neoplasm. The pathophysiology of paraneoplastic syndromes is incompletely understood. 7.  What paraneoplastic syndromes are particularly important in lung cancer?   Patients with SCLC will classically develop SIADH, and patients with squamous cell carcinoma will develop hypercalcemia as a result of PTHrP. Of note, most cases of hypercalcemia in cancer can be attributed to bone metastases leading to osteolysis as a result of PTHrP secretion. Other tumors associated with paraneoplastic syndromes are listed in Table 10.6.

Table 10.6.   Tumors Associated with Paraneoplastic Syndromes PARANEOPLASTIC SYNDROME

ASSOCIATED CANCERS

ECTOPIC PEPTIDE OR ANTIBODY PRODUCED

Cushing syndrome

SCLC

ACTH or ACTH-like peptide

SIADH

SCLC, intracranial neoplasms

ADH

Polycythemia

Renal cell carcinoma, hemangioblastoma, Erythropoietin hepatocellular carcinoma, pheochromocytoma

Lambert-Eaton syndrome

Thymoma, SCLC

Autoantibodies to presynaptic Ca2+ channels at neuromuscular junction

Carcinoid syndrome

Most common cancer is found in the appendix

Serotonin (5-HT) and bradykinin

Hypercalcemia

Squamous cell lung carcinoma, breast carcinoma, renal cell carcinoma

PTHrP

Zollinger-Ellison syndrome

Pancreatic, duodenal tumors

Gastrin

ACTH, adrenocorticotropic hormone; ADH, antidiuretic hormone; PTHrP, parathyroid hormone–related peptide; SCLC, small cell lung cancer; SIADH, syndrome of inappropriate secretion of antidiuretic hormone.

Oncology  249

STEP 1 SECRET Paraneoplastic syndromes, especially those associated with various types of lung cancers, are among the highest yield oncology topics for boards. 8. This patient was initially treated with cisplatin and etoposide. How do these chemotherapeutic agents work?   Cisplatin is thought to have action similar to alkylating agents. It kills cells in all stages of the cell cycle by inhibiting DNA biosynthesis via intrastrand cross-links. Etoposide inhibits topoisomerase II, thereby resulting in DNA damage through strand breakage. It is specific to late S phase and G2 phase of the cell cycle.

STEP 1 SECRET Students always want to know how much they should learn about chemotherapeutic drugs for boards. First Aid has an excellent list of these drugs, but the various uses for these drugs are quite detailed. We recommend that you approach chemotherapeutic drugs in the following manner: • Begin by learning the various drug classes (antimetabolites, alkylating agents, etc.) and their mechanisms of action. Classify each individual drug according to these groups. • Learn the toxicity of each individual chemotherapeutic agent. “Chemo man,” which is available on the Internet, is a terrific resource to help you undertake this task. Step 1 is fond of testing students on toxicities of chemotherapeutic drugs. • Briefly study the drugs that can help neutralize the toxic effects of chemotherapeutic agents. By far the most important one to know is mesna, which can prevent hemorrhagic cystitis in patients receiving cyclophosphamide. • Believe it or not, learning the clinical uses of chemotherapeutic agents is last on our list. You do not have to know which drugs are used for all types of cancers, but there are a few on which you should focus. We recommend knowing the drugs that are useful for testicular cancer (etoposide, bleomycin, and cisplatin), choriocarcinoma (methotrexate and vincristine/vinblastine), acute myelogenous leukemia (cytarabine), and brain tumors (nitrosoureas). You should also know that 5-fluorouracil can be given topically for actinic keratosis.

SUMMARY BOX: PARANEOPLASTIC SYNDROMES • Presentation: Varies depending on tumor type (see Table 10.6) • Pathophysiology: Ectopic secretion of peptides or antibodies by a neoplasm

Case 10.11 A 45-year-old woman presents with concerns for her risk of developing ovarian cancer. Her mother died at age 60 of ovarian cancer, and she has a sister who was recently diagnosed with breast cancer. She smokes two packs of cigarettes per day and drinks alcohol occasionally. She has three children and had an intrauterine device (IUD) inserted 10 years ago. The patient wants to know what her chances of developing ovarian cancer are.   

1. What is the risk of an American woman developing ovarian cancer in her lifetime? How is this risk altered if the woman has a first-degree relative with ovarian cancer?   Ovarian cancer is the fifth leading cause of cancer death among women in the United States and is the leading cause of death among gynecologic malignancies. A woman’s lifetime chance of developing ovarian cancer is roughly 1.6% but increases to 5% with an affected first-degree relative. 2. Older age and a family history of ovarian cancer in a first-degree relative are major risk factors for the development of ovarian cancer. What is the third major risk factor for the development of epithelial ovarian cancer, and how does it relate to theories regarding the pathogenesis of epithelial ovarian cancer?   Nulligravity is a major risk factor for developing epithelial ovarian cancer (EOC). The predominant theory is that repeated ovulation leads to minor trauma to the epithelial surfaces of the ovaries and that this repeated trauma predisposes the epithelium to malignant transformation. In women who have had children, there have been significant periods in their lives during which they were not ovulating; thus, the epithelial surfaces of their ovaries were not disturbed during this time (decreasing their risk of epithelial malignant transformation). 3. What are the protective factors against epithelial ovarian cancer?   Oral contraceptive pill use, multiparity, tubal ligation, and breastfeeding are protective against EOC. Any factors that inhibit ovulation are protective against EOC because there will be less repeated trauma to the epithelial surface of the ovaries that can potentially lead to malignant transformation. Breastfeeding inhibits ovulation by increasing prolactin levels that suppress the pulsatile release of GnRH.

250  Oncology 4. In a female patient with two first-degree relatives with diagnoses of ovarian and breast cancer, what genetic test is appropriate to determine her risk of developing ovarian or breast cancer?   BRCA gene testing. Germ-line mutation of the BRCA genes is one of the few identified genetic risk factors for ovarian cancer and is associated with both ovarian and breast cancer. Women with a BRCA1 mutation have a 45% lifetime risk of developing ovarian cancer, and those with a BRCA2 mutation have a 25% risk. Though it would be inappropriate to use BRCA testing to screen for ovarian cancer risk, it is justified in this patient, who has two first-degree relatives with ovarian and breast cancer. 5. What laboratory test might be used to follow an ovarian carcinoma once it has been diagnosed?   Cancer antigen 125 (CA-125). The use of both annual CA-125 and transvaginal ultrasonography has been advocated for early detection of ovarian cancer. However, these tests are not thought to be effective in screening the general population. Nevertheless, CA-125 levels correlate well with disease progress and are frequently used to assess response to treatment.

STEP 1 SECRET You should expect at least one question on tumor markers. These are listed for you in Table 10.7.

Table 10.7.   Tumor Markers MARKER

TUMORS/CONDITIONS MONITORED

CA-125

Ovarian cancer

PSA

BPH Prostate cancer

α-Fetoprotein

Hepatocellular carcinoma Yolk sac tumor

CEA

Colorectal cancer Pancreatic cancer

β-hCG

Hydatidiform mole Choriocarcinoma

S-100

Neuroendocrine tumors

Bence Jones proteins

Multiple myeloma Waldenström’s macroglobulinemia

TRAP

Hairy cell leukemia

BPH, benign prostatic hypertrophy; CEA, carcinoembryonic antigen; β-hCG, β-human chorionic gonadotropin; PSA, prostate-specific antigen; TRAP, tartrate-resistant acid phosphatase.

Case 10.11 continued: The patient returns to the office to discuss results of the genetic testing that she requested. She is told that she has a BRCA1 mutation and that as a result, she has an 85% to 90% risk of developing breast cancer and a 45% chance of developing ovarian cancer in her lifetime. She decides to undergo prophylactic bilateral mastectomy and oophorectomy. On pathologic examination of the resected ovaries, she is found to have a 1-cm serous cystadenocarcinoma of the right ovary that had not been seen on preoperative ultrasonography.   

6. What are the three pathologic classifications of ovarian cancer?   Epithelial ovarian carcinoma: EOCs are seen as part of a spectrum of tumors that can arise from anywhere on the epithelial surface of the peritoneal cavity but tend to occur with the most frequency in the ovarian epithelium. The five major pathologic types of EOCs are as follows: • Serous • Mucinous • Endometrioid • Clear cell tumor • Brenner tumor

Oncology  251

    Germ cell neoplasms: Germ cell neoplasms occur much less frequently and are typically found in younger patients. Common types are teratoma, dysgerminoma, endodermal sinus tumor, and embryonal carcinoma. These tumors tend to behave aggressively and can often be cured with surgery and chemotherapy. There is much similarity between these tumors and male testicular cancers.     Stromal tumors: Sex cords in the embryonic gonad eventually develop into the ovarian stroma in women. When undifferentiated, the sex cords can develop into either the specific cell types of men (Sertoli and Leydig cells) or women (granulosa and theca cells). Stromal tumors can also proliferate into granulosa-theca cell tumors, Leydig cell tumors, and Sertoli cell tumors. These neoplastic cells tend to produce the same estrogens or androgens that their precursor cells produce. As a result, feminizing or virilizing effects may be seen (Fig. 10.13).

Nonovarian primary tumor

ORIGIN

Overall frequency Proportion of malignant ovarian tumors Age group affected Types

SURFACE EPITHELIAL CELLS (Surface epithelial–stromal cell tumors)

GERM CELL

SEX CORD –STROMA

METASTASIS TO OVARIES

65–70%

15–20%

5–10%

5%

90%

3–5%

2–3%

5%

All ages

Variable

20+ years • Serous tumor • Mucinous tumor • Endometrioid tumor • Clear cell tumor • Brenner tumor • Cystadenofibroma

0–25+ years • Teratoma • Dysgerminoma • Endodermal sinus tumor • Choriocarcinoma

• Fibroma • Granulosa–theca cell tumor • Sertoli –Leydig cell tumor

Figure 10.13.  Derivation of various ovarian neoplasms and some data on their frequency and age distribution. (From Kumar V, Abbas AK, Fausto N, Aster JC. Robbins and Cotran Pathologic Basis of Disease. 8th ed. Philadelphia: Saunders; 2010.)

7. What is a Brenner tumor?   A Brenner cell tumor is a transitional cell tumor mimicking the epithelium found in the bladder. These rare tumors make up only 2% of all EOCs and are mostly benign. 8. In a patient with a granulosa cell tumor that secretes estrogen, what other malignancy would she be at risk for?   Such a patient would be at risk for endometrial carcinoma. 9. What types of cancers have been found to metastasize to the ovary? What is a Krukenberg tumor?   Breast, colon, gastric, and pancreatic cancers metastasize to the ovary. Krukenberg tumor is the name for a mucinsecreting gastrointestinal cancer that metastasizes to the ovaries. This typically occurs bilaterally. Look for the presence of signet ring cells in the ovary to confirm the diagnosis. 0. What is the primary mode of spread in epithelial ovarian cancer? 1   Neoplasms tend to form within cysts and eventually rupture through the surface of the ovary and spread along the peritoneal surfaces. Tumor cells may also spread through the lymphatics or hematogenously, though these tumors occur after peritoneal spread. Because an ovarian cancer tends to be asymptomatic until it has spread outside the affected ovary, most women are diagnosed in the advanced stages of ovarian cancer when it finally becomes symptomatic.

Case 10.11 continued: The patient has no signs of peritoneal spread of the tumor and makes an uneventful recovery.   

252  Oncology

SUMMARY BOX: OVARIAN CANCER • Presentation: Patients tend to be asymptomatic until more advanced stages. • Risk factors: More common in older women with a history of nulligravity, family history of ovarian cancer in a first-degree relative, obesity, and BRCA mutations. Less common in patients with a history of oral birth control use, multiparity, and breastfeeding. • Pathophysiology: Repeated ovulation leads to minor trauma to the epithelial surfaces of the ovaries. The repeated trauma predisposes the epithelium to malignant transformation. • Complications: Peritoneal metastatic spread, potential risk of breast and ovarian cancer development with BRCA mutations

Samantha Chirunomula, Thomas A. Brown, MD, and Sonali J. Bracken

CHAPTER 11

GENETIC AND METABOLIC DISEASE

Insider’s Guide to Genetic and Metabolic Disease for the USMLE Step 1 Understanding biochemistry and genetics is crucial to achieving a good score on the USMLE Step 1. These subjects lay the foundation for many of the diseases that you are expected to know for boards. This is an intimidating thought for many students who think that they will be expected to memorize a bunch of pathways, enzymes, and intermediates, but this is not the case. Although you are not expected to memorize every step of every biochemical pathway, you should understand the implications of abnormalities in these pathways and how they result in various disease symptoms. Focus on the rate-limiting steps and key enzymes of the pathways that you learn as well as reactions targeted by pharmacologic interventions. Pay attention to where the reactions take place (e.g., cytosol, mitochondrial membrane, mitochondrial matrix, or a combination of the aforementioned locations). More important, you should know how these pathways are regulated.

BASIC CONCEPTS 1. What is an enzymopathy, and how does it result in clinical symptoms?   An enzymopathy is a genetic disease in which a deficiency in activity of an enzyme leads to a block in a metabolic pathway. The altered (usually reduced) enzymatic activity can be due to reduced cellular expression of the enzyme or to expression of a dysfunctional enzyme. The pathologic manifestations of the enzyme deficiency are a result of the accumulation of substrate (or its derivatives) before the blockage, a lack of the product(s), or a combination of both (Fig. 11.1).

Substrate

Product(s) Enzyme deficiency due to mutation

Alternate derivative(s)

Figure 11.1.  Mechanisms by which an enzymopathy produces clinical symptoms. ↑ = increased, ↓ = decreased. (From Brown TA, Brown D. USMLE Step 1 Secrets. Philadelphia: Hanley & Belfus; 2004.)

2. What is the typical pattern of inheritance observed in enzymopathies?   Almost all enzymes are produced in excess of minimal requirements. So although heterozygous carriers of an enzyme deficiency typically have only 50% of normal enzyme activity levels, usually they are phenotypically normal. Thus almost all enzymopathies have an autosomal recessive pattern of inheritance, in which a phenotypic abnormality manifests only when there is nearly no enzyme activity. This generalization is extremely useful for the boards. 3. Explain why the pathologic consequences of X-linked enzymopathies are manifested almost exclusively in males.   Again, enzyme deficiencies generally require a near-total loss of enzyme activity to result in phenotypic abnormalities. Because males have only a single X chromosome, inheritance of a single defective copy of an X-linked gene from the mother will result in the pathologic consequences of the enzyme deficiency/abnormality. Because females have two X chromosomes, female heterozygotes are often asymptomatic because the normal copy of a gene on one X chromosome masks the effect of the abnormal copy on the other X chromosome. Therefore, females will generally exhibit the disease only if they are homozygous for the mutated alleles, which is far less likely. (For example, if the odds of a male inheriting a single defective gene is 1/p, the odds of a female inheriting two defective copies will be approximately 1/p 2.)     Important examples of X-linked recessive enzymopathies include hemophilia A (factor VIII deficiency), hemophilia B (factor IX deficiency), glucose-6-phosphate dehydrogenase deficiency, Duchenne (and Becker) muscular dystrophy, and Lesch-Nyhan syndrome (LNS; hypoxanthine-guanine phosphoribosyltransferase [HGPRT] deficiency).

STEP 1 SECRET The USMLE loves to ask students to calculate inheritance risk. This will require you to know the inheritance pattern of the disease in question and apply it to the Hardy-Weinberg principle. Recall that if a population is in Hardy-Weinberg equilibrium and p is the frequency of the normal allele while q is the frequency of the abnormal allele, disease prevalence = p 2 + 2pq + q 2, where p 2 and q 2 represent the prevalence of homozygosity and 2pq is heterozygosity prevalence (see Case 11.2, question 4). 253

254  Genetic and Metabolic Disease 4. What is the process of lyonization, and how may it cause the manifestation of X-linked diseases in females?   Because females have two X chromosomes, they would have twice the level of expression of genes located on the X chromosome were it not for the random inactivation of one X chromosome that occurs in each somatic cell early in embryogenesis. This process is called lyonization (named after the scientist Lyon, who was the first to propose it). One of the manifestations of lyonization is the Barr body (Fig. 11.2), a condensed, often drumstick-shaped body of DNA seen at the periphery of the nuclei of the cells of females, which corresponds to the inactivated X chromosome. Another nonpathologic manifestation of lyonization is the coloration pattern of calico cats.     Owing to the normally random nature of the X chromosome inactivation, some females may happen to have a mutated X-linked allele on the active X chromosome of a large number of cells and, as a result, may exhibit some pathologic features. These rare individuals are termed mosaics or manifesting heterozygotes. For example, some female carriers of hemophilia A will have some degree of anemia if a large enough proportion of their bone marrow cells inactivate the X chromosome carrying the normal factor VIII allele. Barr body

mat

X inactivation pat mat Xi Expresses maternal alleles

or

X

pat

X X inactivation

Xi

A

B

Expresses paternal alleles

Figure 11.2.  A, Three Barr bodies, indicating four X chromosomes. Three masses of densely staining chromatin material (arrows) are present at the periphery of the nucleus in this cytologic smear. The number of sex chromatin masses is one less than the actual number of X chromosomes. B, Random X chromosome inactivation early in female development. Shortly after conception of a female embryo, both the paternal (pat) and maternal (mat) X chromosomes are active. Within the first week of embryogenesis, one or the other X chromosome is chosen at random to become the future inactive X chromosome through a series of events involving the X inactivation center in Xq13.2 (black box in the schematic). That X becomes the inactive X (Xi indicated by the blue shading) in that cell and its progeny, and it forms the Barr body in interphase nuclei. (Part A from Mutter G, Pratt J. Pathology of the Female Reproductive Tract. 3rd ed. Philadelphia: Churchill Livingstone; 2014, Fig. 2-25. Part B from Nussbaum R, McInnes R, Willard H. Thompson & Thompson Genetics in Medicine. 7th ed. Philadelphia: Saunders Elsevier; 2007: 102, Fig. 6-13.)

5. What is the significance of the autosomal dominant disease pattern of inheritance? What types of diseases are typically inherited in an autosomal dominant pattern?   In autosomal dominant diseases, the disease manifests even though there is a normal copy of the gene remaining that produces 50% of the normal amount of gene product. Dominance of a defective gene can be attributed to one of the following reasons: more than 50% of normal gene product is needed for a nondiseased physiologic state; the defective protein adversely affects the normal gene product (a dominant negative effect); or the defective protein has acquired a novel, detrimental property.     Most diseases caused by mutations in nonenzymatic structural proteins (e.g., collagen, fibrillin) or in membrane receptors (e.g., low-density lipoprotein [LDL] receptor) are inherited in an autosomal dominant manner. This again is a useful generalization for the boards. 6. What is the general relationship between the function of a protein and its pattern of inheritance?   See Table 11.1. 7. What are the following molecular biology diagnostic methods used for? Explain briefly how they work. 1. Southern blotting This technique involves detecting the presence of a specific DNA sequence within a mixture of DNA by using a sequencespecific strand of complementary DNA or messenger RNA (a “probe”) that is able to hybridize to the targeted DNA. The specific steps include separating the mixture of DNA fragments by gel electrophoresis, denaturing the DNA (i.e., altering the DNA solution so that the double-stranded DNA separates into single strands), transferring (i.e., blotting) the DNA onto a membrane, and mixing the blotted DNA mixture with radioactively labeled probes to allow for hybridization. In the laboratory, it is often used to detect the presence of large unique DNA sequences (such as a gene mutation) within a patient’s genome.

Genetic and Metabolic Disease  255

Table 11.1.   Proteins in Enzyme Deficiency FUNCTIONAL CATEGORY*

INHERITANCE PATTERN

EXAMPLE DISEASE(S)

DEFECTIVE PROTEIN

Enzymes

Autosomal recessive

Phenylketonuria (PKU)

Phenylalanine hydroxylase

Galactosemia

Galactose-1-phosphate uridyltransferase

Medium-chain acyl-CoA dehydrogenase (MCAD) deficiency

Medium-chain acyl-CoA dehydrogenase

Tay-Sachs disease

Hexosaminidase A

Thalassemias

α- or β-Hemoglobin

Cystic fibrosis

Cystic fibrosis transmembrane conductance regulator

Osteogenesis imperfecta

Type I and type II collagen

Marfan syndrome

Fibrillin

Hereditary spherocytosis

Spectrin (found in the RBC membrane)

Transport proteins

Structural proteins

Autosomal recessive

Autosomal dominant

Developmental gene expression

Autosomal dominant

Achondroplasia

Fibroblast growth factor receptor 3 (FGFR3)

Metabolic receptors

Autosomal dominant

Familial hypercholesterolemia

LDL receptor

*The information presented conveys the general pattern, but a few exceptions can be found in each category. CoA, coenzyme A; LDL, low-density lipoprotein; RBC, red blood cell.







2. Northern blotting Northern blotting is very similar to Southern blotting, except that a specific sequence of RNA (rather than DNA) is detected using a nucleic acid probe. This technique is commonly used to measure expression of a gene in a patient, as determined by its production of messenger RNA (mRNA). 3. Polymerase chain reaction (PCR) PCR allows for detection of a specific DNA sequence (such as a mutant allele) by making billions of copies of that allele from as little as a single DNA molecule. This test is performed by using two primers, which are complementary to the DNA regions at the ends of the sequence of interest that is to be amplified. There are three main steps in the process. First, the target DNA is denatured. Next, excess premade DNA primers hybridize (or anneal) to a specific sequence of DNA on each strand to be amplified. Finally, the DNA sequence following each primer is extended by a heat-stable DNA polymerase. These three steps are repeated over and over to amplify the amount of target DNA. The number of sequences created can be calculated as 2n, where n = the number of rounds of PCR that have been completed. 4. Western blotting This test is similar to Southern or Northern blotting, but rather than detecting a nucleic acid, it measures the level of a specific protein. First, the protein mixture is coated by a negatively charged detergent molecule that denatures the proteins (i.e., unfolds it into linear peptides) such that the proteins can be separated according to size using gel electrophoresis. Next, the proteins are blotted onto a membrane to which an antibody against the protein of interest (the primary antibody) is added. If the protein is present, the primary specific antibody will bind to the membrane and this binding, in turn, will be detected using a secondary antibody that is both directed against the first antibody and labeled in an assayable fashion. (For example, the primary antibody may be a specific sheep antibody, but the secondary antibody is an antisheep antibody linked to an enzyme that produces a colored product upon exposure to the reagents.) The size or intensity of the band produced is proportional to the amount of protein present, so Western blots are used clinically to measure the degree of protein expression of a gene. This is important because diseases can be caused by translational problems, in which transcription of the gene into mRNA occurs normally but the translation of this mRNA is defective.

Case 11.1 A 2-day-old infant boy tests positive for a relatively rare, but simply managed, medical condition. The diagnosis is based on the presence of markedly elevated serum levels of an essential amino acid. A second positive test result is obtained during his 2-week checkup visit. His family history is remarkable for mental retardation in a 45-year-old aunt.   

1. What is the most likely diagnosis in this baby, and how is it inherited?   The most likely diagnosis is phenylketonuria (PKU), which, as with most enzymopathies, is inherited in an autosomal recessive manner.

256  Genetic and Metabolic Disease 2. What is the major defect and underlying pathophysiology of this disorder?   PKU is caused by the defective conversion of phenylalanine to tyrosine and results from mutations in the phenylalanine hydroxylase (PAH) gene (classic PKU). The PAH enzyme deficiency leads to both an accumulation of phenylalanine (substrate) and its phenylketone derivatives, including phenylacetate, phenyllactate, and phenylpyruvate. There is also a decrease in the levels of tyrosine (product) and its derivatives (such as dopa and melanin). A rarer form of PKU, called malignant PKU, results from a defect in dihydropteridine reductase (DHPR). During the process by which phenylalanine is oxidized to tyrosine, PAH must also oxidize tetrahydrobiopterin (BH4) to dihydrobiopterin (BH2). BH4 not only serves as a cofactor for PAH but is also required for the synthesis of l-dopa from tyrosine. l-dopa is then converted to dopamine, which can be used to synthesize the catecholamines norepinephrine and epinephrine (Fig. 11.3). DHPR is the enzyme responsible for regeneration of BH4 from BH2. In patients with malignant PKU who have DHPR deficiency, BH4 levels are reduced, thus resulting in a functional inability (as opposed to deficiency) of PAH to oxidize phenylalanine to tyrosine. Phenylalanine NADP+ NADPH

Dihydropteridine reductase

BH2

Phenylalanine hydroxylase

BH4 Tyrosine

NADP+ NADPH

Dihydropteridine reductase

BH2

Tyrosine hydroxylase

BH4

Dihydroxyphenylalanine (DOPA) Vitamin Be

DOPA decarboxylase

Dopamine Vitamin C

Dopamine hydroxylase

Norepinephrine S-adenosyl methionine (SAM)

Phenylethanolamine Nmethyltransferase (PNMT)

Epinephrine Figure 11.3.  Biochemical pathway involved in conversion from phenylalanine to epinephrine. BH2, dihydrobiopterin; BH4, tetrahydrobiopterin; NADP+, nicotinamide adenine dinucleotide phosphate (oxidized form); NADPH, nicotinamide adenine dinucleotide phosphate.

    In order to distinguish between classic and malignant PKU, one can examine levels of dopamine and prolactin. Recall that dopamine is a negative inhibitor of prolactin release. Classic PKU does not significantly affect dopamine synthesis, and prolactin levels are therefore relatively normal. DHPR deficiency and subsequent BH4 deficiency, on the other hand, will reduce dopamine synthesis. Thus prolactin levels will be elevated in these patients.     Regardless of cause, the pathology of PKU is primarily a result of substrate (phenylalanine) accumulation, which causes severe neuronal damage, mental retardation, growth retardation, and motor dysfunction. The lack of neurotransmitter compounds derived from tyrosine (particularly the catecholamines dopamine, norepinephrine, and epinephrine) may also contribute to damage of the central nervous system (CNS). Other manifestations include a predisposition to eczema, a “musty” or “mousy” body odor (caused by phenylketone excretion into sweat), and fair skin coloring (due to tyrosine deficiency, which normally serves as a precursor to melanin) (Fig. 11.4).

STEP 1 SECRET The biochemical pathway affected in patients with phenylketonuria (PKU) shows up quite frequently on Step 1. Be familiar with the functions of phenylalanine hydroxylase (PAH) and tetrahydrobiopterin and their relevance to this disease. You should also know every step of the catecholamine synthesis pathway—it is extremely high yield!

3. How is phenylketonuria treated?   Patients with classic PKU need to follow a strict diet that restricts phenylalanine intake and is supplemented with tyrosine. If started within the first month of life, this diet is very effective in preventing mental retardation.

Genetic and Metabolic Disease  257 Dietary intake

Dietary intake

(Substrate) Phenylalanine

(Product) Tyrosine Phenylalanine hydroxylase enzyme deficiency

(Substrate derivatives) Phenylpyruvic acid

DOPA Protein metabolism (in vivo)

Melanins

Adrenaline Adrenal hormones

Figure 11.4.  Pathologic mechanisms of phenylketonuria. (From Brown TA, Brown D. USMLE Step 1 Secrets. Philadelphia: Hanley & Belfus; 2004.)

Because phenylalanine is found in breast milk, most babies suffering from PKU must be placed on special phenylalanine-restricted formulas. Phenylalanine is also found in high concentrations in artificial sweeteners such as aspartame, which must thus be avoided.     Patients with malignant PKU often have additional neurologic problems that do not resolve with dietary phenylalanine restriction alone. This occurs because the BH4 cofactor carries out additional roles such as the hydroxylation of tryptophan, a precursor of serotonin. Therefore individuals with a BH4 deficiency generally require supplementation with neurotransmitter precursors, such as l-dopa, and/or with a commercially available form of BH4. 4. Given the fact that phenylketonuria is a relatively rare condition (prevalence rates range from 1 in 2600 to 1 in 200,000 live births), why does it make sense to screen all neonates for this condition?   The screening test (Guthrie test) is inexpensive (as it simply involves measuring plasma phenylalanine levels), and PKU is easily prevented by dietary modifications. Furthermore, early screening detects the disease before irreparable damage (particularly to the CNS) has occurred (i.e., early intervention affects outcome). Screening should take place 2 to 3 days after birth because phenylalanine levels are often normal immediately following birth as a result of the presence of the maternal enzyme during fetal life. 5. If the parents have a female child with this disease, why is it crucial to advise the child about the risks to her baby if she becomes pregnant when she is older?   As patients generally tolerate more dietary phenylalanine with age, most women with PKU abandon the diet therapy by their early teens, well before they reach childbearing age. This termination of dietary therapy generally has limited ill effects on the women themselves at this age but will cause irreparable harm to a developing fetus should they become pregnant. Specifically, high levels of phenylalanine can diffuse across the placenta, causing brain damage in the developing fetus. So, although these babies will (virtually always) be heterozygous for the PAH mutation and are thus born without PKU, they can exhibit severe mental retardation, microcephaly, growth retardation, and congenital heart defects, a condition termed maternal PKU. 

RELATED QUESTION 6. Why is screening for congenital hypothyroidism (cretinism), congenital adrenal hyperplasia, and galactosemia also routinely performed in newborns?   These diseases are similarly screened for because they are additional preventable causes of mental retardation or death. In general, screening is performed on diseases for which treatment is available, for which a rapid and low-cost laboratory test is available, and that are frequent and serious enough to justify the screening cost.

SUMMARY BOX: PHENYLKETONURIA • Epidemiology: Prevalence ranges from 1 in 2600 to 1 in 200,000 live births • Presentation: Mental retardation, growth retardation, motor dysfunction, eczema, a “musty” odor, and fair skin coloring • Pathophysiology: Defect in the phenylalanine hydroxylase gene (classic PKU) or in the dihydrobiopterin reductase (DHPR) enzyme (malignant PKU). Both instances result in the accumulation of the phenylalanine substrate and its phenylketone derivatives as well as the lack of the tyrosine product and its derivatives. • Diagnosis: Guthrie screening test (measurement of plasma phenylalanine levels in neonate) • Treatment: Low-phenylalanine, high-tyrosine diet; additional supplementation with l-dopa or a tetrahydrobiopterin analog for malignant PKU. Women with a history of PKU who are pregnant or may become pregnant should strictly follow such a diet regardless of their personal symptoms because of the risk of fetal neurologic damage (maternal PKU) resulting from embryonic exposure to high phenylalanine levels.

258  Genetic and Metabolic Disease

Case 11.2 A woman and her husband just gave birth to a child with cystic fibrosis (CF). Both the woman and her husband are in their 30s and are completely asymptomatic.   

1. If the parents decide to have another child, what is the probability of that child having cystic fibrosis?   Because CF is an autosomal recessive disease, the chance of the second child having CF remains at 25%. Each parent is a heterozygous carrier of the mutant allele such that each parent has a 50% chance of passing it on to their offspring, and the chance of the child receiving both mutant alleles is 0.5 × 0.5 = 0.25, or 25%.     Note: Each birth is a completely independent event, such that the outcomes of prior pregnancies do not affect the odds of disease transmission in subsequent pregnancies.

STEP 1 SECRET Students are frequently asked to calculate genetic probabilities on the USMLE.

2. If the parents want to have another child, what kinds of genetic screening methods are available for them to consider?   A few genetic screening methods are available:     The first method involves preimplantation diagnosis using in vitro fertilization (IVF). The zygotes resulting from IVF are allowed to develop into an 8-cell or 16-cell blastomere, from which a single cell is removed. The DNA is isolated from this single cell, and PCR is then used to screen for a mutation in the cystic fibrosis transmembrane regulator (CFTR) gene locus. Only the unaffected embryos (wild type or carrier status) are implanted.     Other genetic screening methods involve prenatal diagnosis using either amniocentesis (the withdrawal of 20–30 mL of amniotic fluid at 15–17 weeks of gestation) or chorionic villus sampling (the aspiration of several milligrams of villus tissues at 10–12 weeks of gestation). 3. Despite having mutations in the same gene, why do patients with cystic fibrosis exhibit significant variability in disease severity?   The most common mutation in CF is the deletion of Phe508 from the CFTR gene (it is not important to memorize this!). However, different patients may have different mutations of the same gene, with certain mutations causing less severe phenotypes. For example, mutations of the chloride channel gene that have a smaller detrimental effect on its function result in milder clinical manifestations. There are over a thousand mutations of CFTR that have been identified in patients with CF. This phenomenon of different mutations of the same allele resulting in differing disease manifestations is known as allelic heterogeneity. Interestingly, many patients with CF (>33%) are compound heterozygotes, with a different locus mutated on each copy of their CFTR genes.     Second, even in patients with identical mutations, there is often some degree of clinical heterogeneity. This may be due to other genetic differences or to environmental variables that influence disease expression.     Note: Many genetic diseases have allelic heterogeneity, leading to significant heterogeneity in clinical manifestations. 4. Assuming a cystic fibrosis prevalence rate of 1 in 2500, what is the carrier frequency for this disease?   The carrier frequency for CF is 4%. Here, the Hardy-Weinberg law can be used to describe the genotypic distribution of an abnormal allele (p + q = 1) and the phenotypic distribution of the disorder:

p 2 + 2pq + q 2 = 1   p = frequency of normal allele q = frequency of abnormal allele (1 − p) p 2 = frequency of unaffected individuals 2pq = frequency of carriers (usually asymptomatic in autosomal recessive diseases) q 2 = frequency of disease        Assuming a CF prevalence of 1 in 2500, q 2 = 1/2500 (0.004) such that q = 1/50 (0.02). Because p + q = 1, p is 0.98. Therefore the carrier frequency for CF is 2pq = 2 (0.98) (0.2) = 0.039, or approximately 4% of the population. Thus in this example, 1 in every 25 individuals is a carrier. This is roughly the carrier frequency in Caucasians, whereas the mutation and the disease are less common in non-Caucasians.     The Hardy-Weinberg law can be applied to alleles and populations that are in “genetic equilibrium” (i.e., populations in which the allele frequency is not undergoing rapid change). Genetic equilibrium exists when there are no mutations at the locus of interest, no selection for specific genotypes, no net migration of the population, when population size is large, and when mating is completely random. For the purposes of the USMLE, usually such equilibrium can be assumed.

Genetic and Metabolic Disease  259

SUMMARY BOX: POPULATION GENETICS AND PRENATAL GENETIC SCREENING • Children born to two carriers of an autosomal recessive mutation have a 25% chance of inheriting the mutation, regardless of the outcome of prior pregnancies. • Genetic screening methods include preimplantation diagnosis using in vitro fertilization as well as amniocentesis (done at 15–17 weeks’ gestation) or chorionic villus sampling (done at 10–12 weeks’ gestation). • The prevalence of various genotypes and phenotypes relating to an allele in genetic equilibrium in a population can be predicted using the Hardy-Weinberg equation.

Case 11.3 A 9-month-old Jewish baby girl is brought to the hospital by her parents. The parents report that over the past 3 months, the baby has been having trouble feeding and has become lethargic and “floppy” appearing. They have also noticed that the child startles easily. More recently, the baby has developed worsening motor dysfunction, now with rigid and spastic movements. The parents are also concerned the baby girl is going blind. Fundoscopic examination results are shown in Fig. 11.5.   

Figure 11.5.  Ocular exam of patient in Case 11.3. (From Martyn LJ. Neurometabolic Disease Affecting the Eye. In: Tasman WJE, editor. Duane’s Ophthalmology, Lippincott Williams & Wilkins: Philadelphia; 2010.

1.  What two diagnoses are top considerations in the differential diagnosis at this point?   A child with progressive neurodegeneration and a funduscopic exam revealing a prominent red macular fovea centralis (sometimes called a cherry-red spot ) should automatically make you think of Tay-Sachs disease (TSD) and Niemann-Pick disease (NPD). Both are autosomal recessive lysosomal storage diseases and, more specifically, sphingolipidoses. TSD is caused by a deficiency of hexosaminidase A, and NPD is caused by a deficiency in sphingomyelinase. The prevalence of both these diseases is higher in Ashkenazi Jews, who are primary descendants of a relatively small group of individuals that broke off from a larger population and bred amongst themselves (thus promoting the amplification of certain deleterious alleles within this population).     Note: Many of the lysosomal storage diseases have multiple subtypes based on the underlying biochemical and molecular characteristics. The syndromes described here correspond to the most common subtype (i.e., type 1 or type A) of each disorder.

Case 11.3 continued: Cells from this child are isolated and examined under the electron microscope, revealing “onion-skinning” of lysosomes.   

2. Now what is the most likely diagnosis?   This feature is associated with TSD but not NPD. On the other hand, “foamy histiocytes” (macrophages filled with sphingomyelin) are found in the tissues of patients with NPD (Fig. 11.6).     The exaggerated startle reaction reported in the initial vignette is also particularly suggestive of TSD. The startle reaction is caused by hyperacusis and, because it does not occur in NPD, can be a major clue to the early diagnosis of TSD. The lack of hepatosplenomegaly on abdominal exam is another clue; sphingomyelin accumulation in hepatic and splenic macrophages can result in hepatosplenomegaly in NPD, but this is not usually the case in TSD.

260  Genetic and Metabolic Disease

Figure 11.6.  Macrophages in Niemann-Pick disease. (From Goljan E. Rapid Review Pathology. 4th ed. Philadelphia: Elsevier; 2013, Fig. 14-15B.)

3. What is the pathogenesis of Tay-Sachs disease?   Hexosaminidase A is a lysosomal enzyme that cleaves a cerebral ganglioside (GM2, a sphingolipid). Mutations make this enzyme less effective, leading to massive accumulation of GM2 and its by-products within the lysosomes of neurons. These lysosomes become enormously enlarged such that they begin to interfere with normal cell function and ultimately cause neuronal death. Neuronal death that overlies the fovea centralis of the retina is responsible for the cherry-red spot seen on funduscopic examination. More specifically, ganglion cells in the retina fill with lipid, which imparts an opaque gray color to the retina. Because the optic disk does not contain ganglion cells, it remains red on a gray background, giving the look of a cherry-red spot (see Fig. 11.5). This is a common buzzword used by USMLE test makers.     The pathogenesis of other sphingolipidoses such as NPD or Gaucher disease is similarly due to accumulation of substrates of lysosomal enzymes. The differing disease manifestations of these sphingolipidoses depend upon the organs in which the sphingolipids accumulate and the underlying organ sensitivities. As mentioned previously, visceral sphingolipid accumulation and hepatosplenomegaly are prominent in NPD but not in TSD. TSD, in contrast, is characterized by relatively isolated CNS sensitivity to GM2 accumulation.     Note that for both NPD and TSD, treatment options are limited and both disorders are usually fatal within the first few years of life. 4. Which is the most common sphingolipidosis?   Gaucher disease is not only the most common sphingolipidosis but also the most common lysosomal storage disease (note that lysosomal storage diseases include both sphingolipidoses and mucopolysaccharidoses, which are discussed below). The disease is caused by deficiency of glucocerebrosidase and is characterized by the presence of glucocerebroside-laden Gaucher cells (Fig. 11.7), macrophages with characteristic “crumpled-tissue paper” appearance and nuclei displaced by lipid. It is also more common among Ashkenazi Jews.

Figure 11.7.  Gaucher cells. (From Goljan E. Rapid Review Pathology. 4th ed. Philadelphia: Elsevier; 2013, Fig. 14-15A.)

    Unlike TSD and NPD, Gaucher disease usually spares neuronal tissue. Glucocerebroside instead accumulates in the reticuloendothelial system, namely, the spleen and liver (causing hepatosplenomegaly), bone (causing bone pain and fractures), and bone marrow (causing pancytopenia, with particularly prominent thrombocytopenia). The two main contributors to thrombocytopenia are (1) decreased platelet production due to accumulation of Gaucher cells in the bone marrow and (2) entrapment of platelets in the spleen, which is overactive and enlarged with Gaucher cells.     Note: Despite the characteristic histologic or cytologic findings of many storage diseases, virtually all of these disorders are today diagnosed using testing for the underlying specific genetic defects.

Genetic and Metabolic Disease  261

5. What are the mucopolysaccharidoses?   Mucopolysaccharidoses are a different type of lysosomal storage disease in which the substrates that accumulate in the lysosomes are extracellular matrix molecules called glycosaminoglycans (which were previously known as mucopolysaccharides). Like the sphingolipidoses, these diseases are caused by hereditary deficiency of lysosomal enzymes. The two main examples of this type of disease, Hurler syndrome and the similar but less severe Hunter syndrome, are both caused by accumulation of the glycosaminoglycans heparan sulfate and dermatan sulfate.     To keep these two disorders straight, think of a male HUNTER (X-linked) with aggressive behavior and great vision (no corneal clouding). The only other commonly tested lysosomal storage disease that is X-linked recessive is Fabry disease (α-galactosidase A deficiency, resulting in accumulation of ceramide trihexoside). Look for peripheral neuropathy, angiokeratomas (dark red skin rash), and cardiovascular/renal involvement in a patient presenting with Fabry disease on the USMLE Step 1. 6. Quick review: Cover the three columns on the right side of Table 11.2, and attempt to describe the enzyme deficiency, accumulated substrate, inheritance pattern, pathophysiology, and any high-yield associations for the listed lysosomal storage disorders. Table 11.2.   Lysosomal Storage Diseases LYSOSOMAL STORAGE DISEASE

ENZYME DEFICIENCY/ ACCUMULATED INHERITANCE PATHOPHYSIOLOGY AND HIGHSUBSTRATE PATTERN YIELD ASSOCIATIONS

Sphingolipidoses Tay-Sachs disease

Hexosaminidase A/GM2 ganglioside

Autosomal recessive

Accumulation of cerebral ganglioside causes progressive psychomotor deterioration, macular cherry-red spot, and lysosomes with onion-skinning.

Gaucher disease

Glucocerebrosidase/ glucocerebroside

Autosomal recessive

Gaucher cells (enlarged lipid-laden histiocytes with “wrinkled tissue paper” cytoplasm) accumulate in bone, marrow, liver, and spleen, causing bone pain and fractures (bone crises), osteonecrosis of femoral head, massive HSM, and pancytopenia.

Niemann-Pick disease

Sphingomyelinase/ sphingomyelin

Autosomal recessive

Sphingomyelin accumulation in neurons and liver/spleen causes progressive psychomotor dysfunction, macular cherry-red spots, “foamy histiocytes,” and HSM.

Fabry disease (angiokeratoma corporis diffusum)

α-Galactosidase A/ ceramide trihexoside

X-linked recessive

Accumulation in the vascular endothelium results in “Maltese crosses” (fat bodies) in urine and renal disease, angiokeratomas, burning peripheral neuropathy, stroke, and cardiovascular disease.

Krabbe disease (globoid cell leukodystrophy)

β-Galactosidase/ceramide galactoside (i.e., galactocerebroside)

Autosomal recessive

Demyelination and accumulation of globoid cells in CNS result in optic atrophy, peripheral neuropathy, and psychomotor retardation.

Metachromatic leukodystrophy

Arylsulfatase A/ cerebroside sulfatides

Autosomal recessive

Sulfatide accumulation and central and peripheral demyelination result in ataxia and psychomotor degeneration and dementia in adults.

Mucopolysaccharidoses Hurler syndrome (mucopolysaccharidosis type I H)

α-l-Iduronidase

Autosomal recessive

Accumulation of glycosaminoglycans leads to coarse facial features (gargoylism), hepatosplenomegaly, mental retardation, joint and skeletal abnormalities, cardiac disease, and corneal clouding.

Hunter syndrome (mucopolysaccharidosis type II)

Iduronate sulfatase

X-linked recessive

Same features but with milder mental retardation with aggressive behavior and no corneal clouding

CNS, central nervous system; HSM, hepatosplenomegaly.

262  Genetic and Metabolic Disease

SUMMARY BOX: TAY-SACHS DISEASE • Epidemiology: Most common within Ashkenazi Jews • Presentation: Floppy-appearing baby with lethargy, feeding trouble, visual defects, and signs of neurodegeneration/motor dysfunction; may be easily startled • Pathophysiology: Mutation of hexosaminidase A gene, leading to accumulation of GM2 ganglioside in neuronal lysosomes • Diagnosis: Fundoscopic exam (macular cherry-red spot), abdominal exam (absence of hepatosplenomegaly, as opposed to Niemann-Pick disease), genetic testing, “onion-skinning” of lysosomes under electron microscopy • Prognosis: Fatal in early childhood

Case 11.4 A 2-year-old boy was brought to the clinic with choreoathetosis (constant and involuntary writhing movements of the legs and arms), spasticity (muscular hypertonicity with increased tendon reflexes), impaired cognitive development, and self-mutilation (compulsive biting of the fingers, lips, tongue, and inside of the mouth). The parents also observed the presence of orange “sand” in the child’s diapers when the boy was a few months old.   

1. What is the most likely diagnosis?   The most likely diagnosis is Lesch-Nyhan syndrome (LNS), a rare X-linked recessive disease that is caused by a defective HGPRT (hypoxanthine-guanine phosphoribosyltransferase) enzyme. The HGPRT enzyme is present in most cell types and is involved in the salvage pathway of purine metabolism. The most striking and characteristic neurologic symptom of this disease is self-mutilation.     Recall that the purine bases are adenine and guanine and the respective nucleosides are adenosine and guanosine. A nitrogenous base linked to a sugar ribose or deoxyribose is referred to as a nucleoside, whereas a phosphorylated nucleoside is referred to as a nucleotide.     The mnemonics “PURe As Gold” and “CUT the PY” can be used to remember that Adenine and Guanine are Purine bases, whereas Cytosine, Uracil, and Thymine are PYrimidines. 2. What is the normal function of the purine “salvage” pathway?   The purine salvage pathway functions to “salvage” purine metabolites such as hypoxanthine and guanine, preventing them from being unnecessarily degraded and then renally excreted as uric acid. (Hypoxanthine is another purine that is an intermediate in the synthesis or degradation of adenosine monophosphate [AMP] or guanosine monophosphate [GMP].) As shown in Fig. 11.8, the salvage pathway recycles these metabolites to replenish the purine bases guanine and adenine by the action of the HGPRT enzyme. Normally, the de novo pathway provides only about 10% of the daily purine requirement, whereas the salvage pathway provides the remaining 90%. The amount of net degradation to uric acid is always balanced with the amount of purines synthesized via the de novo pathway. It follows that the loss of the salvage pathway would result in a dramatic increase in de novo purine synthesis and a similarly dramatic increase in uric acid generation. Nucleic acids

Nucleic acids

Guanylic acid (GMP)

HGPRT + PRPP

Inosinic acid (IMP)

Guanosine

Inosine HGPRT + PRPP

Guanine

Hypoxanthine

Xanthine

Adenylic acid (AMP)

Adenosine deaminase

Adenosine

APRT + PRPP

Adenine

Xanthine oxidase (XO)

Xanthine oxidase (XO) Uric acid Figure 11.8.  Purine metabolism. APRT, adenine phosphoribosyltransferase; HGPRT, hypoxanthine-guanine phosphoribosyltransferase; PRPP, 5′-phosphoribosyl-1-pyrophosphate.

Genetic and Metabolic Disease  263

3. How do defects in the purine salvage pathway cause hyperuricemia?   In LNS, HGPRT activity is less than 1% of normal. Owing to the absence of HGPRT, the ability to reuse hypoxanthine and guanine to make the purine nucleotides inosinic acid (IMP) and guanylic acid (GMP) is lost, so these intermediates are degraded to uric acid. As explained in the preceding question, the purine requirements in this case must be met by increased de novo synthesis. Additionally, because of the reduced levels of IMP and GMP, the feedback inhibition normally exerted by IMP and GMP upon the de novo pathway is lost, even further promoting the activity of the de novo synthesis pathway. Because purine synthesis via the de novo pathway must be balanced by purine degradation into uric acid, the dramatic increase in de novo synthesis results in severe hyperuricemia.     In other words, both excessive activation of the de novo pathway and insufficient HGPRT salvage of purine metabolites contribute to the hyperuricemia in LNS. 4. What was the orange “sand” in his diapers observed by his parents?   The sand represents uric acid crystals. Uric acid has limited solubility such that in conditions of extreme hyperuricemia, it will precipitate from urine, forming visible orange “sand.” It can also precipitate from the plasma and accumulate in the joints, causing gouty arthritis. Interestingly, most patients with LNS do not develop gout, presumably because of the patients’ short life span (of about 20 years). However, patients with only a partial deficiency of HGPRT (with 1%–20% of normal activity) have a normal life span but are susceptible to developing severe tophaceous gout. 5. Why do boys with Lesch-Nyhan syndrome typically present with renal dysfunction?   Patients with LNS develop kidney disease primarily from repeated uric acid kidney stones and urinary tract obstruction. In addition to nephrolithiasis, chronic hyperuricemia (from any cause) can result in renal insufficiency caused by urate deposition in the renal parenchyma, a process referred to as urate nephropathy.     Note: In contrast with the renal and joint disease, the cause of the neurologic symptoms in LNS is not well established. 6. How might this patient be managed pharmacologically?   Allopurinol is useful in the treatment of hyperuricemia of any cause. It works by preventing uric acid production by inhibiting the enzyme xanthine oxidase (XO). The xanthine and hypoxanthine that accumulate instead are more soluble and readily excreted than uric acid.     Other, more common uses of allopurinol include the treatment or (more commonly) the prevention of urate nephropathy, uric acid stones, gouty arthritis, and tumor lysis syndrome (which is caused by treatment of acute leukemias or disseminated lymphomas).     For patients with LNS who cannot tolerate allopurinol, a newer drug called febuxostat is a potential alternative. Febuxostat also works by inhibiting XO.

SUMMARY BOX: LESCH-NYHAN SYNDROME • Epidemiology: Prevalent in males (X-linked disorder) • Presentation: Mental retardation, motor dysfunction (choreoathetosis and spasticity), self-mutilating behavior, presence of orange “sand” in diaper • Pathophysiology: Defect in the purine salvage pathway (HGPRT enzyme), resulting in overactivity of the de novo purine synthesis pathway and excess generation of uric acid • Treatment: Allopurinol or febuxostat (xanthine oxidase inhibitors) to reduce uric acid production • Complications: Severe hyperuricemia can lead to tophaceous gout, uric acid kidney stones, and urate nephropathy • Prognosis: Shortened life span (about 20 years)

Case 11.5 A 4-year-old boy is evaluated for profound hypoglycemia and seizures. Examination is remarkable for nontender hepatomegaly. He has a history of multiple hospitalizations for seizures and hypoglycemia since he was 6 months old. His parents have noticed that he has never tolerated even short periods of fasting well. In his previous hospital stays, low blood sugar levels were consistently observed within a few hours after each feeding. He has also repeatedly had lactic acidosis, hyperlipidemia, and hyperuricemia. A liver biopsy indicates excessive accumulation of glycogen and fat.   

1. What is the diagnosis?   The diagnosis is type 1 glycogen storage disease (or von Gierke disease), an autosomal recessive disorder, which is caused by deficiency of the enzyme glucose-6-phosphatase (G6Pase). 2. What type of enzymatic deficiency is present in all types of glycogen storage diseases?   These disorders are caused by a defect in either an enzyme required for glycogen synthesis or an enzyme required for glycogen catabolism (i.e., glycogenolysis) called G6Pase. Glycogen storage diseases principally affect either the liver or skeletal muscle, which are the main sites where glycogen is stored. When they affect the liver, they can lead to hepatomegaly and can predispose to hypoglycemia and its attendant complications (e.g., seizures and, with repeated episodes

264  Genetic and Metabolic Disease of hypoglycemia, neurologic impairment). When they affect the skeletal muscles, they can cause muscle pain and exercise intolerance, but they do not result in hypoglycemic episodes because skeletal muscle plays no role in maintaining plasma glucose. Recall that skeletal muscle lacks G6Pase, so it cannot deliver glucose to the bloodstream because glucose-6-phosphate (G6P) is unable to cross the plasma membrane. 3. How is glycogen normally synthesized and degraded in the liver?   Upon entry into a liver cell, glucose is prevented from diffusing out by phosphorylation to G6P, a reaction catalyzed by the enzyme glucokinase (this function is served by hexokinase in nonhepatic tissues). G6P is then converted into glucose1-phosphate (G1P) by phosphoglucomutase. Next, G1P is converted to uridine diphosphoglucose (UDP-glucose), and UDP-glucose is attached to an existing glycogen molecule by the enzyme glycogen synthetase. Glycogen synthetase joins carbon 1 of UDP-glucose to carbon 4 of a glycogen molecule, creating α-(1,4) linkages between the molecules. Finally, there is a branching enzyme that breaks the α-(1,4) bonds and carries the broken glycogen chain to carbon #6, forming α-(1,6) bonds and giving glycogen its characteristic branched structure.     Glycogen degradation is primarily dependent on the activity of the enzyme glycogen phosphorylase, which catalyzes the breakdown of glycogen into G1P. However, glycogen phosphorylase only acts on nonreducing ends of a glycogen chain that are at least five glucoses away from a branch point. After this, another enzyme called the debrancher enzyme transfers a trisaccharide from an α-(1,6) branch to an adjacent α-(1,4) branch. The remaining glucose molecule at the branch point is then released as free glucose. Therefore the two products of glycogen breakdown are G1P and glucose (Fig. 11.9).

Glucose-6-phosphate Phosphoglucomutase Glucose-1-phosphate UDP-glucose pyrophosphorylase UDP-glucose Glycogen synthetase

Storage form of glycogen Branching enzyme

Glycogen phosphorylase

Debranching enzyme

Debranching enzyme Figure 11.9.  Glycogen synthesis and degradation. UDP-glucose, uridine diphosphoglucose.

Genetic and Metabolic Disease  265

4. How is glycogen breakdown regulated?   Note that the regulation of glycogen breakdown is mediated by multiple substances, including epinephrine and glucagon. Both activate the enzyme adenylyl cyclase to generate cyclic adenosine monophosphate, which in turn activates protein kinase A. Protein kinase A converts an enzyme called glycogen phosphorylase kinase from the inactive to the active form. Glycogen phosphorylase kinase goes on to convert glycogen phosphorylase from the inactive to the active form, so glycogen breakdown can occur. Calcium and calmodulin in skeletal muscle also convert glycogen phosphorylase kinase to the active form, allowing glycogenolysis to be coordinated with muscle activity. It is important to note that insulin also modulates this pathway and has the opposite effect of epinephrine and glucagon; it activates protein phosphatases that convert glycogen phosphorylase kinase and glycogen phosphorylase to their inactive forms, thus preventing glycogen breakdown (Fig. 11.10). Glucagon (liver)

Ca2+/calmodulin in muscle

Glycogen phosphorylase kinase (inactive)

Epinephrine (liver and muscle)

Adenylyl cyclase cAMP

Protein kinase A Protein phosphatase

Pi Glycogen phosphorylase kinase (active) Glycogen phosphorylase kinase (inactive)

Protein phosphatase

Glycogen phosphorylase kinase (active)

Pi

Insulin Figure 11.10.  Regulation of glycogen degradation. cAMP, cytosolic adenosine monophosphate.

5. Why is hepatomegaly seen on examination?   The deficiency of G6Pase causes G6P to accumulate, which stimulates glycogen synthesis and in turn enlarges the liver. 6. What is the explanation for this patient’s severe fasting hypoglycemia and lactic acidosis?   During short-term fasting, liver glycogenolysis is the major pathway that maintains blood glucose. When G6Pase is deficient, glycogenolysis is not effective at releasing glucose into the bloodstream, leading to hypoglycemia. In addition, the release of glucose made by gluconeogenesis during fasting is also impaired because this process is also dependent upon the enzyme G6Pase. This further contributes to hypoglycemia. Excessive accumulation of G6P greatly promotes glycolysis, resulting in high levels of pyruvate production. The pyruvate is then converted into lactate when the mitochondrial uptake of pyruvate is saturated, resulting in lactic acidosis.     For your own review, recall that the production of lactic acid from pyruvate does not cause any additional increase in production of adenosine triphosphate (ATP). The function of this pathway is to simply regenerate the electron carrier NAD+ from NADH. 7. Why is this patient susceptible to hypertriglyceridemia?   Excessive accumulation of G6P overstimulates hepatic glycolysis, supplying substrate for downstream pathways. In addition to lactate synthesis, these pathways also include de novo fatty acid and triacylglycerol (i.e., triglyceride) synthesis. Because the hypoglycemia stimulates glucagon production over insulin release, lipolysis is promoted, providing abundant fatty acids to other tissues for energy production (i.e., beta oxidation). A substantial portion of these fatty acids enters the mitochondria of various tissues to be oxidized, but the excess is repackaged in the liver to form triglyceride-rich, very low density lipoprotein (VLDL) particles to be released into the circulation. High insulin levels normally prevent formation and release of VLDL particles from triglycerides, but in states of profound hypoglycemia, this inhibitory signal is not present. Note that elevated blood lipid levels in patients with von Gierke disease occasionally cause them to develop xanthomas (accumulations of fat beneath the surface of the skin).     You should know how glucagon permits beta oxidation to occur (Fig. 11.11). Note in Fig. 11.11 that glucagon stimulates activity of malonyl-CoA decarboxylase, which catalyzes the breakdown of malonyl-CoA into acetyl-CoA. Insulin, on the other hand, upregulates the enzyme acetyl CoA carboxylase, which stimulates the production of malonyl-CoA along

266  Genetic and Metabolic Disease the pathway of triglyceride synthesis. Malonyl-CoA provides an inhibitory signal to carnitine palmitoyltransferase I (CPTI), a mitochondrial enzyme that shuttles long-chain fatty acids into the mitochondrial matrix for beta oxidation. Without CPTI activity, beta oxidation cannot occur. It makes sense that insulin would prevent beta oxidation from occurring, because it would be a waste of energy for the body to oxidize newly synthesized triglycerides in the fed state. At the same time, it would be advantageous to promote beta oxidation when glucagon is present in the fasting state. See how important it is to know the biochemistry behind the diseases that you study?

Insulin +

Glycerol-3 D

Acetyl CoA Fatty acid ATP-citrate lyase carboxylase synthase Acetyl CoA Malonyl CoA Fatty acyl CoA Citrate Malonyl CoA decarboxylase +



TG

Figure 11.11.  Regulation of fatty acid synthesis and breakdown by glucagon and insulin. ATP, adenosine triphosphate; CPTI, carnitine palmitoyltransferase I; FFA, free fatty acid; TG, triglyceride.

CPTI FFA

(-Oxidation)

Glucagon

STEP 1 SECRET Notice that we did not suggest that you should memorize every step of the pathways that we depicted throughout the case. By simply understanding the major steps, key enzymes, and regulators of the pathways that you learn, you can better reason through disease findings. This is how the USMLE will expect you to think on the examination.

8. What causes the hyperuricemia in this patient?   As previously mentioned, a lack of G6Pase activity leads to an accumulation of G6P in the cell. G6P is shunted into the hexose monophosphate shunt (also known as the pentose phosphate pathway ), leading to accumulation of both ribose 5-phosphate and PRPP (5′-phosphoribosyl-1-pyrophosphate). PRPP is the major allosteric activator of the rate-limiting enzyme (glutaminePRPP amidotransferase) (see Fig. 11.8 in Case 11.4) of the de novo purine synthetic pathway. As discussed in Case 11.4, an increase in de novo purine synthesis leads to an increase in purine degradation, leading to increased uric acid production.     Hypoglycemia also makes cells less able to resynthesize the high-energy molecules adenosine triphosphate (ATP) and adenosine diphosphate (ADP) from adenosine monophosphate (AMP). The resulting increase in cytosolic AMP drives an increase in uric acid production.     Finally, the excess plasma lactate competes with uric acid for urinary excretion, further exacerbating the hyperuricemia. These patients can go on to develop gout as a result of the increase in circulating uric acid. 9. Quick review: Cover the columns on the right side of Table 11.3, and explain how glycogen storage diseases affect the activity of the rate-limiting enzyme in each pathway listed in the table. 

Table 11.3.   Metabolic Pathways in Glycogen Storage Diseases PATHWAY

RATE-LIMITING ENZYME

EFFECT

BASIS OF EFFECT

Glycolysis

Phosphofructokinase-1

Increase

Increased substrate (G6P)

Glycogen synthesis

Glycogen synthetase

Increase

Increased substrate (G6P)

Fatty acid synthesis

Acetyl-CoA carboxylase

Increase

Increased substrate (G6P is converted using pyruvate to acetyl-CoA) through exaggerated glycolysis

Hexose monophosphate shunt

G6P dehydrogenase

Increase

Increased substrate (G6P)

Increase

Increased substrate (glycerol and fatty acids from de novo synthesis or from lipolysis)

Triacylglycerol (triglyceride) synthesis CoA, coenzyme A; G6P, glucose-6-phosphate.

Genetic and Metabolic Disease  267

RELATED QUESTIONS 10. Why does a deficiency of muscle glycogen phosphorylase (seen in type V glycogen storage disease, or McArdle disease) not result in hypoglycemia?   Hypoglycemia doesn’t result because muscle does not contribute to maintenance of plasma glucose levels. Muscle glycogen phosphorylase is required for glycogenolysis (breakdown of glycogen into G6P) in muscle. There is a similar enzyme, encoded by a different gene, in the liver. However, unlike in the liver, there is no G6Pase present in muscle to dephosphorylate glucose. As such, glucose is unable to diffuse out of the muscle cell. Similarly, when glucose enters a muscle cell and is phosphorylated by hexokinase, it remains permanently trapped. In other words, glucose that enters a muscle cell cannot be released and, instead, must be consumed by that cell. For this reason, muscle normally makes no contribution to the maintenance of blood glucose.     In McArdle disease, only muscle glycogen phosphorylase is lost. The corresponding liver enzyme is unaffected. The abnormal accumulation of G6P in muscle results in symptoms such as cramps and muscle fatigue, but there is no impairment in the maintenance of blood sugar. You may also see myoglobinuria (suggested by dark urine) associated with this condition as a result of muscle damage, particularly during strenuous exercise. Confirmation of this disease can be made with muscle biopsy, which will show an accumulation of glycogen and an absence of glycogen phosphorylase. 1. Review the high-yield glycogen storage diseases. 1   See Table 11.4 for a summary of these diseases. Table 11.4.   Summary of High-Yield Glycogen Storage Diseases GLYCOGEN STORAGE DISEASE

ENZYME DEFICIENCY

CLINICAL HALLMARKS

Type I (von Gierke disease)

Hepatic and renal glucose-6phosphatase

Massive hepatomegaly and liver dysfunction, renal enlargement, severe hypoglycemia, growth failure

Type II (Pompe disease)

Lysosomal glucosidase

Cardiomegaly leading to cardiac failure; skeletal muscle weakness leading to respiratory muscle failure

Type III (Cori disease)

Amylo-1,6-glucosidase (debranching enzyme)

Milder disease leading to stunted growth, hepatomegaly, and hypoglycemia

Type V (McArdle disease)

Muscle glycogen phosphorylase

Exercise-induced muscle cramps, myoglobinuria with strenuous exercise

    Note: Although the liver accounts for the majority (about 90%) of gluconeogenesis and blood glucose maintenance, the kidney contributes about 10%. As such, type I disease can also result in less severe renal disease (with kidney enlargement, proteinuria, and renal insufficiency).     Also note that type II (Pompe) disease is both a glycogen storage disease and a lysosomal α-1,4 storage disease. Normally a small percentage (about 2%) of cellular glycogen breakdown is carried out by lysosomal glucosidase. Because hepatic and renal glycogen phosphorylase are still functional, deficiency in the enzyme does not result in hypoglycemia but, instead, causes accumulation of glycogen within the lysosomes. This occurs most significantly in the cardiac and skeletal muscles.

SUMMARY BOX: GLYCOGEN STORAGE DISEASES Type I Glycogen Storage Disease (von Gierke Disease) • Presentation: Seizures, poor growth, neurologic deficits, and low tolerance for fasting due to hypoglycemia • Pathophysiology: Defect in glucose-6-phosphatase resulting in increased synthesis of glycogen, fatty acids, and triglycerides and increased activity of the hexose monophosphate shunt • Diagnosis: Nontender hepatomegaly, lactic acidosis, hyperlipidemia, hyperuricemia, liver biopsy showing accumulation of glycogen and fat, low blood sugar levels within a few hours of fasting • Complications: Xanthomas secondary to hyperlipidemia, gout secondary to hyperuricemia  Type V Glycogen Storage Disease (McArdle Disease) • Presentation: Cramps and muscle fatigue (particularly with strenuous exercise) in the absence of hypoglycemia • Pathophysiology: Absence of muscle glycogen phosphorylase activity leading to abnormal accumulation of glycogen in muscle • Diagnosis: History, myoglobinuria (dark urine), muscle biopsy showing glycogen accumulation Refer to Table 11.4 for a more detailed summary of the glycogen storage diseases.

268  Genetic and Metabolic Disease

Case 11.6 An 8-month-old baby girl is brought to the emergency department by her parents. The baby has been vomiting and irritable over the past 2 days, and in the past 8 hours she has become very lethargic. On examination, her liver is mildly enlarged. Laboratory findings indicate hypoglycemia, moderate hyperammonemia, and abnormally low urine ketones (given the degree of hypoglycemia present). Analysis of the patient’s urine reveals a mixture of organic acids ranging between 6 and 12 carbons long.   

1. What is the most likely diagnosis?   This baby likely has medium-chain fatty acyl-CoA dehydrogenase (MCAD) deficiency, the most common genetic disorder of fatty acid oxidation. 2. What are the three length classifications of fatty acids?   Most edible fats contain a mixture of three types of fatty acids: short-chain, medium-chain (with 6–12 carbons), and long-chain. These (in fatty acyl-CoA form) are oxidized in the mitochondria of the peripheral cells, which metabolize fatty acids by the enzymes LCAD (long-chain acyl-CoA dehydrogenase), MCAD (medium-chain AD), and SCAD (short-chain AD), respectively. 3. What are the reasons for the clinical and laboratory findings exhibited by this patient?   Many tissues (especially heart and skeletal muscle) rely heavily on fatty acid oxidation as the primary fuel source for ATP production during fasting or during times of metabolic stress, such as exercise or illness, especially illness that results in decreased oral intake. In addition, a substantial amount of these fatty acids undergo beta oxidation in the liver, which uses the resulting acetyl-CoA to produce ketone bodies that are released to provide energy for the brain (which is unable to directly oxidize fatty acids) (Fig. 11.12).

Fatty acyl-CoA

+ =

CPT-1

Carnitine shuttle

Fatty acyl-carnitine CPT-2 Mito matrix Fatty acyl-carnitine Fatty acyl-CoA MCAD Ketone

LCAD SCAD

Acetyl-CoA

Intermembrane space

Medium-chain Fatty acid

Cytosol

Long-chain fatty acid

Figure 11.12.  Summary of metabolism of fatty acids. CoA, coenzyme A; CPT, carnitine palmitoyltransferase; LCAD, long-chain acyl-CoA dehydrogenase; MCAD, medium-chain acyl-CoA dehydrogenase; SCAD, short-chain acyl-CoA dehydrogenase. (From Brown TA, Brown D. USMLE Step 1 Secrets. Philadelphia: Hanley & Belfus; 2004.)

    When MCAD is deficient, medium-chain fatty acyl-CoA molecules are unable to undergo beta oxidation in these tissues. In addition, although long-chain fatty acyl-CoA compounds can be oxidized into medium-chain acyl-CoA molecules, beta oxidation is arrested at the 12-carbon fatty acyl-CoA stage. As a result, medium-chain fatty acyl CoA molecules accumulate in the cytosol and mitochondrial matrix. Some of these medium-chain compounds are converted to the organic acid derivatives that can be detected in the urine. These derivatives may also be toxic to tissues that carry out beta oxidation. In the liver, a substantial portion of these medium-chain fatty acyl CoA molecules is used in cytosolic resynthesis of triglycerides, resulting in liver enlargement from fatty infiltration.     MCAD deficiency also causes ATP production and ketogenesis to be greatly decreased. Without an adequate supply of energy, the rate of the urea cycle is decreased, leading to hyperammonemia. The rate of gluconeogenesis is similarly reduced, and the endogenous glucose supply (liver glycogen) is rapidly exhausted, resulting in hypoglycemia. Loss of MCAD function also results in decreased production of acetyl-CoA, which, in turn, leads to decreased ketone production despite the hypoglycemia. This distinctive hypoketotic hypoglycemia pattern is characteristic of MCAD deficiency.     Other clinical features of MCAD deficiency reflect the involvement of organs that are (directly or indirectly) dependent on beta oxidation. These findings include liver damage and liver function test (LFT) elevation; muscle enzyme elevation, hypotonia, and rhabdomyolysis; congestive heart failure; and neurologic impairment and cerebral edema. Interestingly, this constellation of features is similar to that seen in Reye syndrome (which occurs rarely in children after treatment of a viral illness with aspirin). 4. How should this child be treated?   Avoidance of fasting and of medium-chain fatty acids in the diet is essential. By not allowing this child to rely on peripheral lipolysis and beta oxidation for energy needs, hypoglycemia and accumulation of intermediates caused by

Genetic and Metabolic Disease  269

the metabolic block will be minimized. Frequent small meals high in carbohydrate and protein and low in fat (95% of patients with SLE have them), they are a good screening test for the disease. However, they are not specific because they are present in many other autoimmune diseases and are often present in the healthy elderly. Anti-dsDNA and anti-Smith antibodies are very specific for SLE and are useful for confirming the diagnosis. Anti-dsDNA antibodies, in particular, are associated with SLE-induced renal disease and indicate poorer prognosis. Antihistone antibodies are found in drug-induced lupus.

464  Rheumatology     Note: Understanding which antibodies are sensitive and specific for which condition is very high yield for the STEP 1 exam. 9. What is causing this patient’s dry mouth and dry eyes?   The complaint of dry mouth (xerostomia) and dry eyes (xerophthalmia) together constitutes sicca complex, also known as Sjögren syndrome. This is an autoimmune disorder of the exocrine glands that can occur either on its own (primary Sjögren) or alongside another autoimmune disease (secondary) such as lupus, scleroderma, or RA. The anti-Ro and anti-La antibodies are often found in Sjögren syndrome. Anti-Ro is capable of crossing the placenta and can cause thirddegree heart block in neonates born to mothers positive for this antibody. 0. What is the reason for the proteinuria? 1   Immune complexes are deposited in the renal glomeruli, leading to a type III hypersensitivity reaction. This entity is termed lupus nephritis and may progress to varying degrees in different patients with SLE. Renal biopsy is often needed to accurately determine prognosis and therapy. Other type III reactions seen in lupus include pericarditis, pleuritis, endocarditis, and the malar rash.     Note: Libman-Sacks endocarditis is a nonbacterial form of endocarditis seen in SLE. Fibrinous vegetations are formed on valve leaflets in response to immune complex deposition. The mitral valve is most often involved. 1. What is the reason for this woman’s anemia? 1   Lupus is a chronic inflammatory disorder, and as such, it is capable of causing anemia of chronic disease. This sort of anemia is often normochromic and normocytic, but it may be hypochromic and microcytic in some cases. Although anemia of chronic disease is the most frequent hematologic manifestation of lupus, other potential complications include autoimmune hemolytic anemia, leukopenia, lymphopenia, and thrombocytopenia. Lab values for anemia of chronic disease are low serum iron and transferrin, but high ferritin. 2. Why did this patient have a positive Venereal Disease Research Laboratory (VDRL) test? 1   Some lupus patients produce the inaptly named “lupus anticoagulant,” which is an antibody directed against certain phospholipid molecules. While this antibody delays in vitro coagulation assays, it actually predisposes to thrombus formation in vivo. This accounts for the increased incidence of venous and arterial thrombi, fetal loss (first trimester), and thrombocytopenia in lupus patients. The lupus anticoagulant happens to bind the phospholipid used in the VDRL assay, which is the reason for the false-positive result when testing for syphilis.

STEP 1 SECRET The correlation between systemic lupus erythematosus (SLE) and positive Venereal Disease Research Laboratory (VDRL) test results is a high-yield fact to know for boards.

3. What are the treatment options for systemic lupus erythematosus? 1   The mainstay of treatment is systemic glucocorticoids. High doses are used for short periods of active disease, and low doses can be used to prevent flares. The side effects of glucocorticoids are frequently encountered in lupus patients, who may take these medications for years. The cytotoxic drug cyclophosphamide is also useful in the treatment of lupus nephritis.     Note: Side effects of glucocorticoids include hypertension, hyperglycemia, osteoporosis, central obesity, and increased rates of infection. 4. What medications are responsible for drug-induced lupus? 1   Procainamide, quinidine, hydralazine, isoniazid, sulfonamides, methyldopa, and chlorpromazine have all been shown to cause a disease syndrome that mimics SLE. Drug-induced lupus typically resolves once the offending agent is withdrawn. Remember that with drug-induced lupus, antihistone antibodies will be present. 15. Quick review: Cover the right column in Table 19.3 and name the primary disease(s) associated with the autoantibodies listed in the left column.

Table 19.3.   Autoantibody Summary AUTOANTIBODY

PRIMARY DISEASE

Antiacetylcholine receptor

Myasthenia gravis

Anticentromere

Limited cutaneous scleroderma (CREST syndrome)

Anti-dsDNA

SLE (specific)

Rheumatology  465

Table 19.3.   Autoantibody Summary—cont’d AUTOANTIBODY

PRIMARY DISEASE

Anti–glomerular basement membrane

Goodpasture syndrome

Antihistone

Drug-induced SLE

Anti-IgG (RF)

Rheumatoid arthritis

Anti–islet cell

Type 1 diabetes mellitus

Anti-La (SS-B)

Sjögren syndrome

Antimicrosomal

Hashimoto thyroiditis

Antimitochondrial

Primary biliary cirrhosis

Antineutrophil (c-ANCA)

Wegner granulomatosis

Antineutrophil (p-ANCA)

Microscopic polyangiitis

Antinuclear

SLE, scleroderma, dermatomyositis

Antiphospholipid

SLE

Anti-Ro (SS-A)

Sjögren syndrome

Anti-Smith

SLE (specific)

Anti–smooth muscle

Chronic autoimmune hepatitis

Anti–tissue transglutaminase

Celiac

Anti-topoisomerase (Scl-70)

Diffuse cutaneous scleroderma

c-ANCA, cytoplasmic antineutrophil cytoplasmic antibodies; CREST, calcinosis, Raynaud phenomenon, esophageal dysmotility, sclerodactyly, telangiectasia; ds, double-stranded; IgG, immunoglobulin G; p-ANCA, perinuclear antineutrophil cytoplasmic antibodies; RF, rheumatoid factor; SLE, systemic lupus erythematosus.

SUMMARY BOX: SYSTEMIC LUPUS ERYTHEMATOSUS (SLE) • Epidemiology: Common autoimmune disease that primarily occurs in young women • P resentation: Classically a young woman with a rash, constitutional complaints, diffuse arthralgias, and an exam showing synovitis of the metacarpophalangeal (MCP) and proximal interphalangeal (PIP) joints of the hands. A history of photosensitivity, miscarriages, hematologic disorder, renal dysfunction, and neurologic disorder makes the diagnosis that much more likely. • Pathophysiology: Caused by autoantibodies directed against nuclear antigens resulting in pathologic immune complex deposition and tissue damage • Diagnosis: Based on a combination of clinical and laboratory findings. Recall that antinuclear antibodies are highly sensitive, but not specific, for SLE, whereas Anti-Smith and anti-dsDNA antibodies are highly specific, but not sensitive, for SLE. • Treatment • SLE: Steroids and immunomodulators such as azathioprine • Lupus nephritis: Commonly cyclophosphamide • Complications • The lupus anticoagulant is an autoantibody that causes thrombosis, early miscarriages, thrombocytopenia, and ­false-positive results on Venereal Disease Research Laboratory (VDRL) assays. • Drug-induced lupus: Potentially caused by procainamide, quinidine, hydralazine, isoniazid, methyldopa, and ­chlorpromazine • Lupus nephritis is a type III hypersensitivity reaction associated with high morbidity and mortality rates.

Case 19.10 A 58-year-old woman presents to your office for evaluation of a several-month history of progressively worsening muscle weakness. She has had difficulty getting into and out of chairs, climbing the stairs, and lifting things over her head. She also reports recently developing a violet-colored rash around her eyes (Fig. 19.19). Review of systems is positive for fatigue, joint stiffness, and an unintentional 10-lb weight loss over the past 6 months. She takes no medications.   

466  Rheumatology

Figure 19.19.  Periorbital rash from the patient in Case 19.10. (From Habif TP. Clinical Dermatology. 4th ed. Philadelphia: Mosby; 2004.)

1. What is your differential diagnosis?   This patient has several symptoms of systemic disease (weakness, fatigue, weight loss) as well as a rash and joint pain. One could entertain diagnoses of a thyroid disorder, rheumatic arthritis, myasthenia gravis, polymyalgia rheumatica, Cushing disease, paraneoplastic syndrome or other malignancy, or various myopathies (e.g., muscular dystrophy, dermatomyositis, polymyositis).

Case 19.10 continued: You order several laboratory tests and find elevated levels of creatinine kinase (CK), aldolase, and aspartate transaminase (AST). Electromyographic studies are suggestive of myopathy, and a muscle biopsy shows an infiltration of lymphocytes and muscle atrophy.   

2. What is the diagnosis?   This patient has dermatomyositis, one of the idiopathic inflammatory myopathies. This autoimmune disease causes inflammatory damage to muscle fibers, with resultant proximal muscle weakness and elevated muscle enzymes (CK). 3. What causes this disease?   Although the exact cause is unknown, a substantial fraction of patients with dermatomyositis have an underlying malignancy. It is therefore important to consider the presence of a neoplastic process in a patient who presents with dermatomyositis. 4. What is the treatment?   Because this is an inflammatory disorder, immunosuppressant drugs are the mainstay. Prednisone is first-line therapy, and methotrexate can be used if corticosteroids are unsuccessful. 5. What are the other “idiopathic inflammatory myopathies”?   Dermatomyositis and polymyositis are both inflammatory myopathies characterized by symmetric proximal muscle weakness, elevated serum muscle enzymes, and evidence of myopathy on electromyography. Dermatomyositis often presents with skin findings, such as the heliotrope rash, Gottron sign (Fig. 19.20), and shawl sign; polymyositis does not have these dermatologic manifestations. Dermatomyositis and polymyositis appear distinct on muscle biopsy, with dermatomyositis showing immune complex deposition, and polymyositis revealing predominantly T-cell invasion of muscle fibers. Inclusion body myositis is another inflammatory myopathy, which presents with both proximal and distal muscle weakness and normal or mildly elevated muscle enzymes and is associated with distinctive changes on electromyogram (EMG) and biopsy. Fig. 19.20 shows Gottron papules on the hands in dermatomyositis.

Figure 19.20.  Gottron papules, a pathognomonic sign of dermatomyositis, are round, smooth, flat-topped papules that occur over the knuckles and along the sides of the fingers. (From Habif TP. Clinical Dermatology. 4th ed. Philadelphia: Mosby; 2004.)

Rheumatology  467

6. What disease that is transmitted by pork can cause similar muscular symptoms?   Trichinosis is caused by eating raw or undercooked meats that contain the viable larvae of the roundworm Trichinella spiralis. Although most infections are subclinical, exposure to a heavy inoculum of larvae can result in trichinosis, which may present clinically with diarrhea, myositis, fever, and periorbital edema. Laboratory evaluation will typically reveal hypereosinophilia as well.

SUMMARY BOX: INFLAMMATORY MYOPATHIES • Dermatomyositis, polymyositis, and inclusion body myositis are idiopathic inflammatory myopathies. • Dermatomyositis is characterized by proximal muscle weakness, elevated serum muscle enzymes, electromyogram (EMG) abnormalities, dermatologic manifestations, and characteristic muscle pathologic changes. • Polymyositis is a similar myopathic process with distinct muscle disease but without skin involvement. • Inclusion body myositis causes both proximal and distal muscle weakness without markedly raised serum muscle enzymes. • The inflammatory myopathies are treated with steroids. • Trichinosis is a parasitic disease that may cause myopathy.

Case 19.11 The parents of a 3-year-old boy are concerned that he is not walking as well as other boys his age. Both parents are healthy, and there is no family history of neuromuscular disease. He has three older brothers who are healthy. On physical examination, he has large calf muscles and lower extremity proximal muscle weakness, as demonstrated by the need to use his arms and hands to assist in standing from a seated position. Examination is otherwise unremarkable.   

1. What is your differential diagnosis?   This child with muscle weakness may be suffering from a myopathy such as juvenile dermatomyositis, an inflammatory disease such as juvenile RA, an inherited muscular dystrophy, a neurologic disorder such as Guillain-Barré syndrome, or an infection such as Lyme disease or trichinosis.

Case 19.11 continued: Laboratory tests are significant only for a markedly elevated CK. A skeletal muscle biopsy reveals complete absence of dystrophin staining.   

2. What is the most likely diagnosis?   Duchenne muscular dystrophy is most likely. 3. Is this condition more commonly acquired or inherited?   About two-thirds of cases of Duchenne muscular dystrophy are inherited in an X-linked recessive manner. However, approximately one-third of the cases are secondary to spontaneous mutations within the dystrophin gene. The dystrophin gene is subject to a high rate of spontaneous mutations because of its enormous size (>2 × 106 bases). Because his parents were unaffected and he has three healthy older brothers, this condition was likely acquired in this boy following a spontaneous mutation in the dystrophin gene.

STEP 1 SECRET Boards will often relate genetics questions to pedigrees, which means that you should know the inheritance patterns of the genetic diseases that you study. X-linked recessive diseases are a particular USMLE favorite. These include G6PD deficiency, Ocular albinism, Lesch-Nyhan syndrome, Duchenne muscular dystrophy, Wiskott-Aldrich syndrome, Bruton agammaglobulinemia, Chronic granulomatous disease, Hunter’s syndrome, Fabry disease, and Hemophilia (A and B). You can use this mnemonic to remember the X-linked recessive diseases: “Good OLD WBCs Hunt and Fight Heroically.” 4. What is the function of dystrophin?   Dystrophin is a cytoskeletal membrane protein that plays an important structural role in skeletal muscle cells. It is absent in Duchenne muscular dystrophy. 5. How do the manifestations of Becker muscular dystrophy differ?   This disease is also due to mutations in the dystrophin gene, but there is some level of protein present rather than a complete absence, so the clinical manifestations are not as severe as in Duchenne muscular dystrophy. 6. Why does this boy have such large calf muscles on examination? What term is used to describe this finding in Duchenne muscular dystrophy patients?   Patients with Duchenne muscular dystrophy ironically have the appearance of enlarged calf muscles, referred to as pseudohypertrophy of the calf muscles (Fig. 19.21). This hypertrophy occurs initially in response to hypertrophy of muscle fibers but secondarily in response to fatty infiltration of the muscle and abnormal proliferation of connective tissue within the muscle.

468  Rheumatology

Figure 19.21.  Enlarged calf muscles in a patient with Duchenne muscular dystrophy. (From Fenichel GM. Clinical Pediatric Neurology. Philadelphia: WB Saunders; 1997.)

7. How is a Gower sign elicited on examination, and what does it indicate?   A Gower sign can be elicited by asking the child to stand from a sitting position. Children with muscular dystrophy and other disorders involving muscle wasting will not have the muscle strength to simply stand. They may instead first roll over into a prone position, push themselves onto all fours, and then “walk” their hands up their thighs to a standing position (i.e., positive Gower sign). The presence of a Gower sign indicates marked proximal muscle weakness (Fig. 19.22). 

Figure 19.22.  Gower sign. (Redrawn from Siegel IM. Clinical management of muscle disease. In: Canale STS. Campbell’s Operative Orthopedics. 5th ed. London: William Heinemann; 1977.)

Rheumatology  469

A FEW MORE MUSCULAR DYSTROPHIES … Case 19.11.1 A patient complains of a long history of generalized muscle weakness. On examination, his facial muscles show marked atrophy, and when you ask him to shake your hand, he appears unable to relax his grip for an extended period.   

8. What diagnosis might you suspect?   You might suspect myotonic dystrophy, which is the most common adult dystrophy. The term myotonia refers to a sustained involuntary contraction of muscles, which this man is exhibiting by not being able to release his grip.     Other symptoms of myotonic dystrophy include facial muscle weakness, frontal balding, testicular atrophy, cataracts, cardiac conduction defects, and glucose intolerance. 9. What is the mechanism of inheritance of myotonic dystrophy?   Myotonic dystrophy results from impaired expression of the myotonin protein kinase gene. The mechanism causing impaired expression involves expansion of a trinucleotide repeat sequence located in the 3′ untranslated region of the myotonin protein kinase gene. This disorder is inherited as an autosomal dominant disease, and because this mechanism involves expansion of trinucleotide repeat sequences (CTG), the phenomenon of amplification is seen (i.e., family members get the disease at earlier and earlier ages throughout the generations).     Note: Other trinucleotide repeat disorders include Huntington disease (CAG), fragile X syndrome (CGG), and Friedreich ataxia (GAA).

Case 19.11.2 An adult patient with a long history of muscle weakness has maintained a slow, steady course of declining function and is now wheelchair-bound. His weakness is most prominent in proximal muscles, with complete sparing of facial and extraocular musculature. A muscle biopsy shows normal dystrophin expression. The patient’s father and grandfather had similar courses.   

0. What is the diagnosis? 1   This is limb-girdle muscular dystrophy, which is actually a group of myopathies that affect the shoulder and pelvic girdles. Limb-girdle dystrophies can be inherited in both autosomal dominant and recessive fashion and may display a heterogeneous phenotype. The recessive form of the disease tends to have an earlier onset and progresses more quickly, whereas the dominant form follows a slower and more variable course. Several different genes have been implicated in this disease.

SUMMARY BOX: THE MUSCULAR DYSTROPHIES • Duchenne muscular dystrophy is an inherited loss of the dystrophin protein, which is a structural component of skeletal muscle cells. • Duchenne muscular dystrophy is inherited in a recessive X-linked fashion, or it may be an acquired spontaneous mutation. • Becker muscular dystrophy is an inherited defect in dystrophin that results in partial loss of the dystrophin protein. • Myotonic muscular dystrophy is a trinucleotide repeat disorder. • Limb-girdle muscular dystrophy is a heterogeneous group of heritable defects in proteins that result in proximal muscle weakness and atrophy. • Gower sign indicates proximal muscle weakness.

Case 19.12 Dr. Rheumatoid is a specialist widely known for his interest and skill in treating rare disorders of the musculoskeletal system. His particular expertise is in diagnosing and treating metabolic and developmental disorders of bone. A third-year medical student working with him one afternoon is delighted to encounter one “zebra” after another in clinic.   

Case 19.12.1 The first patient was referred to Dr. Rheumatoid with complaints of bone pain and a diagnosis by a hematologist of myelophthisic anemia. A bone scan reveals abnormally thick and dense bones with an “Erlenmeyer flask” deformity (Fig. 19.23).   

470  Rheumatology

Figure 19.23.  Radiograph of the upper extremity in a patient with this condition (see text for discussion). (From Kumar V, Abbas AK, Fausto N. Robbins and Cotran Pathologic Basis of Disease. 7th ed. Philadelphia: WB Saunders; 2005.)

1. What is your diagnosis?   Osteopetrosis (also known as marble bone disease) is the diagnosis. 2. What causes the bones to be dense and thick in this patient?   In osteopetrosis, osteoclasts are less active than normal (or inactive entirely) and therefore do not resorb bone effectively during bone remodeling. This generally is the result of failure of the osteoclasts to acidify the resorption pit (e.g., due to carbonic anhydrase or chloride channel gene mutations). Additionally, the process whereby woven (immature) bone is converted to compact (mature) bone is disrupted in osteopetrosis. This combination of reduced remodeling of bone and inadequate bone “maturation” results in thick and brittle bones. 3. Why might you see anemia in osteopetrosis, and why is it referred to as a myelophthisic anemia?   The term myelophthisis describes the replacement of hematopoietic tissue in the bone marrow with abnormal tissue. Myelophthisic anemia is therefore caused by the replacement of bone marrow by abnormal tissue. In the case of osteopetrosis, the failure of osteoclasts to remodel existing bone allows newly formed bone to encroach on the space of the bone marrow, making hematopoiesis less effective and resulting in pancytopenia (anemia, thrombocytopenia, and leukopenia). 4. Should you observe any laboratory value abnormalities in a patient who has osteopetrosis?   No. Serum calcium, phosphate, alkaline phosphatase, and parathyroid hormone (PTH) levels are normal in osteopetrosis.     Note: You do not need to know exact laboratory value ranges, but you should be able to compare them with normal values using relative terms (increased, decreased, normal).

STEP 1 SECRET Laboratory value abnormalities associated with various bone disorders are high-yield for Step 1. When studying this topic, you should classify these disorders according to their unique clinical and radiographic features as well as their expected laboratory values (i.e., calcium, phosphate, alkaline phosphatase, parathyroid hormone [PTH]).

Rheumatology  471

Case 19.12.2 A 13-year-old boy with sickle cell anemia is referred for persistent right hip pain and intermittent fevers, although he cannot recall any specific trauma to the hip. An x-ray of the hips suggests avascular necrosis of the femoral heads.   

5. Infection with what organisms should be suspected?   Although S. aureus is the most common organism responsible for osteomyelitis, patients with sickle cell anemia are uniquely susceptible to Salmonella bacteremia and osteomyelitis. This susceptibility stems from the impaired splenic and mononuclear cell function associated with sickle cell anemia because Salmonella is an encapsulated organism.

Case 19.12.3 A 42-year-old woman with end-stage renal failure is referred to Dr. Rheumatoid because recent bone scans revealed marked osteopenia throughout her body.   

6. What most likely explains this?   The most likely explanation is osteomalacia caused by vitamin D deficiency secondary to renal failure. Recall that an important endocrine function of the kidneys is the production of 1,25-dihydroxycholecalciferol, the active form of vitamin D. Because vitamin D is necessary for bone mineralization and because bone is constantly being remodeled, impaired mineralization results in an imbalance between mineralization and degradation, causing marked osteopenia (Fig. 19.24).     Note: Additional causes of renal osteodystrophy include bone buffering of excess acid and hypocalcemia from calcium phosphate precipitation in hyperphosphatemia.

7-Dehydrocholesterol

UV light (skin)

Vitamin D

Diet

Liver

25-(OH)-hydroxyvitamin D (calcidiol) PTH

+

+ ↓ Phosphate

1a-hydroxylase

Kidney

1,25-(OH)2-vitamin D3 (calcitriol)

+ Stimulates

Figure 19.24.  Vitamin D synthesis. PTH, parathyroid hormone; UV, ultraviolet. (From Brown T. Rapid Review Physiology. 2nd ed. Philadelphia: Mosby; 2011.)

Case 19.12.4 A 6-year-old boy is brought to the clinic by his mother because he has suffered multiple bone fractures throughout his short life. These fractures were all unexpected because they invariably occurred in response to very minor accidents. Additionally, the boy has been doing poorly in school recently because he is having trouble hearing the teacher. The examination is remarkable only for slightly blue sclerae. X-ray of the lower extremity shows marked bowing of the bones (Fig. 19.25).   

472  Rheumatology

Figure 19.25.  Lateral view of the lower extremities shows marked bowing of the bones due to softening and multiple fractures that occur as a result of this congenital bone dysplasia. (From Mettler FA. Essentials of Radiology. 2nd ed. Philadelphia: WB Saunders; 2005.)

7. What is your diagnosis, and what is the etiology of this condition?   Osteogenesis imperfecta is due to genetic defects that result in structural or quantitative abnormalities of type I collagen, which is the primary component of the extracellular matrix of bones, including the middle ear bone (explaining the hearing loss seen in this child). Type I collagen is also found in corneal tissues. A defect in type I collagen results in translucency of the connective tissue over the vascular choroid layer of the eye such that the veins impart a blue appearance to the sclerae.     Note: A wide spectrum of genotypes and phenotypes is associated with osteogenesis imperfecta, ranging from fairly minor to very severe. 8. Quick review: Cover the right column in Table 19.4 and list the pathophysiologic abnormality associated with each of the rheumatologic disorders listed in the left column.

Table 19.4.   Rheumatologic Disorders DISORDER

PATHOPHYSIOLOGY

Achondroplasia

Mutation in fibroblast growth factor receptor prevents endochondral ossification and limits long bone growth

Gout

Increased uric acid production or decreased uric acid excretion

Osteoarthritis

Degeneration of joint cartilage

Osteogenesis imperfecta

Genetic defects in type I collagen weaken bone

Osteomalacia

Impaired bone mineralization in adults

Osteopetrosis (marble bone disease)

Decreased osteoclast activity, bony invasion of bone marrow leading to myelophthisic anemia

Paget disease

Increased rate of osteoclast activity (perhaps secondary to viral infection of osteoclasts), resulting in increased rates of bone resorption, formation, and mineralization, with consequent deposition of woven rather than lamellar bone

Rheumatology  473

Table 19.4.   Rheumatologic Disorders—cont’d DISORDER

PATHOPHYSIOLOGY

Pseudogout (chondrocalcinosis)

Calcium pyrophosphate deposition

Rheumatoid arthritis

Inflammation of synovial membrane

Rickets

Impaired bone and cartilage mineralization in children

 UMMARY BOX: OSTEOPETROSIS, SICKLE CELL AVASCULAR NECROSIS, S OSTEOGENESIS IMPERFECTA

• Osteopetrosis is a disorder of osteoclasts that results in thick, brittle bones and myelophthisic anemia. • Patients with sickle cell disease are prone to osteomyelitis caused by Salmonella species. • End-stage renal failure results in vitamin D deficiency and osteomalacia, also known as renal osteodystrophy. • Osteogenesis imperfecta is a highly variable disease caused by a defect in type I collagen.

CHAPTER 20

VASCULITIDES Aaron Lee, MD, Robert D’Angelo, MD, Thomas A. Brown, MD, and Sonali J. Bracken

Insider’s Guide to Vasculitides for the USMLE Step 1 Vasculitides on Step 1 are very straightforward if you know the most common signs and symptoms of each diagnosis. The best way to study for this section is to simply read through the information listed in First Aid and then test yourself with the tables and cases in this chapter. Other good resources include USMLE World and Pathoma. It is advisable that you make a list for yourself of the unique features of each disease (e.g., palpable lower extremity skin rash with Henoch-Schönlein purpura, weak upper extremity pulses in Takayasu arteritis, unilateral headache in temporal arteritis). These features will most likely be your biggest clues in the clinical vignettes presented to you on boards. You should also be sure to know the unique laboratory features associated with specific vasculitides, such as elevation in perinuclear antineutrophil cytoplasmic antibody (p-ANCA) or cytoplasmic antineutrophil cytoplasmic antibodies (c-ANCA), elevated erythrocyte sedimentation rate (ESR), and hepatitis B seropositivity, because the USMLE loves to test students on these facts.

BASIC CONCEPTS 1. What are the vasculitides, and how do they typically present clinically?   The best way to think of the vasculitides is as a group of poorly understood autoimmune disorders involving the blood vessels. They are defined by the presence of leukocytes in the vessel walls, and as inflammatory diseases, they typically present with vague constitutional signs such as fever, malaise, and arthralgias or myalgias. A biopsy of affected blood vessels can be very helpful in making a definitive diagnosis, although obtaining a segment of affected vasculature can be difficult. Although it appears that a majority of the vasculitis syndromes are caused by an immune-mediated mechanism, other possible etiologic factors include drug hypersensitivity reactions and viral infections resulting in immune complex deposition within the vasculature. 2. Along with constitutional complaints, what clinical signs and patterns of organ involvement suggest a vasculitic syndrome?   Palpable nonblanching purpura may indicate vasculitides such as hypersensitivity (leukocytoclastic) vasculitis, HenochSchönlein purpura, and microscopic polyangiitis.     Mononeuritis multiplex, a clinical picture that arises from simultaneous disease to multiple individual nerves, typically affects sensory and motor function. In the United States, diabetes is the most common cause of this neuropathy, but in the nondiabetic person, it is very suggestive of vasculitis, particularly polyarteritis nodosa (PAN).     Pulmonary-renal involvement, such as hemoptysis and hematuria, are suggestive of a pulmonary-renal syndrome such as granulomatosis with polyangiitis (Wegener granulomatosis) or Goodpasture syndrome. 3. How are the vasculitides classified?   The vasculitides are generally classified by the size and type of blood vessels that are typically affected in patients with each disorder.     Large vessel vasculitis: • Takayasu arteritis affects the aorta and its major branches. • Temporal arteritis, also known as giant cell arteritis, most commonly affects the branches of the external carotid artery, characteristically including the temporal artery.     Medium-sized vessel vasculitis: • PAN affects medium-sized muscular arteries. • Kawasaki disease actually affects large, medium-sized, and small arteries, but the most important association is that, if untreated, it can affect the coronary arteries.     Small vessel vasculitis: • Eosinophilic granulomatosis with polyangiitis (Churg-Strauss Syndrome)a affects the arteries of the lungs and of the skin. • Granulomatosis with polyangiitis (Wegener)a and microscopic polyangiitis (MPA) affect medium-sized and small arteries, arterioles, and venules, particularly in the respiratory tract and kidneys. • Cryoglobulinemic vasculitis affects capillaries, arterioles, and venules. a The

names “Churg-Strauss syndrome” and “Wegener granulomatosis” were changed to “eosinophilic granulomatosis with polyangiitis” (EGPA) and “granulomatosis with polyangiitis” (GPA), respectively, after the 2012 International Chapel Hill Consensus Conference. While USMLE questions often give both names in the question stem and answer choices, it is best to be familiar with both the new and old naming conventions.

474

Vasculitides  475



• Henoch-Schönlein purpura primarily affects venules. • Vasculitis that is due to hypersensitivity reaction, secondary to viral infection, or secondary to connective tissue disorder typically affects small vessels.

4. Cover the right column in Table 20.1 and attempt to describe the “classic presentation” for each of the listed vasculitides. Table 20.1.   Classic Presentation of the Vasculitides VASCULITIS

TYPICAL PATIENT

CLASSIC PRESENTATION

CHARACTERISTIC LABS

Temporal (giant cell) Caucasian female arteritis older than 50

Fever, unilateral headache, jaw claudication, vision Markedly elevated ESR loss

Granulomatosis Older Caucasian with polyangiitis (Wegener)

Constitutional symptoms, hemoptysis, hematuria, ENT symptoms (saddle nose, sinusitis, epistaxis)

Microscopic polyangiitis (MPA)

Older Caucasian

Constitutional symptoms, hemoptysis, hematuria p-ANCA Elevated ESR No granulomatous changes

Kawasaki syndrome

Asian child less than 5

Unexplained fever, maculopapular rash that Elevated ESR, CRP, starts on hands and feet, bilateral nonexudanormochromic, tive conjunctivitis, cervical lymphadenopathy, normocytic anemia edema of extremities, and mucosal changes such as strawberry tongue

c-ANCA Elevated ESR Granulomatous changes

Polyarteritis nodosa Middle-aged or older Generally affects vessels of the kidney, heart, No ANCA association patient with HBV liver, and GI system Aneurysms in mesenteric Does not affect the pulmonary vasculature or renal arteries on Hepatitis B antigenemia is common; arterial biangiography opsy reveals inflammation of the tunica media Churg-Strauss syndrome

Middle-aged patient, very rare in children and elderly

History of asthma, sinusitis, peripheral neuropathy, Eosinophilia (>10% on skin lesions; can have cardiac involvement WBC differential) p-ANCA (40–60% of cases)

Takayasu arteritis (“pulseless disease”)

Asian women less than 40

Fever, night sweats, arthritis, myalgia, vision problems, different blood pressures in the arms

Henoch-Schönlein purpura

Child from age 3–15, Abdominal pain, arthritis, arthralgia, hematuria No specific testing most commonly with red blood cell casts, maculopapular Elevated IgA in 50–70% in winter, fall, or rash on lower extremities (palpable purpura) of cases spring Most commonly occurs in young children; associated with IgA nephropathy after upper respiratory infection; increased risk of intussusception

Elevated ESR and CRP support clinical findings MRI or CT angiography to confirm

Mixed essential Patient in 50s with cryoglobulinemia HCV

Arthralgias, hepatosplenomegaly, hypocomple- Elevated cryoglobulins mentemia, palpable purpura, proteinuria, Hypocomplementemia and hematuria; confirmed by presence of circulating cryoglobulins; strongly associated with HCV infection

Thromboangiitis Male smoker 40–45 obliterans (Buerger disease)

Young male smoker with distal extremity cold intolerance

Normal ESR/CRP, complement levels

c-ANCA, cytoplasmic antineutrophil cytoplasmic antibodies; CRP, C-reactive protein; CT, computed tomography; ENT, ear, nose, throat; ESR, erythrocyte sedimentation rate; GI, gastrointestinal; HBV, hepatitis B virus; HCV, hepatitis C virus; IgA, immunoglobulin A; MRI, magnetic resonance imaging; p-ANCA, perinuclear antineutrophil cytoplasmic antibodies; WBC, white blood cell.

Case 20.1 A 75-year-old Caucasian woman is evaluated for a 1-week history of anorexia, fatigue, and unilateral severe headache. She denies any recent visual problems or photophobia.   

476  Vasculitides 1. What are the main considerations in your differential diagnosis?   Constitutional complaints such as anorexia and fatigue are suggestive of malignancy, depression, infection, and vasculitis. The headache could be caused by a migraine, meningitis, a brain mass, intracranial hemorrhage, or vasculitis. However, for boards, unilateral headache in an adult over 50 in the absence of fever or head trauma is temporal arteritis until proven otherwise.

Case 20.1 continued: Physical examination is significant for right-sided scalp tenderness. Laboratory tests reveal a markedly elevated erythrocyte sedimentation rate.   

2. What is the likely diagnosis?   Temporal arteritis or giant cell arteritis (GCA) is the likely diagnosis. Temporal arteritis occurs almost exclusively in patients older than 50. Women are more likely than men to be affected, and rates are highest in Caucasians. Unilateral headache and scalp tenderness are classic for temporal arteritis, as is the elevated erythrocyte sedimentation rate (ESR). Temporal arteritis also commonly presents with jaw pain. Vision impairment or blindness may result in the most serious cases.     The name temporal arteritis is derived from the fact that the disease preferentially targets the extracranial branches of the carotid arteries, frequently affecting the superficial temporal artery. The ophthalmic, vertebral, and carotid arteries may also be affected. Involvement of intracranial/intradural arteries is extremely rare, possibly because of fewer elastic fibers in the media and adventitia and absence of vasa vasorum in these arteries (see subsequent discussion regarding events that lead to inflammation). 3. What does the elevated ESR suggest?   The ESR and C-reactive protein (CRP) are the most widely used indicators of the acute-phase protein response. Although these measurements lack specificity (being elevated in vasculitides, infections such as endocarditis, and malignancies), they are useful because the acute-phase protein response may reflect the presence and intensity of an inflammatory process.     Specifically, the ESR represents the rate at which the erythrocytes fall (sediment) through the plasma, which depends largely upon the plasma concentration of fibrinogen, a protein that is seen in higher concentration during an inflammatory process. 4. In temporal arteritis, what events lead to inflammation of the artery?   It is likely that T cells and macrophages enter the artery wall via the vasa vasorum. How they become activated and targeted is unknown. CD4+ T cells release interferon gamma (IFN-γ), and macrophages release interleukins (IL-1, IL-6) and platelet-derived growth factor (PDGF). IFN-γ mediates the inflammatory response in the vessel wall, IL-6 is largely responsible for systemic signs of inflammation, and PDGF promotes proliferation of smooth muscle cells and intimal hyperplasia. This intimal hyperplasia leads to occlusion of the arterial lumen, ultimately leading to symptoms of ischemia.

Case 20.1 continued: The patient is started on high-dose steroids, and the following day a biopsy of a 3-cm section of the right side of the temporal artery returns as negative for signs of inflammation.   

5. Why might it still make sense to treat this patient?   Temporal arteritis affects the temporal artery in a segmental fashion, and this could explain a negative biopsy result even in the presence of the disease. Furthermore, in some cases, GCA may affect other extracranial branches of the carotid artery and spare the superficial temporal artery. Therefore, if the clinician has a high index of suspicion for temporal arteritis, the patient should be treated regardless of biopsy results (making biopsy of questionable clinical value). 6. What severe complication of this disorder may be avoided by initiating immunosuppressive therapy as soon as possible?   Partial or total blindness can occur suddenly and without warning. This is caused by occlusion of the ophthalmic artery, leading to ischemia of the optic nerve. Blindness may be preceded by amaurosis fugax, which is transient visual loss, often with heat or exercise. Blindness is usually permanent but can be prevented by adequate treatment with corticosteroids, making timely diagnosis and treatment of GCA crucial.     As demonstrated in this case, patients with GCA are treated with high-dose steroids to prevent blindness. 7. What other symptomatic manifestations may be expected as a result of arterial inflammation in patients with giant cell arteritis?   Decreased blood flow in the extracranial branches of the carotid arteries caused by inflammation of those vessels can lead to symptoms such as jaw claudication, especially when prolonged talking or chewing causes an increase in oxygen demand. Occasionally, respiratory symptoms such as coughing can be seen with GCA and are believed to be a result of inflammatory involvement of branches of the pulmonary artery.

Vasculitides  477

    In a subset of patients, the predominant symptoms of GCA may be the constitutional signs of systemic inflammation, such as fever, fatigue, and anorexia. Fatigue is often also noted in patients with more typical presentations, such as headache or scalp tenderness. This is evidence that immune activation is not necessarily limited to vascular lesions. Patients with GCA have elevated levels of circulating monocytes, which produce IL-1 and IL-6, and the latter is a potent inducer of the acute-phase response. Release of IL-6 therefore not only leads to the elevation of the ESR that is seen in these patients but also helps explain the nonspecific systemic symptoms. 8. How can response to corticosteroids be monitored?   Remember the ESR? The drop in ESR or CRP, along with the clinical response, can be used to gauge effectiveness of corticosteroid therapy. 9. With what other disease is giant cell arteritis associated?   Polymyalgia rheumatica (PMR), which is a disorder characterized by bilateral pain in the muscles of the neck, shoulder, and pelvic girdle, along with morning stiffness and elevated ESR. PMR is often considered to be a form of GCA that lacks the fully developed vasculitis. Both GCA and PMR respond to steroid therapy. While high-dose steroids are used to treat GCA, low-dose steroids are used to treat PMR.

STEP 1 SECRET Temporal arteritis is a favorite on the USMLE. Be on the lookout for symptoms of unilateral headache, jaw claudication, vision problems, and musculoskeletal pain (PMR association).

SUMMARY BOX: TEMPORAL ARTERITIS • Presentation: Headache, scalp tenderness, jaw claudication, and constitutional signs (fever, fatigue, and weight loss) • Epidemiology: Adults > 50, usually Caucasian women • Pathogenesis: Inflammation of the walls of extracranial branches of carotid arteries • Diagnosis: Elevated erythrocyte sedimentation rate (ESR) and C-reactive protein (CRP) • Biopsy of the superficial temporal artery, showing granulomatous infiltration of the arterial wall, may be helpful but is associated with a high false-negative rate due to skip lesions • Complications: Partial/complete blindness • Treatment: High-dose corticosteroids

Case 20.2 A 4-year-old girl is evaluated for a 5-day history of high fever and a desquamating rash of her palms and soles with swelling of her hands and feet (Fig. 20.1). Examination is significant for cervical lymphadenopathy, injected conjunctiva, and “cherry-red” lips with fissuring and crusting. Laboratory evaluation reveals an elevated ESR and an increased number of platelets.   

Figure 20.1.  Kawasaki disease. (From Shah BR, Laude TA. Atlas of Pediatric Clinical Diagnosis. Philadelphia: WB Saunders; 2000.)

1. What are the considerations in your differential diagnosis?   The desquamating rash is concerning for staphylococcal scalded skin syndrome (SSSS), in which epidermolytic toxins are produced by Staphylococcus aureus infection. Children are thought to be at increased risk for SSSS because they have not previously been exposed to the epidermolytic toxins and thereby lack protective antibodies to the toxins. The

478  Vasculitides presentation is also concerning for toxic shock syndrome, in which superantigen toxins from streptococcal or staphylococcal infections (often following use of tampons, contraceptive devices, or nasal packing) are produced. These toxins overstimulate the immune system, resulting in fever, a rash that may be desquamating, and a myriad of other signs and symptoms. Remember that superantigens nonspecifically activate many T cells at once, resulting in release of large quantities of cytokines (IL-1, IL-2, tumor necrosis factor alpha and beta [TNF-α, TNF-β], interferon gamma) that trigger a massive inflammatory response.     Scarlet fever is another concern in this child. Scarlet fever is an exotoxin-mediated disease associated with streptococcal pharyngitis, impetigo, or other streptococcal infections. It is characterized by fever, rash, and a “strawberry tongue.”     Finally, we need to consider Kawasaki disease (also called mucocutaneous lymph node syndrome), an acute febrile systemic illness of childhood affecting medium-sized vessels. It is more common in Asian children and can also present with many of the previously mentioned symptoms.

Case 20.2 continued: The attending physician is not interested in an impressive differential diagnosis but rather wants a specific diagnosis for the child.   

2. What is your diagnosis?   Kawasaki disease could explain all of this girl’s symptoms. The diagnosis of Kawasaki disease requires unexplained fever for at least 5 days, accompanied by at least four of the five following criteria: 1. Bilateral nonexudative conjunctivitis (redness of the conjunctiva overlying the eye without discharge) 2. Oral mucous membrane changes, such as cracked lips, throat redness, or “strawberry tongue” 3. Peripheral extremity changes such as palmar erythema or edema of the hands and feet 4. Polymorphous rash (different appearance depending on the patient with temporal variation, but generally maculopapular [small, flat discolored areas with small, raised bumps] and involving the trunk) 5. Cervical lymphadenopathy     A popular mnemonic that is used to remember these criteria is “CRASH and burn” (“CRASH” stands for Conjunctivitis, Rash, Adenopathy, Strawberry tongue, Hand swelling and peeling, and “burn” refers to 5 days of fever). If these conditions are not strictly met, the patient may have an “atypical Kawasaki disease,” which can potentially lead to the same cardiac sequelae as typical disease. Of the criteria found in typical disease, mucous membrane changes are most common.     It may be helpful to remember the other term for Kawasaki disease: mucocutaneous lymph node syndrome. This name is derived from the typical signs and symptoms, which include mucosal inflammation, cutaneous maculopapular rash, and lymph node enlargement, all with an unexplained high fever. 3. What is the major concern in patients with this disease who do not receive adequate therapy?   Coronary artery aneurysms are the major cause of morbidity and mortality in Kawasaki disease, occurring in 20% to 25% of untreated children. A much smaller percentage (≈4%) of those adequately treated (see later discussion for treatment) will develop coronary artery aneurysms. Coronary artery inflammation can lead to myocardial inflammation, arrhythmias, or death. In fact, roughly 1% of all children who develop Kawasaki disease will die because of rupture of a coronary artery aneurysm or as a result of coronary artery thrombosis and infarction. Pericardial effusions are seen in approximately 20% of cases, and myocarditis can lead to tachycardia. Otherwise, the course of this disease is selflimiting, with fever and acute manifestations lasting an average of 12 days without therapy. 4. What is the pathogenesis of Kawasaki disease?   The exact pathogenesis of Kawasaki disease is unknown, but it is speculated that an infectious agent may trigger vasculitis in genetically susceptible individuals. Overactivation of immune-competent cells and an overproduction of cytokines cause endothelial cell injury and blood vessel wall damage. Inflammation in blood vessels is primarily mediated by activated T cells, monocytes, and macrophages. Coronary arteries are preferentially affected, which can lead to myocardial infarction. 5. How is this disease treated?   Intravenous immunoglobulins (IVIG) and high-dose aspirin therapy are given to prevent coronary aneurysms. IVIG has antiinflammatory properties, while aspirin has both antiinflammatory and antiplatelet properties. Note that aspirin is generally contraindicated in children with a febrile illness because of the risk of developing Reye syndrome (encephalopathy and hepatic dysfunction), especially in children with varicella or influenza. However, the benefits are thought to outweigh the risks in Kawasaki disease. Aspirin should be discontinued (with continuation of IVIG) if exposure to varicella or influenza occurs during treatment. 6. Describe the epidemiology of this disease.   This is a disease of unknown cause that is seen in children and is most common in Asian populations. Eighty percent of cases occur in children younger than 5 years of age, with the peak incidence at 2 years of age. The disease is more common in boys than in girls. Interestingly, there is a twofold increased risk in a child who has at least one parent who was affected as a child, suggesting a possible genetic component to the disease. Seasonal variation in incidence, with increased incidence in late winter and early spring, and the “epidemic” nature of the disease suggest some environmental component.

Vasculitides  479

SUMMARY BOX: KAWASAKI DISEASE • Presentation: Fever for 5 days or more, erythema of lips and oral mucosa, rash, bilateral nonexudative conjunctivitis • Epidemiology: Eighty percent of cases occur in children under age 5; more common in males and Asians • Pathogenesis: Immune overactivation and excessive cytokine production result in endothelial cell and vascular wall damage • Speculated to be triggered by an infectious agent • Diagnosis: Unexplained fever for 5 or more days, accompanied by at least four of the five following criteria: 1. Bilateral nonexudative conjunctivitis 2. Oral mucous membrane changes, such as cracked lips, throat redness, or strawberry tongue 3. Peripheral extremity changes such as palmar erythema or edema of the hands and feet 4. Polymorphous rash 5. Cervical lymphadenopathy • Complications: Coronary artery aneurysms, leading to myocardial infarction (MI) or fatal arrhythmias • Treatment: Intravenous immunoglobulins (IVIG) and aspirin

Case 20.3 A 40-year-old man is evaluated for a 1-year history of recurrent ear and sinus infections and headache. He has a history of pollen allergy and assumed that the sinus congestion resulted from increased allergies this season. Recently, however, he began to notice blood-tinged sputum and a slight cough.   

1. What is your differential diagnosis?   Upper airway involvement and constitutional complaints (anorexia, fatigue, weakness) are suggestive of a variety of conditions. Recurrent sinusitis with hemoptysis due to acute bronchitis is one possibility, but this seems unlikely. Other diagnoses to consider include Churg-Strauss syndrome, Wegener granulomatosis, Goodpasture syndrome, and bronchogenic carcinoma.     He may simply be experiencing a difficult-to-eradicate sinus infection, and the recent hemoptysis is due to acute bronchitis, the most common cause of hemoptysis. However, other diagnoses to consider include the Churg-Strauss syndrome, which is a systemic vasculitis that occurs in the setting of allergic rhinitis, asthma, and eosinophilia. Pulmonary infiltrates may occur. Asthma generally precedes this disease by many years, and the allergic nasal and sinus disease are generally not destructive.     Another vasculitic syndrome that should be considered is Wegener granulomatosis. Patients with Wegener granulomatosis will often present with sinus, tracheal, or ear complaints. Goodpasture syndrome (also known as anti-glomerular basement membrane disease) is a pulmonary renal syndrome that can also present with cough and hemoptysis.     Finally, pulmonary vascular disorders, such as pulmonary embolism (PE) or elevated pressure in the pulmonary vasculature, can lead to hemoptysis, although nothing else in this patient’s history so far would indicate PE or pulmonary hypertension.

Case 20.3 continued: A chest x-ray study reveals bilateral nodular and cavitary infiltrates, as shown separately. Laboratory workup is significant for an elevated ESR and the presence of antineutrophil cytoplasmic antibodies (c-ANCA) directed against proteinase 3 (PR3).   

2. What is the likely diagnosis?   The presentation of upper and lower respiratory airway symptoms with a workup significant for cavitary infiltrates and elevated inflammatory biomarkers with positive c-ANCA is classic for Wegener granulomatosis. This disease can occur at any age and affects mostly Caucasian individuals. 3. What is the pathogenesis of this disease?   Antibodies to neutrophil cytoplasmic antigens lead to aseptic inflammation and granuloma formation. Inflammation causing vascular injury leads to damage in the respiratory tract and kidneys, specifically causing glomerulonephritis. Granuloma formation occurs both within arterial walls, causing further vasculitic damage, and outside vascular structures, causing granulomatous lesions that may cavitate and damage pulmonary tissue. 4. What is the usual progression of symptoms in this disease? What other organ systems will likely become involved?   Approximately 85% of patients with Wegener granulomatosis will eventually develop pulmonary disease. Symptoms include cough, hemoptysis, and dyspnea. Upper airway involvement may lead to epistaxis or nasal septum perforation. About 75% will develop glomerulonephritis, which will almost always be asymptomatic until the development of advanced uremia. The severity of glomerulonephritis can range from focal and segmental to rapidly progressive (crescentic) glomerulonephritis.     A diagnosis of Wegener granulomatosis before the 1970s meant that the patient had a 50% 5-month survival rate, and 82% of patients died within a year of diagnosis. Besides pulmonary involvement and glomerulonephritis,

480  Vasculitides musculoskeletal symptoms can occur, usually consisting of severe pain that is disproportionate to the signs of inflammation. Peripheral nerves can also be affected in some cases. 5. How is Wegener granulomatosis treated?   The current therapy recommended for Wegener granulomatosis has become the standard for severe vasculitides with significant organ involvement. This consists of a combination of cyclophosphamide and prednisone. Doses are increased until symptoms are reduced and until the leukocyte count returns to normal values. The prednisone is then tapered gradually. Cyclophosphamide is continued, often for a year after symptomatic improvement, although long-term daily cyclophosphamide therapy is associated with bladder cancer and with myelodysplasia. Long-term immunosuppressive therapy also makes patients susceptible to opportunistic disease.

SUMMARY BOX: WEGENER GRANULOMATOSIS • Presentation: Sinus congestion, headache, blood-tinged sputum • Epidemiology: Any age, more common in Caucasians • Pathogenesis: Antibodies to neutrophil cytoplasmic antigens lead to aseptic inflammation and granuloma formation, causing damage to the vasculature • Diagnosis: Positive cytoplasmic antineutrophil cytoplasmic antibodies (c-ANCA), cavitary nodules on chest x-ray • Complications: Necrotizing pulmonary granuloma formation (resulting in hemoptysis) and kidneys (resulting in glomerulonephritis) • Treatment: Cyclophosphamide and prednisone

Case 20.4 A 61-year-old man is evaluated for a 5- to 6-week history of fever, myalgias, fatigue, anorexia, and postprandial abdominal pain. He also complains of painful paresthesias of the hands and has noticed a netlike rash on his legs. His girlfriend adds that she has noticed him dragging his right foot lately. He denies a history of tick bites or unprotected sexual intercourse. Workup reveals that he is positive for hepatitis B surface antigen (HBsAg), although to his knowledge he has never been diagnosed with hepatitis. An arterial biopsy shows inflammation of the tunica media.   

1. What is the diagnosis?   Polyarteritis nodosa (PAN), a necrotizing vasculitis affecting small- to medium-sized arteries with a predilection for the arteries supplying peripheral nerves, skin, the gastrointestinal (GI) tract, and the kidneys. Greater than 80% of people with PAN develop neuropathy, often in the pattern of mononeuritis multiplex (see later discussion). This neuropathy explains the tingling in this patient’s hand and the foot drop. The postprandial abdominal pain or “intestinal angina” is also classic for PAN. 2. What is the significance of the positive hepatitis B surface antigen in this patient?   About 20% of cases of PAN are associated with hepatitis B viral infection. Other microbial pathogens may be a factor in many of the remaining cases, although no definitive links have been established with any infectious agent other than hepatitis B.     It is believed that immune complex depositions with antigens may be a cause of the disease. Inflammatory cells, predominantly neutrophils, form an infiltrate in arterial walls, which eventually leads to fibrinoid necrosis and varying degrees of intimal proliferation. Occlusion and thrombosis of arteries may result, leading to tissue ischemia and the symptoms and complications that have been described here.     The lesions of PAN are segmental and favor the branch points of the smaller arteries. A key pathologic feature is the absence of granulomas or granulomatous infiltration.

STEP 1 SECRET Hepatitis B association with polyarteritis nodosa is a commonly tested fact on Step 1.

Case 20.4 continued: The patient returns to the clinic several weeks later for evaluation of severe hand pain. Examination of his hands reveals severe digital cyanosis and edema, as shown in Fig. 20.2.   

3. What are the clinical manifestations of polyarteritis nodosa?   The rash on this patient’s legs is livedo reticularis, a mottled blue-red discoloration that may affect large areas of the legs, arms, or abdomen. Painful nodules, purpura, and splinter hemorrhages may also be seen. Later in the course of this disease, patients may develop gangrene of the digits following occlusion of the arteries in the hands or feet. GI tract involvement is typically evidenced by postprandial periumbilical pain. Potential life-threatening consequences include

Vasculitides  481

Figure 20.2.  Marked digital cyanosis and swelling. (From Harris ED, Budd RC, Genovese MC, et al. Kelley’s Textbook of Rheumatology. 7th ed. Philadelphia: WB Saunders; 2005.)

rupture of mesenteric aneurysms and perforation of ischemic bowel. Finally, renal involvement, which is almost always seen on autopsy, presents as renin-mediated hypertension caused by occlusion of interlobar renal vessels. 4. How can the diagnosis of polyarteritis nodosa be confirmed?   Diagnosis is based mainly on clinical suspicion. Angiography of the mesenteric or renal arteries may show aneurysms, as is suggested by the name polyarteritis “nodosa.” Arterial biopsy will reveal inflammation of the tunica media and an inflammatory infiltrate without granulomas.

Case 20.4 continued: The patient is placed on high doses of corticosteroids for his PAN. Without immunosuppressive therapy, the prognosis for PAN is poor because of complications from renal failure and mesenteric, cardiac, or cerebral infarction.   

5. Given the high-dose steroids, what prophylaxis needs to be considered?   Prophylaxis against Pneumocystis jiroveci with trimethoprim-sulfamethoxazole should be considered.

SUMMARY BOX: POLYARTERITIS NODOSA • Presentation: Hypertension from stenosis of renal arteries. Skin involvement manifesting as mottled blue-red rash (livedo reticularis) on arms, legs, and abdomen, painful skin nodules, purpura, and splinter hemorrhages. GI involvement manifesting as postprandial periumbilical pain • Epidemiology: Middle-aged or older adult with HBV • Pathophysiology: Immune-complex deposition in arterial walls accompanied by neutrophilic inflammatory infiltrates, leading to fibrinoid necrosis and intimal proliferation that results in artery occlusion and thrombosis • Diagnosis: Clinical suspicion and arterial biopsy showing neutrophilic infiltration of the arterial walls • Complications: Rupture of mesenteric aneurysms, ischemic bowel perforation, gangrene, renin-mediated hypertension • Treatment: Corticosteroids • Prognosis: Poor without treatment from complications related to renal failure, stroke, mesenteric infarction, and cardiac infarction

Case 20.5 A 41-year-old Asian woman is evaluated for a 2-week history of fatigue, diffuse myalgias, anorexia, and unintentional weight loss. She was found to be very mildly anemic, and iron supplementation therapy was begun. She is presenting to the emergency department today with a new, very distinct complaint. She states that since she began to feel fatigued weeks ago, she became more interested in her personal health, attempting to exercise every day and taking her own blood pressure (BP) before and after her morning walks. Yesterday she was unable to get a BP reading in her left arm before exercising. Her BP in her right arm was 110/60 mm Hg, per her report. She had assumed that there had been some problem with the BP cuff, but the same thing happened again this morning. She could not obtain a BP from her left arm, and her right arm read 100/60 mm Hg. She also states, somewhat fearfully, that her left arm feels cool today, and that in retrospect, her left arm has frequently been “tingly” over the past week.   

482  Vasculitides 1. What is the likely diagnosis in this patient?   Given the problem obtaining a BP in this patient’s left arm and the coolness in the left extremity, consider involvement of the arch of the aorta or occlusion of the arteries of the upper extremity. This vascular compromise would also explain the tingling in the arm. Other, nonvascular explanations of paresthesias, such as brachial plexus damage, would not explain the difficulty in obtaining the BP. The patient has no severe back pain, which would raise concern for a ruptured aortic aneurysm. The onset late in life makes a congenital coarctation of the aorta unlikely. Given the very specific presentation, Takayasu arteritis should be considered. 2. What is Takayasu arteritis?   This is a vasculitis of the large elastic arteries, including the aorta and its main branches. It can also affect the coronary and pulmonary arteries. Inflammatory injury to the arterial wall leads to aneurysm formation or occlusion of the arteries, leading to the symptoms of decreased blood flow to the upper extremity in this patient. Takayasu arteritis is also known as pulseless disease, due to the possibility of losing the pulse in one or both upper extremities.     The cause of this disease is unknown. Granulomas and giant cells are characteristically found in the media of the large elastic arteries, and the adventitia is usually profoundly thickened. Destruction of the media by granulomatous inflammation leads to replacement with fibrotic tissue and subsequent aneurysm formation. Thickening of the adventitia, on the other hand, leads to occlusion of the vascular lumen. 3. What makes this patient different from the typical presentation of Takayasu arteritis?   Although a different BP in the two upper extremities is a classic presentation for Takayasu arteritis, this patient does not demonstrate the typical epidemiologic features of this disease. This disease is most common among Asian women (specifically those of Japanese, Chinese, or Korean descent; incidence is also relatively high among Indian women). Furthermore, it is a disease of adolescent girls and young women. Some diagnostic criteria for the disease require that the patient be younger than 40 years of age at onset. Our patient, as a 41-year-old Caucasian woman, is therefore atypical. 4. Ischemic complications due to vascular involvement of the arch of the aorta and its major branches led to this patient’s symptoms. What other symptoms can be expected from further ischemia in a patient with Takayasu arteritis?   The carotid and vertebral arteries can also be involved, leading to symptoms including headache, syncope, or visual disturbance. Stroke can sometimes occur. The following may also be seen: • Involvement of the coronary arteries can produce classic symptoms of myocardial ischemia. • Involvement of the renal arteries can cause renin-induced hypertension, which is classically the presenting symptom in some specific ethnic groups, such as Indians. • Involvement of the mesenteric arteries is less common, but when present it leads to symptoms such as nausea and vomiting. • Progressively enlarging aneurysms can occur, typically along the aorta. These are frequently asymptomatic, however. 5. How is the diagnosis made?   In this patient, physical examination would include listening for a bruit over the subclavian arteries or abdominal aorta and documenting a lower BP in the left arm as compared with the right as well as observing for extremity claudication with activity. Noninvasive magnetic resonance angiography would then be indicated to confirm the diagnosis and to document the extent of arterial wall inflammation. The effects of therapy are usually monitored by documenting change in arterial wall inflammation on angiography and monitoring the diameter of the aortic root. 6. How is this disease treated?   Corticosteroids are used for the management of patients with Takayasu arteritis. The initial dose is usually 60 mg of prednisone per day, and this is tapered as appropriate while monitoring arterial involvement as described previously. Methotrexate may be used to enable a lower dose of steroids to be given.

SUMMARY BOX: TAKAYASU ARTERITIS • Presentation: Initially nonspecific symptoms including fatigue, myalgias, and weight loss, followed by signs of extremity involvement including coolness, pain with use (claudication), and tingling • Epidemiology: Characteristically seen in young Asian women but can be seen in other races as well • Pathophysiology: Mechanism is poorly understood but thought to result from cell-mediated inflammatory injury to the arterial wall that causes fibrosis and adventitial thickening, leading to aneurysm formation and vascular occlusion • Diagnosis: Physical exam findings include decreased brachial pulses, BP difference between the arms, and bruit over subclavian arteries or abdominal aorta. Clinical suspicion is confirmed by noninvasive magnetic resonance angiography or computed tomography (CT) angiography • Complications: Angina or myocardial infarction from involvement of coronary arteries, renin-induced hypertension from involvement of renal arteries, bowel ischemia from involvement of mesenteric arteries, stroke or transient ischemic attack (TIA) from intracerebral artery involvement, and aortic aneurysms • Treatment: Corticosteroids

Deirdre Lewis, MD, Bryan Stenson, Thomas A. Brown, MD, and Sonali J. Bracken

CHAPTER 21

BACTERIAL DISEASES

Insider’s Guide to Bacterial Diseases for the USMLE Step 1 There is no better way to say it: the USMLE loves bacterial diseases! This is one of the highest-yield subjects on the examination, so you must know it well! Our book has divided microbiology into two chapters, but you should note that the breakdown of the examination is not likely to be evenly distributed among bacteria, viruses, fungi, and parasites. Bacterial diseases are tested far more commonly than the other three types, but recently, fungal diseases have been heavily represented on many students’ exams. Fungal diseases are discussed more in Chapter 22, Viral, Parasitic, and Fungal Diseases. As you may know, microbiology is not inherently difficult, but it does take time to learn. The most effective way to study for microbiology on the USMLE is to introduce yourself to this material early on, preferably during your microbiology class in medical school. This is one subject for which multiple resources may be quite helpful to you. For those of you seeking to combine your medical school education with boards studying, we recommend using Clinical Microbiology Made Ridiculously Simple, Microcards, and the microbiology section of Firecracker when you first begin learning the material. Pull the highest-yield facts from these already high-yield materials and write them into First Aid. Many students also find it helpful to make their own flashcards. You can then study from your annotated copy of First Aid and the cases in this book once your focus shifts entirely to boards. How should you be expected to know which facts are the most important to learn for Step 1? That is why you purchased this book! As always, we will be pointing this information out along the way. However, you should keep in mind that the USMLE will expect you to know the major diseases and toxins associated with each and every medically important bacterial species. The Step 1 exam places heavy emphasis on the mechanisms of various bacterial toxins as well as the associated characteristics of individual bacterial species that can be helpful in identifying and differentiating among them in the laboratory.

BASIC CONCEPTS PART I: BACTERIAL MORPHOLOGY AND VIRULENCE FACTORS 1. What makes an organism gram-positive or gram-negative?   Both gram-positive and gram-negative organisms have an internal cell membrane and cell walls made of peptidoglycan. The cell walls of gram-positive organisms tend to be thicker than their gram-negative counterparts and contain teichoic acid. Because gram-positive bacteria have these thicker, mesh-like cell walls, they retain crystal violet dye, causing them to stain purple in color. On the other hand, gram-negative bacteria have very thin cell walls and cannot retain crystal violet dye. They are counterstained with safranin or fuchsine, giving them a pink coloring. However, gramnegative bacteria do have an extra outer membrane outside the cell wall that is largely comprised of lipopolysaccharide (endotoxin) (Fig. 21.1). 2. Why are gram-negative infections more likely to produce bacterial sepsis?   The outer membrane of gram-negative organisms (see previous question) contains lipid A, an endotoxin that is part of the lipopolysaccharide in the cell wall of gram-negative bacteria. Lipid A gets released upon bacterial death and has potent proinflammatory effects.     Lipid A binds to the Toll-like receptor 4 (TLR4)/CD14/MD2 receptor complex on the surface of many different types of immune cells (e.g., macrophages). This stimulates immune cells to secrete a host of inflammatory cytokines including interleukin 1 (IL-1) and tumor necrosis factor alpha (TNF-α), both of which are referred to as acute-phase cytokines. IL-1 stimulates the fever response, while TNF triggers tissue necrosis and shock. Lipid A also stimulates the release of nitric oxide (NO) from endothelial cells, causing vasodilation. Large amounts of lipid A may lead to shock and intravascular coagulation via this stimulatory effect. 3. What are exotoxins?   Exotoxins are proteins released by both gram-positive and gram-negative bacteria during their normal life cycle, as opposed to endotoxin, which is present within the cell wall of gram-negative organisms and is released only after bacterial cells are lysed. Exotoxins released into food can cause poisoning, as in Bacillus cereus and Staphylococcus aureus food poisoning. Pyrogenic exotoxins released by S. aureus and Streptococcus pyogenes can cause rash, fever, and toxic shock syndrome. Enterotoxins act on the gastrointestinal system, whereas neurotoxins act on nerves or motor end plates. For example, infectious diarrhea is caused by enterotoxins released by Vibrio cholerae, Escherichia coli, Campylobacter jejuni, and Shigella dysenteriae.

483

484  Bacterial Diseases Gram-positive (+) cell wall

Peptidoglycan

Lipoteichoic acid

Teichoic acid

Cell wall Cytoplasmic membrane

A

Structural and enzymatic proteins Gram-negative (–) cell wall LPS Porin channel Outer membrane Periplasmic space Cytoplasmic membrane Nutrient-binding protein

B

Transport protein

Lipoprotein

Peptidoglycan

Figure 21.1.  Structure of the cell wall in grampositive and gram-negative bacteria. A, Grampositive bacteria have a thick peptidoglycan layer that contains teichoic and lipoteichoic acids. B, Gram-negative bacteria have a thin peptidoglycan layer that is connected by lipoproteins to an outer membrane. LPS, lipopolysaccharide. (From Rosenthal K. Rapid Review Microbiology and Immunology. 3rd ed. Philadelphia: Elsevier; 2010.)

4. What is a capsule, and what purpose does it serve?   Certain species of bacteria produce a slippery outermost covering called a capsule. This covering consists of highmolecular-weight polysaccharides, which help the bacteria evade phagocytosis by neutrophils and macrophages. Note that Bacillus anthracis has a proteinaceous capsule that consists of d-glutamic acid. The capsule is not essential for growth and serves only in a protective capacity. The most common medically relevant encapsulated organisms are Streptococcus pneumoniae, Klebsiella pneumoniae, Haemophilus influenzae type b, Pseudomonas aeruginosa, Neisseria meningitidis, and Cryptococcus neoformans (a fungus).     Remember that Some Killers Have Perfectly Nasty Capsules. This mnemonic will help you recall the encapsulated organisms that are important to know for boards.     Note: In the Quellung reaction, which tests for the presence of encapsulated bacteria, encapsulated bacteria will swell when exposed to specific antibodies. The latex agglutination assay and India ink stain are two other methods for detecting capsular presence.

STEP 1 SECRET Although the Quellung reaction and several other techniques in this book may be clinically outdated, you should remember that many of the physicians who author Step 1 questions will have relied upon this technology during the course of their careers and will thus expect you to know the names and basic principles behind these tests. As a general rule, you should focus on learning the techniques listed in this book and in First Aid. You are not expected to know complex or cutting-edge technologies that are not mentioned in your USMLE study resources.

5. What sort of individual is susceptible to infection by encapsulated bacteria?   Splenectomized patients or those with opsonization/complement protein defects are susceptible. The spleen normally sequesters encapsulated bacteria. Patients who have undergone splenectomy, such as sickle cell disease patients or sufferers of hemolytic anemias, are at a greater risk for incurring infection by encapsulated bacteria. Patients with opsonization defects and complement protein deficiencies are also vulnerable to infection by encapsulated bacteria (see Chapter 15, Table 15-3). C5-C9 complement deficiency and subsequent predisposition to Neisseria meningitides is a classic vignette on Step 1.

Bacterial Diseases  485

6. Identify the Gram stain and the morphology of the organisms in Table 21.1. 7. Review Tables 21.2 through 21.9 to test your knowledge of the properties of these high-yield bacteria. 

BASIC CONCEPTS PART II: ANTIBACTERIAL PHARMACOLOGY 1. Describe the difference between bacteriostatic and bactericidal antibiotics.   Bacteriostatic antibiotics work by inhibiting the growth or reproduction of the infectious bacteria; they do not kill the organism. These include most ribosomal-acting antibiotics such as tetracyclines and macrolides, which block bacterial protein translation by inhibiting ribosomal subunits.     On the other hand, bactericidal agents actually kill the bacteria. These antibiotics include agents that disrupt the cell wall such as β-lactam antibiotics (penicillins, cephalosporins, carbapenems) as well as aminoglycosides, fluoro­ quinolones, and metronidazole, which generally act via different mechanisms.

Table 21.1.   Bacterial Images IMAGE

GRAM STAIN AND MORPHOLOGY

Gram-positive cocci in pairs: S. pneumoniae

Gram stain of a sputum sample infected with Streptococcus pneumoniae. Gram-positive cocci in clusters: S. aureus

Expectorated sputum with gram-negative rods in a patient with Klebsiella pneumoniae pneumonia. Continued

486  Bacterial Diseases Table 21.1.   Bacterial Images—cont’d IMAGE

GRAM STAIN AND MORPHOLOGY

Gram-negative cocci in pairs: Neisseria spp.

Sputum smear, stained with Gram stain, shows many neutrophils and intracellular gram-negative diplococci suggestive of Neisseria meningitidis infection (oil immersion). Gram-negative rods: many possibilities

Klebsiella pneumoniae image from Bennett JE, Dolin R, Blaser MJ. Mandell, Douglas, and Bennett’s Principles and Practice of Infectious Diseases. 8th ed. Philadelphia: Elsevier; 2015. Images of Streptococcus pneumoniae, Neisseria meningitidis, and gram-negative rods from McPherson RA, Pincus MR. Henry’s Clinical Diagnosis and Management by Laboratory Methods. 22nd ed. Philadelphia: Saunders; 2011.

2. Describe the most common mechanisms by which antibiotic agents work.   Antibiotics generally work via four different mechanisms. 1. Disruption of the cell wall synthesis (β-lactam antibiotics) 2. [Direct] inhibition of bacterial DNA replication (fluoroquinolones) 3. [Indirect] inhibition of bacterial DNA synthesis (trimethoprim, sulfamethoxazole) 4. Impaired function of the bacterial ribosome (macrolides, tetracyclines, aminoglycosides, chloramphenicol, clindamycin, linezolid) 3. What are the β-lactam antibiotics, and what is their mechanism of action?   The β-lactam antibiotics include the penicillins, cephalosporins, and carbapenems (imipenem, meropenem). By virtue of their β-lactam chemical moiety, they all block bacterial cell wall synthesis by inhibiting bacterial transpeptidase, also known as penicillin-binding protein (PBP). Resistance to these antibiotics is mediated by bacterially synthesized β-lactamase enzymes that destroy the β-lactam ring. 4. Why are clavulanic acid, sulbactam, and tazobactam added to some penicillins?   These agents inhibit β-lactamase, thereby reducing resistance of bacterial species to the penicillins. Typical combinations include amoxicillin-clavulanic acid, ampicillin-sulbactam, and piperacillin-tazobactam.

Bacterial Diseases  487

Table 21.2.   Gram-Positive Cocci ASSOCIATED DISEASE(S)

ORGANISM

TOXIN

Protein A (binds Fc portion of IgG) Toxic shock syndrome toxin-1 (TSST-1)

LABORATORY INFO

PEARLS TO REMEMBER

Catalase positive Coagulase positive

Toxin-mediated diseases: Staphylococcal toxic shock syndrome Scalded skin syndrome Staphylococcal gastroenteritis

Staphylococcus aureus

Cellulitis Acute endocarditis (in previously normal valve) Osteomyelitis Pneumonia Carbuncles/furuncles Stye

Staphylococcus epidermidis

Prosthetic valve endocarditis — Bacteremia from indwelling catheters

Novobiocin-sensitive Normal skin flora

Staphylococcus saprophyticus

Cystitis in young women — Second most common cause of UTI (behind E. coli )

Novobiocin-resistant



Streptococcus pneumoniae

Pneumonia Meningitis Sinusitis Otitis media

IgA Protease

α-Hemolysis Bile-soluble Optochin-sensitive

Encapsulated rust-colored sputum

Viridans streptococci

Dental caries (S. mutans) Subacute bacterial endocarditis (S. sanguinis)



α-Hemolysis Optochin-resistant

Normal oral flora

Streptococcus agalactiae (group B streptococci)

Neonatal pneumonia, men- — ingitis, and sepsis Chorioamnionitis

β-Hemolysis Bacitracin-resistant

Normal vaginal flora

Streptococcus bovis

Subacute endocarditis



Strong association with colon cancer

Enterococcus spp.

UTI Bacteremia/sepsis Endocarditis Abdominal abscess

α- or γ-Hemolysis

Part of normal bowel flora that causes disease when host is immunocompromised or gastrointestinal tract has been breached



UTI, urinary tract infection.

Table 21.3.   Gram-Positive Bacilli ORGANISM

Bacillus anthracis

ASSOCIATED DISEASE(S)

Cutaneous anthrax (most common form) Pulmonary anthrax

TOXIN

Lethal factor Edema factor (increases cAMP)

LABORATORY PEARLS TO INFO REMEMBER

Spore-forming

Painless black eschars with cutaneous anthrax Pulmonary anthrax (Woolsorter’s disease)

Corynebacterium Diphtheria Diphtheria toxin Grows on tellurite Normal skin flora spp. Granulomatous lymphadenitis (RNA translaagar Pseudomembrane or esophaPneumonitis tional inhibitor geal web Pharyngitis that inactivates Metachromatic granules Skin infections EF-2 via ADP Endocarditis ribosylation) Listeria monocytogenes

Listeriosis

Listeriolysin O (LLO)

Exhibits characteristic tumbling motility

Perinatal/neonatal infections Immunocompromised persons at risk Associated with raw milk and dairy products

ADP, adenosine diphosphate; cAMP, cyclic adenosine monophosphate; EF-2, elongation factor 2; RNA, ribonucleic acid.

488  Bacterial Diseases Table 21.4.   Gram-Negative Cocci ORGANISM

TOXIN

LABORATORY PEARLS TO INFO REMEMBER

IgA protease



Has a capsule Purpuric, nonblanching rash Vaccine available

Infects superficial mucosal surfaces IgA protease lined with columnar epithelium: Urethra: urethritis (gonorrhea) Vagina: vulvovaginitis in young girls Rectum: proctitis Conjunctiva: ophthalmia neonatorum



No vaccine Main cause of infectious arthritis in sexually active persons

ASSOCIATED DISEASE(S)

Neisseria meningiti- Meningitis dis (meningoSepticemia coccus) Waterhouse-Friderichsen syndrome Neisseria gonorrhoeae (gonococcus)

Table 21.5.   Enteric Gram-Negative Rods ASSOCIATED DISEASE(S)

TOXIN

Campylobacter jejuni

Enteritis



Escherichia coli

Enteritis UTI Meningitis Peritonitis Mastitis Septicemia Gram-negative pneumonia HUS

K Capsule (can cause — pneumonia) Labile toxin (increases cAMP) Stable toxin (increases cGMP)

Normal gut flora E. coli O157:H7—a particularly virulent pathologic strain associated with HUS

Salmonella spp.

Food-borne illness Typhoid fever (Salmonella typhi )



Produces hydrogen sulfide (H2S)

Osteomyelitis in patients with sickle cell anemia

Shigella spp.

Shigellosis (bacterial dysentery)

Shiga toxin (inhibits protein synthesis in target cells)

Does not produce hydrogen sulfide (H2S)

Bloody diarrhea Fecal–oral route of transmission Low inoculum required (toxin-mediated)

Vibrio cholerae

Diarrhea

Cholera toxin (perma- Grows in alkaline nently activates media Gs) Oxidase positive Comma-shaped

“Rice-water” diarrhea Prompt oral rehydration necessary

Helicobacter pylori

Peptic ulcer disease Gastritis Duodenitis Gastric cancer Mucosa-associated lymphoid tissue (MALT) lymphoma



Lives in stomach but common in duodenal ulcers Triple treatment: amoxicillin, clarithromycin, and proton pump inhibitor

ORGANISM

LABORATORY INFO

PEARLS TO REMEMBER

Grows at 42°C Oxidase positive Comma-shaped

Present in animal feces Associated with Guillain-Barré syndrome

Positive urea breath test due to presence of enzyme urease

cAMP, cyclic adenosine monophosphate; cGMP, cyclic guanosine monophosphate; HUS, hemolytic uremic syndrome; UTI, urinary tract ­infection.

Bacterial Diseases  489

Table 21.6.   Other Gram-Negative Rods ORGANISM

ASSOCIATED DISEASE(S)

TOXIN

LABORATORY INFO

PEARLS TO REMEMBER

Bordetella pertussis

Pertussis (whooping Pertussis toxin (inactivates Grown on Bordetcough) Gi, leading to high Gengou agar amounts of cAMP)

Highly contagious; spread by coughing and nasal drops

Brucella spp.

Brucellosis (“undulant fever”)





Transmitted via contaminated or unpasteurized milk

Francisella tularensis

Tularemia (“rabbit fever”)





Reservoir in rabbits; transmitted by tick Symptoms/signs similar to those of plague Culture, drainage contraindicated owing to high virulence

Haemophilus influenzae

Meningitis (type b) Bacteremia Cellulitis Pneumonia Sinusitis Epiglottitis

IgA protease

Grown on chocolate Type b encapsulated and Agar with Factors more virulent V (NAD+) and X Vaccine available for (hematin) type b strain

Pseudomonas aeruginosa

Pneumonia in cardiac failure patients External otitis Osteomyelitis in diabetics Endocarditis UTI Hot tub folliculitis

Exotoxin A (inactivates Produces the blueEF-2, similar to Diphgreen pigment theria toxin) pyocyanin Oxidase positive

Legionella pneumophila

Legionnaire’s disease Pontiac fever



Readily visualized Legionnaire’s disease: with silver stain acute pneumonia with Grown on charcoal multisystem involveyeast extract with ment; from water iron and cysteine source, so no personto-person spread Pontiac fever: similar to flu

Yersinia pestis

Bubonic plague





Transmitted by fleas from rodents to humans Black buboes

Yersinia enterocolitica

Enterocolitis





Pseudoappendicitis (can mimic Crohn’s disease as well) Seen in nursery schools

Think Pseudomonas infection in burn patients and intravenous drug users Can cause black skin lesions Resistant to many antibiotics

cAMP, cyclic adenosine monophosphate; EF-2, elongation factor 2; NAD, nicotinamide adenine dinucleotide; UTI, urinary tract infection.

STEP 1 SECRET Antibiotics that inhibit the bacterial ribosome can be recalled with the mnemonic “These Malicious Antibiotics Cripple Little Critters” (Tetracyclines, Macrolides, Aminoglycosides, Chloramphenicol, Linezolid, Clindamycin).

490  Bacterial Diseases Table 21.7.   Anaerobes ORGANISM

Clostridium perfringens

ASSOCIATED DISEASE(S)

TOXIN

Anaerobic cellulitis Alpha toxin (lecithinase) Gas gangrene (myonecrosis) Food poisoning

Clostridium tetani Tetanus (“lockjaw”)

LABORATORY PEARLS TO INFO REMEMBER

Spore-forming

Exotoxin causes spastic paraly- Spore-forming sis by blocking inhibitory glycine release from Renshaw cells in spinal cord

Crepitus is associated with gas gangrene

Vaccine is available

Clostridium botulinum

Botulism Food poisoning that causes flac- Spore-forming Floppy baby syndrome cid paralysis (ingestion of spores Preformed toxin prevents in honey) release of acetylcholine (Ach) at presynaptic terminals

Classic scenario from consumption of dented canned goods or honey

Clostridium difficile

Pseudomembranous colitis

Caused by antibiotic use, especially clindamycin or ampicillin Treat with metronidazole or oral vancomycin

Toxin A (acts on brush border of gut) Toxin B (causes pseudomembranes)

Spore-forming

Table 21.8.   Spirochetes ORGANISM

ASSOCIATED DISEASE(S)

Borrelia burgdorferi

LABORATORY INFO

PEARLS TO REMEMBER

Lyme disease

Best visualized with Wright or Giemsa stain

Bull’s-eye rash Late stage of infection is associated with arthritis and neurologic symptoms

Borrelia recurrentis

Relapsing fever



Organism switches surface proteins to evade immune response, leading to intermittent fevers

Treponema pallidum

Syphilis

Visualized via dark-field microscopy

Spread through sexual contact or through vertical transmission

Leptospira interrogans

Leptospirosis



Transmitted by water that is contaminated by animal urine through cracks in the skin, eyes, or mucous membranes

5. What percentage of patients allergic to penicillin is also allergic to cephalosporins?   Only a small percentage of individuals with a penicillin allergy will also have a hypersensitivity reaction to cephalosporins that results in pruritus, urticaria, bronchospasm, laryngeal edema, and hypotension. There is no cross-reactivity between penicillins and aztreonam. 6. What is the antibacterial spectrum of the various subclasses of penicillins and cephalosporins?   The spectrum of action of antibiotics is complicated, and the USMLE does not expect you to have a specialist’s understanding. Instead, you should focus on overall themes. Note that new antibiotic classes were created as organisms became increasingly resistant to “older” drugs. Thus, each new antibiotic class that was developed typically demonstrated broader or improved coverage.     Natural penicillins have largely gram-positive coverage. Extended spectrum penicillins such as ampicillin and amoxicillin were developed to include better gram-negative coverage. (Importantly, they also cover enterococcus!) In order to combat the growing issue of penicillinase-producing bacteria, penicillinase-resistant penicillins were developed (e.g., methicillin, oxacillin, nafcillin, and dicloxacillin). These antibiotics are used largely for treatment of methicillin-sensitive Staphylococcus aureus (MSSA). Finally, antipseudomonal penicillins were developed. These include ticarcillin and piperacillin.

Bacterial Diseases  491

Table 21.9.   Intracellular Organisms ORGANISM

ASSOCIATED DISEASE(S) LABORATORY INFO

PEARLS TO REMEMBER

Mycoplasma pneumoniae

Atypical (“walking”) pneumonia

Best grown on Eaton’s agar Blood shows IgM “cold agglutinins”

No cell wall Treat with macrolides Chest radiograph demonstrates diffuse interstitial infiltrates; radiographic changes often more extensive than expected from patient’s symptoms

Chlamydia trachomatis

Urethritis Pelvic inflammatory disease Blindness Lymphogranuloma venereum Neonatal conjunctivitis

Visualized with Giemsa stain Cell wall lacks muramic acid

Treat neonates with erythromycin eye drops for conjunctivitis

Chlamydia psittaci

Psittacosis (flu-like syndrome)



Transmitted from bird droppings via aerosol

Chlamydia pneumoniae

Atypical pneumonia



Transmitted via aerosols

Mycobacterium tuberculosis

Tuberculosis



Associated with granulomas and caseous necrosis

Mycobacterium leprae

Leprosy (Hansen’s disease)



Tuberculoid form: milder with few organisms in lesions Lepromatous form: severe with many organisms in lesions Grows in cool temperatures, so affects distal sites Treat with dapsone and rifampin

Rickettsia rickettsii

Rocky Mountain spotted fever (rash starts on palms and soles and migrates centrally)

Weil-Felix test will be positive for rickettsial diseases

Rash that starts on palms and soles and migrates centrally (centripetal migration) Treat with tetracyclines

    Cephalosporins can be thought of as “stronger” penicillins, as they have a similar mechanism of action but are less susceptible to the effects of β-lactamases. There are a multitude of drugs in this class with more names than you are expected to know, but important cephalosporins are highlighted in Table 21.10. First-generation and second-generation cephalosporins largely have gram-positive coverage, while third-generation cephalosporins also have robust gramnegative coverage. Fourth-generation cephalosporins (cefepime) have a similar coverage spectrum as third-generation cephalosporins but also include pseudomonal coverage. 7.  What is the antibacterial spectrum and mechanism of action of vancomycin?   Vancomycin is mostly effective against gram-positive bacteria and is often used for drug-resistant organisms such as methicillin-resistant Staphylococcus aureus (MRSA). It is also a common treatment for Clostridium difficile infections. It is important to note that vancomycin is poorly absorbed from the intestinal tract. Therefore, it is primarily administered orally for infections of the colon such as C. difficile colitis and diverticulitis; otherwise, vancomycin must be given intravenously.     Vancomycin acts by inhibiting bacterial cell wall synthesis. Its specific mechanism of action involves binding to the d-Ala-d-Ala site of the gram-positive bacteria. However, some bacteria have developed resistance to vancomycin by altering their binding site to d-Ala-d-Lac, thereby preventing the antibiotic from working.     Intravenous vancomycin is commonly used in the hospital for treatment of MRSA. Other antibiotics used to cover MRSA include linezolid and daptomycin; however, you are more likely to be tested on vancomycin. 8. What is the antibacterial spectrum of the fluoroquinolones, and what is their mechanism of action?   This class has a broad spectrum of activity, including both gram-positive and gram-negative organisms. They also cover Pseudomonas, making them fairly similar in spectrum to the antipseudomonal penicillins. These antibiotics work by inhibiting bacterial DNA synthesis through inhibition of the bacterial topoisomerase (DNA gyrase) protein. You can recognize them by their suffix “floxacin” (e.g., levofloxacin, ciprofloxacin, and moxifloxacin). They are frequently used to treat respiratory infections such as pneumonia and urinary tract infections.

492  Bacterial Diseases 9. What are the antimicrobial spectrum and mechanism of action of the macrolides?   These antibiotics have good gram-positive coverage and also cover Mycoplasma, Legionella, and Chlamydia (recall that this is one of “our favorite drugs for intracellular bugs”). Macrolides are also used for prevention of Mycobacterium avium complex (MAC) species in HIV patients with CD4 counts below 50 cells/mm3. They work by inhibiting bacterial protein synthesis via the 50s ribosomal subunit. Examples include azithromycin and erythromycin. An important side effect of macrolides for Step 1 is that they can prolong the QT interval. 0. What are the mechanism of action, side effects, and spectrum of activity of tetracyclines? 1   Tetracyclines work by inhibiting bacterial protein synthesis by binding to the 30s ribosomal subunit. These agents are bacteriostatic (stop bacterial reproduction without harming the organism in any other way). Important side effects of tetracyclines include the discoloration of teeth in children, photosensitivity, and renal impairment. These drugs are the most important agents for the treatment of infection with intracellular organisms such as Chlamydia and the rickettsial species. 1. What are the mechanism of action and spectrum of the aminoglycosides? 1   Aminoglycosides are irreversible inhibitors of protein synthesis (via the 30s ribosomal subunit) that are generally only effective against gram-negative rods. However, they may be used in combination with penicillins for enterococcal endocarditis (a gram-positive organism). Aminoglycosides are frequently combined with ampicillin for broad gram-positive and gram-negative coverage. They are also the only bactericidal ribosomal antibiotics. Important side effects to know for this class are ototoxicity and nephrotoxicity.

STEP 1 SECRET Aminoglycosides are the only bactericidal ribosomal agents. They also bind to the 30s subunit. Remember this by recalling that April (A for Aminoglycoside) has 30 days. You can think of a mean kid shouting in your ear to remember that a-MEAN-oglycosides are toxic to the KIDney and the EAR. 2. How does chloramphenicol work, and why is it rarely used? 1   This antibiotic also inhibits protein synthesis via the 50s ribosomal subunit, but because of the risk of aplastic anemia, it is not commonly used in industrialized nations. Another must-know side effect of chloramphenicol is “gray baby syndrome,” which is potentially fatal. It occurs in premature infants and neonates who are unable to fully metabolize the drug, leading to shock and cyanosis (hence the name “gray baby syndrome”). 13. How does trimethoprim work? Why is it commonly given in combination with sulfamethoxazole, as TMP-SMX?   Both trimethoprim and sulfamethoxazole inhibit the formation of tetrahydrofolic acid, an essential precursor of thymidine. This inhibits synthesis of bacterial DNA. Because these agents inhibit tetrahydrofolic acid synthesis at different steps, their combination is synergistic. Trimethoprim inhibits dihydrofolate reductase while sulfamethoxazole inhibits dihydropteroate synthetase (Fig. 21.2).

Dihydropteroate diphosphate + p-aminobenzoic acid (PABA) Dihydropteroate synthetase

X

Sulfonamides

Dihydropteroic acid

Dihydrofolic acid Dihydrofolate X reductase

Trimethoprim

Tetrahydrofolic acid

Figure 21.2.  Mechanism of trimethoprim and sulfamethoxazole in folate synthesis inhibition. (From Wikipedia available at http://en.wikipedia.org/wiki/ Trimethoprim/sulfamethoxazole, accessed November 3, 2016.)

Bacterial Diseases  493

14. Test yourself on the coverage, mechanism of action, resistance, and common side effects for the antibiotic classes using Table 21.10.

STEP 1 SECRET The list of antibiotics to know for Step 1 is quite extensive, and students often wonder how in depth their knowledge must be to learn this subject for boards. Our best guess is that you should expect anywhere from one to three questions on antibiotics. First Aid has a great review of this topic, but there is still quite a bit of information in these pages. If you can learn it all, great! If you find yourself short on time, go for the highest-yield points. For each antibiotic you should therefore learn this information in the following order: • Mechanism of action and mode of resistance • Unique side effects and toxicity symptoms: Note that we said you should learn the unique side effects of each drug. Boards will not test you on the fact that certain antibiotics can cause occasional gastrointestinal (GI) upset or headache. These symptoms are characteristic of too many drugs to make for good test questions. Focus on the toxicities listed in Table 21.10. • Clinical uses: Note that you should know the general uses for each drug (e.g., vancomycin is used for gram-positive organisms only, aminoglycosides for serious gram-negative infections, aztreonam for gram-negative rods, metronidazole and clindamycin for anaerobes), but you do not necessarily need to learn the individual organisms affected by every antibiotic. We do not mean to imply that this material is not important for your clinical years or fair game for boards, but it is less likely to be tested than the previous two points. However, you should be sure to know which drugs can be used for select bacterial species—namely, Pseudomonas (piperacillin-tazobactam, ciprofloxacin, cefepime, imipenem), methicillin-resistant Staphylococcus aureus (MRSA) (vancomycin), and Enterococcus (ampicillin, vancomycin).

Case 21.1 A 64-year-old man is evaluated for a 3-day history of sudden onset productive cough, fever, and chills. He describes his phlegm as “rust colored” and notes that his ribs hurt when he takes a deep breath. On exam, the patient is febrile with a temperature of 101.5°F and an O2 saturation of 89%. Crackles are heard in the right lower posterior lung field. Laboratory workup reveals a significant leukocytosis. Chest x-ray study and sputum culture are pending.   

1. What is the most likely diagnosis?   The combination of fever, chills, pleuritic chest pain, hypoxemia, and productive cough is very suggestive of pneumonia. Furthermore, the rust-colored sputum suggests streptococcal pneumonia.

STEP 1 SECRET Gram-positive cocci in pairs and rust-colored sputum are common buzzwords for Streptococcus pneumoniae. You should know the buzzwords associated with various microorganisms. Currant-jelly sputum should suggest Klebsiella infection, while frank blood or hemoptysis should suggest tuberculosis. We will continue to draw attention to these buzzwords throughout the microbiology chapters.

2. What defense mechanisms prevent pneumonia in the healthy individual?   The respiratory tract has many defenses in place to prevent access to the lungs by potential pathogens. The nasal hairs, mucosa, and dynamics of airflow all act early to prevent inhalation of microorganisms. The epiglottis and cough reflex both act to prevent particulate matter from traveling into the deeper airways. The respiratory tract is lined with mucus until the terminal bronchioles are reached. This mucus is propelled upward by the ciliated epithelium, eliminating foreign material as expectorant. The last line of defense is in and around the alveolar complex and is composed of alveolar macrophages, infiltrating leukocytes (e.g., lymphocytes, neutrophils), immunoglobulin, and complement. These components will become hyperactive during an infectious process.     Note: Any state that alters the level of consciousness (anesthesia, seizure, intoxication, sedation, and neurologic disorders such as coma) predisposes to aspiration pneumonia because of suppression of the cough reflex. The organisms causing this type of infection are usually anaerobes from the mouth or refluxed gastric contents (e.g., Bacteroides species). 3. Why might a patient in the intensive care unit who is intubated be at increased risk for developing pneumonia?   Mechanical ventilation bypasses the normal host defenses (e.g., mucociliary clearance) for preventing contamination of the sterile lower respiratory segments. For each day on mechanical ventilation, it is estimated that the patient has a 1% increased chance of acquiring nosocomial pneumonia. The expected duration of intubation must therefore be a consideration in deciding whether or not to place a patient on mechanical ventilation. In any question where you suspect nosocomial or ventilator-associated pneumonia, consider Pseudomonas aeruginosa infection.

DRUG CLASS EXAMPLES

COVERAGE

Penicillin-based Antibiotics Natural penicillin Mostly gram-positive Penicillin ­organisms

Extended-spectrum penicillins Ampicillin Amoxicillin

Gram positive with improved gram negative coverage, includes enterococcus

Antistaphylococcal (penicillinase-resistant) penicillins Dicloxacillin Cloxacillin Methicillin Oxacillin

MSSA

Antipseudomonal penicillins Ticarcillin Piperacillin

Increasing gram-negative coverage includes ­Pseudomonas

MECHANISM OF ACTION

MECHANISM OF RESISTANCE

Inhibit transpeptidase and stimulation of autolysis

Formation of β-lactamases that break the β-lactam ring

ADVERSE DRUG EFFECTS

NOTES

Increasing resistance limits use, antibiotic of choice for syphilis (intramuscular or intravenous penicillin G)

Better resistance to β-lactamases due to bulk side chain

Penicillin plus β-lactamase β-Lactam-resistant bacteria. inhibitor Piperacillin-tazobactam Ampicillin-sulbactam covers Pseudomonas Amoxicillin-clavulanic acid Piperacillin-tazobactam First-generation cephalosporins Cephalexin Cefotetan Cefazolin

Mostly gram positive

Ten percent cross-reactivity with penicillin allergy (all generations)

494  Bacterial Diseases

Table 21.10.   Antibacterial Pharmacology

Second-generation ­cephalosporins Cefuroxime Cefaclor Cefoxitin

Mostly gram positive

Third-generation ­cephalosporins Ceftazidime Ceftriaxone Cefotaxime

Mostly gram-negative coverage, including invasive infections such as meningitis and pneumococcal pneumonia as well as gonorrhea. Ceftazidime covers Pseudomonas

Fourth-generation ­cephalosporins Cefepime Monobactam

No penicillin cross-reactivity

Carbapenems

Imipenem is administered with cilastatin to inhibit metabolism by renal dehydropeptidase I

Ribosomal Antibiotics Aminoglycosides Gram negative Gentamicin Tobramycin Streptomycin Neomycin Amikacin Lyme, Rickettsia, Chlamydia

Acetylation, adenylation, phosphorylation

Nephrotoxicity and ototoxicity Only bactericidal ribosomal antibiotic

Bind to the 30S subunit of the bacterial ribosome, inhibiting protein synthesis

Decreased transport into the cell and increased transport out of the cell

GI upset, toxicity in renal impairment, photosensitivity; alters bone growth and discolors teeth in children

Demeclocycline more commonly used to treat SIADH

Continued

Bacterial Diseases  495

Tetracyclines Doxycycline Tetracycline Demeclocycline

Impairs proper assembly of the ribosome, causing the 30S subunit to misread the genetic code

DRUG CLASS EXAMPLES

COVERAGE

Clindamycin

Gram positive, anaerobes

Chloramphenicol

Gram positive, gram negative, Reversibly inhibits protein anaerobes synthesis by binding to the 50s subunit

Macrolides Azithromycin Erythromycin

Gram positive, atypicals (Mycoplasma, Legionella, Chlamydia), newborn conjunctivitis prophylaxis, MAC prophylaxis in HIV

Unique Antibiotics Vancomycin

MECHANISM OF ACTION

MECHANISM OF RESISTANCE

Binds 50s subunit to prevent peptide bond formation

Binds to 50S subunit of ribosome, inhibiting translocation

Gram-positive coverage only; Inhibits cell wall synthesis MRSA (IV), C. difficile (oral) by binding d-alanine

Metronidazole

Giardia, Entamoeba, Trichomonas, Gardnerella, anaerobes, C. difficile

Converts to a toxic metabolite that prevents cell wall synthesis

Fluoroquinolones Ciprofloxacin Levofloxacin Moxifloxacin

Gram-negative rods, UTIs, respiratory infections

Inhibits DNA gyrase, preventing DNA replication

TMP-SMX

UTIs, MRSA, PCP and toxoplasmosis and PCP prophylaxis in HIV

Inhibits folic acid synthesis

Linezolid

MRSA

Binds to 50s subunit to prevent protein synthesis

ADVERSE DRUG EFFECTS

NOTES

Common cause of antibioticassociated Clostridium difficile Acetylation

Aplastic anemia, gray baby syndrome

P450 inhibitor. Not used in the United States because of gray baby syndrome

Methylation

GI upset, acute cholestatic hepatitis, prolonged QT

P450 inhibitor. Bacterostatic alone. Erythromycin also used for GI motility

Nephrotoxicity, ototoxicity, thrombophlebitis, Red Man syndrome

Administer antihistamine and slow infusion rate to treat Red Man syndrome

Disulfiram-like reaction, metallic taste

P450 inhibitor

d-Alanine replaced d-lactate

with

Efflux pump and mutated DNA gyrase

Cartilage damage in children, P450 inhibitor tendon rupture in adults

Megaloblastic anemia, leuko- Treat bone marrow suppenia, granulocytopenia. pression with leucovorin Sulfonamides component rescue can cause allergic reaction, hemolysis in G6PD, photosensitivity

496  Bacterial Diseases

Table 21.10.   Antibacterial Pharmacology—cont’d

Daptomycin

MRSA, VRE

Disrupts bacterial plasma membrane by altering electrical charge

Myopathy, elevated CPK

Activated by mycobacterial KatG, product inhibits mycolic acid synthesis

Hepatotoxicity, peripheral P450 inhibitor. Administer neuropathy, drug-induced with B6 to prevent pelupus ripheral neuropathy

Ethambutol

Obstructs formation of mycobacterial cell wall by inhibiting arabinosyl transferase

Red-green color blindness, hepatotoxicity

Pyrazinamide

Mechanism unknown

Hepatotoxicity

Rifampin

Inhibitor of bacterial DNA-dependent RNA polymerase

Hepatotoxicity, turns body fluids orange

Streptomycin

Blocks 30s subunit of bacte- Acetylation, adenylation, or rial ribosome phosphorylation

Nephrotoxicity and ototoxicity See also aminoglycosides

Mycobacterial Drugs Isoniazid Mycobacterium tuberculosis

P450 inducer

CPK, creatine phosphokinase; GI, gastrointestinal; HIV, human immunodeficiency virus; IV, intravenous; KatG, catalase-peroxidase enzyme; MAC, Mycobacterium avium complex; MRSA, methicillin-resistant Staphylococcus aureus; MSSA, methicillin-sensitive Staphylococcus aureus; PCP, Pneumocystis pneumonia; SIADH, syndrome of inappropriate secretion of antidiuretic hormone; TMP-SMX, trimethoprim-­ sulfamethoxazole; UTI, urinary tract infection; VRE, vancomycin-resistant enterococci.

Bacterial Diseases  497

498  Bacterial Diseases 4. Why is it important to distinguish between community-acquired and nosocomial pneumonia?   There is generally a different spectrum of organisms that cause these two types of pneumonia, so empirical selection of antibiotics is different. The most common pathogens causing community-acquired pneumonia include S. pneumoniae, H. influenzae, Legionella pneumophila, and Mycoplasma pneumoniae. The most common pathogen causing nosocomial pneumonia is S. aureus, but P. aeruginosa should always be considered. 5. What is atypical (“walking”) pneumonia, and is the patient in this case more likely to have a typical or an atypical pneumonia?   Atypical or “walking” pneumonia has a more insidious onset than the sudden onset described in this case. It classically occurs in younger individuals who live in close proximity to others (e.g., college dormitory or military barrack ­inhabitants). It is characterized by headache, nonproductive cough, low-grade fever, and a nonspecific diffuse interstitial infiltrate on chest x-ray study that looks worse than might be expected from the patient’s appearance. Atypical pneumonia is generally caused by viruses or intracellular bacteria such as L. pneumophila, M. pneumoniae, and species of Chlamydia such as Chlamydia psittaci. M. pneumoniae is the classic causative organism and can be differentiated from other causes based upon a high titer of cold agglutinins (IgM). Most of the bacterial causes can be treated with a macrolide or tetracycline. Remember that these are “our favorite drugs for intracellular bugs!”     This patient most likely has a typical pneumonia based on the rapidity of onset and productive cough.

STEP 1 SECRET Note: The term cold agglutinins refers to the fact that IgM antibodies bind optimally to red blood cells (RBCs) at low temperatures and cause them to agglutinate, or stick together. This can be demonstrated at the bedside when a blood sample becomes clumpy when placed in ice and fluid again when rewarmed. Boards commonly test students on the association between cold agglutinins and M. pneumoniae.

Case 21.1 continued: Chest x-ray film is shown in Fig. 21.3A. Sputum Gram stain reveals large numbers of slightly elongated, gram-positive cocci in pairs and chains (Fig. 21.3B).   

A

B

Figure 21.3.  A, Chest film showing classic pneumococcal pneumonia (arrows). B, Gram-stained sputum from a patient in Case 21.1 at 1000× magnification. (A from Brown TA, Brown D. USMLE Step 1 Secrets. Philadelphia: Hanley & Belfus; 2004. B from Goldman L, Schaffer A. Goldman-Cecil Medicine. 25th ed. Philadelphia: Elsevier; 2016.)

6. What is the diagnosis?   Gram-positive cocci in pairs are suggestive of streptococcal infection. For the sake of completeness, the chest x-ray film shows opacification (consolidation) of the right upper lobe, consistent with a lobar pneumonia. In contrast, atypical pneumonia would show diffuse interstitial infiltrates without evidence of lobar consolidation (Fig. 21.4).

Bacterial Diseases  499

Figure 21.4.  Lobar pneumonia and bronchopneumonia. Chest x-ray film showing right upper lobe pneumonia. (From Husain AN. High-Yield Thoracic Pathology. Philadelphia: Elsevier; 2012.)

STEP 1 SECRET Although interpretation of complex chest x-ray films is beyond the purview of the second-year medical school curriculum, you should be able to recognize some common chest x-ray findings such as lobar and interstitial pneumonia, pneumothorax, pleural effusion, and congestive heart failure (CHF)-associated pulmonary congestion. We recommend that you peruse an anatomy atlas or credible online sites to study these images. Step 1 is becoming increasingly clinical, and writers will insert pathologic images and perhaps even some radiographs throughout your test. It may not always be necessary to use the provided images to arrive at the correct answer, but you should not take this gamble. You should also be aware that the USMLE Step 1 occasionally inserts pathologic and radiographic images as answer choices to its questions, so you would be wise to prepare for this possibility.

7. How should this patient be treated pharmacologically?   Although penicillin G has been first-line therapy for community-acquired pneumonia, a rising incidence of penicillin resistance among strains of S. pneumoniae often necessitates the use of an alternative agent such as ceftriaxone. Notice that as a third-generation cephalosporin, ceftriaxone can cover the more common gram-positive and gram-negative organisms that lead to community-acquired pneumonia (e.g., pneumococci and H. influenzae, respectively). 8. Use Table 21.11 to quiz yourself on the most common causes of pneumonia in different age groups.

Table 21.11.   Causes of Pneumonia by Age NEONATES (0–6 WEEKS)

Group B streptococci Escherichia coli

CHILDREN (6 WEEKS–18 YEARS)

Viruses Mycoplasma Chlamydia pneumonia Streptococcus pneumonia

ADULTS (18–40 ADULTS (40–65 YEARS) YEARS)

Mycoplasma C. pneumoniae Streptococcus pneumoniae

S. pneumoniae Haemophilus influenzae Anaerobes Viruses Mycoplasma

ELDERLY (> 65 YEARS)

S. pneumoniae Viruses Anaerobes H. influenzae Gram-positive rods

500  Bacterial Diseases 9. Use Table 21.12 to quiz yourself on the important characteristics of the organisms that are known to cause pneumonia. Table 21.12.   Characteristics of Organisms That Cause Pneumonia Streptococcus pneumoniae Seen in:

Community-acquired pneumonia

Stain

Gram-positive

Morphology

Cocci in pairs

Catalase

Negative

Hemolysis

Alpha

Optochin

Sensitive

Quellung reaction

Positive

Bile solubility

Soluble

Sputum

Rust-colored

Staphylococcus aureus Seen in:

Nosocomial pneumonia

Stain

Gram-positive

Morphology

Cocci in clusters

Catalase

Positive

Coagulase

Positive

Hemolysis

Beta

Klebsiella spp. Seen in:

Alcoholics Diabetics Aspirations

Stain

Gram-negative

Morphology

Rods

Lactose fermentation

Positive

Sputum

Red currant jelly

Pseudomonas aeruginosa Seen in:

Cystic fibrosis, nosocomial infection

Stain

Gram-negative

Morphology

Rod

Lactose fermentation

Negative

Oxidase

Positive

Group B Streptococci Seen in:

Neonates

Stain

Gram-positive

Morphology

Cocci in chains

Catalase

Negative

Hemolysis

Beta

Bacitracin

Resistant

Mycoplasma spp. Seen in:

Atypical pneumonia

Stain

None

Bacterial Diseases  501

Table 21.12.   Characteristics of Organisms That Cause Pneumonia—cont’d Growth medium

Eaton’s agar

Blood test

Cold agglutinins

Escherichia coli Seen in:

Neonates

Stain

Gram-negative

Morphology

Rod

Lactose fermentation

Positive

Chlamydia pneumoniae Seen in:

Atypical pneumonia

Stain

Giemsa

SUMMARY BOX: PNEUMONIA • Presentation • Lobar pneumonia: Sudden onset productive cough, fever, chills, pleuritic chest pain • Rust-colored sputum often accompanies streptococcal pneumonia • Atypical pneumonia: More insidious onset with a classic clinical presentation of headache, nonproductive cough, and low-grade fever • Epidemiology • Lobar pneumonia typically affects older patients and immunocompromised individuals • Atypical pneumonia classically affects young, healthy patients (e.g., adolescents living in dormitory or barracks settings) • Pathophysiology • Lobar pneumonia is most commonly caused by Streptococcus pneumonia • Most common causes of nosocomial pneumonia are Staphylococcus aureus and Pseudomonas aeruginosa • Atypical pneumonia is most commonly caused by Mycoplasma pneumonia • Diagnosis • Crackles on physical exam, leukocytosis, chest x-ray (lobar opacification [lobar pneumonia] or diffuse patchy infiltrates [atypical pneumonia]), decreased O2 saturation, sputum culture • Treatment: Antibiotics appropriate for causative organism

Case 21.2 A 26-year-old woman presents to your office complaining of nausea, vomiting, and severe diarrhea for the past day. She informs you that her bowel movements are watery, but she denies the presence of blood in her stool. She just returned from a week-long trip to Mexico, where she drank only bottled water supplemented with ice from her hotel room. She has no other complaints or problems. Examination is remarkable for tachycardia and dry mucous membranes.   

1. What is the most likely diagnosis?   Traveler’s diarrhea, which is caused by enterotoxigenic E. coli (ETEC). Despite her best efforts to drink only bottled water, she has made a very common mistake among travelers: she used ice made with local water. 2. What other types of diarrhea can be caused by Escherichia coli ?   Enterohemorrhagic E. coli (EHEC) and enteroinvasive E. coli (EIEC) both cause a dysentery-like syndrome with fever and bloody stools, which distinguishes them from the watery stools of ETEC. Enteropathogenic E. coli (EPEC) is a common cause of diarrhea in infants, and enteroadherent E. coli is another cause of traveler’s diarrhea. Use Table 21.13 to review the types of E. coli and the syndromes they cause.

STEP 1 SECRET It is important to remember that the enterohemorrhagic E. coli (EHEC) serotype O157:H7 is strongly associated with hemolytic uremic syndrome (HUS).

502  Bacterial Diseases Table 21.13.   Escherichia Coli Strains STRAIN

SYNDROME

Enterotoxigenic

Traveler’s diarrhea

Enteroadherent

Traveler’s diarrhea

Enteropathogenic

Infantile diarrhea

Enterohemorrhagic

Bloody diarrhea; hemorrhagic colitis and hemolytic uremic syndrome (O157:H7)

Enteroinvasive

Bloody diarrhea (dysentery)

Enteroaggressive

Persistent diarrhea in children and HIV-infected patients

HIV, human immunodeficiency virus.

3. What is the difference between osmotic and secretory diarrhea? Name a cause for each type.   Secretory diarrhea is caused by active secretion of fluids by the intestines. Examples of the causes of this type of diarrhea are V. cholerae (Fig. 21.5) and ETEC, the cause of traveler’s diarrhea. However, these two organisms have different mechanisms of action. V. cholera has an exotoxin that permanently activates Gs receptors, leading to chloride ion efflux and subsequent secretion of water and other ions into the intestinal lumen. ETEC has a heat-labile toxin (LT) that overactivates cyclic adenosine monophosphate (cAMP) (remember “Labile like the Air”) and a heat-stable toxin (ST) that overactivates cyclic guanosine monophosphate (cGMP) (remember “Stable like the Ground”). Both toxins promote excess fluid secretion by intestinal epithelial cells into the intestinal lumen (Table 21.14). Osmotic diarrhea is caused by osmotically active agents within the gut lumen that result in passive movement of water into the intestinal lumen along osmotic gradients. An example in which this may occur is nutritional malabsorption (e.g., in celiac sprue or pancreatic insufficiency), in which the osmotically active nutrients pull water into the intestines. V. cholerae Diarrhea

Cholera toxin B B Ganglioside receptor

B A1 A2

B Na+

B

H2O

Loss of cell nutrients Cl– K+ HCO3–

A2 A1

Increased adenylate cyclase activity

cAMP

Cell membrane

Figure 21.5.  Mechanism of cholera toxin, an A-B type toxin. cAMP, cyclic adenosine monophosphate. (From Rosenthal K. Rapid Review Microbiology and Immunology. 3rd ed. Philadelphia: Elsevier; 2010.)

4. What predisposes patients to Clostridium difficile colitis, and what sort of diarrhea does this cause?   The use of antibiotics such as ampicillin and clindamycin must be carefully monitored to avoid inducing C. difficile colitis, also known as pseudomembranous colitis. Pseudomembranous colitis occurs when a member of the normal intestinal flora (C. difficile) proliferates in excess after elimination of competitor species following broad–spectrum antibiotic use, resulting in superinfection. This bacteria is associated with two exotoxins, referred to as toxins A and B that result in secretory diarrhea and damage the gut mucosa. Toxin A is an enterotoxin that causes diarrhea, while toxin B is a cytotoxin that acts on colonic cells. Colonoscopy generally reveals inflamed mucosal surfaces and presence of pseudomembranes (layers of exudate resembling membranes) following C. difficile infection. Detection of toxin B in the stool can be used to confirm C. difficile infection.

Bacterial Diseases  503

5. How is diarrhea treated?   Generally, supportive therapy to replace lost fluids and electrolytes is all that is needed, and prognosis is typically quite good under these circumstances. For the more serious bugs such as those causing bloody diarrhea, broad-spectrum antibiotics may be helpful, although this runs the risk of inducing C. difficile infection (Tables 21.14 and 21.15).

Table 21.14.   Causes of Watery Diarrhea INFECTIOUS AGENT

COMMENTS

TREATMENT

ETEC

Causes traveler’s diarrhea and is an important cause of diarrhea in children younger than 2 years of age in the developing world; heatlabile toxin acts on adenylate cyclase; heatstable toxin acts on guanylate cyclase

Fluid and electrolyte replacement

Vibrio cholerae

Acts on G protein to stimulate adenylate cyclase, leading to increased Cl− release into lumen of gut; possible “rice water” diarrhea

Fluid and electrolyte replacement

Giardia (protozoan)

Transmitted by cysts in water and diagnosed by trophozoites in stool

Metronidazole

Norwalk virus

A calcivirus

Fluid and electrolyte replacement

Rotavirus

Cause of fatal diarrhea in children and often found Fluid and electrolyte replacement in day care centers

Cryptosporidium (protozoan)

Can be severe in AIDS

None

AIDS, acquired immunodeficiency syndrome; ETEC, enterotoxigenic Escherichia coli.

Table 21.15.   Causes of Bloody Diarrhea INFECTIOUS AGENT

COMMENTS

Shigella

Low inoculum (101); nonmotile; transmitted by 4 Fs TMP-SMX (fingers, food, feces, flies); does not invade beyond gut mucosa

TREATMENT

Salmonella

Higher inoculum (105); motile; transmitted from animal TMP-SMX products, especially poultry and eggs; can become disseminated

EHEC

Shiga-like toxin that can cause hemolytic uremic syndrome (HUS), especially O157:H7

EIEC

Signs/symptoms similar to those of shigellosis; begins Fluid and electrolyte replacement. as watery and can proceed to bloody diarrhea Antibiotics for severe infections.

Campylobacter

“Thermophilic” (optimal growth temperature is 42°C); characteristic comma or S shape; oxidase- and catalase-positive

Usually self-limiting; give fluid and electrolyte replacement

Clostridium difficile

Causes pseudomembranous colitis; can be seen after the administration of clindamycin or ampicillin

Metronidazole or oral vancomycin

Yersinia enterocolitica

Transmitted via pet feces, milk, or pork; causes day care outbreaks with symptoms/signs similar to those of appendicitis, called pseudoappendicitis

Fluid and electrolyte replacement (although antibiotics are indicated if infection is invasive)

Entamoeba histolytica (protozoan)

Transmitted by cysts in water

Metronidazole

Fluid and electrolyte replacement (with glucose)

EHEC, enterohemorrhagic Escherichia coli ; EIEC, enteroinvasive E. coli ; TMP-SMX, trimethoprim-sulfamethoxazole.

504  Bacterial Diseases

SUMMARY BOX: TRAVELER’S DIARRHEA • Presentation: Abrupt onset of nausea, vomiting, and watery diarrhea in a patient with recent travel history • Risk factors: Consumption of contaminated water in endemic areas • Pathophysiology: Mediated by the enterotoxigenic (ETEC) or enteroadherent strains of E. coli • ETEC-associated heat-labile toxin (LT) and heat-stable toxin (ST) promote excess fluid secretion by intestinal epithelial cells into the intestinal lumen • Diagnosis: History, physical exam (dry mucous membranes, tachycardia), stool culture • Treatment: Supportive (replacement of lost fluids and electrolytes) • Prognosis: Good prognosis with appropriate fluid and electrolyte replacement

Case 21.3 A 65-year-old Polish woman with a history of hypertension and rheumatic fever as a child is evaluated for a 2- to 3-week history of night sweats, fever, malaise, and myalgias. Cardiac auscultation reveals a previously undetected faint diastolic murmur. Findings on inspection of the fingers and funduscopic examination are as shown in Figs. 21.6 and 21.7. Echocardiogram and blood culture results are pending.   

1. What is the most likely diagnosis?   This case describes the presentation of acute bacterial endocarditis, an infection of the endothelial lining of the heart (Fig. 21.8).

Figure 21.6.  Finger inspection of patient in Case 21.3. (From Korzeniowski OM, Kaye D. Infective endocarditis. In Braunwald E, ed. Heart Disease. 4th ed. Philadelphia: WB Saunders; 1992.)

Figure 21.7.  Funduscopic examination of patient in Case 21.3. (From Newman NJ. Neuro-Ophthalmology. Philadelphia: Elsevier; 2008.)

Bacterial Diseases  505

A

B

Figure 21.8.  A, Acute rheumatic endocarditis. Gross photograph of an aortic valve with small vegetations (verrucae) along the lines of valve closure. B, Chronic rheumatic endocarditis. Gross photograph of a mitral valve with massive fibrosis and distortion of the leaflets and fusion of the chordae tendineae. (From King T. Elsevier’s Integrated Pathology. Philadelphia: Mosby; 2007.)

STEP 1 SECRET Bacterial endocarditis is a high-yield topic for Step 1. Students will often see images similar to those in Fig. 21.8 on their examinations.

2. What are the major risk factors for developing endocarditis?   The major risk factor for the development of endocarditis is a structurally abnormal heart valve causing aberrant flow streams. Common structural abnormalities are prosthetic valves or native valve lesions, calcifications, rheumatic heart disease, and congenital abnormalities. A majority of infections occur in the left side of the heart (the mitral valve is the most frequently affected valve in bacterial endocarditis, but the aortic valve may also be involved), but with intravenous (IV) drug use, right-sided tricuspid valve lesions may occur as a result of introduction of the pathogens into the venous system. Bacterial species associated with IV drug abuse include S. aureus and P. aeruginosa. Candida albicans is a fungal cause of right-sided endocarditis. 3. What are the clinical signs of bacterial endocarditis?   Bacterial endocarditis commonly presents with low-grade to high fever, new-onset heart murmur, chills, night sweats, weight loss, fatigue, and mild anemia of chronic disease. The timeline of this presentation depends on whether the endocarditis is acute or subacute. Bacterial endocarditis also presents with Roth’s spots (white dots on the retina surrounded by areas of hemorrhage; see Fig. 21.7), Osler nodes (painful, elevated lesions on the pads of the fingers and toes), Janeway lesions (painless, flat discolorations on the palms and soles), and splinter hemorrhages (see Fig. 21.6). Roth’s spots, Osler nodes, Janeway lesions, and splinter hemorrhages are all manifestations of small bacterial emboli (Table 21.16) and can aid in the diagnosis of bacterial endocarditis. Echocardiography and blood cultures are also useful diagnostic tools for this condition.

Table 21.16.   Symptoms and Signs of Bacterial Endocarditis SYMPTOM/SIGN

DESCRIPTION

Fever

Can be spiking

Roth spots

Retinal hemorrhages with pale, white centers composed of fibrin

Osler nodes

Tender, raised lesions of finger or toe pads

Murmur

New or changing due to valvular damage

Janeway lesions

Nontender, erythematous macules on palms or soles

Anemia

Anemia of chronic disease

Nail bed hemorrhages

Often called splinter hemorrhages and can be seen under the nail bed; due to microemboli blocking smaller vessels

Emboli

Can lead to stroke or gangrene of distal extremities

506  Bacterial Diseases 4. What are the clinical signs of rheumatic fever?   Rheumatic fever is an inflammatory sequela of S. pyogenes (group A, β-hemolytic) pharyngitis that is thought to result from cross-reactivity of streptococcal-specific antibodies against the myocardium and joints (type II hypersensitivity reaction; see Chapter 15, Table 15.7). Rheumatic heart disease is a risk factor for subsequent development of bacterial endocarditis as a result of damage inflicted to the heart valves. Acute rheumatic fever most commonly occurs in children but has been seen in adults too. The symptoms of acute rheumatic fever usually occur 2 to 3 weeks following pharyngitis, making prompt treatment of S. pyogenes pharyngitis an important part of rheumatic fever prevention.     Rheumatic fever is diagnosed using the Jones criteria (Table 21.17). The diagnosis of rheumatic fever is made when at least two major criteria or one major criterion plus two minor criteria are met. The five major criteria can be easily remembered using the acronym JONES, which stands for Joints (migratory arthritis), carditis (O is circular like a heart), subcutaneous Nodules, Erythema marginatum, and Sydenham chorea.     Note: Aschoff bodies are the pathognomonic histologic finding in rheumatic heart disease. They are found in the myocardium and consist of regions of fibrinoid necrosis with mononuclear and multinucleated giant cell infiltrates (Fig. 21.9). Also, the antistreptolysin O (ASO) titer is used to detect a recent S. pyogenes infection and should be elevated in cases of acute rheumatic fever. Table 21.17.   Acute Rheumatic Fever SYMPTOM/SIGN

DESCRIPTION

Major Jones Criteria Migratory arthritis

Multiple joint involvement, but each only persists for a short period of time; arthritis is usually the initial manifestation

Carditis

New or changing murmurs may appear; pericardium, epicardium, myocardium, and endocardium are all affected; may see cardiomegaly on radiologic studies

Subcutaneous nodules

Most commonly seen over bony prominences; nonpainful and noninflammatory

Erythema marginatum

A rash similar to that of Lyme disease, in which the erythematous region extends outward as the center becomes pale, forming a ring; most often seen on trunk and not the face; occurs early in the disease and persists throughout its course

Sydenham chorea

“St. Vitus dance”—sudden, nonrhythmic, purposeless movement; may be associated with muscle weakness and behavioral changes

Minor Jones Criteria Fever Arthralgias Previous episode of rheumatic fever Elevated inflammatory markers (ESR/CRP) Prolonged PR interval on ECG Leukocytosis CRP, C-reactive protein; ECG, electrocardiogram; ESR, erythrocyte sedimentation rate.

Figure 21.9.  Microscopic appearance of an Aschoff body in a patient with acute rheumatic carditis; there is central necrosis with a circumscribed collection of mononuclear inflammatory cells, some of which are activated macrophages (Anitschkow cells) with prominent nucleoli (arrowheads). (From Kumar V, Cotran R, Robbins S. Robbins Basic Pathology. 8th ed. Philadelphia: WB Saunders; 2008.)

Bacterial Diseases  507

STEP 1 SECRET Rheumatic fever is a high-yield diagnosis for Step 1. A clinical vignette featuring rheumatic fever in childhood will mention a recent streptococcal infection (classically strep throat). Clinical vignettes highlighting the sequelae of rheumatic fever in adulthood may mention that the patient was from a resource poor area (classically Eastern Europe). You should also be able to identify Fig. 21.9 as an Aschoff body, a pathognomonic finding in this disease.

5. Which bacteria are most commonly associated with bacterial endocarditis?   Endocarditis can be classified into acute or subacute types depending on the time course. Acute infections occur within days to weeks, and patients are extremely sick during this time; they are most often due to Streptococcus or Staphylococcus. Subacute infections present with milder symptoms and are characterized by a consistently low-grade illness for 3 to 4 weeks; they are frequently caused by Streptococcus viridans and group D streptococci such as Streptococcus bovis (Table 21.18).     Note: Bacterial endocarditis that occurs soon after prosthetic valvular surgery is commonly due to Staphylococcus epidermidis and is believed to result from intraoperative contamination. Table 21.18.   Acute Versus Subacute Endocarditis CHARACTERISTIC

ACUTE

SUBACUTE

Organisms

Staphylococcus aureus

Streptococcus viridans (S. sanguis) after dental procedures

Onset

Rapid (days to weeks)

Insidious (3–4 weeks)

Clinical manifestations

Severe sickness

Mild sickness

Vegetation size

Large

Smaller

Types of valves affected

Previously normal valves

Damaged or congenitally abnormal valves

6. What drugs could be used to treat this patient?   Because acute endocarditis can be caused by Streptococcus species as well as S. aureus, the drug chosen will need to cover both of these organisms. If there is no suspicion of MRSA, then either oral dicloxacillin or intravenous nafcillin would work on both, but if there is suspicion of MRSA, then intravenous vancomycin would be the drug of choice. In some cases, surgical intervention may also be necessary. 7. How does bacterial endocarditis differ from Libman-Sacks endocarditis?   Libman-Sacks (LS) endocarditis, which is seen in systemic lupus erythematosus (SLE), is an aseptic inflammation of the heart valves. The vegetations typically involve both sides of the valve, whereas the vegetations occur primarily on the “downstream” side of the valve in bacterial endocarditis. Finally, the vegetations in LS endocarditis will not embolize.

SUMMARY BOX: ENDOCARDITIS • Presentation: Fever, chills, night sweats, weight loss, and fatigue • Risk factors: Structurally abnormal heart valves, history of rheumatic fever, intravenous drug use (affects right-sided heart valves) • Pathophysiology • Causes: Staphylococcus aureus is the most common cause of acute bacterial endocarditis. Streptococcus viridans is an important cause of subacute endocarditis that should be considered in a patient who has undergone a recent dental procedure. • Complications: Significant heart damage, anemia of chronic disease • Diagnosis: Physical exam (new-onset heart murmur, Roth spots, Osler nodes, Janeway lesions, splinter hemorrhages), positive blood cultures, echocardiography • Treatment: Antibiotic therapy and potential surgery

Case 21.4 A 20-year-old man is evaluated for a new genital lesion. The patient returned from spring break last week and noticed a painless ulcer on his scrotum. He is quite concerned and admits to several instances of unprotected intercourse. On examination, there is a well-demarcated, 2-cm painless lesion with a raised border on the shaft of the penis (Fig. 21.10). The remainder of the examination is unremarkable.   

508  Bacterial Diseases

Figure 21.10.  Physical examination of patient in Case 21.4. (From Habif TP. Clinical Dermatology. 6th ed. Philadelphia: Elsevier; 2016, Fig. 10-9.)

1. What is the likely diagnosis?   Syphilis resulting from Treponema pallidum infection. The organism enters the body through broken epithelium or direct mucosal contact. The classic syphilitic chancre is painless and has a clean, nonpurulent base with a sharply defined border, as shown in Fig. 21.10. 2. Based on this man’s presentation, in which “stage” of syphilitic infection is he most likely to be?   Syphilis progresses through three stages: primary, secondary, and tertiary. This patient displays the classic painless genital chancre of primary syphilis, which appears 3 to 6 weeks after contact. This lesion is highly infectious and continuously sheds motile spirochetes. The primary stage will last 4 to 6 weeks and then resolve, often fooling patients that they are cured. 3. What stage of syphilis would you suspect in a patient with a diffuse maculopapular rash?   This presentation is classic for secondary syphilis. The secondary stage of syphilis will begin approximately 6 weeks after the primary chancre has healed. This phase is characterized by a generalized maculopapular rash, often involving the palms and soles, with or without the fleshy, painless genital warts termed condylomata lata. The secondary stage of syphilis resolves in 6 weeks and enters the latent phase. If the infection is not treated, it will progress to tertiary syphilis in approximately one-third of these patients.

STEP 1 SECRET You should know which bacteria and viruses cause genital lesions and whether these lesions are painful or painless. Remember that the two bugs associated with painful genital lesions are herpes simplex virus 2 (HSV-2) (genital herpes) and Haemophilus ducreyi. An easy way to keep this in mind is to remember that “those with genital herpes do cry (ducreyi) in pain.” By contrast, infections associated with syphilis, gonorrhea, chlamydia, lymphogranuloma venereum, human papillomavirus (HPV), trichomoniasis, and bacterial vaginosis are painless.

STEP 1 SECRET When distinguishing the likely cause of a rash, involvement of the palms and soles is an important clue. Very few rashes involve these areas, and include syphilis, Rocky Mountain spotted fever (Rickettsia ricketssii), hand-foot-and-mouth disease (coxsackievirus A), and the noninfectious Kawasaki disease.

Bacterial Diseases  509

Case 21.4 continued: This patient does not seek treatment and presents to your office 10 years later with a regurgitant murmur heard best over the right second intercostal space and an ataxic gait.   

4. What is the likely diagnosis?   This patient is presenting with symptoms of tertiary syphilis. This stage can develop anywhere from 5 to 35 years after the initial infection. Tertiary syphilis is a systemic disease with three major components: granulomatous change (gummas), cardiovascular syphilis, and neurosyphilis. Inflammatory destruction is the pathophysiologic mechanism that is inherent to all three components. Know that cardiovascular syphilis may result in aortic valve insufficiency and thoracic aortic aneurysm (caused by involvement of the vasa vasorum), and neurosyphilis can cause tabes dorsalis, a condition that causes dorsal column disease of the spinal cord and subsequent ataxia. Another common association is ArgyllRobertson or “prostitute’s” pupil, which is a pupil that accommodates but does not react. 5. Use Table 21.19 to quiz yourself on the three stages of syphilitic infection. Table 21.19.   Stages of Syphilis PARAMETER PRIMARY

SECONDARY

TERTIARY

CONGENITAL

Timing

Three weeks of incuba- Weeks to months after tion followed by emergence of papule emergence of papule

One to 30 years after Transmitted to fetus primary infection (because of latent period between secondary and tertiary)

Characteristic symptoms/ signs

Painless papule on genitals

Disseminated disease with Gummas (granulomas), Stillbirth, “saber constitutional sympaortitis, tabes dorshins,” saddle-nose toms; possible rash salis (neurosyphilis deformity, deafness that can involve palms of dorsal columns), and soles; condylomata Argyll Robertson lata are white lesions pupil (constriction on genitals; most infecto accommodation tious stage but not to light)

Treatment

Penicillin G

Penicillin G

None

Symptom-dependent

6. What diagnostic tests could be done to definitively diagnose syphilis in this man?   Direct visualization by dark-field microscopy can be done during the active phases of stage 1 and stage 2 syphilis. This is conducted by obtaining a sample from the lesion and observing the motile spirochetes. Serologic tests were also developed to satisfy the need for a syphilis screen. The Venereal Disease Research Laboratory (VDRL) and the rapid plasma reagin (RPR) tests were developed to detect antibodies present against certain components released after cell death. These tests are nonspecific treponemal tests and, if positive, require a more specific measure, the fluorescent treponemal antibody absorption (FTA-ABS) test. The key point is that the VDRL and the RPR tests are effective for screening high-risk patients. The VDRL test is easier and less expensive, so it is usually done first. However, it can have false-positive results because it cross-reacts in the presence of various Viruses, Drugs, Rheumatologic diseases, and Lupus or Leprosy. These can be easily remembered because they start with the letters V, D, R, and L. The VDRL test will become positive in late primary syphilis and becomes negative again in late secondary syphilis. In addition to being more specific, the FTA-ABS test also becomes positive earlier and stays positive longer. Therefore, the FTA-ABS test can be used to diagnose tertiary syphilis and to confirm a positive screening VDRL test (Table 21.20). Remember: the VDRL test is extremely sensitive and is used for screening, while the FTA-ABS test is very specific and used for confirmation of the diagnosis. Table 21.20.   Syphilis Tests TEST

USE

Dark-field microscopy

Test of choice when a chancre is present and a biopsy of the lesion can be taken for direct observation

VDRL

First test used when secondary syphilis is suspected; may need to be confirmed by FTA-ABS testing because of high number of false positives

FTA-ABS

Test of choice for tertiary syphilis; used to confirm a positive result on VDRL test

FTA-ABS, fluorescent treponemal antibody absorption; VDRL, Venereal Disease Research Laboratory.

510  Bacterial Diseases 7. How would you treat this patient?   Fortunately, syphilis is one of the easiest diseases to treat. Administer penicillin G, and if the patient is penicillin-allergic, offer tetracycline or doxycycline. It is important to remember that only primary and secondary syphilis can be cured with medication. Antibiotics do nothing for tertiary syphilis. 8. Later that night, the patient calls you at home with serious concerns about a reaction to penicillin. He states that several hours after being treated he developed a new rash, along with fever, headache, and muscle aches. What are you concerned about in this patient?   This patient has likely suffered from a common reaction to the penicillin treatment of syphilis known as the Jarisch-Herxheimer reaction. This side effect of treatment is due to the immune system’s reaction to the lysis of treponemes. When exposed to the tremendous load of foreign antigens, the body releases IL-1 and TNF-α, causing fever and possibly shock. This entity should not be confused with an allergy to penicillin and requires only treatment of symptoms and close monitoring.

SUMMARY BOX: SYPHILIS • Presentation • Primary: Painless genital chancre in an individual with a history of unprotected sexual intercourse • Secondary: Diffuse maculopapular rash and condylomata lata • Pathophysiology: Results from Treponema pallidum infection through direct mucosal contact • Diagnosis: The Venereal Disease Research Laboratory (VDRL) assay followed by confirmatory fluorescent treponemal antibody absorption (FTA-ABS) test, direct visualization by dark-field microscopy • Complications: Progression to tertiary syphilis if untreated (gummas, aortic valve insufficiency, thoracic aortic aneurysm, tabes dorsalis, Argyll Robertson pupil) • Treatment: Intravenous penicillin G • Jarisch-Herxheimer reaction may occur with treatment as a result of widespread treponemal lysis. • Prognosis: Tertiary syphilis is not curable with antibiotics.

Case 21.5 A frantic mother has brought her 8-year-old son in for an emergent visit. She is concerned about an enlarging rash located on the child’s back where she had found an attached tick. She adds that he has been complaining of a flu-like illness since the family’s return from a hiking trip in New England. On examination, you appreciate a large, well-demarcated 20-cm erythematous rash with central clearing (Fig. 21.11) and some regional adenopathy.   

Figure 21.11.  Lesion from patient in Case 21.5. Note the variation in color and target-like appearance of the lesion. The bite site is visible in the center.

1. What is the most likely diagnosis?   Lyme disease, which is caused by the spirochete Borrelia burgdorferi, is the most likely diagnosis. This bug is transmitted from the bite of an Ixodes tick, endemic to the woodlands of New England. The image in Fig. 21.11 shows an expanding erythematous lesion known as erythema chronicum migrans (“bull’s-eye” rash).     Note: The Ixodes tick is also the vector for Borrelia burgdorferi (Lyme disease), Babesia (babesiosis), and Anaplasma phagocytophilum (granulocytic ehrlichiosis), and co-infection is possible. Other arthropod vectors include the dog

Bacterial Diseases  511

tick (Dermacentor variabilis) and Rocky Mountain wood tick (Dermacentor andersoni ), which carry Rocky Mountain spotted fever, as well as the lone star tick (Amblyomma americanum), which carries Ehrlichia chaffeensis (human monocytic ehrlichiosis).

STEP 1 SECRET The USMLE commonly asks students about vectors for various bacterial and parasitic infections. Co-infection with Lyme disease and a parasitic infection such as babesiosis or anaplasmosis is a particular favorite because both bugs share the same vector, the Ixodes tick.

2. What stage of Lyme disease would you suspect in this child?   Our patient has manifestations consistent with stage 1 or “early localized” Lyme disease. Lyme disease is similar to syphilis in that both illnesses are caused by the dissemination of an infectious spirochete and progress through three stages: an early localized stage, an early disseminated stage, and a late stage (stages 1, 2, and 3, respectively). This patient is in stage 1, which consists of the expanding erythematous lesion known as erythema chronicum migrans. A flu-like syndrome and regional adenopathy often accompany the rash of stage 1 Lyme disease and can clue one into the diagnosis. Diagnosis of Lyme disease is typically performed via enzyme-linked immunosorbent assay (ELISA), which can detect antibodies against B. burgdorferi. Western blot is used to confirm the diagnosis if the ELISA test is positive. 3. How would your diagnosis change if this patient presented with a similar history but had complaints of various painful swollen joints and a diffuse macular rash all over his body?   He would then mostly likely be suffering from stage 2 or early disseminated Lyme disease. This stage is characterized by the spread of B. burgdorferi to four components of the body: joints, heart, nervous tissue, and skin. Migratory musculoskeletal pains occur and usually affect the large joints such as the knee. These joints become swollen and tender. Cardiac complications can vary, ranging from conduction block to myocarditis, and neural issues range from viral meningitis to nerve palsies, most classically a bilateral Bell’s palsy. The skin lesions of stage 2 Lyme disease are similar to stage 1 rashes but are smaller and more widely distributed over the body surface. A summary of the symptoms of stage 2 Lyme disease can be remembered by the acronym CANE (Cardiac block, Arthritis, Neural issues, Erythema migrans). 4. If this patient does not receive appropriate treatment, what is the likelihood that the infection will progress to stage 3 Lyme disease?   The late stage of Lyme disease (stage 3) occurs in only 10% of untreated patients and is characterized by the development of a chronic arthritis, which involves multiple large joints and a progressive central nervous system (CNS) disease (Table 21.21). Table 21.21.   Stages of Lyme Disease STAGE

CHARACTERISTIC SYMPTOMS

Early local (stage 1)

Erythema chronicum migrans; flu-like symptoms; occurs within 1 month of tick bite

Early systemic (stage 2)

Monoarticular or oligoarticular arthritis, Bell’s palsy or other cranial nerve palsy, and atrioventricular conduction blocks; can occur days to months after tick bite

Late (stage 3)

Migratory polyarthritis and neurologic symptoms; occurs months to years after initial infection

5. What is the treatment for Lyme disease? Name a preventive measure that can be taken to protect against development of Lyme disease.   Lyme disease is effectively treated with doxycycline. Later stages of Lyme disease should be treated with ceftriaxone. Recently, an effective vaccine has been developed and is routinely administered to pets in Lyme-endemic areas. The vaccine for humans is no longer available.     Note: Lyme disease is most commonly transmitted during the summer, so the vaccine should be given in the spring. 6. Describe the Ixodes life cycle.   Remember, the Ixodes tick is only the vector for the infectious spirochete B. burgdorferi. The Ixodes life cycle extends over 2 years. Eggs are laid in the spring and will develop into larvae that feed in the summer, preferably on mice. The mice act as the reservoir for B. burgdorferi, and it is here that Ixodes acquires the spirochete that it can later transmit. Ixodes is dormant in the fall and winter and will become a nymph in the following spring. It will feed on a mouse or a human (but note that the human is not necessary for the life cycle of the tick), and Borrelia can be transmitted at this time. After feeding, the tick becomes an adult and will mate, often on a deer (Fig. 21.12). Note that ticks typically require a minimum 24-hour attachment period to transmit Lyme disease, although other diseases can be transmitted more quickly.

512  Bacterial Diseases

ll Fa

r inte dw n a

Sp r in

g

Eggs laid Adults mate Larva develops in 1 month ?

Ye

Larva feeds Summer

a

r1

Nymph molts to adult tick

Nymph feeds

Ye

Summer

/

ar

2

Nymph Larva dormant

Sp rin g

l an Fal

er int w d

Figure 21.12.  Life cycle of Ixodes scapularis (also known as Ixodes dammini). (Adapted from an illustration by Nancy Lou Makris in Rahn DW, Malawista SE. Lyme disease. West J Med 1991;154(6):708.)

SUMMARY BOX: LYME DISEASE • Presentation: Appearance of a bull’s-eye rash and development of flu-like symptoms that occur shortly after a tick bite • Risk factors: Increased prevalence in hikers, particularly those located in New England • Pathophysiology: Caused by Borrelia burgdorferi, a spirochete that infects the Ixodes tick • Diagnosis: History and physical exam (rash, regional adenopathy), enzyme-linked immunosorbent assay (ELISA), and confirmatory Western blot • Complications: Progression to stage 2 or stage 3 Lyme disease, co-infection with other bugs that infect the Ixodes tick • Stage 2 Lyme disease: CANE symptoms (cardiac block, articular disease, neural issues [Bell’s palsy], erythema migrans) • Stage 3 Lyme disease: Migratory arthritis and neurologic symptoms • Treatment: Doxycycline or ceftriaxone

Case 21.6 While you are in Pakistan on a medical mission, a patient presents with an 8-week history of fever, night sweats, and a productive cough, at times tinged with blood (hemoptysis). He has lost 20 lb during this time and has been generally fatigued and weak. A chest x-ray film reveals a pulmonary infiltrate, and a purified protein derivative (PPD) skin test is positive. The patient reports that the same test was negative a year ago. A sputum stain for acid-fast bacilli is positive.   

1. What is the presumptive diagnosis?   Tuberculosis (TB), which is caused by Mycobacterium tuberculosis. Note that this is a presumptive rather than a definitive diagnosis and needs to be confirmed with DNA testing because several other occasionally pathogenic mycobacteria such as Mycobacterium avium-intracellulare (MAC) can produce a similar clinical presentation and positive acid-fast stain result. Epidemiologic clues may help you. Tuberculosis often occurs in immigrants from developing nations or in persons who have spent significant time in prisons or shelters. MAC commonly occurs in elderly women who are otherwise healthy and in HIV patients who are severely immunocompromised.

Bacterial Diseases  513

2. How is this disease primarily transmitted?   TB is primarily transmitted through aerosolization of contaminated respiratory secretions (e.g., coughing). 3. Why is the acid-fast stain required to visualize this bacterium?   Mycobacterium species, such as tuberculosis and MAC, do not stain well with the Gram stain because they contain mycolic acids in their cell wall rather than peptidoglycan. However, they do stain well with the acid-fast stain, which is why mycobacterium is referred to as an acid-fast bacterium.     Note: Acid-fast bacteria are visualized with Ziehl-Neelsen stain, and M. tuberculosis is grown on LowensteinJensen agar. 4. Does this patient most likely have primary tuberculosis, latent tuberculosis, or recrudescent (secondary) tuberculosis?   Because his previous PPD test was negative, he most likely has primary TB, which results from initial infection with the organism. More specifically, he probably has a “progressive” primary infection, in which symptoms manifest. This latter distinction is made because most patients who become infected with the mycobacterium do not develop symptoms. Latent TB develops after symptoms have resolved from primary TB (if there were any symptoms) and is due to tubercle bacilli residing in macrophages. Recrudescent TB develops after some form of immunologic compromise that allows the latent tubercle bacilli to begin proliferating again (Fig. 21.13).     Note: About 10% of patients infected with TB in the United States will eventually have a recrudescence. Miliary TB occurs when the bacilli are transmitted and cause foci of infection throughout the body.     A Ghon complex refers to a region of the lung and associated perihilar lymph nodes that have been exposed to TB and have become granulomatous. A Ghon complex indicates that there has either been an exposure to TB that the body was able to resolve immunologically or that there is a current primary infection.

Inhalation of M. tuberculosis

Uptake into alveolar macrophages Primary TB

Escape Local spread TH1 response

Healed lesion

Fibrosis

Granuloma (caseous)

Immunity/hypersensitivity (tuberculin-positive) Into lung Cough (contagion) and pulmonary disease

Reactivation later in life

Secondary TB

Liquefaction and release of bacilli Into blood

Miliary TB Renal GI

CNS Bone

Figure 21.13.  Pathogenesis and clinical course of tuberculosis (TB) caused by Mycobacterium tuberculosis. CNS, central nervous system; GI, gastrointestinal. (From Rosenthal K, Tan J. Rapid Review Microbiology and Immunology. 2nd ed. Philadelphia: Mosby; 2007.)

5. What are the first-line drugs for treating tuberculosis, and why are they always used in combination?   These drugs include Rifampin, Isoniazid (also used for prophylaxis), Pyrazinamide, Ethambutol, and Streptomycin (just remember the mnemonic “RIPES”). They are used in combination because there is a high incidence of resistance to these drugs. In the United States, in fact, about 10% to 15% of isolates have resistance to one these drugs even before treatment is started. 6. If this patient is treated with isoniazid as part of his regimen, why should he also receive supplemental pyridoxine (vitamin B6)?   One of the main side effects of isoniazid is a peripheral neuropathy, which is caused by the drug stimulating pyridoxine excretion and creating a relative pyridoxine deficiency. Remember that one of the features of pyridoxine deficiency is peripheral neuropathy.

514  Bacterial Diseases     Note: Isoniazid is well known for its hepatotoxicity. It can even cause a full-blown hepatitis with nausea, vomiting, jaundice, and right upper quadrant pain. Isoniazid is also known to cause a lupus-like syndrome and can lead to hemolysis in glucose-6-phosphate dehydrogenase deficiency. It is an inhibitor of the P-450 system. 7. If this patient is treated with rifampin as part of his regimen, why may he need larger doses of opioid analgesics for pain control in other illnesses/injuries?   Rifampin induces hepatic P-450 enzymes, including those that metabolize opioids. 8. Three weeks after starting a therapeutic regimen with rifampin and isoniazid the patient complains of orange urine. What is probably causing this?   This is a well-known and common side effect of rifampin. Rifampin also often turns sweat, tears, and contact lenses an orange color. 9. If this patient begins complaining of vision problems, what would you suspect is the cause?   Ethambutol. This drug has a side effect of optic neuropathy (decreased visual acuity and red-green color blindness). 0. Why is the standard treatment regimen that this patient will be put on so prolonged? 1   Several characteristics of the tubercle bacillus make it difficult to control quickly. One problem is its intracellular location, where drugs do not penetrate well. In addition, the bacillus is often found in large cavities with avascular centers, into which drugs do not penetrate well either. Finally, the tubercle bacillus has a very slow generation time. 

RELATED QUESTIONS 1. Is cell-mediated immunity or humoral immunity more important for fighting tuberculosis? Why? 1   Because the tubercle bacillus resides intracellularly in macrophages, cell-mediated immunity is more important because it can better target intracellular pathogens. 2. How does the purified protein derivative skin (Mantoux) test work? 1   PPD is made from the bacterial cell wall of M. tuberculosis. When injected into an individual whose immune system has been exposed to the tubercle bacilli, the PPD elicits a type IV hypersensitivity response, which manifests as an indurated area at the site of injection within about 48 hours.     Certain patient groups may have false results with this test. Patients with a compromised immune system (e.g., HIV-infected patients) may not mount the appropriate immune response and could thus have a falsely negative PPD. On the other hand, patients who have been previously infected with TB or who have previously received the BCG vaccine (used abroad in many nations) will always have a positive result despite lack of infection. 13. Why is reactivation tuberculosis more likely to occur in the apical lungs rather than in the lower lobes?   Because mycobacteria are obligate aerobes, the higher oxygen tension in the apex of the lung facilitates their growth there. However, primary infections are more likely to occur in the lower segments where the bacteria are initially deposited. 4. What type of necrosis is associated with granulomatous cell death in tuberculosis? 1   Caseous necrosis, which has a cheesy white appearance. For boards, other types of necrosis include liquefactive (e.g., stroke), coagulative (e.g., myocardial infarction [MI]), fat (e.g., pancreatitis), and gangrenous (e.g., bacterial nfection) necrosis.     Note: TB is the only granulomatous disease associated with caseous necrosis. Other granulomatous diseases (e.g., syphilis, cat scratch fever, leprosy, Crohn’s disease, chronic granulomatous disease [CGD], Wegener granulomatosis, berylliosis, sarcoidosis, systemic fungal infections, Listeria infection, and foreign bodies) are noncaseating. 15. What type of secondary infection can be seen in pulmonary cavitations such as those associated with tuberculosis?   Aspergillus, which can colonize in previously formed lung cavities, is an associated secondary infection. These colonies are often called aspergillomas, aspergillus balls, or fungus balls. 16. How can tuberculosis cause a urinalysis to show microscopic pyuria and hematuria (with red blood cell casts) in the face of a “sterile” culture (“sterile pyuria”)?   Hematogenous spread of TB to the kidneys can cause pyelonephritis. TB is notoriously difficult to culture, and the urine is not cultured routinely unless specifically requested. 17. Why might Pott’s disease be suspected in a patient with tuberculosis who has new-onset back pain but denies any trauma that might explain the pain?   Hematogenous spread of TB to the spine can lead to vertebral osteomyelitis, referred to as Pott’s disease.

Bacterial Diseases  515

SUMMARY BOX: TUBERCULOSIS (TB) • Presentation: Fever, weight loss, night sweats, hemoptysis • Epidemiology: Often occurs in immigrants from developing nations, individuals who have traveled to endemic areas, or persons who have spent significant time in prisons or shelters • Pathophysiology: • Mycobacterium tuberculosis is inhaled and resides inside alveolar macrophages (primary infection), stimulating granuloma formation • If infection is reactivated later in life secondary to immunologic compromise, this can stimulate development of caseous necrosis, pulmonary symptoms, and signs of disseminated infection • Diagnosis: Positive purified protein derivative (PPD) or acid-fast stain, chest x-ray findings (Ghon complex), diagnosis confirmed via DNA testing • Complications • Reactivation of TB leads to apical lung disease • Hematogenous spread can cause Pott’s disease and pyelonephritis • Aspergillus can infect old tubercles, leading to aspergillomas • Treatment: Drug combination (Rifampin, Isoniazid, Pyrazinamide, Ethambutol, Streptomycin [RIPES]) to prevent antibiotic resistance

Case 21.7 A 21-year-old woman presents to your clinic because of abdominal discomfort, which she describes as becoming increasingly severe over the past week. She has also noticed a yellow, malodorous vaginal discharge as well as occasional vaginal bleeding following sex. It is becoming more uncomfortable for her to urinate, but there has been no change in urgency or frequency. She has had three sexual partners over the last 3 months and uses an intrauterine device (IUD) as contraception, which was most recently changed 2 weeks ago. On speculum exam, she has purulent drainage from the cervical os, and on bimanual exam, she has significant cervical motion tenderness.   

1. What is the most likely diagnosis?   Pelvic inflammatory disease (PID), which is most often caused by Chlamydia trachomatis or Neisseria gonorrhoeae. 2. How are C. trachomatis and N. gonorrhoeae transmitted?   By contact with infected genitals, most commonly via sexual contact or at birth. 3. How is pelvic inflammatory disease transmitted, and why can it lead to pelvic discomfort, vaginal discharge, and vaginal bleeding?   PID is the result of a cervical or vaginal infection that ascends the female reproductive tract to cause endometritis or salpingitis. The inflammation of the uterine lining or the fallopian tubes leads to the pelvic discomfort. The original infection of the lower reproductive tract and the resulting inflammatory response can result in discharge. The infected epithelium is more likely to bleed with even mild contact.

Case 21.7 continued: The laboratory results report the presence of cytoplasmic inclusions but no gram-negative diplococci.   

4. What is the definitive diagnosis?   C. trachomatis infection. A Gram stain is helpful to differentiate between C. trachomatis and N. gonorrhea because the former is an obligate intracellular organism, whereas N. gonorrhoeae is a gram-negative diplococcus (Fig. 21.14A and B).     Chlamydiae are obligate intracellular organisms specifically because they cannot make their own adenosine triphosphate (ATP). Therefore, when stained with Giemsa, they will be seen in the cytoplasm of the infected cell. Rickettsia is another example of an obligate intracellular organism. 5. What should be prescribed as a treatment for your patient?   Chlamydial infections respond best to antibiotics that target the 30S ribosomal subunit and work best against intracellular organisms (e.g., macrolides and tetracyclines). Azithromycin is commonly used to treat C. trachomatis infections. Note that the chlamydial peptidoglycan lacks muramic acid. This renders β-lactam antibiotics useless against Chlamydia. Because patients with chlamydial infection are at risk for simultaneous gonorrheal infection, you should also treat them with ceftriaxone.

516  Bacterial Diseases

A

B

Figure 21.14.  A, Neisseria gonorrhoeae. Gram stain of urethral exudate in gonorrhea, showing intracellular gram-negative reniform diplococci. (From Hochberg M, Silman, A, Smolen J, et al. Rheumatology. 6th ed. Philadelphia: Hochberg; 2015.) B, Gram stain of Chlamydia trachomatis. (Courtesy Centers for Disease Control and Prevention; from Bailey. Colorectal Surgery, Phildadelphia: Elsevier; 2012.)

6. If the patient’s current and past partners do not have any symptoms, should they also be considered for treatment?   Yes. Anyone who has had sexual contact with the patient in the 60 days leading up to her symptoms should also be treated. C. trachomatis genital infections are often asymptomatic and are an important reservoir for continuing the infectious cycle. Despite the fact that they can be asymptomatic, chlamydial infections can still lead to sterility in women, most often because of the inflammatory effects on the fallopian tubes (salpingitis). 7. Why is the fact that the patient was using an intrauterine device significant in this case?   An IUD may help the infection to ascend from the lower reproductive tract into the endometrium of the uterus when the IUD is inserted. This was particularly a problem with older models of IUDs, which had a braided string that allowed easy ascension of bacteria into the upper reproductive tract. Current IUDs have a straight string to prevent this problem. 8. What are other risk factors for the development of pelvic inflammatory disease?   Any act that may help the passage of an infection from the lower genital tract into the upper genital tract, including douching, aborting a pregnancy, and parturition, is a risk factor. 9. If this patient was not using any birth control and had been trying to become pregnant, what other concerns would you need to take into account?   PID increases the risk of ectopic pregnancy and can also lead to infertility due to scarring of the fallopian tubes (a sequela of the inflammatory response). 0. What is Reiter syndrome? 1   This autoimmune disease is caused when the antibodies formed against C. trachomatis react against antigens on the urethra, joints, and uveal tract. This results in the classic triad of uveitis, urethritis, and arthritis. (“Can’t see, can’t pee, can’t climb a tree.”) Remember that while Reiter syndrome is commonly associated with chlamydial infection, it can occur with other bacterial infections (e.g., Salmonella, Shigella, Campylobacter, and N. gonorrhoeae). It is also an HLAB27 linked condition. 1. Describe the unique life cycle of a chlamydial infection. 1   Infection begins when an elementary body attaches to and enters an epithelial cell. The elementary body will then transform into a reticulate body, which will divide many times by binary fission. The many reticulate bodies will then be organized into elementary bodies, and it is at this point that the cytoplasmic inclusion bodies may be seen microscopically. The elementary bodies will be released from the cell, and each is then capable of infecting another epithelial cell (Fig. 21.15). 2. What are the serotypes of C. trachomatis that can cause pelvic inflammatory disease? 1   PID is caused by serotypes D through K. See Table 21.22 for diseases caused by other C. trachomatis serotypes. 3. What are the other species of Chlamydia, and what diseases do they cause? 1   Table 21.23 presents the diseases, modes of transmission, and recommended treatments for Chlamydia species.

Bacterial Diseases  517 Elementary body

Attachment phagocytosis

Inclusion Replicating reticulate bodies

Figure 21.15.  Life cycle of Chlamydia spp. (From Cohen J, Powderly WG, Berkley SF, et al. Infectious Diseases. 3rd ed. Philadelphia: Saunders; 2010.)

Table 21.22.   Serotypes of Chlamydia Trachomatis SEROTYPES

DISEASE

A, B, C

Blindness in Africa due to chronic infections

D–K

Pelvic inflammatory disease, neonatal pneumonia, neonatal conjunctivitis

L1, L2, L3

Lymphogranuloma venereum

Table 21.23.   Chlamydial Species and Associated Diseases SPECIES

DISEASE

TRANSMISSION

TREATMENT

Chlamydia trachomatis

Reactive arthritis, nongonococcal urethritis (NGU), conjunctivitis, blindness, lymphogranuloma venereum

Sexual or passage through birth canal

Tetracycline Azithromycin Erythromycin eye drops for neonatal conjunctivitis

Chlamydia pneumoniae

Atypical pneumonia

Aerosol

Tetracycline or erythromycin

Chlamydia psittaci

Atypical pneumonia with avian reservoir

Aerosol

Tetracycline or erythromycin

SUMMARY BOX: CHLAMYDIA • Presentation: Abdominal discomfort, malodorous vaginal discharge, vaginal bleeding, discomfort while urinating • Epidemiology: Suspect in sexually active patients who do not use protection • Diagnosis: History, physical exam (cervical motion tenderness, drainage from cervical os), Gram stain to rule out gonorrhea, Giemsa stain • Pathophysiology • Obligate intracellular bug • Two phases of the chlamydial life cycle: 1. Elementary bodies infect new cells 2. Reticulate bodies divide within cells • Serotypes L1 to L3 cause lymphogranuloma venereum (LGV), serotypes A to C cause blindness, and serotypes D to K cause pelvic inflammatory disease (PID) • Complications: PID, infertility, ectopic pregnancy, Reiter syndrome • Treatment: Azithromycin and ceftriaxone (for potential gonorrheal co-infection) • All sexual contacts within past 60 days should be treated as well.

518  Bacterial Diseases

Case 21.8 A 20-year-old university music major is brought to your clinic by one of his roommates, who reports that the patient was complaining of a headache last night and was confused when he was awakened this morning. Upon questioning, the patient knew his name but thought that he was in a different city and that the year was 2008. He reports having a severe headache and asks for the lights to be turned down in the office. His roommate says that he has no history of migraines and that they have known each other for the last 3 years. His temperature is taken and shown to be elevated at 38.7°C. On examination, he has positive Brudzinski and Kernig signs.   

1. What is the most likely diagnosis?   Meningitis is the most likely diagnosis. 2. What is the “classic triad” of symptoms associated with meningitis?   Fever, nuchal rigidity, and altered mental status (confusion). However, only about one-third of patients with meningitis will present with all three of these symptoms. Photophobia and headache can commonly be seen in meningitis but are not considered part of the triad. 3. What are the most common causes of meningitis by age group? Use Table 21.24 to quiz yourself. Table 21.24.   Causes of Meningitis by Age 0–2 YEARS

Escherichia coli Group B streptococci Listeria monocytogenes

2–18 YEARS

18–60 YEARS

Neisseria meningitidis Streptococcus pneumoniae Haemophilus influenzae

N. meningitidis S. pneumoniae H. influenzae L. monocytogenes

60+ YEARS

S. pneumoniae L. monocytogenes N. meningitidis Group B streptococci H. influenzae

Case 21.8 continued: Lumbar puncture is performed on the patient to obtain cerebrospinal fluid (CSF; see Chapter 26, Case 26.4, Part A, for more details). A Gram stain of the CSF is provided in Fig. 21.16.   

Figure 21.16.  Gram stain of a cerebrospinal fluid sample that grew Listeria monocytogenes. The short bacilli are seen inside white blood cells. (From Murray P, Rosenthal K, Pfaller, M. Medical Microbiology. 8th ed. Philadelphia; 2016.)

4. What is the most likely cause of the meningitis?   N. meningitidis is the most likely causative agent based upon this Gram stain (gram-negative cocci in pairs) and the patient’s age. 5. What other potential complications should we be aware of in this patient?   In patients with meningococcus, it is important to look out for Waterhouse-Friderichsen syndrome, which results from hemorrhage into the adrenal glands and subsequent adrenal failure.

Bacterial Diseases  519

6. When would be an appropriate time to initiate antibiotic therapy in this patient, and what antimicrobial agent could be used?   Antibiotic therapy must be initiated immediately when bacterial meningitis is suspected due to its severity. Based on the age of the patient and the morphology on the Gram stain, an appropriate antimicrobial can be chosen. Often a combination of intravenous vancomycin and ceftriaxone is used because of their central nervous system (CNS) penetration and broad coverage. Contacts of a patient with bacterial meningitis should be treated prophylactically with rifampin. 7. In a patient with human immunodeficiency virus, what infective agents may be more likely to cause meningitis than in a patient who has a fully competent immune system?   In a patient with human immunodeficiency virus (HIV), opportunistic infections such as toxoplasmosis, Cryptococcus, and JC virus must be considered in the differential diagnosis. If these organisms are discovered to be the causative pathogens in healthy individuals, they should be worked up for possible causes of immunodeficiency.

SUMMARY BOX: MENINGITIS • Presentation: Fever, nuchal rigidity, altered mental status, headache, and photophobia • Pathophysiology: Causes of meningitis are divided based on age (see Table 21.24). • Diagnosis: Positive Brudzinski and Kernig signs, lumbar puncture • Complications: Waterhouse-Friderichsen syndrome • Treatment: Vancomycin and ceftriaxone are often used to treat bacterial meningitis because of good central nervous system (CNS) penetration.

CHAPTER 22

VIRAL, PARASITIC, AND FUNGAL DISEASES Christopher Del Prete, MD, Maritza Montanez, MD, Thomas A. Brown, MD, and Sonali J. Bracken

Insider’s Guide to Viral, Parasitic, and Fungal Diseases for the USMLE Step 1 Preparing for microbiology does not end after you master the bacterial diseases that were discussed in the previous chapter. These remaining critters are important, too. Fungi are becoming particularly high yield on Step 1. In fact, some students report having more questions on fungi than on bacteria, so you should learn this section particularly well. Viruses and protists also show up rather frequently on boards, though not nearly as often as do bacteria and fungi. In comparison, helminths and helminth-related drugs are relatively low-yield topics. We recommend that you study for this section in the same way that you learned the bacteria in Chapter 21, Bacterial Diseases. A resource many students find particularly useful is Clinical Microbiology Made Ridiculously Simple. You may not have time to read the book cover to cover; however, it offers many excellent tables, figures, and classification schemes that students find visually appealing and memorable. If you prefer case-based learning or flashcards, we recommend Lippincott’s MicroCards, which offer images, cases, and classification flowcharts on one side, with diagnosis, treatment, and pathobiology material on the other. Our strongest suggestion is to try to study this material early and often—there are many specific details that need to be remembered for test day.

BASIC CONCEPTS—VIROLOGY 1. What structural components are used to categorize viruses?   Viruses can be classified according to the following: • Nucleic acid • Ribonucleic acid (RNA) vs. deoxyribonucleic acid (DNA) • Single-stranded vs. double-stranded • Segmented vs. nonsegmented • Capsid symmetry (icosahedral vs. helical) • Size • Presence or absence of an envelope     There are many RNA viruses; all contain single-stranded RNA (ssRNA) except rotavirus, which is a member of the reovirus family and contains a double-stranded RNA (dsRNA) genome. They are most easily categorized by their capsid symmetry and nucleic acid polarity. The nucleic acid polarity is either + or − sense. Viruses with + sense polarity have RNA strands that function directly as messenger RNA (mRNA). Viruses that contain − sense genomes (like influenza and parainfluenza) rely on RNA-dependent RNA polymerases to make a + sense template for transcription.     All DNA viruses except the Parvoviridae family contain dsDNA. All are enveloped except Parvoviridae, Adenoviridae, and Papovaviridae. If you can memorize the viruses that are all medically relevant DNA viruses, then you will know by default that any other virus must contain RNA (Fig. 22.1).

STEP 1 SECRET Although it is of little clinical relevance, you are expected to know how to classify viruses according to their genomes (DNA vs. RNA, single- vs. double-stranded, linear vs. circular), capsids (helical or icosahedral), and presence or absence of a membrane. For instance, you may be given a clinical vignette with a question that asks you to complete the following sentence: “The causative agent of this patient’s disease is _____.” Instead of naming viruses, you might see answer choices that reflect the composition of the virus (e.g., “a dsDNA virus with a circular genome”). Remember, the USMLE test makers love to ask second- and third-order questions! Unfortunately, the only way to learn this information is to simply memorize it. Tricks and mnemonics are of great use here. Of the DNA viruses, any virus core name ending with an “a” (papilloma, polyoma, hepadna) has a circular genome, and the rest are linear (adeno, parvo, pox, herpes). Hepadna virus is especially easy to remember—it has DNA in its name! All of these viruses have double-stranded genomes except parvovirus, because it is the smallest DNA virus (“parvo” is derived from the Latin word for small ). All of the DNA viruses replicate in the nucleus except poxvirus, which is large enough to carry its own DNA-dependent RNA polymerase. Enveloped viruses include herpesviruses and poxvirus; the rest are naked. You can remember this because poxvirus is large, so it has an envelope, and the other two enveloped DNA viruses both begin with “H.” 520

Viral, Parasitic, and Fungal Diseases  521 DNA viruses

Double-stranded

Enveloped

A

Single-stranded

Unenveloped

Unenveloped

Hepadnaviruses (C) Herpesviruses (L) Poxviruses (L)

Parvoviruses (L)

Adenoviruses (L) Papovaviruses (C) RNA viruses

(+) RNA

Unenveloped

B

Enveloped

(–) RNA

(+/–) RNA

(+) RNA via DNA

Enveloped

Double capsid

Enveloped

Retroviruses Coronaviruses Arenaviruses (S) Noroviruses Bunyaviruses (S) Reoviruses (S) Picornaviruses Flaviviruses Togaviruses Filoviruses Orthomyxoviruses (S) Paramyxoviruses Rhabdoviruses

Figure 22.1.  Classification of major viral families based on genome structure and virion morphology. A, DNA viruses. C, circular genome; L, linear genome. B, RNA viruses. S, segmented genome. + or − refers to nucleic acid polarity (see text). (From Rosenthal K, Tan J. Rapid Review Microbiology and Immunology. Philadelphia: Mosby; 2007.)

2. Name the disease associated with each DNA virus listed in Table 22.1. Table 22.1.   DNA Viruses FAMILY

MEMBER VIRUS(ES)

DISEASE(S)

Parvoviridae

Parvovirus B19

Fifth disease, aplastic anemia, arthritis, hydrops fetalis in utero

Herpesviridae

Herpes simplex virus types 1 and 2 (HSV-1 and HSV-2)

Oral and genital lesions Temporal lobe encephalitis

Varicella-zoster virus (VZV)

Chickenpox (varicella), herpes zoster

Epstein-Barr virus (EBV)

Infectious mononucleosis

Cytomegalovirus (CMV)

CMV retinitis, congenital deafness

Human herpesvirus types 6 and 7 (HHV-6 and HHV-7)

Roseola infantum (exanthema subitum)

HHV8

Kaposi sarcoma in human immunodeficiency virus (HIV) infection

Variola

Smallpox

Molluscum contagiosum virus

Umbilicated papules (spread among wrestlers and sexual partners)

Hepadnaviridae

Hepatitis B

Hepatitis, hepatocellular carcinoma

Adenoviridae

Adenovirus

Conjunctivitis, pneumonia, gastroenteritis and pharyngitis

Papovaviridae

Human papillomavirus (HPV)

Cervical dysplasia (high-risk serotypes 16 and 18) Note: Can be prevented by the quadrivalent vaccine

John Cunningham (JC) virus

Progressive multifocal leukoencephalopathy Note: Is commonly tested for before initiating immunosuppressive drugs

Poxviridae

522  Viral, Parasitic, and Fungal Diseases

STEP 1 SECRET Herpes simplex virus 1 (HSV-1) can cause genital lesions but more commonly causes oral lesions, as opposed to HSV-2, which is the more common cause of genital herpes. Just remember “Head to groin—1, 2.”

3. Cover the right-hand column in Table 22.2, and using the clinical description given, name the most likely virus. 

Table 22.2.   Classic Clinical Manifestations of Infection with Different Viruses DESCRIPTION

VIRUS(ES)

Rash with “slapped cheek” appearance

Parvovirus B19

Descending maculopapular rash, Koplik spots

Measles (rubeola) virus

Typically causes gastroenteritis but may cause paralysis by destruction of anterior horn cells

Poliovirus

Cervical cancer in sexually active smoker

HPV (serotypes 16, 18)

Parotitis, orchitis, and possible sterility in males

Mumps virus

Cataracts leading to blindness in newborns

Rubella virus

Painful vesicular lesions in dermatomal pattern; virus remains dormant in dorsal root ganglion

Varicella-zoster virus

Acute retinitis in patient with AIDS

CMV

Genital warts

HPV (serotypes 6, 11)

Painful genital vesicular lesions

HSV-2 (occasionally HSV-1)

Hepatitis in pregnant women, with high mortality rate

Hepatitis E virus

Fatigue, splenomegaly, and atypical lymphocytosis in a teenager; positive heterophile antibody test result

Epstein-Barr virus (EBV)

Gastroenteritis on cruise ship

Norovirus

Common cause of gastroenteritis in children

Rotavirus

Common cold viruses

Coronaviruses, rhinoviruses

Most common cause of bronchiolitis in children

RSV

Segmented genome can undergo reassortment, causing epidemic shift pneumonia

Influenza A virus

Severe encephalitis after an animal bite; intracytoplasmic Negri bodies in neurons

Rabies virus

Neonatal encephalitis

HSV or CMV

“Barking” cough in children

Parainfluenza virus

AIDS, acquired immunodeficiency syndrome; CMV, cytomegalovirus; HPV, human papillomavirus; HSV, herpes simplex virus; RSV, respiratory syncytial virus.

BASIC CONCEPTS—PARASITOLOGY 1. What are protozoa?   Parasites can be classified as protozoa or metazoa. Protozoa are single-celled eukaryotic organisms. The medically important protozoa and their associated diseases are listed in Table 22.3.

Viral, Parasitic, and Fungal Diseases  523

Table 22.3.   Protozoa Commonly Causing Disease in Humans PROTOZOAN

ASSOCIATED DISEASE

Entamoeba histolytica

Amebic dysentery (may cause sterile, “flask-shaped” liver abscess)

Giardia lamblia

Giardiasis (foul-smelling steatorrhea after drinking contaminated water)

Cryptosporidium spp.

Severe watery diarrhea and wasting in an immunocompromised patient

Trichomonas vaginalis

Trichomoniasis (vaginitis with green discharge)

Plasmodium falciparum, Plasmodium vivax, Plasmodium ovale, Plasmodium malariae

Malaria; spread by Anopheles mosquito (only P. vivax and P. ovale cause recurrent infection due to liver hypnozoite stage) Treat with primaquine to eradicate; adverse reaction in patients with G6PD deficiency is hemolytic anemia

Toxoplasma gondii

Toxoplasmosis (from cat feces) Can cause multiple cystic brain lesions in HIV-infected individuals

Leishmania spp.

Leishmaniasis (cutaneous and visceral)

Trypanosoma brucei

African sleeping sickness (African trypanosomiasis) Transmitted by tsetse fly

Trypanosoma cruzi

Chagas disease (American trypanosomiasis), reduviid kissing bug Dilated cardiomyopathy, dementia, and megacolon

G6PD, glucose-6-phosphate dehydrogenase; HIV, human immunodeficiency virus.

2. What is the difference between cestodes, nematodes, and trematodes?   They are all helminths (worms). Cestodes are flatworms (tapeworms), nematodes are roundworms, and trematodes are flukes. 3. Cover the left column in Table 22.4, and from the description of the infection at the right, name the helminth that causes it.  Table 22.4.   Helminths Commonly Causing Disease in Humans HELMINTH

Cestodes (Flatworms) Taenia solium (pork tapeworm)

MANIFESTATION(S)/MECHANISM(S) OF INFECTION

Infection from eating pork leads to intestinal worm; infection from egg ingestion causes cysts to encrust in brain

Taenia saginatum (beef tapeworm)

Transmitted by undercooked beef (mostly asymptomatic)

Diphyllobothrium latum (fish tapeworm)

Extremely long intestinal tapeworm that causes vitamin B12 deficiency and anemia; can be acquired by eating raw fish

Echinococcus granulosus

Ingestion of eggs in dog feces, causing cysts in liver, lungs, and brain Rupture of cysts causes allergic reaction

Nematodes (Roundworms) Enterobius vermicularis (pinworm)

Anal itching with white worms visible in perianal region; positive result on Scotch tape test

Ascaris lumbricoides (giant roundworm)

Intestinal infection, but worms pass from intestine to lungs Marked eosinophilia Eggs have rough, bumpy surface

Ancylostoma duodenale or Necator ameri­ canus (hookworms)

Larvae directly penetrate the skin and attach to intestinal mucosa, causing chronic blood loss and anemia

Trematodes (Flukes) Schistosoma haematobium

Hematuria after swimming in the Nile Egg has small terminal spine Increased risk of bladder squamous cell cancer

Schistosoma mansoni or Schistosoma japonicum

Free-swimming cercariae released from snails infect the human host Eggs are antigenic and induce granuloma formation Pipestem fibrosis of liver

Clonorchis sinensis

Biliary obstruction in patient from southeast Asia

Paragonimus westermani

Transmitted by eating raw crab meat, resulting in gastrointestinal and pulmonary disease

524  Viral, Parasitic, and Fungal Diseases

BASIC CONCEPTS—MYCOLOGY 1. What are the two morphologic types of pathogenic fungi?   Filamentous mold and unicellular yeast are the two types. An example of a filamentous mold is Aspergillus. Inhalation of spores, often found in hay and dead organic matter, is responsible for allergic bronchopulmonary aspergillosis, angioinvasive aspergillosis, and pneumonia with “fungus balls.” The pathognomonic microscopic appearance of Aspergillus is septate hyphae with a 45-degree branching pattern (Fig. 22.2). Other pathogenic filamentous molds include Mucor and Rhizopus, which branch at wide angles.     Cryptococcus neoformans is a unicellular (yeast), encapsulated fungus that can cause cryptococcal meningitis in immunocompromised patients. The capsule is antiphagocytic, is responsible for conferring virulence, and characteristically excludes India ink.

Figure 22.2.  Aspergillus in tissue showing acute-angle branching. (From Murray P, Rosenthal K, Pfaller M. Medical Microbiology. 8th ed. Philadelphia: Elsevier; 2016, Fig. 60-3.)

STEP 1 SECRET You should know the appearances of all the medically relevant fungi. It is common for the USMLE test makers to ask students to identify fungi based on images.

2. What is meant by the term dimorphic fungi?   Dimorphic fungi can exist in either the filamentous mold form or the unicellular yeast form, depending on conditions. In the environment, they live as mold. In the host, they live as yeast. As a general rule of thumb, all dimorphic fungi are responsible for systemic infections that mimic tuberculosis (i.e., they often lead to granuloma formation). The exception to this rule is Candida albicans, which does not result in granulomas.     Histoplasma capsulatum and C. albicans are both dimorphic fungi that can cause pathogenic infections. H. capsu­ latum causes an atypical pneumonia (occasionally with cavitations) endemic to the Ohio and Mississippi river valleys. C. albicans causes a variety of mucocutaneous and systemic infections (thrush, intertrigo, diaper rash, paronychia, vaginitis, urinary tract infections [UTIs], endocarditis in intravenous [IV] drug users, and pneumonia). Treat superficial Candida infections with nystatin and systemic infections with fluconazole or amphotericin B.     Note: Certain systemic fungal infections typically occur only in patients with severely compromised immune systems, especially with defects in cell-mediated immunity. These fungi include C. albicans, C. neoformans, and Aspergillus fumigatus.

STEP 1 SECRET It may be helpful to know the regions (Fig. 22.3) in which each fungus is found. You will often receive this information in the question stem.

3. How do the antifungal “-azole” agents work?   These agents all inhibit the synthesis of ergosterol, a key component of fungal cell membranes. Examples of this class of drug are ketoconazole, itraconazole, and miconazole.     Note: Ketoconazole inhibits hepatic enzymes as well as adrenal and gonadal steroid synthesis. This latter effect may explain the frequent reversible gynecomastia that develops in men who take this drug. 4. What is the mechanism of action for amphotericin B and nystatin?   Both of these agents bind to ergosterol in the fungal membrane, creating pores that affect membrane permeability and stability. Note that amphotericin B is very nephrotoxic and can cause distal (type 1) renal tubular acidosis.

Viral, Parasitic, and Fungal Diseases  525

Blastomycosis Coccidioidomycosis Histoplasmosis Cryptococcus gattii

Figure 22.3.  Map of fungi infections.

5. Cover the right-hand column in Table 22.5 and determine the most likely fungal organism based on the clinical description in column 1.

Table 22.5.   Clinical Manifestations of Fungal Infections DESCRIPTION

FUNGAL PATHOGEN(S)

Diffuse interstitial markings on chest radiograph in HIV-seropositive patient who presents with shortness of breath Positive silver staining Responds to trimethoprim-sulfamethoxazole

Pneumocystis jiroveci Note: Most common opportunistic infection in HIV-infected individuals

Thrush in cancer patient receiving high-dose chemotherapy

Candida albicans

Signs and symptoms of meningitis in an HIV-infected patient Positive India ink staining of CSF obtained by lumbar puncture

Cryptococcus neoformans

Lung granulomas in former or present resident of Ohio River Valley Intracellular yeast

Histoplasma capsulatum

Tinea cruris, corporis, and pedis Hyphae on KOH preparation

Trichophyton, Epidermophyton, or Microsporum

Lung granulomas in San Joaquin Valley (coccidioidomycosis) Spherule with endospores

Coccidioides spp.

Fungus ball in cavitary lung lesion Can be angioinvasive

Aspergillus

Systemic mycoses involving lungs, bone, and skin Broad-based, budding yeast

Blastomyces

Ascending lymphangitis after puncture with a thorn

Sporothrix schenckii

Severe rhinocerebral infection in diabetic ketoacidosis

Agents of mucormycosis (any of several different fungi)

Nonseptate hyphae with wide 90-degree branching pattern

Angioinvasive fungi

CSF, cerebrospinal fluid; HIV, human immunodeficiency virus; KOH, potassium hydroxide.

526  Viral, Parasitic, and Fungal Diseases

Case 22.1 A 45-year-old woman presents to the clinic with complaints of a flu-like illness. The patient is currently employed as a nurse and has recently had to take several days off for sick leave. She states that approximately 1 month ago, she started feeling fatigued and feverish. Soon she developed an “achy” abdominal pain in the right upper quadrant (RUQ). Last week she noticed that her urine was darker than usual.   

1. With this initial history, what is your differential diagnosis?   There are several causes of acute RUQ pain—biliary disease (colic, cholecystitis, ascending cholangitis), acute pancreatitis, peptic ulcer disease, dyspepsia, lower lobe pneumonia, or an atypical presentation of myocardial infarction. In a 45-year-old woman, a gallstone should be high on your differential list. Her complaint of darkened urine suggests conjugated bilirubinuria and is further support for an obstruction. It is also possible that this clinical picture could be caused by an intrahepatic process.

Case 22.1 continued: On further questioning, you learn that she is quite concerned about being infected with human immunodeficiency virus (HIV) due to a needlestick exposure a few months earlier. On examination you appreciate a jaundiced, ill-appearing woman with RUQ tenderness to palpation. You remind yourself about Charcot’s triad (fever, jaundice, and RUQ pain) and realize that she has all three features.   

2. How does the preceding information alter the differential diagnosis?   Given the needlestick exposure, the concern now should be an infection such as HIV or, even more likely, hepatitis B. One would want to check liver enzymes and serologic findings for viral hepatitis as well as HIV at this point. These tests may also help clarify whether her pain is related to gallbladder disease and if an abdominal ultrasound is necessary.

Case 22.1 continued: Alanine aminotransferase (ALT) and aspartate aminotransferase (AST) are markedly elevated, but the alkaline phosphatase and γ-glutamyltransferase (GGT) are within normal limits. The hepatitis serologic assays return positive for hepatitis B core IgM antibody (HBcAb IgM) and hepatitis B surface antigen (HBsAg).   

3. What is the diagnosis?   Acute hepatitis B infection is the diagnosis. Hepatitis B can be acute or chronic (>6 months). Acute hepatitis B often manifests weeks to months after infection with constitutional symptoms, RUQ abdominal pain, and jaundice. Because hepatitis B can be transmitted parenterally, she was likely infected by the needlestick. 4. Why are the aspartate aminotransferase and alanine aminotransferase values elevated in this patient?   Hepatitis is an inflammatory disease of the liver. The viral particles infect hepatocytes, and in an effort to clear the infection, the host immune system destroys infected cells. Hepatocyte necrosis causes a massive leakage of hepatic enzymes. The AST and ALT are markers of hepatocyte death, not liver function. Thus, the term transaminitis indicates an increased AST and ALT. In comparison, a “cholestatic” pattern suggests an obstructive process (intra- or extrahepatic) with elevated alkaline phosphatase and GGT. 5. What are the 3 different antigens in hepatitis B, and which antibody shows up first in an acute infection?   The three different antigens in hepatitis B are surface antigen (HBsAg), core antigen (HBcAg), and e antigen (HBeAg) (Table 22.6).     HBcAb IgM is the first to be made. The presence of this antibody and the HBsAg indicates acute infection. The presence of this antibody and absence of HBsAg indicates a recent, resolved acute infection. Note that nearly 95% of all acute hepatitis B infections are resolved; in contrast, only 15% to 45% of hepatitis C infections are resolved. Thus, hepatitis C is more likely to cause chronic disease. Table 22.6.   Serologic Markers in Hepatitis B Infection MARKER

ABBREVIATION

SIGNIFICANCE

Hepatitis B surface antigen

HBsAg

Indicates active infection (acute or chronic)

Hepatitis B surface antibody

HBsAb

Indicates successful eradication of infection or immunized status

Hepatitis B core antibody IgM

HBcAb IgM

First antibody produced in acute hepatitis B infection

Hepatitis B e antigen

HBeAg

Indicates high level of viral infectivity

IgM, immunoglobulin M.

Viral, Parasitic, and Fungal Diseases  527

6. What other viruses cause hepatitis, and what is their usual course of infection?   Hepatitis A, C, D, and E viruses are all RNA viruses. Hepatitis B virus is the only DNA virus that commonly causes hepatitis. Hepatitis A and E cause acute infections and are transmitted by the fecal-oral route, typically through contaminated water or food. Hepatitis B, C, and D can cause acute or chronic infections and are transmitted sexually, parenterally (IV drug use, transfusion, or needlestick), and vertically (mother to baby). Remember that the “chronic” viruses are so labeled because they persist beyond 6 months; it is important to realize that any of the hepatitis viruses can cause an acute infection with the classic presenting signs. Hepatitis D is a uniquely defective virus: It requires coinfection with hepatitis B before it can cause disease. Thus, a serum test for anti-hepatitis D Ab is indicated only if the HBsAg is positive. 7. How is viral hepatitis treated?   Viral hepatitis is treated via passive immunization through the administration of pooled IgG and supportive care. A vaccine is available. Active hepatitis B is treated with interferon alpha and the reverse transcriptase inhibitor lamivudine. The hepatitis B vaccine is now routinely administered to children and high-risk individuals such as yourself (health-care workers). Hepatitis C is treated with combination therapy consisting of interferon alpha and ribavirin. There is currently no vaccine available for hepatitis C. The best treatment for hepatitis D consists of the prevention of hepatitis B infection. At this time, hepatitis E is treated with supportive care (Table 22.7). Table 22.7.   Hepatitis Viruses VIRUS

MODE OF TRANSMISSION

TIME COURSE

TREATMENT

Hepatitis A virus

Fecal-oral

Acute

Pooled intravenous immunoglobulin (IVIG); vaccine for travelers to endemic areas

Hepatitis B virus

Usually sexual contact, but also parenteral and vertical

Chronic (>6 months)

Vaccine for persons at high risk Interferon alpha and lamivudine

Hepatitis C virus

Usually parenteral, but also sexual contact and vertical

Chronic

Interferon alpha and ribavirin

Hepatitis D virus

Sexual contact, parenteral, and vertical

Chronic

Prevention of hepatitis B infection through vaccination

Hepatitis E virus

Fecal-oral

Acute

Symptomatic relief

8. Describe the association between viral hepatitis and hepatocellular carcinoma.   Both hepatitis B and hepatitis C have been strongly associated with hepatocellular carcinoma (HCC). Hepatitis C is estimated to be the causative agent behind approximately one-third of HCC cases.

SUMMARY BOX: HEPATITIS • Epidemiology • Fecal-oral transmission (hepatitis A and E) • Sexual, parenteral, or vertical transmission (hepatitis B, C, and D) • Hepatitis D requires co-infection with hepatitis B. • Symptoms: Charcot’s triad: fever, jaundice, and right upper quadrant (RUQ) pain • Pathophysiology: Immune-mediated destruction of virally infected hepatocytes • Diagnosis: Impressively elevated AST and ALT levels in patient with symptoms of viral hepatitis with confirmatory viral titers • Treatment: See Table 22.7. • See Chapter 7 for a more detailed analysis of hepatitis B.

Case 22.2 A 51-year-old Mexican-American man presents to the emergency department (ED) in a moderately stuporous condition with new-onset seizure and headache. He is confused and is unable to answer questions. His wife describes an approximate 1-month history of worsening headache that has not been relieved with ibuprofen. Three days before admission, he awoke feeling nauseated in the middle of the night and collapsed on the way to the bathroom. He was found unconscious on the bedroom floor and regained consciousness minutes later. On the day of admission, the patient complained of an acrid, burning smell at breakfast, and his right arm began to twitch uncontrollably. He then slumped in his chair and seized violently.   

528  Viral, Parasitic, and Fungal Diseases 1. What is the differential diagnosis for new-onset seizure in adults?   New-onset seizure in an adult is an ominous sign. It can be associated with a space-occupying lesion, head trauma, medications, alcohol withdrawal, illicit drug use, or intracranial infection.

Case 22.2 continued: On admission, the patient is obtunded and oriented to person only. His speech is disorganized and incoherent. He is afebrile with no lymphadenopathy or nuchal rigidity. Further history is elicited from his wife. It turns out that his daughter has recently been treated for a Taenia solium infection.   

2. What is the significance of a close contact with previous taeniasis?   Humans can serve as the intermediate or definitive host, depending on which stage of the parasite is ingested. Taeniasis is caused by consumption of cysticerci (larvae) in undercooked pork, with subsequent growth of the adult tapeworm in the intestine. Thus, the patient’s daughter most likely ate infected meat and represents the index case. When ova shed in the feces of a human carrier are ingested, a distinct disease, cysticercosis, develops. Here, the organism disseminates hematogenously and encysts in the skin, striated muscle, and brain. 3. How is the diagnosis of neurocysticercosis made?   Diagnosis is typically made clinically on the basis of radiographic findings, symptomatology, and exposure history. Images can show a single lesion that is often calcified, serving as a substrate for seizures, or hundreds of lesions distributed diffusely through the cortex. There are serologic tests for antiparasite antibodies, but they are somewhat unreliable and rarely available in endemic areas. Examination of the stool for parasite eggs may detect concurrent taeniasis, but a positive finding is not diagnostic of cysticercosis.

Case 22.2 continued: The patient is given IV lorazepam for seizure prophylaxis. Blood work and chest x-ray film are unrevealing. A computed tomography (CT) scan of the head shows viable cysts as diffuse radiolucent defects (small arrow in figure) and as calcified (nonviable) cysts (large arrow in figure) (Fig. 22.4).   

    Based on unequivocal radiographic evidence, his possible exposure history in an endemic area, and clinical symptoms of new-onset seizures, the diagnosis of neurocysticercosis is made. Stool sample for Taenia solium ova is positive, indicating active gastrointestinal (GI) infection.

Figure 22.4.  Computed tomography head scan demonstrating cysts in patient from Case 22.2. (From Cohen J, Powderly WG, Berkley SF, et al. Infectious Diseases. 2nd ed. Edinburgh: Mosby; 2004.)

4. How is neurocysticercosis managed?   Asymptomatic disease is not treated. Symptomatic neurocysticercosis with few parenchymal lesions is managed only with anticonvulsant therapy. Cysticidal agents like albendazole and praziquantel, paired with prophylactic IV corticosteroids, are reserved for symptomatic disease with a high burden of cysts.

Case 22.2 continued: An infectious disease (ID) consult suggests that the patient should be started on IV dexamethasone to limit the anticipated inflammatory response to anthelminthic therapy. Within 12 hours, the patient’s mental status improves. His 15-day course of albendazole, a cysticidal agent, is initiated.   

5. What does neurocysticercosis look like pathologically?   The brain parenchyma is infiltrated with fluid-filled cysts surrounded by a dense fibrotic capsule. Inflammation is scant and mainly lymphocytic.

Viral, Parasitic, and Fungal Diseases  529

6. What is the epidemiology of neurocysticercosis?   Neurocysticercosis is the most common parasitic disease of the central nervous system (CNS). It is caused by the cestode Taenia solium, a parasite endemic to Central and South America, sub-Saharan Africa, and parts of Asia. It is the leading cause of late-onset seizure in these regions. Infections in the United States have also been reported, primarily in large urban centers among immigrants and travelers.

SUMMARY BOX: NEUROCYSTICERCOSIS • Epidemiology: Most common parasitic disease of the central nervous system (CNS) • Symptoms: Headache, late-onset seizure • Pathophysiology: Taenia solium infection • Ingestion of ova shed in the feces of a human carrier leads to cysticercosis with hematogenous dissemination of the organism. • Diagnosis: Cortical cysts on computed tomography (CT) scan, symptomatology, exposure history • Treatment • Anticonvulsant therapy for symptomatic neurocysticercosis • Cysticidal paired with prophylactic intravenous (IV) corticosteroids for symptomatic disease with high cyst burden

Case 22.3 A 35-year-old man with acquired immunodeficiency syndrome (AIDS) presents to the ED complaining of fever, stiff neck, and a mild but persistent headache. One month ago he experienced an upper respiratory tract infection that resolved on its own without specific therapy. A chest x-ray film at that time was normal. Over the last 48 hours he has vomited twice, and he complains that it hurts his eyes to go outside because “it is too bright.” He denies night sweats or recent weight loss. Examination is significant for nuchal rigidity and positive Kernig and Brudzinski signs. Laboratory tests from his last visit show a CD4 count of 105 cells/mm3 (normal is 500–1500 cells/mm3).   

1. What diagnosis do you suspect?   Fever, nuchal rigidity, and photophobia are classic for meningitis. The causes of meningitis vary considerably based on age and immune status, and it is important to know likely pathogens in the different age groups (Table 22.8).     In immunocompromised patients, CNS symptoms can be caused by cytomegalovirus (CMV) encephalitis, toxoplasmosis, cryptococcosis, CNS lymphoma, and progressive multifocal leukoencephalopathy (PML). CMV typically occurs in HIV patients with a CD4 count below 50. PML is a rare, typically fatal development in AIDS patients following recrudescence of John Cunningham (JC) virus (a polyomavirus). It presents with signs of increased intracranial pressure and focal neurologic deficits, rather than true meningismus, as described here. Primary CNS lymphoma is almost always accompanied by night sweats and weight loss. In this patient the clinical picture is best accounted for by toxoplasma encephalitis or cryptococcal meningitis.     Note: For purposes of review, Kernig sign is positive when a straight leg raise in the supine position elicits severe neck pain. Brudzinski sign is positive when passive flexion of the neck causes involuntary knee and hip flexion (to reduce stress on the spine).

Table 22.8.   Causes of Meningitis in Different Age Groups NEWBORNS (0–6 MONTHS)

Group B streptococci E. coli Listeria

CHILDREN

Pneumococci N. meningitidis Haemophilus influenzae type b Enterovirus (echovirus, coxsackievirus B)

ADULTS

Pneumococci N. meningitidis Enterovirus (echovirus, coxsackievirus B)

ELDERLY (65+)

Pneumococci E. coli Listeria

Case 22.3 continued: A head CT scan is performed and no contraindications to lumbar puncture are identified. An elevated opening pressure is noted.   

530  Viral, Parasitic, and Fungal Diseases 2. How do the cerebrospinal fluid findings differ among viral, fungal, and bacterial meningitis?   See Table 22.9 for a comparison of these findings. Table 22.9.   Cerebrospinal Fluid Findings in Meningitis OPENING PRESSURE

INFECTION

COLOR

WBC DIFFERENTIAL

GLUCOSE

PROTEIN

Viral (aseptic)

Clear

Increased lymphocytes

Normal

Normal

Normal or mildly increased

Fungal/TB

Clear

Increased lymphocytes

Low

Normal to elevated

Elevated

Bacterial

Cloudy

Predominantly neutrophils

Low

High (>40 mg/dL)

Elevated

TB, tuberculosis; WBC, white blood cell.

Case 22.3 continued: Cerebrospinal fluid (CSF) analysis reveals a lymphocytosis, with normal to mildly decreased glucose, increased opening pressure, and slightly elevated CSF protein. Gram stain is negative. Serum is negative for toxoplasma antibodies, ruling out previous exposure. An India ink preparation of the CSF is as shown in Fig. 22.5.   

Figure 22.5.  India ink preparation of cerebrospinal fluid revealing encapsulated cryptococci. (From Andreoli TE. Cecil Essentials of Medicine. 4th ed. Philadelphia: WB Saunders; 1997.)

3. What is the diagnosis?   Cryptococcal meningitis is the diagnosis. The India ink stain (see Fig. 22.5) shows encapsulated cryptococci; note the large capsules surrounding the smaller organisms. More commonly, a latex agglutination assay for the cryptococcal capsular antigen is done. Cryptococcus can also be cultured on Sabouraud agar. 4. What is aseptic meningitis?   Aseptic meningitis is inflammation of the meninges caused by nonbacterial pathogens. Over 80% of aseptic meningitis diagnoses are due to viral infections (commonly by enteroviruses such as echovirus and coxsackievirus B), but other causes include mycobacteria, fungi, rickettsiae, spirochetes, malignancy, and medications (e.g., IV immunoglobulin and Bactrim [trimethoprim-sulfamethoxazole]). 5. Why is the distinction between aseptic meningitis and bacterial meningitis important?   The prognosis and treatment vary tremendously depending on whether the cause of the meningitis is viral, fungal, or bacterial. Acute bacterial meningitis can be a life-threatening disease and often responds well to antibiotics. Fungal meningitis likewise requires emergent therapy. In contrast, aseptic viral meningitis is usually self-limited. After 48 hours of negative CSF cultures, patients will be taken off empiric antibiotics and monitored for any change in course. Viral encephalitis, in which both the meninges and the brain parenchyma itself become inflamed, frequently has devastating outcomes. 6. What is the treatment of cryptococcal meningitis?   Amphotericin B plus the antimetabolite 5-flucytosine for induction therapy is the treatment for cryptococcal meningitis. Maintenance therapy requires a minimum of 10 weeks of fluconazole; depending on the severity of infection, fluconazole may also be continued for life if well tolerated.

Viral, Parasitic, and Fungal Diseases  531

SUMMARY BOX: MENINGITIS • Epidemiology: Causes vary by age group (see Table 22.8). • Symptoms: Fever, nuchal rigidity, photophobia • Diagnosis: Cerebral spinal fluid (CSF) analysis (see Table 22.9), positive Kernig and Brudzinski signs • Diagnosis of cryptococcal meningitis requires India ink stain or latex agglutination assay. • Treatment: Dependent on cause of infection • Bacterial disease: Antibiotics • Cryptococcal meningitis: Amphotericin B plus 5-flucytosine

Case 22.4 A 6-year-old boy is evaluated for a 2-day history of headache, runny nose, and nausea. He also complains of diffuse muscle aches but denies neck stiffness or photophobia. He does not know of any sick contacts but was recently transferred to a new elementary school. Examination is significant for fever and pink and edematous (“boggy”) nasal turbinates but is otherwise unrevealing.   

1. Are you concerned, given the history, examination, and laboratory test findings?   This is a common story for young children. The cause of new-onset fever accompanied by headache and nausea varies significantly. This clinical picture could be caused by something as commonplace as a viral infection (common cold, influenza, or gastroenteritis) or an ear infection as well as something more serious like meningitis. Influenza should be eliminated from the differential with a rapid flu test if there is a high index of suspicion, and ear infections and meningitis should be eliminated by clinical examination. Most commonly, reassurance and clinical follow-up are all that is necessary, particularly once life-threatening illnesses have been ruled out. 2. What is the most common presentation of influenza in children? In adults?   In both adults and children, influenza virus infection classically presents with acute-onset, high-grade fever (>102.2° F), myalgia, malaise, and headache. Other common symptoms include nonproductive cough and rhinitis. In children, the symptoms can vary greatly depending on the age of the child and previous exposure to influenza, though generally in a child without comorbidities, influenza virus causes a self-limited infection. The same is true in adults, and most will recover within a week or two without significant sequelae.     An important complication of influenza to keep in mind for Step 1 is bacterial co-infection with organisms that cause secondary bacterial pneumonia. Secondary bacterial pneumonia most commonly affects older adults (ages 65+) and presents with a worsening of respiratory and febrile symptoms after initial improvement. Pneumococcus is responsible for the majority of these cases, although Staphylococcus aureus accounts for an increasing proportion of cases. 3. What is the significance of H and N typing? How do these proteins help influenza virus infect host cells?   H and N (hemagglutinin [HA] and neuraminidase [NA]) refer to the major antigenic determinants found on the surface of influenza viruses and provide a means of categorizing virus subtypes. The specific type of HA also determines where in the respiratory tract the virus will bind, with certain subtypes having tropism for sites in more proximal or distal airways.     Virus aerosols or droplets are inhaled and HA binds to sialic acid on the surface of epithelial cells in the bronchioles, bronchi, and trachea. This facilitates endocytosis of the virus so that viral RNA can be released into the cytoplasm. From there, it is transported into the nucleus for replication and transcription. New virus buds from the host cell membrane in a process enabled by NA. NA catalyzes the hydrolysis of sialic acid from the virions and host cell receptors, thus allowing viruses to leave and infect other cells. Oseltamivir (Tamiflu) and zanamivir (Relenza) competitively inhibit viral NA, which prevents viral budding.

Case 22.4 continued: A rapid flu test is negative. Several days later, an erythematous macular rash develops in the malar distribution. His abdomen and extremities are also covered diffusely by a reticular pattern. No desquamation is noted. Antistreptolysin O (ASO) titer is negative.   

4. What is the most likely diagnosis now?   Erythema infectiosum is the likely diagnosis. Given the description of the diffuse rash, it is important to rule out exotoxinmediated scarlet fever with an arteriosclerosis obliterans ASO titer as well as rarer vasculitic entities like Kawasaki disease via clinical examination and history (absence of desquamation, conjunctivitis, and cervical lymphadenopathy). Clinical Pearl In practice, the sensitivity of the rapid flu test ranges from 30% to 70%. Whereas a positive result can be very helpful in ruling in disease because of the test’s high specificity (≈98%), a negative test should not be used to rule out influenza infection if clinical suspicion is high.

532  Viral, Parasitic, and Fungal Diseases 5. What is erythema infectiosum?   Erythema infectiosum (also known as fifth disease or slapped cheek disease) is a self-limited illness most often affecting school-aged children. It is a viral exanthema caused by parvovirus B19. The virus infects erythroid progenitor cells in the bone marrow and peripheral blood, resulting in defective erythropoiesis.     Often it presents as a biphasic illness, with a viremic period marked by fever, headache, and myalgia. Up to a week later, the characteristic slapped cheek rash can appear and evolve to include the whole body. The illness may persist for several weeks to months and is exacerbated by stress, increased physical activity, and exposure to sun. 6. What are the other viral exanthems?   See Table 22.10. 7. What are other clinical manifestations of parvovirus B19 infection?   Remember the other presentations of B19 as the three As—anemia, arthritis, and abortion: • Aplastic anemia (transient anemic crisis) usually causes pure red blood cell aplasia but can also affect other hematopoietic cell lines. Severe anemia most often occurs in patients with extant hematologic abnormalities such as sickle cell disease, thalassemia, and hereditary spherocytosis. Severity varies, and transfusions are occasionally necessary. • Arthritis is chronic monoarticular or pauciarticular in nature. Symptoms are usually symmetric and involve the small joints of the hands, knees, and feet. The arthritis is nondestructive. • Abortion: In pregnant women, parvovirus can cause miscarriage, intrauterine fetal death, and nonimmune hydrops fetalis. When infection occurs before 20 weeks’ gestation, outcomes are worse. 8. Describe the structure of the B19 virion. How is it transmitted?   Parvovirus B19 is a nonenveloped ssDNA virus. It is transmitted primarily by respiratory aerosols but can also pass hematogenously and transplacentally. Approximately 50% to 70% of Americans older than 18 years of age are seropositive. 9. How is infection diagnosed?   Serologic testing for B19-specific IgM is used to detect acute infection. Detectable levels can be found within 7 to 10 days of exposure and remain elevated for several months. Another method to diagnose acute infection is detection of B19 DNA by polymerase chain reaction (PCR). This test, however, has the drawback of remaining positive for several years and does not reliably indicate acute infection. 0. What is the treatment for parvovirus infection? 1   There is no specific treatment for B19 infection. Passive immunity via immunoglobulin transfer may be beneficial to compromised hosts with chronic infection but plays no role in acute disease. Currently, there is no vaccine available. Prevention of disease by good infection control practices is the best method to decrease transmission.

SUMMARY BOX: PARVOVIR US B19 • Epidemiology: Most commonly affects school-aged children • Symptoms: Fever, nausea, and myalgia, with subsequent development of the slapped cheek rash characteristic of erythema infectiosum • Other clinical manifestations include aplastic anemia, arthritis, and abortion. • Pathophysiology: Caused by single-stranded DNA virus that infects early-stage red blood cells (RBCs) • Diagnosis: Serum immunoglobulin (Ig) M levels or polymerase chain reaction (PCR) • Treatment: No specific treatment or vaccine is available.

Case 22.5 A 28-year-old male presents to the ED with a 3-day history of high fever, headache, fatigue, and muscle aches. Over the last 24 hours, he has developed vomiting and diarrhea and “hasn’t been able to keep anything down.” His eyes appear reddened.   

1. With this initial history, what is your differential diagnosis?   Influenza, typhoid, malaria, and Ebola are all infections to be considered in this patient. Because some of these are extremely serious infections, travel history should be obtained at this point.

Case 22.5 continued: Upon further questioning, the patient reports travel to Liberia 2 weeks ago.   

Table 22.10.   Childhood Exanthems DISEASE

ETIOLOGY

AGE RANGE

Measles (first disease)

Rubeola (paramyxovirus; enveloped-ssRNA virus)

3–5 years

Chickenpox (second disease)

Varicella-zoster virus Usually before (enveloped herpesviage 10 rus dsDNA, also known as HHV-3 )

RASH

ASSOCIATED SYMPTOMS

COMPLICATIONS

Maculopapular erythematous rash that spreads from face to trunk and extremities over ≈3 days.

Cough, coryza, conjunctivitis, Koplik spots (white spots on oral mucosa). Often associated with fever and toxic appearance.

Otitis media, encephalitis, PNA, rarely subacute sclerosing panencephalitis months to years after infection.

“Dew drops on rose petal” appearance— (papular/vesicular rash on an erythematous base). Starts as localized groups of erythematous macules, progresses to papules that eventually become vesicular and rupture with associated crusting. Often intensely pruritic with lesions in various stages of healing across entire body, though palms and soles are usually spared. Usually occurs 24 hr or less after mild prodrome.

Malaise, decreased appetite, low-grade fever.

Shingles from reactivation of dormant virus. Causes dermatomal rash in elderly with postherpetic neuralgia (severe pain in distribution of rash). Effective vaccine since 1995 has reduced rates of infection and complications.

Low-grade fever, generally not In pregnancy, can cause congenital toxic appearing. Tender rubella syndrome (MR, cataracts, suboccipital and posterior PDA) or miscarriage. Also a auricular lymphadenopathy notable cause of encephalitis are unique features. and/or TTP in neonates. Older patients may have arthralgias as predominant symptom.

School age to young adult

Rapidly progressive rash (vs. measles). Starts on face and progresses to trunk and extremities. Pruritic/maculopapular, but fainter than measles.

Scarlet fever (fourth disease)

School age to young adult

Perioral pallor with flushed face, fine red sand- Abrupt-onset high fever, headache, paper rash on abdomen and trunk. Lasts and severe sore throat. about a week followed by peeling skin.

Rheumatic fever, glomerulonephritis, peritonsillar abscess.

4–10 years, pregnant patients

Bright red “slapped-cheek” pruritic facial rash. When facial rash fades, can progress to trunk and extremities as an erythematous reticular rash.

Hydrops fetalis, fetal demise. Aplastic crisis especially in sickle cell patients.

Streptococcus pyogenes

Erythema Parvovirus B19 infectiosum (nonenveloped (fifth disease) ssDNA virus)

Roseola infantum HHV-6 and -7 6–36 months (sixth disease) (Enveloped herpesvirus dsDNA)

50% of affected individuals will have no symptoms. Malaise, low-grade fever, sore throat. Joint pain more common in young women.

Fever followed by pink macules and papules Abrupt-onset high-grade fever but Rapidly developing high-grade fever with white halos appearing on trunk and generally not toxic appearing. (>104°F) can lead to febrile spreading to neck, face, and proximal Just as abrupt defervescence, with seizures. extremities. Rash often improves or no new symptoms for a few days. Key to recognizing this disease is the disappears entirely within 24 hr. Rash follows as fever resolves. timing. Think: High fever followed by transient trunk, neck, facial rash.

dsDNA, double-stranded DNA; HHV-6, -7, human herpesvirus types 6 and 7; MR, mental retardation; PDA, patent ductus arteriosus; PNA, pneumonia; ssDNA, single-stranded DNA; TTP, thrombocytopenic purpura.

Viral, Parasitic, and Fungal Diseases  533

German measles Rubella (third (togavirus; disease) enveloped +ssRNA)

534  Viral, Parasitic, and Fungal Diseases 2. What diagnosis are you now most concerned about?   Considering the patient’s history of recent travel to West Africa, Ebola infection is a strong possibility. Ebola virus is a nonsegmented, single-stranded RNA virus that belongs to the Filoviridae family.     Note: The 2014 Ebola outbreak in West Africa was the largest Ebola outbreak in history. The highest number of cases was recorded in Liberia. 3. How is Ebola virus transmitted?   The Ebola virus is transmitted through direct contact with body fluids from an infected individual displaying signs and symptoms of illness. Patients with Ebola typically develop acute onset of symptoms within 2 to 21 days after initial exposure. 4. What are the typical symptoms of Ebola infection?   Typical symptoms include fever, myalgias, weakness, vomiting, diarrhea, and rash. Some patients experience unexplained bleeding (i.e., blood in the stool or ecchymoses); major bleeding is a manifestation of the terminal phase of the infection. Bleeding results from massive, virally-triggered cytokine release (a phenomenon known as cytokine storming ), which increases the permeability of blood vessels.

Case 22.5 continued: The patient is quickly isolated, and the local health department is notified of the possible Ebola case. Rapid blood testing for Ebola virus by reverse transcription polymerase chain reaction (RT-PCR) is positive.   

5. How is Ebola infection managed?   Ebola is managed with supportive care because there is no known cure. Important measures include repletion of fluids lost through vomiting, bleeding, and diarrhea; correction of electrolyte abnormalities; and prevention of shock.

SUMMARY BOX: EBOLA INFECTION • Epidemiology: The most widespread epidemic in history is ongoing in West Africa. The highest number of cases during the 2014 outbreak was reported in Liberia. • Transmission occurs through direct contact with bodily fluids of infected individuals. • Symptoms: Flu-like symptoms with vomiting, diarrhea, and bleeding Diagnosis: Exposure history, reverse transcription polymerase chain reaction (RT-PCR) • Treatment: Supportive care (i.e., fluid and electrolyte repletion)

Alex M. Hennessey, MD, Thomas A. Brown, MD, and Sonali J. Bracken

CHAPTER 23

PHARMACOLOGY AND TOXICOLOGY

Insider’s Guide to Pharmacology and Toxicology for the USMLE Step 1 We recognize that pharmacology and toxicology are not trivial Step 1 subjects. Each section has its own extensive list of drugs to know, and after a while, it can become very overwhelming. We hope that the following list of tips will help you prepare for this subject with the minimum amount of stress: • Although there are no shortcuts for learning all the drugs that you are expected to know for boards, you do not necessarily need to know an extensive amount of information about each and every drug. If a drug pops up over and over again in First Aid and in your question bank software, you should learn more about it because it has a greater chance of showing up on your exam. For example, the USMLE is not likely to ask you about the adverse effects of an infrequently used drug such as bosentan, but you will be expected to know its use and basic mechanism of action. On the other hand, you should know the mechanism of action, uses, adverse effects, and toxicity treatment for a drug such as digitalis, which is commonly used in practice and shows up repeatedly within the context of boards questions. • Focus on unique aspects and properties of the drugs you learn. The USMLE Step 1 is not as likely to ask you about whether a drug causes headache or gastrointestinal (GI) upset because these reactions are quite common. You will be tested on drugs that cause seizures, sexual dysfunction, reflex tachycardia, pulmonary fibrosis, etc. • You should expect to have a few questions on the fundamental principles of pharmacology included in the Basic Concepts section of this chapter. The equations for pharmacology are particularly high yield. It is a great idea to practice using these equations as much as possible. You may consider writing these on your marker board immediately before you begin your exam. • Do not try to cram pharmacology. The more you practice, the better you will get at classifying drugs by groups and acquiring a feel for the drug categories that are most important to know for Step 1. Five-star topics include drugs of the autonomic nervous system (ANS). In fact, you should expect to get three to five questions on your exam specifically about ANS drugs. Four-star topics include antibiotics, analgesics, cardiovascular drugs, and neurologic/psychiatric drugs.

BASIC CONCEPTS 1. How does the route by which a drug is administered affect its metabolism?   Most drugs that are taken orally enter the bloodstream at the level of the portal circulation and encounter the liver almost immediately. Here, they undergo first-pass metabolism, which renders them less efficacious than if they had reached their target organs first. As such, the rate of portal blood flow is a major determinant of first-pass metabolism. First-pass metabolism can be circumvented by administering the medication parenterally (e.g., intravenously, intramuscularly, or subcutaneously). Because the drug is able to reach its target faster and relatively unaltered, its onset of action is more rapid. In comparison with the oral route, the advantage of parenteral administration is that the clinician is able to observe the effects of the drug almost instantaneously and manage dosing appropriately. Disadvantages include more rapid induction of drug side effects, complications associated with venipuncture (hematoma, phlebitis, extravascular injection), and the need for frequent dosing in cases where a sustained effect is desired.     Other routes of delivery include inhalation (rapid effect, targeted delivery); intrathecal (into the cerebrospinal fluid [CSF]); sublingual (rapid onset, avoids first-pass metabolism); rectal (note that some of the delivered drug will undergo first-pass metabolism because the superior rectal vein drains into the portal circulation but the middle and inferior rectal veins drain to the systemic circulation); topical (for local effect); and transdermal (sustained delivery).     The concentration of a drug in the body is dependent on the route of administration because this influences the bioavailability (F) of the drug. A drug administered intravenously has 100% bioavailability (F = 1), but drugs administered orally have a bioavailability of less than 1 (F heart

Dilates coronary arteries, suppresses SA/AV nodes Causes peripheral vasodilation without reflex tachycardia (due to its suppression of SA/AV nodes)

Vasospastic (Prinzmetal’s) angina Obstructive (exertional) angina Arrhythmias (especially supraventricular tachyarrhythmias) Hypertension

Nifedipine

Vasculature

Dilates coronary arteries Causes peripheral vasodilation with reflex tachycardia (no chronotro­ pic/inotropic effects on the heart)

Vasospastic (Prinzmetal’s) angina Not useful in obstructive (exertional) angina because reflex tachycardia increases oxygen demand Ideal for hypertension with bradycardia because it causes a reflex tachycardia

AV, atrioventricular; SA, sinoatrial.

4. What are some general side effects of calcium channel blockers?   Although CCBs are fairly selective for cardiomyocytes and vascular smooth muscle, they do have limited activity at GI smooth muscle, which usually manifests as constipation. Their vascular effects in the brain can cause headaches. Excessive hypotension may manifest as generalized fatigue or lead to reflex tachycardia. Peripheral edema and flushing, which are caused by peripheral vasodilation, are commonly tested side effects. Gingival hyperplasia is also an important side effect of CCBs.

SUMMARY BOX: CALCIUM CHANNEL BLOCKERS • Indications: Antihypertensive, heart rate control, antiarrhythmic, prevention of cerebral vasospasm in subarachnoid hemorrhage • Mechanism of action: Block the L-type calcium channel, thus decreasing the initial influx of calcium into the cell and reducing both cardiac contractility as well as peripheral vascular resistance • Side effects: Peripheral edema, flushing, and gingival hyperplasia • Toxicity: Profound bradycardia and hypotension (especially with verapamil and diltiazem)

Case 23.6 An otherwise healthy 35-year-old man is brought to the ED by his coworker, who found him to be excessively drowsy and slurring his speech when he showed up to work this morning. The friend mentions that the patient’s past medical history is notable for anxiety and insomnia, for which he was recently prescribed “some medication.” There is no history of trauma, neurologic or metabolic anomalies, or alcohol/drug abuse. On physical examination, the patient has a heart rate of 55 beats/min, BP of 120/70 mm Hg, RR of 9 breaths/min, and temperature of 37°C. The remainder of the examination is unremarkable.   

546  Pharmacology and Toxicology 1. Given the preceding presentation, which types of items are at the top of the differential diagnosis and would be worth exploring?   This patient presents in a generally depressed cognitive state, and his vital signs are similarly somewhat diminished. The process appears to be acute in onset, but there is no history of trauma, and the patient is afebrile. This history immediately moves injuries, infections, and chronic processes down the differential list and shifts other items such as alcohol intoxication, hypoglycemia, and medication involvement up the list. History also reveals that he was recently started on pharmacologic therapy for his anxiety and insomnia. Medications typically used for this purpose tend to be general depressants (e.g., Xanax) that, if misused, could precipitate a presentation similar to this one. This patient should have his blood glucose checked as well as his levels of alcohol and other depressants (e.g., barbiturates and benzodiazepines). 2. What is the mechanism of action of benzodiazepines?   Benzodiazepines bind sites on the cell membrane that are adjacent to but separate from γ-aminobutyric acid (GABA) receptors. Their presence enhances the affinity that GABAA receptors have for their ligand, GABA. (Recall that increased affinity between receptor and substrate leads to a decreased Km value—benzodiazepines are a commonly used example of this pharmacodynamic principle!) This translates into a higher frequency of GABA-GABAA receptor interaction, and therefore more frequent opening of chloride channels in the cell membrane. The chloride influx hyperpolarizes the cell, making it “more difficult” to reach the firing threshold. The end effect is that benzodiazepines enhance the actions of a major inhibitory neurotransmitter in the CNS—namely, GABA—and bring about depressive effects overall.     It is important to note that benzodiazepines themselves do not cause the chloride channel to open—this can only be done if GABA is already present and bound to the GABAA receptor. Benzodiazepines facilitate this process, but their effects are limited by the amount of GABA that is released. Since there is a finite supply of GABA in the body, the effects of benzodiazepines usually plateau before life-threatening respiratory depression, coma, or death ensues. This is not to say that respiratory depression does not occur with benzodiazepines, but it is less likely when compared to drugs such as barbiturates or ethanol. As such, benzodiazepines are considered to be “safer” drugs.     Incidentally, if this patient were a woman, and if the history were more fitting, then intoxication with a different ­GABAergic substance would be very high on the differential list. The name of this agent is γ-hydroxybutyrate (GHB), and it is a metabolite of GABA. GHB acts as a fast-acting sedative-hypnotic that is a popular choice as a drug of abuse and has also been implicated as a date rape drug. The sedative effects of benzodiazepines are additive with other CNS depressants, so they can have dangerous effects if taken with alcohol, barbiturates, first-generation antihistamines, or other depressants.

STEP 1 SECRET It is important to note that benzodiazepines increase the affinity of the γ-aminobutyric acid A (GABAA) receptor for GABA. Activation of the GABAB receptor causes opening of a nearby potassium (not chloride) channel, leading to K+ efflux and cellular hyperpolarization. Baclofen (an antispasmodic) is a notable example of a drug that binds the GABAB receptor. 3. What are a few clinical indications for using benzodiazepines?   See Table 23.3.

Table 23.3.   Benzodiazepines INDICATION/USE

COMMENTS

Anxiolytic

Alprazolam (Xanax) helps calm patients with intense fear of flying before boarding plane

Sedative-hypnotic

Induces sleep

Anticonvulsant

Diazepam and lorazepam can terminate seizures and are used for treatment of status epilepticus Clonazepam may be used for long-term treatment of epilepsy

Alcohol withdrawal

Chlordiazepoxide, diazepam, and oxazepam can be used in acute withdrawal

   

One means of categorizing benzodiazepines is based on their duration of action (Table 23.4).

Table 23.4.   Categories of Benzodiazepines Based on Duration of Action Short-acting

Triazolam, oxazepam (Serax), midazolam (Versed)

Intermediate-acting

Clonazepam (Klonopin), alprazolam (Xanax), temazepam (Restoril), lorazepam (Ativan), estazolam

Long-acting

Chlordiazepoxide (Librium), diazepam (Valium)

Pharmacology and Toxicology  547

4. What are some common adverse effects of benzodiazepines?   Given that benzodiazepines are generally depressants, it is no surprise that they can produce oversedation, predisposing the patient to falls, fractures, or work injuries; cause cognitive impairment (e.g., memory loss); exacerbate respiratory problems (e.g., emphysema); and lead to dependence. Given the risk of dependence, benzodiazepines should not be prescribed for prolonged durations.     Benzodiazepine intoxication can be reversed with flumazenil (a GABAA receptor competitive antagonist); withdrawal should be treated symptomatically using long-acting benzodiazepines. 5 What are the symptoms of benzodiazepine intoxication and withdrawal?   See Table 23.5. Table 23.5.   Benzodiazepine Intoxication and Withdrawal Intoxication

Slurred speech, drowsiness, decreased respiratory rate and tidal volume, bradycardia

Withdrawal

Shaking, diaphoresis, anxiety, irritability, insomnia, cardiac palpitations, painful abdominal cramps

SUMMARY BOX: BENZODIAZEPINES • Indications: Anxiolytic, antiepileptic, sedative (for insomnia), and treatment of alcohol withdrawal • Mechanism of action: Enhances the affinity of γ-aminobutyric acid A (GABAA) receptors for GABA, resulting in chloride influx that hyperpolarizes the cell and leads to depressive effects • Side effects: Oversedation, memory loss, and dependence • Toxicity • If taken alone, effects of overdose are minimal (drowsiness). • Respiratory depression is more likely if taken in combination with barbiturates or ethanol. • Antidote: Flumazenil, a GABAA receptor competitive antagonist

Case 23.7 A 48-year-old woman is brought to the ED by her son, who found her to be unarousable this morning from last night’s sleep. There is no history of trauma or drug abuse. Her past medical history is notable for insomnia, for which she was started on secobarbital recently. Her son brought the pill bottle to the ED with him, and all the pills are accounted for. She takes no other medication and has no known allergies. When questioned, the patient’s son notes that his mother did have a few beers last night, although she was not overtly intoxicated. Physical examination reveals a somnolent woman with slurred/unintelligible speech, constricted pupils, diminished deep tendon reflexes, a heart rate of 48 beats/min, a BP of 100/60 mm Hg, and an RR of 8 breaths/min.   

1. Given this clinical picture, what scenario best explains this patient’s presentation?   This patient has a history of insomnia and was therefore recently started on a barbiturate (secobarbital). To this, she added another depressant (alcohol) and is now presenting with CNS, respiratory, and cardiac depression.     Although alkalinizing her urine using intravenous bicarbonate may help rid the body of secobarbital (barbiturates are weak acids—see Basic Concepts, question 3), there is no specific antidote for barbiturates. In this case, the best approach is to monitor her respiration (and intubate if warranted) and prevent cardiovascular collapse (give intravenous fluids and possibly administer inotropics/vasopressors such as dopamine or norepinephrine). 2. What is the mechanism of action of barbiturates?   Although their binding site is different, barbiturates act very similarly to benzodiazepines in that they potentiate the effect of GABA on the chloride channel. The result is a hyperpolarized cell that is less excitable. However, instead of increasing the frequency of chloride channel openings as benzodiazepines do, barbiturates increase the duration for which the channel is open. It is critical to remember this distinction!     In addition, barbiturates also diminish activity of the excitatory neurotransmitter glutamate. Barbiturates do this by blocking a type of glutamate receptor that is found almost exclusively in the CNS. Although its actual name is much longer (and relatively unimportant), this receptor is commonly referred to by the acronym AMPA. 3. What are some common indications for using barbiturates?   Although barbiturates have largely been replaced by benzodiazepines because they have a better side effect profile and less potential for abuse, barbiturates continue to have uses in some clinical settings. For example, they are still used in induction of anesthesia, as anticonvulsants to treat seizures, and as anxiolytics, and they have also been useful in treating alcohol withdrawal and insomnia.     One means of categorizing barbiturates is based on the duration of action—long-acting, short-acting, or ultrashortacting. Phenobarbital is a long-acting agent that can be used to treat seizures on a long-term basis; pentobarbital is

548  Pharmacology and Toxicology short-acting and used as a sedative-hypnotic; thiopental is ultrashort-acting and used to induce anesthesia. Thiopental is highly lipid-soluble and will quickly diffuse out of the bloodstream and into adipose tissue (thus lowering its plasma concentration). 4. What are some common adverse effects of barbiturates?   Similar to benzodiazepines, barbiturates are generally depressants. At the level of the CNS, this manifests as drowsiness. In cases of overdose, respiratory depression due to blockade of the body’s response to hypoxia/hypercapnia is also a very real risk. Barbiturates are much more likely to cause respiratory depression than the similarly acting benzodiazepines for reasons explained earlier in this chapter (see Case 23.6, question 2). At toxic doses, barbiturates can also cause severe bradycardia to the point of causing a shock-like condition. In contrast with benzodiazepines, there is no pharmacologic treatment for barbiturate overdose. Treatment consists of supportive care and symptom management.     As noted earlier, barbiturates do have hypnotic/anxiolytic effects that can precipitate dependence and abuse. On withdrawal, symptoms such as tremors, anxiety, seizures, delirium, and cardiac arrest can result. This ability to cause death with both intoxication and withdrawal is a feature that really sets barbiturates and benzodiazepines apart from other drugs of abuse and is all the more reason not to combine the two with other general depressants such as alcohol.     A noteworthy complication of barbiturate use is their exacerbation of porphyria in susceptible individuals. This occurs because barbiturates induce the activity of d-ALA synthase, which is the rate-limiting enzyme in the heme synthesis pathway. In individuals with porphyria, this causes an increased accumulation of heme precursors and a flare of porphyria symptoms. Many cytochrome P-450 inducers are capable of doing this, but barbiturates are by far the most commonly tested of the bunch.

STEP 1 SECRET Barbiturates and benzodiazepines are commonly encountered drugs on Step 1. You should know their mechanisms of action, clinical uses, and adverse effects and treatment for overdose.

SUMMARY BOX: BARBITURATES • Indications: Induction of anesthesia, anticonvulsant, anxiolytic, and treatment of alcohol withdrawal • Mechanism of action: Modulate γ-aminobutyric acid (GABA) receptors to increase the duration of chloride channel opening, which hyperpolarizes the cell and decreases excitability • Side effects: Drowsiness • Toxicity: Respiratory depression, severe bradycardia. Contraindicated in porphyria due to their induction of d-ALA synthase in the heme synthesis pathway • Antidote: None • Can provide supportive care and sodium bicarbonate to alkalinize the urine

Case 23.8 You are on call overnight, and are paged about a 54-year-old patient who is reportedly having a grand mal seizure. The patient underwent an uncomplicated emergent appendectomy approximately 72 hours earlier. Over the last 48 hours, the nursing staff reports the patient to have deteriorated from being fairly pleasant soon after his operation to being quite anxious, diaphoretic, tremulous, and unable to sleep. More recently, the patient is said to have been experiencing visual and auditory hallucinations. His past medical history is notable for a simple hand fracture sustained during a bar fight; he is not currently on any medications, nor does he have any allergies. He has no history of prior seizures. He lives alone and is unemployed and twice divorced. A comparison of his day-to-day vital signs shows that his heart rate, BP, and temperature have been progressively increasing over the last 48 hours.   

1. Given his recent uncomplicated hospital course and his benign past medical history, what condition is this patient likely experiencing?   This patient has no history of epilepsy and is not taking any medications that would predispose him to seizures. On the other hand, he has several risk factors for depression and substance abuse (e.g., living alone, inability to hold a job or establish close relationships). The most likely cause of his seizure is alcohol withdrawal syndrome, which usually presents 2 to 3 days after a chronic alcohol abuser stops his intake of alcohol. Symptoms of withdrawal can range from insomnia and tremulousness to severe complications such as seizures and even delirium tremens (visual/tactile/auditory hallucinations, autonomic hyperactivity, disorientation, agitation, nightmares, and uncontrollable tremors).

Pharmacology and Toxicology  549

2. What are the symptoms of acute alcohol toxicity, and how can these effects be explained at the molecular level?   Alcohol has a depressive effect on the brain. It impairs motor function, cognition, judgment, speech, and respiration, and it disinhibits behavior.     At the molecular level, think of the brain as being a teeter-totter that is balanced by an inhibitory neurotransmitter (GABA) at one end and an excitatory neurotransmitter (glutamate) at the other end. Alcohol enhances the effects of GABA on the GABAA receptor and blunts the effects of glutamate on the NMDA receptor (thus weighing the teeter-totter in favor of a net inhibitory effect).     If exposure to alcohol is chronic, then the brain will enact certain compensatory measures in an attempt to reestablish balance. On the GABA end, the brain will downregulate the number of GABAA receptors, and on the glutamate end, the brain will upregulate the number of NMDA receptors. Not only is more alcohol needed to attain the same degree of inhibitory effect (i.e., tolerance), but if alcohol exposure is halted suddenly, then there will be pronounced hyperexcitability due to the increased number of NMDA receptors (i.e., withdrawal). 3. What are symptoms of chronic alcohol abuse?   Hepatic manifestations are fairly common with chronic exposure and include fatty liver, hepatitis, cirrhosis, and liver failure. Other findings include pancreatitis, nutritional deficiencies, peripheral neuropathy, and cerebellar degeneration. 4. What is the relationship between alcohol and benzodiazepines in terms of their effect on the brain?   They both enhance the effects of GABA on the GABAA receptor. As described earlier, if the duration of exposure to alcohol is prolonged, then the number of GABAA receptors will be downregulated to reestablish homeostasis. As a result, just as a chronic abuser of alcohol builds a tolerance to alcohol so that a higher dose is required to achieve the same effect, the same patient will require a higher than normal dose of benzodiazepines to achieve sedation in a medical setting (i.e., cross-tolerance).     The notion of cross-reactivity between alcohol and benzodiazepines can be used therapeutically to manage alcohol withdrawal. For example, an intermediate-acting benzodiazepine (e.g., lorazepam) can be used as a substitute for alcohol to ameliorate the hyperexcitable state that is characteristic of alcohol withdrawal. Once the teeter-totter is balanced, the intermediate-acting benzodiazepine can be tapered slowly or replaced with a longacting benzodiazepine (e.g., chlordiazepoxide) that has less abuse potential. Eventually, the number of GABAA and NMDA receptors will be recalibrated to levels present before alcohol exposure, and the benzodiazepines can be stopped altogether. 5. How is alcohol metabolized in the body?   Most of the metabolism occurs in the liver, where alcohol is oxidized by alcohol dehydrogenase (ADH) to acetaldehyde, which in turn is oxidized by aldehyde dehydrogenase (aldehyde-DH) to acetate (Fig. 23.2).     Disulfiram is a drug used to manage alcoholism. It inhibits aldehyde-DH, thus causing accumulation of acetaldehyde. Acetaldehyde, in turn, causes nausea, vomiting, severe headaches, and flushing. These negative effects are intended to discourage further alcohol use.     Certain medications can also inadvertently inhibit aldehyde-DH, causing a “disulfiram-like reaction” with the aforementioned side effects. As such, patients are instructed not to consume alcohol while taking such drugs. This is a commonly tested feature of the antibiotic metronidazole. Other medications such as griseofulvin and firstgeneration sulfonylureas also have this effect, although this is less commonly tested. Polymorphisms in acetaldehyde dehydrogenase result in accumulation of acetaldehyde and are particularly common among people of Asian descent.

Alcohol dehydrogenase (ADH)

Alcohol (Ethanol) H3C-CH2-OH

Aldehyde dehydrogenase (ALDH)

Acetaldehyde H3C-CH=O Figure 23.2.  Metabolism of Alcohol

Acetate (Acetic acid) H3C-COO

550  Pharmacology and Toxicology

SUMMARY BOX: ALCOHOL • Indications: Drug of abuse • Mechanism of action: Enhances the effects of γ-aminobutyric acid (GABA) on the GABAA receptor and blunts the effects of glutamate on the N-methyl-d-aspartate (NMDA) receptor, leading to depressive symptoms • Withdrawal: Usually 2 to 3 days after a chronic abuser stops intake; symptoms include insomnia, tremulousness, seizures, and delirium tremens. • Toxicity: Chronic exposure can damage the liver, pancreas, and nervous system and can also lead to nutritional deficits. • Management • Benzodiazepines are used to manage withdrawal. • Disulfiram can be used to manage alcoholism by inhibiting aldehyde-DH and causing unpleasant symptoms. • Several drugs (e.g., metronidazole) can have a disulfiram-like effect if alcohol is consumed.

Case 23.9 A 52-year-old woman is brought to the ED by ambulance 3 hours after an apparent suicide attempt by ingestion of medication. Her symptoms include nausea, vomiting, and right upper quadrant pain. Her past medical history is positive for depression and chronic alcohol abuse. Her physical examination reveals normal vital signs and some mild tenderness in her right upper quadrant. A serum panel is positive for acetaminophen, but levels are below the toxic threshold.   

1. Given this presentation, which other laboratory values would prove informative? Should this patient be treated based on the information provided?   Acetaminophen is metabolized by the liver, and one of its byproducts (NAPQI) is toxic to hepatocytes. Therefore, any time a toxic level of this drug is ingested, liver necrosis is a real danger. Fortunately, NAPQI can be inactivated by glutathione, which is an antioxidant regenerated in the body by the enzyme glutathione reductase. However, patients whose reserves of glutathione are decreased (e.g., alcoholics, diabetics) or those who ingest massive quantities of acetaminophen that overwhelm the glutathione system are at risk for hepatocyte damage by NAPQI. To assess the degree of damage to the liver, levels of liver function enzymes and coagulation panels are typically acquired. Although this patient’s serum level of acetaminophen is below the toxic threshold, given her past history of alcohol abuse, it is wise to treat her with N-acetylcysteine to prevent liver failure. Recall that N-acetylcysteine replenishes glutathione supply. 2. What is the mechanism of action of acetaminophen?   Central (CNS) COX pathway inhibition is the mechanism of action. Cell membranes are composed of fatty acids, and one of these fatty acids is arachidonic acid. When the membrane is damaged, arachidonic acid begins to be broken down along one of two pathways—the COX pathway or the LOX pathway. The COX pathway results in prostaglandin production; the LOX pathway produces leukotrienes.     Prostaglandins are among a handful of molecules that mediate inflammation, pain, and fever. If the objective is to decrease inflammation, pain, and fever, then inhibiting the COX pathway is a good start. Acetaminophen reversibly inhibits this pathway. Ibuprofen and aspirin (both NSAIDs) act on the same enzyme, COX, albeit peripherally. Steroids (e.g., hydrocortisone, prednisone) inhibit a different enzyme, phospholipase A2, which plays a role earlier along this same pathway.     Although it inhibits COX, acetaminophen is not considered an NSAID. Acetaminophen acts centrally (in the CNS) to inhibit prostaglandin synthesis by COX, whereas NSAIDs inhibit the same pathway in peripheral tissues. This difference in location accounts for the analgesic/antipyretic effects of acetaminophen and the antiinflammatory effects of NSAIDs.

SUMMARY BOX: ACETAMINOPHEN • Indications: Pain and fever reduction • Mechanism of action: Reversibly inhibits the cyclooxygenase (COX) pathway in the central nervous system (CNS), leading to decreased prostaglandin synthesis • Toxicity: Liver necrosis • Antidote: N-acetylcysteine (replenishes glutathione supply)

Case 23.10 A 74-year-old man with Alzheimer’s disease is brought to the ED by ambulance. His grandson had found him on the bathroom floor, along with a half-empty bottle of his “Alzheimer’s medication.” The patient apparently ingested the medication within the last 3 hours because this was the last time his grandson saw the patient before finding him unconscious. He reports his grandfather to have vomited a few times before being found, and several pills could be seen in the vomitus. The patient’s other symptoms include diaphoresis, hypersalivation, and urinary incontinence. Physical examination is notable for a low heart rate and RR, mydriasis, and fasciculations.   

Pharmacology and Toxicology  551

1. Given this patient’s presentation, which group of medications is the likely culprit?   Alzheimer’s disease is thought to be due in part to a deficiency of acetylcholine (ACh) in the CNS. One way of managing an ACh deficiency is to employ a group of medications generally referred to as cholinergics. In the case of Alzheimer’s disease specifically, this means using drugs that are able to cross the blood-brain barrier and block acetylcholinesterase (AChE), thus decreasing the rate of breakdown of ACh. A few AChE inhibitors used in this way include donepezil, rivastigmine, and tacrine.     However, if AChE inhibitors are used in excessive amounts, ACh levels can reach toxic proportions. This in turn can cause more global effects due to the interaction of ACh with nicotinic receptors (fasciculations, muscle weakness), as well as muscarinic receptors (e.g., nausea, vomiting, diarrhea, urinary incontinence, mydriasis, diaphoresis, hypersalivation, bradycardia, hypotension).     Using an anticholinergic agent such as atropine to reverse these effects is a reasonable therapeutic approach. 2. In general terms, how is the nervous system organized?   The nervous system can be divided according to function (sensory/motor) or anatomy (central/peripheral). For discussion of anticholinergics, it is best to adopt the former approach and consider only the motor half of the nervous system.     The motor branch can be further categorized into autonomic and somatic nervous systems (ANS and SNS, respectively). The SNS and ANS can also be viewed in terms of either anatomy or function. In terms of function, the SNS is the voluntary portion of the nervous system, whereas the ANS is the involuntary portion. In terms of anatomy, the SNS is relatively simple in that a single neuron leaves the CNS and travels directly to the target organ, where it delivers ACh to a nicotinic receptor that is located on striated muscle.     In terms of anatomy, the ANS is a bit more specialized than the SNS in that instead of relying on a single neuron, it involves two neurons connecting the CNS to target organs. These two neurons are connected to each other via a synapse. The specifics of this synapse are easy to remember because its anatomy is very similar to that of the SNS; that is, it always involves delivery of ACh to a nicotinic receptor.     The anatomic feature that really sets the ANS apart from the SNS is how the postsynaptic neuron delivers the message from the synapse to the target organ. In fact, the ANS is divided into three groups based on the anatomy of this second neuron: 1. Parasympathetic division delivers ACh to muscarinic receptors. 2. Sympathetic division delivers norepinephrine to adrenergic receptors. 3. Third division is actually partially endocrine and delivers epinephrine (from the adrenal medulla) to an adrenergic receptor. Because this division also involves a catecholamine as its primary neurotransmitter, it is usually considered part of the sympathetic nervous system. The organization of the sensory and motor components of the nervous system is outlined in Table 23.6.

Table 23.6.   Organization of the Nervous System SENSORY

MOTOR

Autonomic nervous system (ANS)

Sympathetic nervous system (SNS)

Involuntary

Voluntary

2 neurons/1 synapse

1 neuron/no synapse

Synapse = ACh → Nct

ACh → Nct (at NMJ)

PARASYMPATHETIC

ACh → M

SYMPATHETIC

NE → Adr

ENDOCRINE

Epi (from adrenals) → Adr

ACh, acetylcholine; Adr, adrenergic; Epi, epinephrine; M, muscarinic; Nct, nicotinic receptor; NE, norepinephrine; NMJ, neuromuscular junction.

3. Which neurotransmitter can be said to be pivotal to the function of the entire motor nervous system?   Because it is the lone neurotransmitter in the SNS and thus the only means of connecting the two neurons in a typical ANS pathway, ACh is pivotal to the function of the motor nervous system. If ACh release is hindered (as occurs in botulinum toxicity), the entire motor nervous system can be blocked, thus producing flaccid paralysis. 4. Is there a way to selectively affect the parasympathetic nervous system?   Yes, the PNS can be selectively affected by stimulating/inhibiting muscarinic receptors, which are exclusive to this branch of the ANS. The two most common stimulants include bethanechol (used to induce urination in nonobstructive urinary retention) and pilocarpine (used to reduce intraocular pressure in glaucoma).     The two most common antimuscarinics are atropine and ipratropium. Atropine drips can be used in the eye to induce mydriasis (cholinergic input into the eye causes miosis or pinpointing; if this function is blocked by

552  Pharmacology and Toxicology antimuscarinics, then sympathetic input goes unchecked to induce mydriasis or dilation). Atropine is typically used during evaluations for corrective lenses. At high doses, atropine causes tachycardia by blocking muscarinic receptors at the SA node. Ipratropium is a derivative of atropine and comes in an inhaled form that is used to treat asthma and chronic obstructive pulmonary disease (COPD).     Several other drugs have antimuscarinic side effects that mimic the side effects of atropine. These drugs include antihistamines, antipsychotics, quinidine, and tricyclic antidepressants (TCAs). TCAs are particularly notable for their antimuscarinic effects and classic “three C’s” triad—coma, cardiotoxicity (torsades de pointes), and convulsions (seizures).

STEP 1 SECRET Antimuscarinic/atropine toxicity is commonly tested on Step 1. Symptoms of atropine toxicity can be remembered with the mnemonic “Hot as a hare, dry as a bone, red as a beet, blind as a bat, mad as a hatter, and full as a flask,” which describes increased body temperature (hot), dry skin due to decreased sweating (dry), flushed skin (red), cycloplegia or inability to accommodate vision for close-up objects (blind), disorientation secondary to atropine entering the CNS (mad), and urinary retention (full). The antidote for atropine toxicity is physostigmine, an acetylcholinesterase inhibitor that causes an increased concentration of ACh to displace atropine from muscarinic receptors. Physostigmine is the only AChE inhibitor capable of crossing the blood-brain barrier and alleviating the CNS side effects of antimuscarinic drugs; other AChE inhibitors such as neostigmine are not used as antidotes for antimuscarinic toxicity.

5. What is one way to reduce the side effects of a drug that stimulates both nicotinic and muscarinic receptors?   By limiting its physical distribution in the body, one can localize the effects of a drug that would otherwise act on a global level. An example of this approach is using carbachol eye drops to reduce intraocular pressure. 6. Are there nicotinic receptor blockers that are selective for the autonomic nervous system or somatic nervous system rather than blocking the entire motor nervous system?   Yes. Nicotinic receptor antagonists that are selective for the ANS (those receptors located in synapses) are called ganglionic blockers. Unfortunately, their effect is still too expansive to serve an effective therapeutic role. An example of such a blocker is nicotine, which initially depolarizes the postsynaptic neuron (resulting in hypertension (HTN), tachycardia, and increased peristalsis) before blocking the nicotinic receptors (which causes a drop in BP, heart rate, and GI motility).     Nicotinic receptor blockers that are selective for receptors located on skeletal muscle (i.e., are SNS-selective) are called neuromuscular blocking agents. There are two classes of neuromuscular blockers—depolarizing and nondepolarizing (Fig. 23.3). Nondepolarizing neuromuscular blockers compete with ACh (i.e., they are competitive antagonists). The parent compound for nondepolarizing blockers is curare, and as a result, drugs in this class have some variation of this word incorporated into their names (e.g., tubocurarine). These agents are typically used as general anesthetics to achieve skeletal muscle relaxation during surgical procedures. Their effects can be overcome by increasing the concentration of their competitor (ACh). This can be achieved with AChE inhibitors (Table 23.7), particularly neostigmine because it cannot cross the blood-brain barrier and thus reduces any risk of CNS effects. Table 23.7.   Acetylcholine Agonists and Antagonists CATEGORY

MUSCARINIC RECEPTORS

NICOTINIC RECEPTORS

Agonist

Bethanechol, pilocarpine Carbachol

Nicotine

Antagonist

Atropine, ipratropium

Nicotine (a ganglionic blocker), succinylcholine (a depolarizing neuromuscular junction blocker), tubocurarine (a nondepolarizing neuromuscular junction blocker)

    Depolarizing neuromuscular blockers bind the sodium ion channel at the neuromuscular junction (NMJ), thereby prolonging depolarization and preventing the myocyte from repolarizing. The net effect of this inability to repolarize is flaccid paralysis. The only depolarizing blocker you need to know for Step 1 is succinylcholine, which is employed in brief procedures (e.g., endotracheal intubations just before surgical procedures). Unlike nondepolarizing neuromuscular blockers, a succinylcholine NMJ block occurs in two phases. During Phase I, the ACh receptor channel undergoes “prolonged depolarization,” and the block cannot be reversed. In fact, giving cholinesterase inhibitors will only cause continued depolarization and will prolong the paralytic effects of succinylcholine. Succinylcholine can only be reversed during Phase II, at which point the channel has repolarized but is still blocked by succinylcholine. At this point, AChE inhibitors can be administered and the block can be reversed. Certain individuals can have an autosomal recessive pseudocholinesterase deficiency, meaning that they metabolize succinylcholine at a much slower rate. These individuals can remain paralyzed for hours. (A normal dose of succinylcholine should only paralyze a

Pharmacology and Toxicology  553 Depolarizing block (Succinylcholine) Normal responses (No drugs present)

Nondepolarizing block

Phase I

Phase II

1 4

TOF 0.9: needed for safe extubation & recovery after surgery Figure 23.3.  Neuromuscular blockade.

typical individual for about 5 minutes.) Such patients can remain in Phase I for prolonged periods of time, so it is especially important not to administer AChE inhibitors until it is certain that they have reached Phase II. This can be determined by administering nerve stimulation four consecutive times and evaluating the muscle responses (“train of four” responses). In Phase I, all four responses should have the same amplitude, which will increase over time. Once in Phase II, the muscle responses demonstrate a “fading” pattern (this fading pattern is also seen at all times with nondepolarizing agents). The Phase II pattern indicates it is now safe to administer AChE inhibitors to reverse the NMJ blockade. Giving AChE inhibitors too early can prolong paralysis even further and possibly lead to respiratory failure and death. 7. How can a cholinergic drug help diagnose myasthenia gravis?   Myasthenia gravis is a neuromuscular disease marked by autoantibodies that occupy ACh receptors at the NMJ (type II hypersensitivity reaction). The typical patient tends to fatigue unusually quickly with increasing activity. The reason for this is that the autoantibodies that occupy ACh receptors prevent the neurotransmitter from contacting its target, and ACh is degraded by AChE before it has a chance to displace the autoantibody. Ptosis and diplopia are common initial findings. Weakness in proximal muscles and the diaphragm often follows. Patients may also present with dysphagia to solids and liquids.     With use of a fast-acting AChE inhibitor such as edrophonium, the exposure time of the receptor to ACh is increased, and the autoantibodies are therefore displaced. A completely fatigued patient may then momentarily regain his or her strength and energy. This is in contrast to a genuinely fatigued patient, who will not respond because the muscle itself is fatigued. This test, called the Tensilon test, is used to diagnose myasthenia gravis. Pyridostigmine is used for long-term treatment of myasthenia gravis.

STEP 1 SECRET Myasthenia gravis is a popular Step 1 topic. Be on the lookout for patients who experience muscle fatigue with increasing use (in contrast with Lambert-Eaton syndrome). Note that myasthenia gravis is also associated with thymoma, which may be presented to you on a chest x-ray.

SUMMARY BOX: CHOLINERGICS/ANTICHOLINERGICS • Muscarinic receptors are exclusive to the parasympathetic branch of the autonomic nervous system (ANS). • Botulinum toxin inhibits release of ACh, thus causing flaccid paralysis. • Muscarinic agonists: Bethanechol (induces urination) and pilocarpine (reduces intraocular pressure) • Muscarinic antagonists: Atropine (induces mydriasis, causes tachycardia at high doses) and ipratropium (bronchodilator) • Nondepolarizing neuromuscular blockers (“curares”): Competitive ACh antagonists (skeletal muscle relaxation during surgical procedures) • Antidote: Acetylcholinesterase (AChE) inhibitors (e.g., neostigmine or edrophonium) • Depolarizing neuromuscular blockers: Succinylcholine (flaccid paralysis for brief procedures, i.e., intubation) • Antidote: AChE inhibitors only after nerve stimulation demonstrates a “fading” pattern (Phase II pattern). Giving AChE inhibitors too early will prolong paralysis.

554  Pharmacology and Toxicology

Case 23.11 A 22-year-old man is brought to the ED by his parents, who report that he recently ingested his father’s antihypertensive medication in an apparent suicide attempt. His past medical history is notable for depression and asthma. On physical examination, the patient is bradycardic, hypotensive, tachypneic, and in respiratory distress. The patient’s mental status is also depressed.   

1. Given this presentation, to which group of antihypertensives does the likely culprit belong?   Very few antihypertensive medications precipitate respiratory crisis in a patient with a history of asthma. From that standpoint, the drug in question is likely a beta-blocker (more on beta-blockers later). Management of this patient requires intravenous fluid, adrenergic agents, and inotropic/chronotropic drugs that bypass the beta receptors altogether and work to restore heart rate and BP. If this patient’s respiratory crisis is unresponsive to beta-agonists, endotracheal intubation may be warranted as well. 2. In the sympathetic nervous system, what are the two types of neurotransmitters and the two main adrenergic receptors?   The two main neurotransmitters are norepinephrine (NE) and epinephrine (Epi). Both are derived from the amino acid tyrosine (Fig. 23.4).     The two main types of receptors in the sympathetic nervous system are α-adrenergic receptors and β-adrenergic receptors. Dopaminergic receptors also exist but are not the predominant subtype. 3. Describe alpha receptors in terms of their distribution in the body and a few of their agonists/ antagonists.   There are actually two types of α-adrenergic receptors: α1 and α2.     α1-Receptors are located in the radial muscle of the eye (causes mydriasis), in the smooth muscle of the vasculature (causes vasoconstriction), and in the penis (causes ejaculation). An example of an α1-agonist is phenylephrine, which is used to treat nasal congestion by inducing vasoconstriction. An example of an α1-blocker is prazosin, which is used to treat urinary tract obstruction in benign prostatic hyperplasia (BPH) and hypertension by causing smooth muscle relaxation.

Dopa decarboxylase

Tyrosine hydroxylase NH2 HO

COOH ρ-tyrosine

NH2

HO HO

COOH DOPA

NH2 COOH N H Tryptophan

HN

N

NH2

COOH

Histidine

Dopamine β-hydroxylase NH2

HO HO

HO

COOH N H 5-hydroxytryptophan NH2 HN

OH

NH2

HO Norepinephrine

Dopamine

NH2

HO

PhenylethanolamineN-methyltransferase HO

OH H N CH3

HO Epinephrine

NH2 HO N H 5-hydroxytryptamine

N Histamine

Choline acetyltransferase O || + + HO–CH2–CH2–N(CC3)3+CH3CO–CoA CH3–C–O–CH2–N(CH3)3+CoA Choline ACh Glutamate decarboxylase COOH | HOOC–CH2–CH2–CH2–NH2 HOOC–CH2–CH2–CH | GABA Glutamate NH2

Figure 23.4.  Synthesis of neurotransmitters. Pathways of synthesis of neurotransmitters are simple. (From Baynes J, Dominiczak M. Medical Biochemistry. 4th ed. Philadelphia: Saunders; 2014.)

Pharmacology and Toxicology  555

    α2-Receptors are located centrally, and stimulating them actually inhibits norepinephrine release from synaptic vesicles. As a result, these receptors can be regarded as part of the inhibitory arm of the sympathetic nervous system. α2-Agonists include clonidine and α-methyldopa. Clonidine is used to treat severe HTN, nicotine withdrawal, heroin withdrawal, alcohol dependence, and migraines. α-Methyldopa is used to treat HTN. An example of an α2-blocker is mirtazapine, which is used in the treatment of depression. By inhibiting the inhibitory arm of the sympathetic nervous system, it increases catecholamine activity (Table 23.8). Table 23.8.   α-Adrenergic Receptors CATEGORY

α1-RECEPTORS

α2-RECEPTORS

Agonists

Phenylephrine

Clonidine, α-methyldopa

Antagonists

Prazosin, terazosin Phentolamine, phenoxybenzamine

Mirtazapine

4. Describe beta receptors in terms of their distribution in the body and a few of their agonists/ antagonists.   There are also two types of β-adrenergic receptors: β1 and β2.     β1-Receptors are located in the heart, and stimulating them increases cardiac output by increasing conduction velocity and contractility. An agonist selective for this receptor is dobutamine, which is used to increase cardiac output in CHF. Blocking these receptors decreases cardiac output and hence BP. A few antagonists selective for this receptor are metoprolol, esmolol, and atenolol.     β2-Receptors are located in the lungs, and stimulating them causes dilation of the bronchi. An example of a selective agonist is albuterol, which is used in the management of asthma. Blocking these receptors can constrict the airways and precipitate an asthma attack in those predisposed to such an event. Because there is little therapeutic advantage to selectively blocking these receptors, there are few such agents available. The drugs that do happen to block these receptors do so as a side effect of their nonselective beta-blocking activity. These agents include timolol, nadolol, and propranolol, and they all are contraindicated in asthmatic patients (Table 23.9). Table 23.9.   β-Adrenergic Receptors CATEGORY

β1-RECEPTORS

β2-RECEPTORS

Agonists

Dobutamine Isoproterenol

Albuterol

Antagonists

Metoprolol, esmolol, atenolol Timolol, nadolol, propranolol

5. What antidote should be given to this patient?   Glucagon is a must-know antidote for beta-blocker toxicity—it is commonly asked on Step 1. Glucagon increases heart rate, contractility, and AV conduction possibly via a mechanism that bypasses the beta-receptor.

SUMMARY BOX: BETA BLOCKERS • α1-Receptors: Phenylephrine is an α1-agonist (used to treat nasal congestion) and prazosin is an α1-blocker (used to treat hypertension and benign prostatic hyperplasia [BPH]). • α2-Receptors: Clonidine (used to treat severe hypertension, nicotine withdrawal, heroin withdrawal, alcohol depen­ dence, and migraines) and α-methyldopa (used to treat hypertension) are α2-agonists. Mirtazapine is an α2-blocker (used in the treatment of depression). • β1-Receptors: Dobutamine is a β1-agonist (used to manage CHF). Metoprolol, esmolol, and atenolol are β1-blockers (used to treat hypertension). • β2-Receptors: Albuterol is a β2-agonist (used to manage asthma). Timolol, nadolol, and propranolol are nonselective beta-blockers, which are contraindicated in asthmatic patients because they can precipitate an asthma attack. • Antidote: Glucagon is a must-know antidote to beta-blocker toxicity.

556  Pharmacology and Toxicology

Case 23.12 A 40-year-old farmer arrives at the ED complaining that he cannot breathe. Physical examination reveals pulse of 48 beats/min and BP of 94/58 mm Hg. He also demonstrates excessive lacrimation, salivation, and pinpoint pupils.   

1. What is the likely cause of this man’s symptoms?   Organophosphate poisoning is the likely cause. Organophosphates are components of insecticides, and toxicity is often suspected in farmers who present with symptoms of excessive cholinergic release.

STEP 1 SECRET Organophosphate poisoning in a farmer is one of the most commonly encountered clinical vignettes on Step 1.

2. How does organophosphate poisoning result in this patient’s symptoms?   Organophosphates are AChE inhibitors. AChE is an enzyme that is responsible for degradation of ACh. Inhibition of AChE results in accumulation of ACh. Excessive ACh results in the DUMBBELSS symptoms: • Diarrhea • Urination • Miosis • Bradycardia • Bronchospasm • Excitation of skeletal muscles (muscle fasciculations, twitches, and trembling) • Lacrimation • Salivation • Sweating 3. What is the treatment for organophosphate poisoning?   Atropine and pralidoxime are used. Atropine is a cholinergic antagonist, which directly inhibits ACh receptors. Pralidoxime is used to regenerate AChE.     Note: Atropine toxicity is treated with physostigmine, an indirect agonist of ACh (inhibits AChE). 4. Atropine administration relieves all DUMBBELSS symptoms except one. Which symptom is not relieved by atropine?   Excitation of skeletal muscle is not relieved by atropine. The synapses at the NMJ are nicotinic receptors, and atropine is a muscarinic antagonist. Therefore, atropine will not block the nicotinic receptors at the NMJ.

STEP 1 SECRET Question 4 is a perfect example of the type of tricky question you can expect to see on boards. At first, you may panic if you cannot recall seeing the direct answer in a textbook. However, if you take a deep breath and just think about what the question is asking, you will realize that you can integrate and apply your knowledge to arrive at the correct answer. The purpose of boards, after all, is to see if you can apply the basic science knowledge you have gained in the first and second years of medical school toward clinical problem solving.

SUMMARY BOX: ORGANOPHOSPHATES • Indications: No medical uses. Found in insecticides and herbicides (suspect in farmers!) • Mechanism of action: Block action of acetylcholinesterase (AChE), leading to increased circulating quantities of ACh • Toxicity: Excessive acetylcholine (ACh) release results in the DUMBBELSS symptoms • Antidote: Atropine (blocks ACh receptors) and pralidoxime (regenerates AChE)

Case 23.13 A 25-year-old woman presents with 3-day history of malaise, headache, low-grade fever and a rash on her left calf. Her social history reveals that she was hiking in the woods near her Connecticut home about 10 days ago. Vital signs show temperature of 100.3°F, HR 100 beats/min, BP

Pharmacology and Toxicology  557 118/82 mm Hg, RR 16 breaths/min, and SaO2 99% on room air. Exam reveals a 3-cm circular red rash on her left posterior calf. You make the presumptive diagnosis of Lyme disease. Anti-Borrelia burgdorferi antibodies are found on serology. The patient has a history of tetracycline allergy, so you begin a course of rifampin. On follow-up several weeks later, the patient reveals that her Lyme symptoms resolved, but she still feels “off.” She took multiple home pregnancy tests, which were positive. Serum β-hCG is 400 mIU/mL. She is sexually active with her boyfriend of 2 years but tells you it is impossible for her to be pregnant because she takes oral contraceptive pills daily as instructed. She asks you how this could have happened.   

STEP 1 SECRET The purpose of this excessively detailed history is to demonstrate a critical test-taking strategy for Step 1. Always read the question’s last sentence before reading the entire vignette! If you had read the vignette from the beginning, you may have found yourself being distracted by certain buzzwords and clues—the patient’s Lyme symptoms, the history of hiking in the woods, the erythema chronicum migrans rash, etc. You may have also been scanning your memory for an alternative antibiotic for Lyme disease if doxycycline cannot be prescribed. If so, you were probably disappointed to find that the vignette had done all of the work for you—diagnosed the illness, identified the organism, and prescribed the appropriate treatment. Step 1 questions can contain lots of irrelevant information. Knowing the question before reading the vignette allows you to hone in on the salient points without being distracted by extraneous details.

1. What is the most likely explanation for this patient’s pregnancy?   Rifampin-induced CYP-450 activation and subsequent consumption of oral contraceptive pill (OCP) metabolites. You must know some common examples of CYP-450 substrates, inhibitors, and inducers (Table 23.10). A CYP-450 substrate is simply consumed by the enzyme but does not alter its activity. Inducers, as the name suggests, will increase the activity of CYP-450 and thereby decrease the concentration of the substrate. Inhibitors will have the opposite effect. In the case of this patient, the rifampin she took for Lyme disease acted as a CYP-450 inducer and caused her OCP metabolites to be consumed at a higher-than-normal rate. As such, the OCPs did not last long enough to have their intended effect, and she became pregnant as a result.     Warfarin is an often-cited example of a CYP-450 substrate. The classic Step 1 vignette will mention a patient who is being treated for atrial fibrillation (they may not explicitly say that the patient is taking warfarin—this is something they may expect you to infer) and is then put on azithromycin for atypical pneumonia. The vignette will then reveal that the patient’s INR is too high (the target range for atrial fibrillation is 2–3) and ask you why. You, of course, will know the answer: macrolides (including azithromycin) act as CYP-450 inhibitors that will prevent CYP-450 consumption of warfarin, leading to a greater anticoagulant effect.

Table 23.10.   CYP-450 Interactions SUBSTRATES

INHIBITORS

INDUCERS

Warfarin

“Azole” antifungals

Barbiturates

Oral contraceptive pills (OCPs)

Macrolides

Rifampin

Statins

Sulfonamides

Phenytoin

Antiepileptics

Cimetidine

Quinidine

Antidepressants

Ciprofloxacin Grapefruit juice Protease inhibitors (“navirs”) Acute alcohol abuse Isoniazid

Carbamazepine Griseofulvin St. John’s Wort Chronic alcohol abuse

STEP 1 SECRET While other CYP-450 substrates exist, the ones included in Table 23.10 are the most important to know. There are a number of mnemonics people use to memorize the CYP-450 inducers and inhibitors. We recommend that you memorize one list, but not both. If you learn the list of inducers, you can safely presume that all other drugs are inhibitors, and vice versa. Try to economize on brain space whenever you can!

558  Pharmacology and Toxicology

SUMMARY BOX: CYP-450 DR UG METABOLISM • CYP-450 is involved in Phase I of drug metabolism. • Key substrates to know: Oral contraceptive pills (OCPs), warfarin, statins, antiepileptics • Key inhibitors to know: Macrolides, azole antifungals, sulfonamides, ciprofloxacin • Key inducers to know: Phenytoin, barbiturates, rifampin, chronic alcohol abuse

Eric Han, MD, Thomas A. Brown, MD, and Sonali J. Bracken

CHAPTER 24

BEHAVIORAL SCIENCES

Insider’s Guide to Behavioral Sciences for the USMLE Step 1 Behavioral science is often overlooked by medical students taking the USMLE Step 1, but in our opinion, this is a huge mistake. Most students say that they wish they had studied more for this section because it can be a huge score booster for those comfortable with the material. Expect to see multiple questions that will present ethical dilemmas and then ask you what you would do in those situations. The ideal way to prepare for such questions is to practice reading through as many ethical scenarios as possible. Your best resources will be the cases in this chapter and those presented to you in question bank software programs. Be sure to pay special attention to exceptions for any rules that apply to ethical situations. Other high-yield behavioral sciences topics include developmental milestones and the physiology and pathophysiology of sleep. You should also know about informed consent, advanced directives, and care for minors.

BASIC CONCEPTS 1. Describe the four principles of biomedical ethics. • Autonomy: personal rule of the self that is free from controlling influences; physician must honor each patient’s preferences in accepting medical care and create conditions that facilitate individuality, autonomous choice, and informed consent (see question 2) • Beneficence: fiduciary duty to act in the patient’s best interest regardless of what is best for one’s self or for society • Nonmaleficence: responsibility to “do no harm”; must be balanced against beneficence if benefits and risks are both present • Justice: responsibility to treat all patients fairly 2. What is informed consent?   Informed consent is the process for getting permission before conducting a health care procedure or initiating a particular treatment. In order to give informed consent, all pertinent information (including risks, benefits, and alternative options) must be disclosed to the patient, and the patient must have the ability to comprehend this information, reason through and make his or her own decision, and be free of coercion. The patient must also be told that he or she can revoke consent at any time.     Notable exceptions to informed consent include patients who lack decision-making capacity (see question 3), patients who waive the right to informed consent, and therapeutic privilege (withholding information if informing the patient could harm the patient or deter his or her decision-making capacity). Implied consent can be granted in the case of an emergency. 3. How is decision-making capacity determined?   In order to be psychologically and legally capable of making a health care decision, the patient must be: • An adult or an emancipated minor • Informed • Able to make and communicate a choice • Able to make a stable decision that is consistent with his or her values • Free of hallucinations, delusions, delirium, or mood disorders that may directly influence decision-making capacity 4. What should a physician do if a patient requires a treatment not covered by his or her insurance?   It is never appropriate to deny a patient proper care because of limitations in time or money. You must discuss all treatment options with the patient regardless of insurance status. 5. Newborns are commonly assessed via Apgar scores. How is this score determined?   Apgar scores are based on a 10-point scale that is evaluated at 1 minute and 5 minutes after birth. These scores take into account Appearance, Pulse, Grimace, Activity, and Respiration. Apgar scores greater than or equal to 7 are considered to be good, while scores between 4 and 6 indicate that some assistance (e.g., in breathing) or stimulation may be required. Scores less than 4 require resuscitation and increase the risk that the child will sustain long-term neurologic damage.

Case 24.1 A 37-year-old man presents to a psychiatrist for evaluation of symptoms he believes might indicate depression. He reports difficulty sleeping for the last 6 months and that he doesn’t feel rested after 7 to 8 hours of sleep. He also notices difficulty concentrating at work.   

559

560  Behavioral Sciences 1. What is the differential diagnosis?   Depression, sleep apnea, sleep disorders (dyssomnias), adjustment disorder, hypothyroidism, substance abuse or withdrawal, and anxiety should be considered.

Case 24.1 continued: He has had relationship troubles with his wife and is recently divorced. He reports that her primary reason for leaving him was that he no longer seemed to care about her, as he never wanted to go out or do the things they used to do. She even went so far as to accuse him of having an affair. They stopped sleeping in the same room 2 years ago because of his excessive snoring, with intermittent bursts of awakening short of breath, which kept her up at night. He says he just doesn’t have energy to do things anymore. He also relates having been recently reprimanded at work for falling asleep. On examination, he is a moderately obese, otherwise healthy-appearing middle-aged man. His mental status examination is unremarkable. He denies any thoughts of suicide, appetite disturbances, or feelings of guilt or hopelessness but feels as though he has had a depressed mood since his wife has left.   

2. In addition to a diagnosis of adjustment disorder with depressed mood, what sleep-related disorder likely explains most of his symptoms?   This patient is suffering from sleep apnea and would appropriately be diagnosed with a breathing-related sleep disorder. These patients are often obese (a collar size >17 inches should be a red flag); presumably, the weight of the fat around the neck collapses the airway. Sleep is often interrupted at night because of the occluded airway, leading to excessive daytime sleepiness and fatigue. Chronic poor sleep can lead to irritability, poor concentration, and the need to “nap” during the day.

STEP 1 SECRET Associate “excessive daytime sleepiness” with narcolepsy and obstructive sleep apnea. Both diseases are favorites on the USMLE Step 1.

3. What treatment can be employed to allow this man to sleep at night?   Continuous positive airway pressure (CPAP), which involves pressurizing the airway to keep it patent could be used. The patient wears a mask that provides positive airway pressure to keep the airway from being obstructed. CPAP is only one treatment option for those patients who suffer from obstructive sleep apnea. As always, lifestyle modifications are important as well. This patient should be encouraged to lose weight, which should reduce the compressive forces on the airway and thereby decrease the airway obstruction. Uvuloplasty or nasal surgery may also be indicated.     Note: Sleep studies will show apneic episodes with increasing breathing effort against an obstructed airway, frequent arousals, and decreased rapid eye movement (REM) sleep.

SUMMARY BOX: SLEEP APNEA • Presentation: Loud snoring, difficulty concentrating, poor memory, and waking up feeling unrested after sleep • Epidemiology: Most commonly occurs in obese individuals • Diagnosis: Sleep studies will show apneic episodes, frequent arousals, and decreased rapid eye movement (REM) sleep • Treatment: May consist of lifestyle modifications and nasal continuous positive airway pressure (CPAP)

Case 24.2 A 29-year-old woman presents following an automobile accident in which she fell asleep at the wheel. She notes that she frequently falls asleep during the day and feels rested after these episodes.   

1. What is the differential diagnosis?   Sleep deprivation, primary hypersomnia, narcolepsy, sleep apnea, substance abuse or withdrawal, hypothyroidism, and anemia are considerations.

Case 24.2 continued: The patient states that sometimes she awakens but is utterly “unable to move a muscle.” She confirms that she has always been able to fall asleep quickly. She denies any use of drugs or medications. You excuse yourself to answer a page and find her asleep when you return to your office. On awakening she is startled at first but then seems to regain her orientation and asks, “What is wrong with me?”   

Behavioral Sciences  561

2. What is the likely diagnosis?   Narcolepsy, a condition that involves poor control of sleep-wake cycles. Individuals with narcolepsy often experience excessive daytime sleepiness that can disrupt normal activities. 3. What are the stages of sleep, and what happens physiologically in these stages?   Sleep is divided into non-REM (NREM) and REM sleep. NREM sleep is divided into four stages, each being a deeper sleep. The stages are further described by fast wave or slow wave sleep. The earliest two stages are associated with fast wave sleep, and stages 3 and 4 are termed slow wave sleep based on the electroencephalogram (EEG) appearance of brain waves. REM refers to rapid conjugate eye movement. As a person falls asleep, he or she passes through stages 1 to 4 and then enters REM sleep the first time (this takes approximately 90 minutes). The first REM episode lasts typically less than 10 minutes, and then the person cycles through the stages again, with further REM episodes of about 15 to 40 minutes each.     Physiologically, during NREM sleep, a person’s pulse, respiration rate, and blood pressure are decreased and show less minute-to-minute variation. Resting muscle tone is somewhat relaxed, and there are episodic body movements. Males do not experience erection, and blood flow, including cerebral circulation, is somewhat lowered. By contrast, REM sleep is characterized by higher pulse rate, respiratory rate, and blood pressure; EEG patterns are similar to those of one who is awake. REM sleep is also termed paradoxical sleep because its associated EEG findings appear similar to those found in a person who is awake. Men and women will experience penile/clitoral tumescence (engorgement). Additionally, a person in REM sleep experiences near total skeletal muscle paralysis, and movement is quite rare. Abstract and surreal dreams occur during this phase of sleep. Most REM sleep occurs in the last one-third of the night. 4. How do nightmares differ from night terrors?   Nightmares occur almost exclusively in REM sleep. Patients who experience nightmares are able to recall the details of these frightening events, which usually involve threats to life, security, or self-esteem. Upon awakening, the person rapidly becomes oriented. Night terrors occur in deep NREM sleep (stages 3 and 4). Often, the person wakes in a panicky scream. These patients are often unresponsive upon awakening, have amnesia for the episode, and show signs of autonomic arousal, such as tachycardia, tachypnea, and diaphoresis. Night terrors can be treated with benzodiazepines. 5.  Using Table 24.1, cover the columns to the right, and for each stage of sleep listed in the left column, describe the electroencephalogram’s appearance and associated findings.

Table 24.1.   Electroencephalographic (EEG) Characteristics of Sleep Stages STATE

EEG APPEARANCE

FREQUENCY

VOLTAGE

Awake

β waves

Random fast waves

Low

ASSOCIATED FINDINGS

Eyes closed

α waves

8–12 cycles/sec

Low

Stage 1

θ waves

3–7 cycles/sec

Low

Stage 2

Sleep spindles K complexes

12–14 cycles/sec Slow, triphasic waves

Low High

Stage 3

δ waves

2–4 cycles/sec

High

Stage 4

δ waves

0.5–2 cycles/sec

High

Bedwetting, sleepwalking, talking, night terrors

REM sleep

β waves

Random fast waves

Low

Dreaming, muscle paralysis, penile/clitoral tumescence

Easy to rouse

REM, rapid eye movement.

6. Describe the expected electroencephalographic findings in this patient.   The EEG in a sleep study would likely show a decreased REM latency, meaning that she rapidly progresses into REM sleep. This accounts for the restfulness these patients feel upon falling asleep.     Note: Patients with primary hypersomnia have a completely normal sleep architecture. 7. What treatments are available for patients with narcolepsy?   A regimen of regularly scheduled or “forced” naps during the day can be a successful treatment for some patients. In severe cases of narcolepsy, amphetamines such as methylphenidate (Ritalin) are also used. These agents cause the release of norepinephrine, dopamine, and serotonin, but all have some abuse potential. A newer agent, modafinil, has been added that has lower abuse liability. Modafinil appears to selectively decrease somnolence in narcoleptic patients; however, the mechanism of action is unknown.

562  Behavioral Sciences 8. This woman had been given a benzodiazepine to assist her sleep, which improved for a while, but now she complains of poor sleep once more. Why have her sleep problems returned?   She is experiencing tolerance to the effects of her medication. Benzodiazepines may be used for short-term management of insomnia, especially when there is an identifiable precipitant, but not for long-term management, because tolerance and dependence may result. Reevaluation should follow a 7- to 10-day trial of benzodiazepine use, and other agents should be considered. 9. How do benzodiazepines manifest their pharmacologic effect?   Benzodiazepines are agonists of γ-aminobutyric acid (GABA) receptors, which are bound to chloride channels. GABA is the primary inhibitory neurotransmitter in the central nervous system (CNS). This CNS inhibition leads to decreased alertness, drowsiness, and less agitation.     Note: Benzodiazepines should be avoided in the elderly because the aged population has a markedly increased (about 25%) incidence of falls when given benzodiazepines due to drowsiness and impaired balance. This effect would be especially concerning in elderly postmenopausal woman, who may have underlying osteopenia or frank osteoporosis. 10. There are now a number of drugs other than benzodiazepines that also act on the γ-aminobutyric acid benzodiazepine receptor and that reach hypnotic effects with less tolerance and less daytime sedation. What are some examples of these?   Zaleplon (Sonata), zolpidem (Ambien), and eszopiclone (Lunesta) are examples of these drugs.     Note: Sedating antidepressants such as trazodone and nefazodone (remember the zzzzzzzz group) may also be used. 11. When evaluating a person for sleep problems, perhaps the first and most important step is to make sure that the patient has good sleep hygiene. What does good sleep hygiene entail? • No alcohol • No caffeine or nicotine • Regular exercise (but not too late in the day) • Relaxing activity before bed (e.g., bath, reading) • Only sleep and sex in the bedroom (no TV) • No clockwatching • No daytime naps • No late meals 2. What changes in sleep are typical as people age? 1   Though this is somewhat controversial, for the purpose of boards you should assume that as people age, they experience a decrease in the amount of time in slow wave sleep (stages 3 and 4) and REM sleep. This typically results in a reduced need for time spent sleeping. Insomnia is common in the elderly population.

SUMMARY BOX: NARCOLEPSY • Presentation: Excessive daytime somnolence, sleep attacks, cataplexy, hypnagogic hallucinations, hypnopompic hallucinations, sleep paralysis • Diagnosis: Sleep studies will show decreased sleep latency (falls asleep faster) and earlier entry into REM sleep. • Complications: Can negatively affect quality of life, social isolation/withdrawal • Treatment: Scheduled naps during the day, stimulants, and modafinil

Case 24.3 An 80-year-old man with severe pulmonary disease requires a lung transplant. Soon after the surgery, the patient develops respiratory failure, which requires him to receive mechanical ventilation. When the patient’s family asks the doctor how long the patient will require ventilation, the physician responds that he is unsure, but that it is likely to be maintained for an extended period of time. The patient’s wife expresses to you that her husband has told her many times that he would not wish to be kept alive on mechanical ventilation for more than a day or two. At this moment the patient’s oldest son, who currently supports his parents, demands that his father be kept on mechanical ventilation until an alternative solution can be found.   

1. What should the physician do?   The physician should terminate mechanical ventilation. Although the patient cannot give proper informed consent, terminating ventilation is in line with the patient’s own wishes. 2. What are advance directives?   Advance directives are instructions provided by a patient in anticipation of the need for a decision to be made regarding his own medical care. They can be oral, written (e.g., living will), or in the form of a durable power of attorney. A durable power of attorney is responsibility assigned to a person by the patient to make medical decisions on his or her behalf in the event that he or she loses the capacity to do so. Statements made to others by the patient can qualify as oral advance directives. They gain more validity if they were repeated, heard by multiple persons, and recent. Although oral advance directives provide more flexibility than written directives, problems may arise from inaccurate communication of the patient’s wishes or deviations in interpretation.

Behavioral Sciences  563

3. How is competence (decision-making capacity) defined?   The patient must be informed (provided with adequate insight regarding all options), able to make and communicate a stable choice, and free from the influence of others (voluntary). The decision cannot result from delusions or hallucinations. 4. What is substituted judgment?   If a medical decision must be made on behalf of an incompetent patient who does not have any advance directives in place, the rule of substituted judgment can be used. The physician and the patient’s surrogate decision makers (individuals who know the patient well enough to determine what he or she would have done) can make a decision for the patient based on what they would expect that the patient would have wanted. The personal wishes of the physician or family members should not affect this decision.

STEP 1 SECRET If substituted judgment is required, priority of surrogate decision makers is as follows: spouse, adult children, parents, adult siblings, other relatives.

5. What is the best interests standard, and when should it be used?   The best interests standard refers to the decision that most competent people would make in a given scenario. Unlike substituted judgment, it is used when the patient’s preferences are completely unknown. Decisions made under the best interests standard should follow the principle of beneficence, which refers to a physician’s responsibility to always act in the patient’s best interest.

STEP 1 SECRET Medical decisions should be made according to the following algorithm: Autonomy (patient’s own preference) → advance directives (instructions communicated by the patient to another individual) → substituted judgment (patient’s anticipated desire) → best interests standard.

SUMMARY BOX: ADVANCED DIRECTIVES • A patient is considered to be competent if he or she is informed, able to make and communicate a stable decision, and free from the influence of delusions, hallucinations, or other individuals. • Advanced directives can be written, oral, or in the form of a durable power of attorney. • Substituted judgment or the best interests standard can be used when an incompetent patient does not have any advanced directives in place. Substituted judgment anticipates the decision the patient would be expected to make if he or she were able. The best interests standard is used when the patient’s wishes are unknown.

Case 24.4 A 15-year-old girl comes into your office asking for birth control. She admits that her parents do not know that she is sexually active, and she implores you not to tell them.   

1. What should you do?   Write the prescription and agree not to tell her parents. However, you should discuss the risks and benefits of using oral contraceptives with the patient. You should also encourage the patient to communicate with her parents. 2. What are the rules regarding parental consent for minors?   Parental consent is required for minors under the age of 18, unless the minor is emancipated (married, self-supporting, or in the military). There are, however, several situations in which parental consent is not required. These situations include emergencies, prescription of oral contraceptives, pregnancy-related medical care, and treatment of sexually transmitted diseases (STDs) and drug problems. Abortion generally requires parental consent.

SUMMARY BOX: CONSENT FOR MINORS • Parental consent must be obtained unless the minor is emancipated. • Exceptions to this rule include emergency situations, drug abuse, pregnancy-related medical care, prescription of oral contraceptives, or treatment of sexually transmitted diseases (STDs).

564  Behavioral Sciences

Case 24.5 A patient comes into your office with depressive symptoms. You work with her over the next few months to treat her for her depression. During a follow-up visit, she expresses her gratitude for your devotion and assistance and says she would like to make it up to you by taking you out to dinner. She winks, and you understand that she intends it to be a date. Although you do not admit it to her, you find that you are indeed attracted to her as well.   

1. What do you do?   It is never acceptable for you to have a romantic relationship with your patients. You should politely decline her invitation and continue to see her as your patient. It is not necessary to refer her to another physician if you can continue to be professional, but it would be a good idea to invite a chaperone into the office during any future visits. 2. Is it a good idea for you to be honest and tell her that you cannot have a relationship with her while she is your patient?   No. This would send the message that if your professional relationship were terminated, you would be willing to pursue a personal relationship with her.

STEP 1 SECRET Whenever the USMLE asks you what to do in a situation similar to the one in Case 24.5, it will often try to entice you with an answer choice that suggests you refer the patient to another physician. For the purpose of boards, this will almost never be correct. The correct choice will require you to be an active participant in the solution.

SUMMARY BOX: THE PHYSICIAN-PATIENT RELATIONSHIP • The physician-patient relationship should never extend beyond professional boundaries. • Under no circumstances is it acceptable to pursue a romantic relationship with a patient. • In the instance that a patient breaches this boundary, your best course of action is to continue to see the patient but to clarify the professional nature of your relationship. • It is not necessary to refer the patient to another physician, but you may want to bring a chaperone into the office during future appointments with this patient.

Case 24.6 A patient confides to you that he has been cheating on his wife and now suspects that he may be infected with human immunodeficiency virus (HIV). You perform the appropriate tests, which turn out to be positive. You tell the patient that you will treat him for HIV, but that it is his responsibility to tell his partner. He immediately breaks down and tells you that he cannot tell his wife and all other sexual partners because his wife will leave him once she finds out that he acquired HIV while cheating on her.   

1. What do you do?   Patients who are HIV-positive have a duty to protect their sexual partners from acquiring the infection. If the patient fails to do so, the physician is generally allowed (or even mandated, depending on the state of residence) to inform the partner(s).     Note: Some states have prohibition against warning, where the physician faces liability for any disclosures without the patient’s permission. 2. Under what other conditions is it acceptable to violate patient confidentiality?   Patient confidentiality should be maintained unless the patient is at significant risk for suicide or poses a risk to another individual. The physician can also intervene in the instance of child or elder abuse. Decisions to disclose information to family and friends should be made according to the patient’s best interest if the patient is not present or is incapacitated.     Note: The Tarasoff decision provides physicians with the legal ability to warn a targeted victim and notify the appropriate officials if a patient poses significant risk to another individual.

SUMMARY BOX: PATIENT CONFIDENTIALITY • Patient confidentiality should be maintained unless a patient is at risk for suicide or harming another individual. • The physician’s role in partner notification for HIV-positive patients depends on the state.

Behavioral Sciences  565

Case 24.7 A 70-year-old obese man with a history of congestive heart failure and newly diagnosed depression comes into your office because he can no longer sustain an erection. He seems upset, because this is greatly affecting his sex life. He admits that he is too embarrassed to discuss this problem with his wife.   

1. What is the differential diagnosis for this patient’s sexual dysfunction?   Drug effects (beta-blockers, selective serotonin reuptake inhibitors [SSRIs], ethanol), diseases (atherosclerosis, depression, diabetes, decreases in testosterone levels), and psychological effects (e.g., performance anxiety) can lead to sexual dysfunction. Given this man’s history of congestive heart failure, it is likely that he has been taking beta-blockers for some time. He was also newly diagnosed with depression and may have been given an SSRI. Side effects of both of these drugs include sexual dysfunction. This man’s age and obesity put him at risk for atherosclerosis and diabetes, which can also contribute to sexual dysfunction. Performance anxiety must be included in the differential diagnosis, particularly if he can sustain erections at certain times of the day (e.g., in the absence of his partner). As the physician, you should include this question in your medical history taking. 2. What changes occur in the elderly with regard to sexual health?   Men are slower to achieve erections and ejaculation and have longer refractory periods. After menopause, women experience vaginal dryness and irritation. Unless patients are on particular medications, libido does not decrease. Never assume that your elderly patients are not interested in sex. If you do not include sexual health in your history and physical examination, they may be too timid to bring up their concerns on their own!

SUMMARY BOX: SEXUAL HEALTH IN THE ELDERLY • Elderly men may be slower to achieve erections/ejaculation and may experience increased refractory time. • Postmenopausal women may experience vaginal dryness and irritation. • For the purpose of boards, sexual interest does not decrease in the elderly. • Sexual dysfunction may be attributed to drug effects, disease, or psychological effects.

Case 24.8 A 24-year-old patient comes into your office with flu-like symptoms. You suspect a viral infection and tell the patient to rest and take plenty of fluids. He becomes irritated with this advice and demands that you prescribe him antibiotics so that he can get over his sickness before his vacation the following week. You hesitate because you know that antibiotics would be of no benefit to the course of this patient’s illness.   

1. What should you do?   Ask the patient why he feels he needs the antibiotics, and politely explain why you feel that it is unnecessary to prescribe them. Though the patient may become argumentative, always keep in mind that it is your decision whether or not to prescribe a medication to a patient. Avoid writing unnecessary prescriptions.

SUMMARY BOX: PATIENT-REQUESTED PRESCRIPTIONS • Avoid writing a prescription for a patient if you as the physician do not consider the medication to be an appropriate treatment.

Case 24.9 You are working alongside a second-year resident during your inpatient medicine rotation. Over the past 2 weeks, you have noticed abrupt changes in the resident’s dress and behavior. He often arrives to work late and ungroomed. You have also noticed that his breath frequently smells like alcohol. You suspect that he has been drinking heavily.   

1. What do you do?   It is your responsibility to protect patients from receiving inadequate or negligent care from an impaired or incompetent medical professional. You may choose to directly confront the resident in a nonthreatening manner (preferable). Alternatively, you may take a more formal approach and voice your concerns to the attending physician or the residency director.

566  Behavioral Sciences

STEP 1 SECRET For the purpose of Step 1, you should never select an answer that suggests you do nothing.

2. What is the CAGE questionnaire?   The CAGE questionnaire is a widely used method for screening for alcohol abuse. You are expected to know this acronym for boards. If a patient responds with “yes” to more than one of the following questions, the patient should be examined further for alcoholism. • Have you ever felt like you should Cut down on your drinking? • Are you ever Annoyed by people criticizing you for drinking? • Have you ever felt Guilty because of your drinking? • Have you ever needed a drink first thing in the morning (Eye-opener) to get out of bed or start your day?

SUMMARY BOX: ALCOHOL ABUSE • It is your responsibility to protect patients from receiving care from any medical professional who is under the influence of alcohol or drugs. • You may choose to directly confront the colleague in question or involve others. • The CAGE questionnaire is often used as a screening tool for alcoholism. • You should know the components of this acronym (see text).

Case 24.10 A mother brings her 2-year-old child to the pediatrician’s office for a well-child visit. She is concerned that her child still does not speak in full sentences. She also says that despite numerous attempts, she has been unable to toilet-train her child even though her neighbor’s 2-year-old child has had success.   

1. Is this child developing normally?   Yes. See Table 24.2 for developmental milestones.

Table 24.2.   Developmental Milestones AGE

GROSS MOTOR

FINE MOTOR

LANGUAGE

OTHER

Infant Birth–3 months

Rolls over (3 months)

Rooting reflex



Orients to voice Social smile

3–6 months

Sits up (6 months)

Puts hands together Strings syllables (3 months) together Passes items (6 months)

Moro reflex disappears Stranger anxiety

6–9 months

Crawls

Pincer grasp



Feeds self Separation anxiety Orients to name and gestures

Toddler 12 months

Walks

Stacks 3 blocks

Speaks 1–3 words

Drinks from a cup

15 months

Runs Walks backward



Speaks 6 words

Babinski reflex disappears Separation anxiety

18 months

Climbs stairs Kicks ball

Stacks 4 blocks

Combines words

Brushes teeth with help

2 years

Jumps (upward)

Stacks 6 blocks Feeds self with utensils

Speaks >200 words Uses 2-word sentences

Washes hands Begins to engage in parallel play

Behavioral Sciences  567

Table 24.2.   Developmental Milestones—cont’d AGE

GROSS MOTOR

FINE MOTOR

LANGUAGE

OTHER

Jumps (forward) Rides tricycle

Stacks 9 blocks Draws circles and dashes

Completely understandable

Brushes teeth Plays board games Toilet training Develops gender identity Comfortable spending a few hours away from parents

4 years

Hops on one foot

Copies stick figure

Uses prepositions Dresses self and complete sen- Plays cooperatively and with tences imaginary friends Tells detailed stories

5 years



Draws squares and triangles Ties shoes



Preschool 3 years

Identifies colors Counts to 5 Grooms self

2. What should you tell this concerned parent?   The mother should be told that every child develops differently and that her child is on track for normal development. Do not automatically dismiss the mother’s concerns; be sure that she feels comfortable coming to you if she notices “anything else that she considers unusual.”

SUMMARY BOX: DEVELOPMENTAL MILESTONES • You should know the information listed in Table 24.2. This is an extremely high-yield topic for boards.

Case 24.11 A 24-year-old patient with type 1 diabetes is admitted to the hospital after an insulin overdose that resulted in hypoglycemic seizures. You go in to see the patient once she is stabilized. You ask her whether she uses her insulin regularly, and she tells you that she gives herself injections twice a day according to the doctor’s instructions. When you ask her how much insulin she injects, she shrugs and tells you that it varies, depending on the food she eats. You ask her to clarify, and she tells you, “I give myself less if I skip meals and more whenever I eat junk food.”   

1. How should you handle this situation?   This is a clear example of a noncompliant patient who is not properly following the instructions of her treatment plan. Not only is this patient administering her insulin incorrectly, but she is not adhering to a proper diabetic diet. The most important thing to remember when dealing with a noncompliant patient is that scolding will be ineffective in preventing future mishaps (and will never be the correct answer on boards!). Instead, you must have a discussion with the patient to figure out the reason for the noncompliance and work together to fix the problem.     Note: In severe cases, patients may be dismissed by a physician for noncompliance. For the purpose of boards, this is not likely to be the correct answer. 2. How can compliance be increased in the future?   As mentioned previously, it is crucial to determine the reason for the patient’s noncompliance. Therefore, it is important to figure out whether this patient is neglecting the physician’s instructions because (a) she does not understand them, (b) it is difficult for her to adhere to them, or (c) she does not know the importance of following them. If you get the feeling that a patient does not understand the directions, do your best not to embarrass the patient. Instead, tell the patient that this could happen to anyone and simplify your instructions. Have the patient repeat the instructions back to you when you are done so that you know she has understood correctly (teach-back technique). Write the instructions down whenever possible. This is especially important to consider whenever the patient is not a native English speaker.     Sometimes, it is difficult for a patient to adhere to the treatment plan. Insulin, for example, must be refrigerated. Consider a scenario in which a diabetic travels a lot for work and does not always have access to a refrigerator. He or she might skip insulin dosages frequently. Once again, simplify the treatment regimen whenever possible.     It is also a good idea to make sure that the patient understands why it is important to follow a specified treatment plan. Perhaps this patient does not understand why junk food is especially harmful to a diabetic, or why skipping meals can lead to hypoglycemia. Perhaps she does not understand the reason behind regulated insulin doses. Educating the patient will most likely motivate her to follow the treatment plan correctly.

568  Behavioral Sciences     Do not attempt to scare the patient into complying with a treatment plan. (For example, it is unethical to show the patient graphic pictures of gangrene and say, “This will happen to you if you don’t shape up!”)

SUMMARY BOX: THE NONCOMPLIANT PATIENT • Nonadherence is a common hurdle faced by all physicians. • Patients should not be scolded for their noncompliance. It is more important to determine the reason for the noncompliance and attempt to fix the problem. • Never use scare tactics in an attempt to improve a patient’s compliance.

Case 24.12 A 68-year-old man is brought to your office by his wife because of abdominal pain, jaundice, and unintentional weight loss. A computed tomography (CT) scan of the abdomen reveals adenocarcinoma of the head of the pancreas. When you walk into the office to break the news to the patient, his wife asks to speak to you alone outside. The two of you step out of the office and she confesses that she has a feeling you are returning with bad news. “Please tell me first,” she begs. “If it’s really bad, I know my husband won’t be able to handle it. If I know what it is, I can help break the news to him in time.”   

1. How should you handle this situation with the patient’s wife?   It is unlawful to disclose a patient’s medical information to family or friends without the permission of the patient. Therefore, you should avoid revealing any information to the patient’s wife at this time. You should, however, find out why the patient’s wife is so concerned about her husband’s ability to handle the news. Her concerns will perhaps guide your approach to handling this patient. 2. What should you say to the patient when you walk into the room?   You should tell the patient that you have some news to discuss with him and politely dismiss his wife for the time being. At this point, you can ask the patient whether he would like his wife to be present. If he agrees, you can invite her back into the room. Asking the patient’s permission for his wife to remain in the room in her presence might pressure his decision.

SUMMARY BOX: DISCLOSURE OF PATIENT INFORMATION • It is unlawful to disclose any patient information to family or friends without explicit permission from the patient. • Always ask to speak to a patient privately before discussing confidential medical information in front of others.

Case 24.13 A 50-year-old female presents to your clinic complaining of burning pain during urination and increased urinary frequency. Already several patients behind, you quickly diagnose a urinary tract infection and prescribe a course of trimethoprim-sulfamethoxazole. The patient returns the next day with a new maculopapular rash and pruritus. Upon closer review of her chart, you discover she had a similar reaction in the past to trimethoprimsulfamethoxazole. The patient asks if this reaction could have been caused by the antibiotic you prescribed.   

1. What should you do?   Physicians have an ethical obligation to inform a patient if a medical error that has caused harm has been made. Acknowledging and apologizing for medical errors improves the physician-patient relationship and can potentially decrease the likelihood of litigation. Primary motivators for patients who file lawsuits are perceived dishonesty on the part of the physician and lack of explanation for the incident.     Note: For the purposes of the boards, you should never choose the answer choice that avoids disclosure.

SUMMARY BOX: MEDICAL ERROR • Medical errors that have caused harm should be disclosed to the patient.

Case 24.14 A 45-year-old patient comes into your office with his wife and complains that he has been experiencing a frequent sensation of his “legs falling asleep.” His discomfort causes an urge to constantly move his limbs because he feels much better when he is active. His wife testifies that her husband continually jerks his legs in his sleep. This activity disrupts both his and her sleep patterns, and both profess feeling tired throughout the day.   

Behavioral Sciences  569

1. What is the most likely diagnosis?   Restless legs syndrome (RLS), a disorder of unknown etiology that causes a constant urge to move in attempt to relieve unpleasant sensations in the lower limbs, is likely. It has been linked to several conditions, including Parkinson’s disease, rheumatoid arthritis, diabetes, kidney failure, and iron deficiency anemia. Use of certain medications may also trigger RLS. However, RLS can be idiopathic in nature. 2. What are the most common symptoms of restless legs syndrome?   Symptoms include an unpleasant sensation in the legs, urge to constantly move, relief upon movement, and worsening of symptoms when inactive. Typical leg movements associated with RLS are jiggling, pacing, tossing, rubbing, and stretching. Limb movements often occur during sleep. 3. How is restless legs syndrome treated?   Although there is no direct cure for RLS, the treatment plan involves correcting the underlying cause of the condition whenever possible. Treatment also focuses on symptom relief, and includes sleep improvement, alcohol avoidance (alcohol may trigger RLS symptoms), walking, and heat/cold packs on the affected limbs.

SUMMARY BOX: RESTLESS LEGS SYNDROME • Presentation: Unpleasant leg sensations, urge to move constantly, worsening of symptoms when inactive, limb movements during sleep • Epidemiology: Linked to several conditions (Parkinson’s disease, rheumatoid arthritis, diabetes, kidney failure, iron deficiency anemia, use of certain medications) but can be idiopathic in nature • Treatment: No direct treatment is currently in existence. Focus on treating the underlying cause of the disease and providing patients with symptom relief.

CHAPTER 25

BIOSTATISTICS Long Tu, MD, Joravar Dhaliwal, Thomas A. Brown, MD, and Sonali J. Bracken

Insider’s Guide to Biostatistics for the USMLE Step 1 Like behavioral sciences, biostatistics is a topic that most medical students do not spend nearly enough time studying because it appears to be “less important” than some of the other subjects. Unfortunately, because of this, students often miss a lot of straightforward biostatistics questions. Biostatistics is one of the highest-yield subjects for the Step 1 exam and an easy way to earn points on your exam if you take the time to understand the concepts. This chapter introduces you to the types of questions you are likely to see on your exam and prepares you to solve them in the most efficient manner possible. Note that you will be given a whiteboard to use during your exam before the start of your test. You may take up to 5 minutes before you begin your exam to write anything you would like on your whiteboard. The formulas that students find most helpful to add to their whiteboard are highlighted throughout this chapter. It would be a good idea to review these formulas and the information you plan to include on your whiteboard the day before your exam.

BASIC CONCEPTS 1. What does the sensitivity of a diagnostic test measure?   The sensitivity of a diagnostic test measures how effectively the test can detect disease in a patient who truly has the disease. Study Table 25.1. The number of individuals who truly have the disease of interest (i.e., in whom disease is present) can be calculated by summing a + c. The sensitivity of a test determines the proportion of these individuals with disease (a + c) who indeed test positive for the disease (a). The remainder of diseased individuals who test negative for the disease are false negatives (c). In other words, sensitivity can be calculated by dividing the number of true positives by the total number of people in a population who have the disease: true positives/(true positives + false negatives), or a/(a + c). Sensitivity is also referred to as the true positive rate and improves when the percentage of a diseased population who test positive for the disease increases. Note that sensitivity is inversely related to the false negative rate (c/(a + c)) by the formula shown in Eq. 25.1: Sensitivity = 1 − false negative rate [25.1]



Table 25.1.   Sample 2 x 2 Table for Determining Sensitivity and Specificity PRESENCE OF DISEASE (+)

ABSENCE OF DISEASE (−)

(+) Test result

True positives (a)

False positives (b)

(−) Test result

False negatives (c)

True negatives (d)

STEP 1 SECRET Practice setting up tables like Table 25.1 whenever you encounter a biostatistics problem that involves sensitivity or specificity. Doing so will help you immensely on the USMLE! It will be helpful to copy this specific table onto your whiteboard before the start of your exam.

2. What does the specificity of a diagnostic test measure?   The specificity of a diagnostic test measures the ability of a test to detect absence of disease in patients who truly do not have the disease. In other words, specificity measures the percentage of nondiseased individuals who test negative for the disease. Let us again turn our attention to Table 25.1.    

570

Specificity = true negatives/(true negatives + false positives = d/ (d + b)[25.2]

Specificity is inversely related to false positive rate as follows: Specificity = 1 − false positive rate[25.3]

Biostatistics  571

    Given this relationship, a more specific test will result in fewer false positive results, which gets to the real meaning of specificity in interpreting medical tests: the more specific the test, the more likely a positive result indicates the presence of disease.

STEP 1 SECRET SPIN and SNOUT are useful mnemonics to remember the differences between specificity and sensitivity. SPIN reminds us that SPecific tests rule IN disease. That is, the more specific a test, the more likely it is that a positive test result indicates real disease, because a highly specific test will have a very small number of false positive results. SNOUT reminds us that SeNsitive tests rule OUT disease. That is, the more sensitive a test, the more likely that a negative test result rules out disease, because a highly sensitive test will have a very small number of false negative results. SPIN: SPecific tests rule IN SNOUT: SeNsitive tests rule OUT In serious diseases where significant differences in outcome may result from a failure of early detection, greater test sensitively is often desired even at the expense of lower specificity.

3. Let’s review: Cover the right column in Table 25.2 and define each of the terms in the left column. Table 25.2.   Basic Terminology TERM

DEFINITION

True positive

A positive test result in someone who truly has the disease

False positive

A positive test result in someone who truly does not have the disease

True negative

A negative test result in someone who truly does not have the disease

False negative

A negative test result in someone who truly does have the disease

4. How does the sensitivity of a test relate to its specificity?   Their relationship is inverse: as sensitivity increases, specificity decreases and vice versa. Consider the example of using prostate-specific antigen (PSA) levels as a screening tool for prostate cancer. Serum levels above 10 ng/dL are typically considered to be high risk (positive test result); approximately 50% of these individuals will have prostate cancer. On the contrary, only 1 in 4 individuals with levels between 4 to 10 ng/dL will have evidence of prostate cancer on biopsy. If the threshold for a positive result were lowered from 10 ng/dL to 6 ng/dL, more individuals with prostate cancer would likely be detected, and sensitivity of the test would thus increase. However, specificity of the test would surely decrease because at this lower threshold, there would be a greater chance that other factors unrelated to prostate cancer (e.g., benign prostatic hyperplasia) may be driving PSA levels up and contributing to false positive test results (Fig. 25.1). Recall that specificity is equal to 1 – false positive rate. When the false positive rate increases secondary to lowered threshold values, specificity conversely decreases.

0

2

4

6

8

10

12

14

16

18

20

22

24

26

PSA (ng/ml) Patients w/o prostate cancer

Threshold for positive result

Patients with prostate cancer

Figure 25.1.  Relationship of sensitivity to specificity using the example of prostate-specific antigen (PSA) measurements to detect prostate cancer. Note that as sensitivity of the test is increased (threshold for a positive result lowered from 10 ng/dL to 6 ng/dL), more individuals with prostate cancer are detected. However, this compromises specificity as more false positive results are detected.

572  Biostatistics 5. What information is given by the relative risk?   The relative risk informs you of the ratio of the probability of a disease occurring in a group exposed to a particular risk factor versus in a nonexposed group. Relative risk (RR) is calculated by dividing the incidence of disease in the exposed group by the incidence of disease in the unexposed group. Examine Table 25.3. Table 25.3.   Sample 2 x 2 Table for Determining Relative Risk and Odds Ratio PRESENCE OF DISEASE (+)

ABSENCE OF DISEASE (−)

(+) Exposed

a

b

(−) Nonexposed

c

d

    The percentage of exposed individuals who have developed disease can be determined by a/(a + b). Likewise, the percentage of nonexposed individuals who have developed disease can be determined by c/(c + d). Thus, RR = a/(a + b) ÷ c/(c + d).     Note that this table is identical to Table 25.1 except that test result has simply been substituted for exposure status. See how easy it is?     How is RR typically applied? For example, assume that the incidence of lung cancer in smokers is 90/100 or 0.90 (Table 25.4). The incidence of lung cancer in nonsmokers is 10/100 or 0.10. Therefore, the RR for smoking is 0.90/0.10, or 9. An RR of 9 implies that smokers are 9 times more likely to get lung cancer than nonsmokers. Table 25.4.   Incidence of Lung Cancer in Smokers Versus Nonsmokers LUNG CANCER (+)

LUNG CANCER (−)

Smoker

90

10

Nonsmoker

10

90

    An RR of greater than 1 implies that the disease is more likely to occur in the exposed group. An RR of less than 1 implies that the disease is less likely to occur in the exposed group (i.e., that the exposure may be protective). An RR of 1 implies that there is no difference in disease occurrence between the groups. 6. What information is given by the odds ratio?   Similarly to RR, the odds ratio (OR) provides a measure of association between exposure status and disease outcome. However, OR is typically used in case-control studies, whereas RR is more commonly applied to cohort studies and randomized controlled trials. (We discuss the differences between these study types later on in this chapter.) The OR is used to determine whether a past exposure was a potential risk factor for a present outcome of interest (as opposed to RR, which determines the likelihood of disease occurrence given a particular exposure status).     The OR is calculated by dividing the odds of exposure in the diseased group by the odds of exposure in the nondiseased group. Examine Table 25.3.

OR = (a/c) ÷ (b/d) = ad/bc[25.4]

    Let’s say that one wanted to determine the odds that individuals with or without lung cancer were previously smokers (see Table 25.4). The OR would be calculated by dividing the odds of smoke exposure in individuals with lung cancer (90/10) versus in those without lung cancer (10/90). OR =

90/10 10/90

=

90 × 90 10 × 10

=

8100 100

= 81

    In other words, the odds of smoke exposure is 81 times higher in individuals with lung cancer versus those without lung cancer.    An OR > 1 means that the exposure occurred more frequently in the disease group. An OR < 1 means that the exposure occurred less frequently in the disease group. An OR = 1 means that the exposure occurred with the same frequency in the disease group and control group.        For diseases that are rare and for case-control studies where the exposure rate of the studied population is similar to that of the general population, the odds ratio approximates the relative risk.

Biostatistics  573

7. What is the difference between probability and odds, and how are they measured?   Both measure the likelihood that a particular event will occur. Probability is expressed as a ratio from 0 to 1 and is defined by the number of results that could produce a particular outcome divided by the number of all potential outcomes. For example, the probability of rolling a 6-sided die and having it land on the number 3 is 1/6 or 0.167; in this case, there are 6 potential outcomes (numbers 1 to 6) and only one (3) produces the desired result.     Odds are defined as the chance of an outcome occurring divided by the chance that a single different outcome will occur. In the 6-sided die example, the odds of rolling a 3 (versus not rolling a 3) are 1 to 5 or 0.2. Make sense?     The two concepts may be interconverted using the formulas shown in Eqs. 25.5 and 25.6: Odds = Probability/(1 − Probability) [25.5]

and

Probability = Odds/ 1 + Odds [25.6]

   

Do the math with the 6-sided die example and see for yourself that these formulas really work!

8. What is positive predictive value? Negative predictive value?   The positive predictive value (PPV) is the probability that, given a positive test result, the disease in question is actually present. In other words, PPV = (true positives)/(all positives).     The negative predictive value (NPV) is the probability that, given a negative test result, the disease in question is in fact absent. NPV = (true negatives)/(all negatives). If a 2 × 2 table is created (Table 25.5), the PPV can be calculated by dividing the number of true positives by the total number of positive test results: PPV = a/(a + b). NPV is calculated by dividing the number of true negatives by the total number of negative test results: NPV = d/(c + d).     You should note that both PPV and NPV vary with disease prevalence. If disease prevalence increases, the numbers in the (+) disease column (true positives [TP] and false negatives [FN]) will both increase. Because PPV = TP/(TP + FP), PPV will increase if TP increases. NPV, on the other hand, will decrease as disease incidence increases, because the number of FN will increase. Recall that NPV = TN/(TN + FN), where TN = true negatives. Table 25.5.   S  ample 2 x 2 Table for Determining Positive Predictive Value and Negative Predictive Value PRESENCE OF DISEASE (+)

ABSENCE OF DISEASE (−)

(+) Test result

True positives (a)

False positives (b)

(−) Test result

False negatives (c)

True negatives (d)

9. What are likelihood ratios?   Likelihood ratios help us determine how much we should shift our degree of suspicion for a particular disease based on a given test result. Unlike PPV and NPV, likelihood ratios are not influenced by disease prevalence. The positive likelihood ratio (PLR) reflects how much a positive test result increases the probability of disease being present over the baseline prevalence. The higher the ratio, the more likely the positive test result indicates the presence of disease.

PLR = sensitivity/ 1 − specificity [25.7]

    For example, if the sensitivity of a particular test is 85% and the specificity is 90%, the PLR is 0.85/(1 − 0.9) = 8.5. In this example, a positive test result means that the individual is 8.5 times more likely to have the disease in question. The more sensitive and specific the test, the higher the PLR. PLRs of 2 to 5 often indicate a small to moderate increase in the likelihood of disease given a positive test result, while a PLR of 10 or more generally offers conclusive increases in the likelihood of disease given a positive test result.     Negative likelihood ratio (NLR) indicates how much the probability of disease decreases if a test is negative.

NLR = 1 − sensitivity /specificity[25.8]

    In the previous example, NLR = (1 − 0.85)/0.9 = 0.17, indicating that a negative test result means that the individual is 5.9 times more likely not to have the disease in question. 0. What is meant by the reliability of a test? 1   Reliability (precision) is the ability to reproduce similar results with each iteration of the same test. The USMLE Step 1 examination would, for example, be considered reliable if a student could take it repeatedly and get close to the same score each time. It would not be considered a reliable examination if the same student received significantly different scores when taking the test on different dates. Precision is improved by reducing the random error in a test. Random error describes deviations from the true value of a measurement that result from imperfections in the measuring tool.

574  Biostatistics 1. What is meant by the validity of a test? 1   Validity (accuracy) is a measure of how closely a test’s results come to the “true value.” The “true value” is defined by the gold-standard test. Validity is reduced by minimizing systematic error. For example, an intelligence quotient (IQ) test that depends in part on reading comprehension is invalid if IQ is not intended to reflect literacy. This reflects a systematic error in designing or conducting the test. Systematic error refers to deviations from the true value of a measurement that result from poor instrument calibration or flawed observation methods. 12. In statistical analyses of differences between groups, a P value is often included to reflect how significant the difference is. What is the meaning of this P value?   P value reflects the probability that the difference observed between the experimental and control groups could occur by chance alone.     For example, if the P value between an experimental and control group is 0.05, there is a 5% chance that the difference observed between these groups was due entirely to chance rather than the variable being tested in the experiment. In the modern medical literature, most results are considered “significant” if there is a less than 5% chance (P < .05) that the observed difference between experimental and control groups could have occurred by chance alone. Some studies will report even more stringent restrictions of P < 0.01 or P < 0.001. These indicate an increasing confidence that the observed results stemmed from the intended difference in intervention rather than mere chance.

STEP 1 SECRET P values are very important for you to understand both for Step 1 as well as when interpreting scientific literature ­during your medical career. In addition to P values, type I and type II errors (see question 13) are commonly tested board topics.

13. What are the differences between type I and type II errors? How is power related to type II error?   Type I (α) error is the false positive error. It is the probability of claiming that a true difference exists between two groups when, in reality, none exists. The null hypothesis (H0) refers to the assertion that no meaningful difference exists between two observed groups. If a researcher claims a statistical difference between two groups when none exists and falsely rejects the null hypothesis, the researcher has committed a type I error.     By contrast, type II (β) error is false negative error. It refers to the probability of stating that no difference exists between two groups or populations when in fact there truly is a difference. Type II error is an acceptance of the null hypothesis when it should indeed be rejected.     Say we wish to determine the factors that influence USMLE Step 1 scores among medical students. We hypothesize that more time spent studying for the USMLE leads to significantly increased exam scores, and decide to test this concept by comparing scores between students who studied for 6 weeks and students who studied for 6 months. After conducting this study, let us suppose that we do not find that test scores significantly differ between the two groups of students despite differences in study time. If USMLE Step 1 score truly is influenced by time spent studying among the general population of medical students, we have made a type II error.     The term statistical power is the likelihood that a study can detect a difference between means (i.e., between groups) if one truly exists. Power analysis is often performed before conducting a research study to verify that the sample size is large enough to detect a significant effect of an independent variable. You certainly do not need to know how to perform a power analysis for the USMLE Step 1, but you should note that as power is increased, the likelihood of committing a type II error decreases according to the following formula: Power = 1 − β .[25.9]



    Makes sense, right? Since β is the probability of committing a type II error (not detecting a difference that truly exists), power is inversely related to this value. This inverse relationship can be clearly observed in Table 25.6.

Table 25.6.   Relationship Among Type I Error, Type II Error, and Power

Significant experimental difference observed (P < 0.05)

ACTUAL DIFFERENCE EXISTS BETWEEN GROUPS

NO ACTUAL DIFFERENCE EXISTS BETWEEN GROUPS

Correct finding; study adequately powered

Type I (alpha error)

No significant experimental difference Type II (beta error) observed (P > 0.05)

Correct finding; increasing sample size will not alter results

Biostatistics  575

14. What are some determinants that can be used to evaluate the existence of a causal relationship between two variables? • Consistency: multiple studies independently support the same conclusion (e.g., six reports from different institutions provide evidence that angiotensin-converting enzyme [ACE] inhibitors reduce blood pressure) • Biologic plausibility: proposed relationship between cause and outcome is consistent with current scientific knowledge (e.g., excessive cell phone use leads to a slightly enhanced risk of brain tumor development) • Temporality: cause must precede disease outcome (e.g., asbestos exposure precedes malignant mesothelioma of the pleura) • Coherence: correlations between epidemiological and laboratory studies increase the likelihood of an effect (e.g., results from animal studies support the hypothesis) • Dose-response relationship: greater exposure leads to more severe phenotype or increased likelihood of disease (e.g., increased number of pack years in smokers directly correlates with risk of lung cancer) • Reversibility: lack of exposure results in reduced likelihood of contracting disease; particularly powerful if a strong dose-response relationship has also been observed (e.g., reducing exposure to radiation reduces the likelihood of developing acute leukemia) • Specificity: more specific associations between an exposure and outcome with no additional explanations enhance probability of causation (e.g., one-to-one relationship is noted between exposure to drug X and a particular side effect) 5. What is the difference between prevalence and incidence? 1   Prevalence is the percentage of the population that currently has the disease. For example, 35% of Americans are obese, so the prevalence of obesity is 35%. Incidence refers to how many people develop a disease within a given time frame (usually annually). For example, if 300,000 people are newly diagnosed with diabetes each year, the annual incidence of diabetes is 300,000. 6. How do the incidence and duration of a disease affect its prevalence? 1   The higher the incidence and the longer the duration of the disease, the greater the prevalence of the disease.

Prevalence = Incidence × Duration of disease [25.10]

    Chronic diseases such as arthritis are unlikely to rapidly result in death, so their prevalence is typically high due to long duration of disease. The opposite trend is true of diseases that have a short duration (e.g., meningitis), either because they rapidly result in death or because they resolve quickly. 

MEASURES OF SPREAD 17. The following sample distribution pattern lists the ages of 11 patients seen by a physician on a given day: 1





2

3

3

3 , 4 , 5 , 5 , 7 , 8 , 80

A. What are the mean, median, and mode for the ages of the patients seen by the physician on this day? The mean is simply the average of the sample, which is calculated by adding together all of the results and then dividing by the sample size. In this example, the mean would be 11 (121/11 = 11). The median is the number in the middle of the data set when ordered sequentially. Half of your data should lie above the median and half will lie below. In this example, the number 4 has equal numbers of subjects on either side. In cases in which there is an even number of results, the median is calculated by taking the average of the middle two numbers. The mode is the number present with the highest frequency. In this case, the mode is 3. B. Is the mean or the median more representative of central tendency? The median is less affected by outliers (e.g., 80) than the mean and is often a better representation of central tendency than the mean, particularly for small sample sizes containing multiple outliers. C. Is this sample “skewed” at all, and if so, in which direction? Yes, this sample population is positively skewed to the right because of the outlier value 80. In Fig. 25.2, graph A represents a positive skew and graph B a negative skew. Positive skews occur when there are outliers that have a higher value than the numbers closer to the mean. Negative skews occur when the outliers have a lower value then the numbers close to the mean.

Frequency

576  Biostatistics

x Mode Median Mean

Frequency

A

x

Mean Median Mode

B

Figure 25.2.  Skewness. A, Positive Skew, B, Negative Skew.

STEP 1 SECRET When looking at a distribution graph, an easy way to determine where the mode is located is to look at the peak of the distribution. The mode is the value at the location of the peak because it is the outcome that appears most frequently. The mean will be located on the same side as the longer “tail” of the distribution, while the median is typically found in between the mean and the mode. Recall that in a normal (Gaussian) statistical distribution, mean, median, and mode will all be equal. 8. What does the standard deviation of a population represent? 1   The standard deviation (σ) is a measure of how spread out a test population is. If most of the values are close to the mean, the standard deviation will be small. However, if many of the values are far from the mean, the standard deviation will be larger. You will not be asked to calculate standard deviation for any data set on the USMLE Step 1.     In a normal distribution, all the members of a population within one standard deviation of the mean (both above and below) will constitute approximately 68% of the total population. All members within two standard deviations will constitute approximately 95% of the total population. Three standard deviations will contain approximately 99% of the data set. Fig. 25.3 shows the distribution of a bell curve.

13.5%

13.5% 34%

0.5% –4

34%

2.0%

–3

2.0%

–2

–1

0

+1

+2

0.5% +3

+4

68% 95% 99%

Figure 25.3.  Distribution of a bell curve.

Biostatistics  577

STEP 1 SECRET You are expected to know the normal-shaped bell curve distribution for Step 1. Recall that the data are distributed evenly to both sides! Remembering the 68-95-99 rule for normal distributions may be extremely helpful on test day, although many students get confused by this concept on the exam. If, for example, you are asked to calculate how much of the data fall out of the range of two standard deviations in a normal distribution, the answer is 5%, because 95% of the data will fall within the range of two standard deviations. If you are asked to calculate how much of the data fall above two standard deviations, the answer is 2.5% (half of the 5% will fall above two standard deviations and half will fall below). As obvious as this may sound to you, pay close attention to the question being asked! It is easy to fall into these types of traps when you are under pressure on test day.

STUDY DESIGNS 9. What is meant by the term bias, and which study design best eliminates bias? 1   Bias is systematic error that affects one study group more than the other. This differs from random error, which typically affects both groups equally and should not adversely affect the study if there are enough participants. Randomized clinical trials typically control most effectively for bias, whereas case-control studies typically control least effectively for bias. Other types of study designs (e.g., cohort, cross-sectional) fall somewhere between these two extremes in their ability to eliminate bias. Table 25.7 reviews concepts pertaining to study design in more detail.

Table 25.7.   Study Designs STUDY DESIGN

SETUP

STRENGTHS

LIMITATIONS

Cohort study

A “cohort” of subjects is Relatively easy to set up Confounding variables: The classified according to compared with randomexposure being studied exposure status and then ized studies may correlate with the followed to determine Allows for the study of expodisease outcome but may the effect of exposure on sures that are known or not be the cause. There disease outcome suspected to be harmful may be other “confoundEstablishes a causal relationing” variables that are ship between exposure more causative. and outcome variables

Case-control study

Subjects are classified acEasiest study type to set up cording to the presence Allows for the study of expoor absence of disease, sures that are known or and correlations are suspected to be harmful made between past exposures and the presence of disease

Randomized study

Participants are randomly as- Provides evidence for cause Costly and time-consuming signed to exposure groups and not just correlaCannot be used to study and followed for the tion because only one exposures that are development of disease “exposure” is manipulated known or suspected to be at a time harmful

May be affected by recall bias and interviewer bias (see Case 25.1, question 4) Does not allow for calculation of the relative risk or the percentages of those in the exposed versus unexposed groups who go on to develop disease Confounding variables are often present

Case 25.1 In the mid-1800s, London was plagued by recurrent outbreaks of cholera, which extracted a high death toll. Although the cause of these outbreaks was unknown, the prevailing hypothesis was that cholera was spread by “miasma,” a poisonous odor emitted from decaying organic material found in open graves, sewers, and swamps. The now-famous epidemiologist John Snow disagreed with the miasma theory and postulated instead that cholera was spread by contaminated water. He believed this in part because the initial symptoms of cholera were intestinal in nature, and he reasoned that an inhaled poisonous odor would not manifest symptoms in this way.



578  Biostatistics To study this hypothesis, he reviewed death certificates and plotted addresses for each person in whom the death certificate implied death from cholera infection. On a map of London, he then mapped out where these people had lived before their death and compared their location to those who died of causes unrelated to cholera infection. What he found was that the incidence of cholera was much higher in London residences that obtained their water supply from a particular water pump.   

1. What sort of study design was this?   This was a case-control study because participants were selected on the basis of either having or not having the disease of interest (cholera). 2. How does a retrospective case-control study differ in design from a retrospective cohort study?   These studies differ largely with respect to how subjects are classified and selected. In a case-control study, subjects are classified according to the presence or absence of disease. By contrast, in a retrospective cohort study, subjects are classified based on the presence or absence of exposure, and only then is disease status determined. 3. What are the strengths of a case-control study?   Case-control studies are relatively easy to set up because the researcher simply has to locate people who have been affected by a disease. Another strength is that case-control studies can be used to look at the effect of exposures that are known or suspected to be harmful. For example, a randomized study could not determine whether having a previous history of child abuse increases the likelihood that the victim will in turn abuse his or her own children, because it would be unethical and illegal to randomize participants to be abused. However, with a case-control study, a researcher could look at child abusers as well as nonabusers and compare the incidence of abuse during their childhoods. 4. What are the limitations of a case-control study?   Although case-control studies are easy to design, they have a number of flaws. As mentioned, case-control studies simply uncover an association between two variables but do not establish a causal relationship. For example, if you did a case-control study and found that those with lung cancer have higher rates of alcoholism, you might conclude that alcoholism leads to lung cancer. However, alcoholics might be more likely than nonalcoholics to smoke cigarettes, and smoking could be the actual cause of their lung cancer.     Another flaw in case-control studies is “recall bias,” which is the tendency of those with a disease to exaggerate their exposures and those without a disease to minimize their exposures. For example, a woman with a child who has been born with a birth defect might recall many more chest x-ray studies during her pregnancy than might a woman with healthy children. “Interviewer bias” is also an issue. This occurs when an interviewer assumes a person with the disease of interest has been exposed to risk factors that the healthy person has not and thus changes the manner in which he or she asks a particular question. For example, the interviewer might ask a person with lung cancer the question “How many packs per day did you smoke?” whereas a healthy person might be asked the question “You never smoked, did you?” Even though both questions ask for similar information, the sense of judgment imposed by the latter may deter the interviewee from providing accurate responses. Additional types of statistical bias are reviewed in Table 25.8. Table 25.8.   Statistical Bias EXAMPLE

METHODS TO REDUCE BIAS

BIAS TYPE

DEFINITION

Selection bias

General term for nonrandom Administering a survey on hospital sat- Randomize participants assignment of participants to isfaction to the “less sick” patients Use the correct control group various groups within a study

Berkson’s bias

A type of selection bias involving When studying the incidence of celluli- Use the correct control hospitalized patients, who tis in diabetic patients, only hosgroups (in this ­example, tend to be sicker than the pitalized patients are examined; one should survey general population this rate is likely to be inflated diabetics in an outparelative to that of the general tient setting as well as diabetic population because these nondiabetic, hospitalized patients required some reason for patients) hospitalization

Recall/­ responder bias

Knowledge of having a disorder alters one’s ability to recall data regarding past exposures

When surveyed for a study that looks at the effect of fiber intake on colon cancer risk, patients with colon cancer provide a more detailed dietary history than patients without colon cancer

Reduce time from exposure to follow-up Avoid leading questions and ask specific follow-up questions (in this example, ask all subjects if they consumed specific foods in addition to asking more open-ended questions)

Biostatistics  579

Table 25.8.   Statistical Bias—cont’d EXAMPLE

METHODS TO REDUCE BIAS

BIAS TYPE

DEFINITION

Confounding bias

The effect of an independent The association between eating more Crossover trials (subjects variable on a dependent varivegetables and living longer may receive a sequence of able is distorted by a third, be confounded by the fact that different exposures and unmeasured variable people who eat a lot of vegetables act as their own controls) tend to exercise more, and regular Adequately control for exercise promotes longer life span confounding variables (in this example, one should include a group that consumes lots of fruit and vegetables but does not exercise)

Hawthorne effect

Patients change their normal behavior once they know they are being studied

Participants enrolled in a dietary recall Use long-term methods that study to examine normal fiber make it more difficult intake within the general populato sustain unnatural tion may increase their fruit and behaviors vegetable intake during the study Be discreet with expectations period to obtain more fiber to prevent subjects from altering their behaviors in order to conform to the hypothesis

Lead-time bias Early detection of a disease is mistaken for increased survival

Diagnosis of breast cancer with a Normalize survival length to ­novel technology claims to increase the severity of disease at patient survival by 6 months, but the time of diagnosis this may be due to earlier stage diagnosis rather than any change in the course of the disease

Observer bias

Observer’s reporting is biased because of knowledge of exposure status (not doubleblinded)

Researcher reports a reduced incidence of depression in a group that he knows is currently placed on a new SSRI

Pygmalion effect

A type of observer bias in which a researcher’s own expectations for the outcome of a study (i.e., personal belief in the efficacy of a therapy) influence the study outcome

In investigating the effect of a new Use placebo groups and pain medication, the researcher double-blinded methods may talk to subjects in the treatment group with greater respect than those in the placebo group, which may ultimately influence the survey results

Procedure bias Subjects in different study groups are not treated equally

Sampling bias

Use placebo groups and double-blinded methods

In a weight loss study in which one Use double-blinded methods group is placed on a diet pill + exercise routine and the second is placed on a placebo + exercise routine, the former group is trained more intensely than the latter; differences in exercise routines may account for some of the observed weight loss

Selecting participants who do Subjects from an upper-class town are Use random samples (ideal) not represent the makeup of selected for participation in a study Use population-based the overall population, so that that examines factors contributing controls findings will not be generalizto heart attack risk. These subjects able may be influenced by financial factors that do not influence the general population but may nevertheless affect heart attack risk Continued

580  Biostatistics Table 25.8.   Statistical Bias—cont’d METHODS TO REDUCE BIAS

BIAS TYPE

DEFINITION

EXAMPLE

Late-look bias

Acquisition of data at an incorrect or inappropriate time may influence the study results

In a study investigating the impact of Stratify groups by disease stroke on quality of life, those with severity the most fatal form of disease (i.e., those who have passed away from stroke) will not be able to participate in the study, thus skewing the results

Measurement bias

Acquisition of data in a manner that distorts it

In a study that examines the effect Use standardized methods of of cold weather on weight gain, data collection participants are weighed with Avoid leading questions clothes and shoes on; participants are more likely to be wearing bulky, heavier clothing in the winter than in the summer

SSRI, selective serotonin reuptake inhibitor.

STEP 1 SECRET Bias occurs when one outcome is favored over another due to systematic error. The various types of statistical bias are commonly tested on Step 1. You should know the types of bias defined in Table 25.8. As you can see, including the correct control groups is extremely important for eliminating bias!

5. What ratio can be used to compare event rates in a case-control study?   The OR can be used to compare event rates in a case-control study (know this!). Case-control studies cannot be used to calculate the RR of an exposure on the disease outcome or to define the percentage of people with a certain exposure who will go on to develop a disease. This is because the calculation of RR requires the incidence rate, which cannot be calculated from a case-control study because there is no follow-up time. OR is used for case-control studies, and RR is used for cohort studies.

Case 25.1 continued: When John Snow charted the water supply of 100 people who died of cholera and 100 people who died of other causes, he found that 80 of the 100 people who died of cholera lived in homes supplied by a particular water pump, but only 10 of the 100 people who died of other causes lived in homes supplied by this same water pump.   

6. What is the odds ratio of developing cholera if the water supply was provided by the particular water pump in question?   36 80/20 80 × 90 7200 OR = = = = 36 10/90 20 × 10 200     An OR greater than 1 implies that the disease is more likely to occur in the exposed group, so cholera occurred more often in those who received their water supply from this particular water pump. This does not prove that the water was the source of the infection, because there may have been confounding variables that were the actual cause. For example, it may have been that those who lived in homes supplied by this water pump were poorer than those who had their own wells, and therefore they had jobs where they were exposed to less sanitary conditions. These unsanitary working conditions, rather than the water in their homes, may have then exposed them to cholera.

SUMMARY BOX: CASE-CONTROL STUDIES • Subjects are classified according to presence or absence of disease. • Strengths: Easy to set up, cost efficient, can be used to study exposures that are known or suspected to be harmful • Limitations: Leave room for recall bias, interviewer bias, and confounding variables • Cannot be used to define relative risk (RR) but can be used to calculate an odds ratio (OR)

Biostatistics  581

Case 25.2 Blood pressure is monitored regularly in a group of 500 adult men. The mean blood pressure for the group is reported as 130 ± 10 mm Hg, and the blood pressure measurements within the group are described as having a “normal distribution.”   

1. What does it mean when the blood pressure in this population is said to be “normally distributed”?   To be normally distributed means that if a plot of the magnitude of the variable being analyzed (in this case, blood pressure) against the frequency of each magnitude is made, the curve takes on a “bell-shaped” form that is well described by a specific mathematical equation, which can be used to accurately calculate the standard deviation. A normal distribution (see Fig. 25.3) represents one in which the majority of participants have measurements close to the mean, because mean, median, and mode are all equal in a normal curve. Curves are often assumed to be normal for the sake of easy calculations, but some curves differ largely from a normal curve. For example, if you asked a group of people what temperature they like their coffee, most would say either very hot or very cold. Almost no one would say that they like lukewarm coffee. This study would produce a “bimodal” distribution (Fig. 25.4) with two “humps” or modes, which would not fit the normal curve. f(x)

Mode

Mode

x

Figure 25.4.  Bimodal distribution.

2. What percentage of participants had blood pressures in the range of 120 to 150 mm Hg?   Eighty-one and a half percent of the participants had blood pressures in the range of 12 to 150 mm Hg.     Recall that in a normal distribution, 68% of the data falls within 1 standard deviation of the mean, 95% of the data falls within 2 standard deviations of the mean, and 99.7% of the data falls within 3 standard deviations of the mean. Therefore, 68% of data will fall within the range of 130 ± 10 mm Hg (120–140 mm Hg) and 95% of the data will fall within the range of 130 ± (2 × 10) mm Hg (110–150 mm Hg). But this tricky question asks what percentage of participants is in the range of 120 to 150 mm Hg. You must therefore realize that the lower value of this range (120 mm Hg) falls within 1 standard deviation of the mean, and the upper value of this range (150 mm Hg) falls within 2 standard deviations of the mean. Therefore, 34% of people will have blood pressures within the range of 120 and 130 mm Hg, while 47.5% of people will have blood pressures within the range of 130 to 150 mm Hg (95/2 = 47.5). 34 + 47.5 = 81.5%.

SUMMARY BOX: NORMAL DISTRIBUTION AND STANDARD DEVIATION • Normal distributions represent bell-shaped curves in which mean, median, and mode are all equal. • Standard deviation is used to estimate the percentage of a population that falls into a certain range, as long as the population fits a normal distribution: • 68% of individuals fall within 1 standard deviation of the mean • 95% of individuals fall within 2 standard deviations of the mean • 99.7% of individuals fall within 3 standard deviations of the mean

Case 25.3 A 27-year-old man complains of fatigue and general malaise beginning several months earlier. Although his past medical history is unremarkable, his more recent history is significant for the use of intravenous drugs and for unprotected sex with prostitutes. With the patient’s consent, you screen him for human immunodeficiency virus (HIV) infection using a test with a reported sensitivity of 95% and specificity of 75%.   

582  Biostatistics 1. Why does it make sense to use a screening test with a high sensitivity, even at the cost of specificity, for this patient?   Screening tests in general, and particularly for life-threatening diseases such as HIV infection, should have a high sensitivity so that they are likely to detect the disease if it is present. Because screening tests must generally be inexpensive, this high sensitivity may come at the cost of a suboptimal specificity. However, because it is much more important to avoid missing a life-threatening disease (i.e., few false negatives) than it is to inconvenience (or even traumatize!) someone with a false positive result, this is considered acceptable.     Not all screening tests have high sensitivity. When used as a single data point, mammograms and Papanicolaou (Pap) smears, for example, have low sensitivity. However, when used on a regular basis (e.g., annually), they become much more effective screening tools because of a high cumulative sensitivity. 2. If this patient tests positive, is it reasonable to tell him that you are 95% confident that he is infected with human immunodeficiency virus?   No. Sensitivity and specificity values simply represent how good a test is at ruling in or ruling out a disease, and perhaps whether the test is ideal for screening large populations for a given disease. Although a positive test result will undoubtedly be concerning to the clinician and the patient, with the information provided there is no way of determining if this is a true positive or a false positive result. What is needed to calculate the PPV for this test depends on additional information (prevalence of the disease in the specific population within which the patient falls), as discussed in the next case. 3. What if the test comes back negative? Can you tell this patient that you are 75% confident that he does not have human immunodeficiency virus infection?   Again, such a statement cannot be made unless you know the NPV of the test, which was not provided. 4. Now let’s assume that a 90-year-old woman and our young drug-abusing citizen in this vignette both test positive for human immunodeficiency virus using this test. Are they both equally likely to have the disease?   No, and this question addresses the important concept of utilizing screening tests appropriately. The goal of clinicians is to selectively screen only those individuals at higher risk for developing a given disease. This is because the PPV of a test depends on the prevalence of the disease in the given population being tested as well as on the specificity and sensitivity of the test. The prevalence of HIV in a 90-year-old woman is much lower than in young intravenous drug users. Therefore, if the elderly woman tests positive for HIV, she is much more likely to have a false positive than our other patient is, because she had a smaller pretest probability of having HIV.     Consider the havoc that would be created if physicians screened all women starting at the age of 20 for breast cancer by performing annual mammograms. Given that the prevalence of breast cancer in young women is low, such testing would yield numerous false positives, necessitating unnecessary referrals and expensive workups by specialists (not to mention a lot of unneeded anxiety!). Using this same test to screen only women over 40 makes a bit more sense, as the number of true positives will increase and the number of false positives will decrease owing to the increased prevalence of breast cancer with aging.

SUMMARY BOX: LIKELIHOOD RATIOS, PREDICTIVE VALUES, AND PRINCIPLES OF SCREENING • Sensitivity of screening tests should be high to avoid missing individuals with disease. • Positive predictive value (PPV) and negative predictive value (NPV) can determine the likelihood that a person has a disease if he or she tests positive or negative for that disease. Sensitivity and specificity do not do this. • PPV and NPV depend on disease prevalence.

Case 25.4 In a town of 1000 individuals, the prevalence of coronary artery disease across all age groups is 20% (as determined by angiography, the “gold standard”). You have created a wonderfully inexpensive screening test that you believe is both highly sensitive and specific for detecting coronary artery disease. Based on your test results, you create the following 2 × 2 table (Table 25.9).   

Table 25.9.   Testing for Coronary Artery Disease CORONARY ARTERY DISEASE (+)

CORONARY ARTERY DISEASE (−)

(+) Test Result

180 (a)

80 (b)

(−) Test Result

20 (c)

720 (d)

Biostatistics  583

1. Given the data presented in the 2 × 2 table in Table 25.9, what is the sensitivity of this new test?   The sensitivity is 90%. Sensitivity can be calculated by dividing the number of true positives by the total number of persons tested with the disease, or a/(a + c) in the 2 × 2 table. There were 180 true positives of the 200 patients tested who had disease, yielding a sensitivity of 180/200, or 90%. This means that this test detects the disease in 90% of people who have the disease. 2. What is the specificity of this new test?   The specificity is 90%. Specificity can be calculated by dividing the number of true negatives by the total number of people tested who do not have the disease (true negatives plus false positives), or d/(b + d) in the 2 × 2 table. There were 720 true negatives and 80 false positives, so the specificity of this test is 720/800, or 90%. 3. What information can be obtained from calculating the positive likelihood ratio?   As described in the Basic Concepts section, the PLR reflects how much a positive test result increases the probability of the presence of disease (i.e., indicates posttest probability of disease). The higher the ratio, the more likely it is that disease is present. This ratio is calculated as the sensitivity divided by 1 − specificity. PLR = sensitivity/1 − specificity

   

For this example, given that sensitivity and specificity are both 90%, the PLR can be calculated as: PLR = .90/1 − .90 = .9/1 = .9

    A positive test result tells us that this patient’s posttest probability for having the disease in question is 9 times greater than his pretest probability. It is often up to the clinician to determine how to factor this new information into his or her management plan following the outcome of such a test. 4. How can the positive predictive value of this test be calculated?   It can be calculated (by using Table 25.9) as the number of true positives divided by the total number of positive results (true and false). This calculation requires knowledge of sensitivity and specificity of the test, as well as the prevalence of the disease (acquired in this case from Table 25.9), to generate the number of true positives and false positives. Positive predictive value = TP/(TP + FP) = 180/180 + 80 = 69%

    So you would tell your patient that he is only 69% likely to have coronary artery disease based on his positive test result.

STEP 1 SECRET It is very likely that you will not be provided with a ready-made 2 × 2 table on the USMLE Step 1. How can you calculate PPV if all you are given is sensitivity (90%), specificity (90%), and disease prevalence (20%)? The trick here is to create your own table by inserting sample numbers that fit the data. We recommend using very simple numbers to do this. We will work through this together. Let’s presume that our sample population consists of 500 people. If disease prevalence is 20%, then 100 people in our population have coronary artery disease, while 400 people do not (Table 25.10, Part A). If the sensitivity of our test is 90%, then the value for (a) must be 90, because 90% of 100 is 90. This means that the value for (c) must equal 10 (see Table 25.10, Part B). If the specificity is 90%, then the value of (d) must be 360 (90% of 400 = 360), and we can subtract 360 from 400 to yield 40, the value of (b) (see Table 25.10, Part C). Now simply calculate the PPV based on these values. PPV = TP/(TP + FP) = 90/(90 + 40) = 69% You will see that we yield the same answer that we did in question 4!

5. What is the negative predictive value for this test?   The negative predictive value is 97%. The NPV is the probability that the disease is absent if the test is negative. It is calculated as true negatives divided by both true negatives and false negatives: Negative predictive value = TN/(TN + FN) = 720/(720 + 20) = 0.97 or 97%

   

So, 97% of the people in this sample who had a negative test result would not have the disease.

584  Biostatistics Table 25.10.   Setting Up a Sample Table to Determine Positive Predictive Value CORONARY ARTERY DISEASE (+)

CORONARY ARTERY DISEASE (−)

(+) Test Result

___ (a)

___ (b)

(−) Test Result

___ (c)

___ (d)

Total

100

400

(+) Test Result

90 (a)

___ (b)

(−) Test Result

10 (c)

___ (d)

Total

100

400

(+) Test Result

90 (a)

40 (b)

(−) Test Result

10 (c)

360 (d)

Total

100

400

TOTAL

A

500

B

500

C

500

SUMMARY BOX: SENSITIVITY, SPECIFICITY, LIKELIHOOD RATIOS, AND PREDICTIVE VALUES • Sensitivity, specificity, positive predictive value (PPV), and negative predictive value (NPV) can be calculated using a 2 × 2 table. It is very likely that you will have to create this table for yourself using sample numbers! • PPV is used to express the likelihood that a positive test result indicates presence of disease. NPV is used to express the likelihood that a negative test result indicates absence of disease. • Positive likelihood ratio (PLR) reflects the degree to which a positive test result increases the risk of disease. PLR = Sensitivity/(1 – specificity).

Case 25.5 In the 1940s, a study was performed on employees at a nuclear power plant to determine if an association exists between radiation exposure and cancer rates. In this study, 500 employees with high-level radiation exposure and 500 employees with very limited exposure were followed for 10

Table 25.11.   Radiation Exposure and Cancer Risk CANCER (+)

CANCER (−)

(+) Radiation exposure

50 (a)

450 (b)

(−) Radiation exposure

5 (c)

495 (d)

years, and the incidence rates for cancer were compared in the two groups throughout this time. The results are depicted in the 2 × 2 table shown in Table 25.11.   

1. What type of study design is this?   This is a (prospective) cohort study because individuals are classified on the basis of exposure, not disease (as with a case-control study). Recall that cohort studies compare groups with or without exposure to a variable of interest to determine how that variable affects development of disease. Furthermore, this was an ongoing study in which the complications associated with radiation exposure were analyzed as they occurred. 2. What is the difference between a prospective cohort study and a retrospective cohort study?   In a prospective cohort study, individuals with a risk factor (i.e., exposure) for disease are followed over time to see whether they do or do not develop disease. In a prospective study, disease outcome is to be determined. In a

Biostatistics  585

retrospective cohort study, a group of individuals who had been exposed to a variable of interest are retrospectively examined for development of disease. In other words, the study examines who already developed disease based on their past exposure status. Note that in both cases, the study population is grouped according to exposure. 3. What is the major limitation of cohort studies?   The major limitation of cohort studies is the inability to distinguish correlation from causality. Although the groups may be distinct from each other according to the factor being studied, there are many other factors that may be different between the groups that could influence outcome (confounding variables). For example, a cohort study found that people who eat more β-carotene have a lower incidence of lung cancer. However, this did not take into account that people who eat more β-carotene may eat substantially more fruits and vegetables in general, which itself may be protective from cancer. In fact, when a randomized trial was done, β-carotene supplementation actually increased the risk of lung cancer. 4. On the basis of data presented in Table 25.9, what is the relative risk for cancer in the exposed group?   The RR for cancer in the exposed group is 10. Recall that RR is determined by comparing incidence rates in exposed individuals (IE) to incidence rates in nonexposed individuals (INE), as shown in Eq. 25.11. Thus, the RR for the employees exposed to radiation is 10 times greater than for the nonexposed employees. RR =



IE INE

=

a/(a + b) c/(c + d)

=

50/500 5/500

= 10[25.11]

Incidence

AR

IE INE

Nonexposed

Exposed

Figure 25.5.  Attribute risk (AR), where INE is the incidence of disease within a nonexposed group and IE is the incidence of disease within an exposed group. AR is the difference in disease incidence between these two groups and can be calculated by subtracting INE from IE.

5. What is meant by attributable risk and attributable risk percent? Calculate both for the preceding example.   Attributable risk (AR), also referred to as the absolute risk, represents the difference in disease risk between two groups based solely on exposure status. It can be determined by the difference in incidence rates between exposed and nonexposed groups (Fig. 25.5).     Let’s calculate AR for the preceding example: AR = IE − INE = 50/500 − 5/500 = 45/500 = 0.09     This AR of 0.09 implies that 9% of people exposed to radiation developed cancer as a result of that exposure (i.e., which could be attributed to that exposure).

586  Biostatistics     The attributable risk percent (AR%) is a measure of the percentage of people who were exposed and developed the disease, and in whom the development of disease was due to the exposure. It can be calculated by dividing the AR by the incidence of disease in the exposed group: AR% = AR/IE × 100 = 0.09/0.10 × 100 = 90%     This AR percent of 90% implies that 90% of people who were exposed to radiation and developed cancer developed their cancer as a result of the radiation.     Absolute risk reduction (ARR) is calculated in the same way as for AR but is used in reference to exposures that reduce one’s chances of acquiring the disease outcome. ARR = INE − IE[25.12]



    For example, if you found a reduction in cholesterol levels in a group taking statins compared with a group not currently on cholesterol-lowering medication, you would use the term absolute risk reduction to describe the difference in risk of developing high cholesterol between exposed and unexposed groups.     The absolute risk reduction percent (ARR%) is similar to AR% in the same way that ARR is similar to AR. It is a measure of the percentage of people who were not exposed and developed the disease, and in whom the development of disease was due to lack of exposure. ARR% = ARR/INE × 100[25.13]



    Number needed to treat (NNT) refers to the average number of patients that must be treated (exposed) to prevent one from developing disease. It is inversely related to ARR. NNT = 1/ARR [25.14]

   

Number needed to harm is calculated using the same concept, except that it equals 1/AR.

STEP 1 SECRET You should know how to calculate odds ratio, relative risk, absolute risk reduction, attributable risk, and number needed to treat/harm. These are helpful formulas to add to your whiteboard before the start of your exam!

6. What experimental design overcomes the shortcomings of the cohort study?   The experimental design that is least susceptible to confounding factors and bias is the randomized controlled trial. In this trial design, individuals are randomly allocated to treatment or control groups, thereby reducing considerably the effects of any confounding factors.     The most highly regarded kind of randomized study is a double-blind placebo-controlled trial. Double-blind means that neither the investigators nor the study subjects know who is receiving the treatment. Placebo-controlled means that those who do not receive the treatment being tested receive a placebo instead, which should be similar enough to the treatment that the participants cannot tell whether they are receiving the placebo or the treatment.     The data from randomized controlled trials may be analyzed by a number of different methods. Among the most common are intention-to-treat (ITT) and as-treated (non–intent-to-treat) analyses. The ITT approach includes all randomized participants in the groups to which they were initially assigned regardless of adherence with the study criteria or withdrawal from the study. This approach reduces bias that may be introduced by patients dropping out of the study, noncompliance with treatment, or patient reassignment to another treatment arm by nonrandom means. The as-treated approach analyzes patients based on the final treatment regimen received rather than the initial randomization. This approach is susceptible to bias from the aforementioned factors but is intended to provide a better estimate of treatment efficacy within general practice given inevitable treatment nonadherence.

SUMMARY BOX: COHORT STUDIES AND RANDOMIZED CONTROL TRIALS • Classifies subjects based on exposure status • Limitation: Presence of confounding variables • Uses relative risk to express the magnitude of association between exposure and disease status (Only cohort studies and randomized control trials use relative risk (RR); case-control studies do not use RR since incidence of disease cannot be determined.) • Randomized double-blind placebo-controlled trial is the gold standard of experimental design.

Matthew N. Anderson, MD, Melina Benson, Thomas A. Brown, MD, and Sonali J. Bracken

CHAPTER 26

CLINICAL ANATOMY

Insider’s Guide to Clinical Anatomy for the USMLE Step 1 Students often wonder how to study anatomy for boards. You may have noticed that the anatomy sections of First Aid are rather sparse in comparison with the depth at which you may have studied this subject in your medical school curriculum. Do not interpret this to mean that anatomy will not be present on your examination. Although some students have reported having relatively few anatomy questions, others have found 5 to 6 questions per block. We share this information not to frighten you but rather to give you the idea that boards considers anatomy to be important. Unfortunately, it is much more difficult to prepare for anatomy than it is for some of the other subjects on boards. There is simply too much material for you to be able to learn everything at this point. In consideration of this reality, how should you go about tackling this subject? Simple: be realistic and do not expect to know it all. Focus on high-yield topics. In other words, this is not the time to relearn each branch of every nerve and all of the origins and insertions of every muscle in the body. You are welcome to do this if you would like, but you would be compromising time that you could be spending on more “test-worthy” topics. The most important thing to keep in mind is that board anatomy questions are clinically based. You will be given clinical vignettes for which you will be asked to relate patients’ symptoms to anatomic lesions and deformities. Expect to see x-ray films, magnetic resonance imaging (MRI) studies, computed tomography (CT) scans, and angiograms. You may be given a question in which you must first determine the site of the lesion based on the patient’s symptoms and then locate the deformed structure on a radiograph. Spend some time perusing an anatomy atlas and some credible online sites for these types of images. No matter how much you prepare for boards, you will encounter questions on material that you failed to study. Do not let this frustrate you. Make your best guess and move on. You can still earn a terrific score if you miss these random questions. Center the majority of your study time on the topics with the best odds of appearing on your examination. This chapter will help guide you to high-yield anatomy topics for boards. Neuroanatomy-specific tips are discussed in Chapter 17, Neurology.

Case 26.1 A 68-year-old retired man presents with a 2-year history of pain and cramping of the lower extremities (LEs) with walking. This has not been much of a problem, but for the past month, he has noticed pain in his right foot that awakens him from sleep. He has a history of myocardial infarction (MI), type 2 diabetes mellitus, erectile dysfunction, and a 40-pack-year history of smoking.   

1. What is the differential diagnosis for his foot and lower extremity pain?   Peripheral vascular disease (PVD), neurogenic causes (e.g., disk herniation), arthritis causing spinal stenosis, diabetic neuropathic pain, and coarctation of the aorta are considerations.

Case 26.1 continued: Upon further questioning, you find that the pain and cramping are absent at rest and begin after about 5 minutes of walking. Stopping for a short period relieves the pain. The right foot pain occurs only at night and is relieved by hanging the foot over the side of the bed. On physical examination, light touch, pinprick, vibration, and temperature sense are intact on the LEs bilaterally. He is noted to have intact femoral pulses, weak popliteal pulses, weak posterior tibial pulses, a weak left dorsalis pedis pulse, and an absent right dorsalis pedis pulse.   

2. What is the most likely diagnosis?   PVD, which is characterized by claudication symptoms (pain and cramping of the LEs with walking) that appear after a specific walking distance and resolve after a specific duration of rest. The peripheral pulse findings on physical examination also are strongly suggestive of PVD. 3. Which historical features in this patient increase the likelihood of a peripheral vascular disease diagnosis?   Smoking and diabetes mellitus are strong PVD risk factors. Patients with PVD often have evidence of atherosclerosis elsewhere, as demonstrated by this patient’s past MI and erectile dysfunction. Other PVD risk factors include 587

588  Clinical Anatomy hypertension, hypercholesterolemia (most notably increased low-density lipoprotein [LDL]), obesity, sedentary lifestyle, and a family history of atherosclerotic disease. 4. Why does his nocturnal right foot pain resolve when he hangs the affected foot over the bedside?   Rest pain commonly occurs in the feet at night in PVD. When a patient is supine, there is no gravitational assistance in foot blood flow. Reduced blood flow results in ischemia and pain. Hanging the foot over the side of the bed places the foot below the level of the heart; then gravity increases the flow of blood to the ischemic areas, reducing the pain. 5. Describe the path of arterial blood from the heart to the femoral sheath.   Blood leaves the heart through the aortic valve to enter the ascending thoracic aorta, arch of the aorta, and descending thoracic aorta (Fig. 26.1). The thoracic aorta passes through the aortic hiatus of the diaphragm at the level of T12 to become the abdominal aorta. The abdominal aorta bifurcates into the right and left common iliac arteries at the level of L4. Each common iliac artery bifurcates into the internal and external iliac arteries just anterior to the sacroiliac joint. The internal iliac primarily supplies pelvic structures, while the external iliac runs deep to the inguinal ligament to become the common femoral artery.

Right common carotid artery Right subclavian artery Innominate artery Ascending aorta

Left common carotid artery Left subclavian artery Arch

Descending thoracic aorta

Abdominal aorta

Common iliac artery Femoral artery Femoral vein Femoral nerve

Figure 26.1.  Path of arterial blood from the heart to the femoral sheath.

Clinical Anatomy  589

6. Outline the borders of the femoral triangle.   The femoral triangle is bordered by the inguinal ligament (superiorly), the sartorius (laterally), and the adductor longus (medially). From lateral to medial, it contains the femoral Nerve, common femoral Artery, femoral Vein, femoral canal (Empty space containing lymph nodes that is the site of femoral hernias), and the deep inguinal Lymph nodes. The classic mnemonic is NAVEL (Fig. 26.2). A way to remember that NAVEL is lateral to medial is that you go from a lateral to medial direction to find your NAVEL.

External iliac artery and vein Internal iliac artery

Inguinal ligament Lateral femoral cutaneous nerve

Femoral nerve Iliopsoas Pectineus Long saphenous vein

Femoral nerve Profunda femoris artery Femoral artery Tensor fasciae latae

Femoral vein Adductor longus Rectus femoris Gracilis

Sartorius

Vastus lateralis

Vastus medialis

Articular branch (descending genicular artery) Patellar anastomosis

Patellar ligament

Figure 26.2.  Femoral triangle. (From Paulsen F, Waschke J. Sobotta Atlas of Human Anatomy. Vol. 1. Urban & Fischer: Elsevier; 2013.)

590  Clinical Anatomy 7. Describe the path of arterial blood from the femoral sheath to the feet.   The common femoral artery quickly bifurcates at L4 into the profunda femoris (supplying primarily the thigh) and the superficial femoral artery, which runs through the adductor canal to the popliteal fossa (via the adductor hiatus) to become the popliteal artery. In the posterior compartment of the leg, the popliteal artery bifurcates into the tibiofibular trunk and the anterior tibial artery, which perforates the superior most portion of the interosseous membrane and descends in the anterior compartment until it crosses the ankle joint to become the dorsalis pedis artery, which can be palpated on the dorsum of the foot lateral to the tendon of the extensor hallucis longus. The tibiofibular trunk bifurcates to become the fibular artery, which runs downward in the deep posterior compartment of the leg, and the posterior tibial artery, which can be palpated between the medial malleolus and the calcaneus before bifurcating to form the lateral and medial plantar arteries, which supply the plantar aspect of the foot (Fig. 26.3).

Common iliac

External iliac Common femoral Profunda femoris Superficial femoral Proximal popliteal Definitive anatomy

Distal popliteal

Anterior tibial

Posterior tibial Peroneal

Remnants of axial artery

Figure 26.3.  Remnants of the axial artery and arteries that develop with later differentiation are indicated. (From Levien LJ, Benn C. Adventitial cystic disease: a unifying hypothesis. J Vasc Surg. 1998;28(2):193-205.)

8. At which sites is arterial plaque formation most likely?   The most likely sites of plaque formation are arterial branch points, such as the bifurcation of the tibiofibular trunk and anterior tibial artery, and tethered arteries, such as the superficial femoral artery in the adductor canal. Pathophysiologically, the turbulent blood flow occurring at branch points and tethering sites causes shear forces on the endothelium, increasing the likelihood of endothelial damage and potentially leading to atherosclerosis. Such changes are evident in this patient, who appears to show significant bilateral superficial femoral atherosclerotic narrowing (given intact pulses at the femoral triangle and weak pulses at the popliteal fossa) and marked right anterior tibial narrowing (supported by the absence of the dorsalis pedis pulse).

Clinical Anatomy  591

9. In Table 26.1, cover the two columns on the right, and attempt to list the drug class and mechanism of action for each of the drugs commonly used in treatment of peripheral vascular disease. Table 26.1.   Selected Drugs Used to Treat Peripheral Vascular Disease DRUG

CLASS

MECHANISM OF ACTION

Aspirin

Nonselective cyclooxygenase inhibitor

Irreversibly inhibits COX-1, decreasing platelet production of thromboxane A2, a vasoconstrictor and promoter of platelet aggregation

Clopidogrel

P2Y12 antagonist

Irreversibly inhibits P2Y12, a platelet ADP receptor necessary for activation of the glycoprotein IIb/ IIIa pathway of platelet aggregation

Statins (e.g., atorvastatin, rosuvastatin, fluvastatin, lovastatin, pravastatin, simvastatin)

HMG-CoA reductase inhibitors

Inhibit HMG-CoA reductase, the rate-limiting step in endogenous production of cholesterol, to reduce the buildup of cholesterol in atherosclerotic plaques. Results in upregulation of LDL-receptor and increased clearance of circulating LDL.

Cilostazol

Phosphodiesterase III inhibitor

Inhibits cAMP phosphodiesterase III to reduce cAMP degradation, vasodilating peripheral arteries and inhibiting platelet aggregation

Pentoxifylline

Phosphodiesterase inhibitor

Inhibits cAMP phosphodiesterase; increases platelet flexibility and decreases blood viscosity

ADP, adenosine diphosphate; cAMP, cyclic adenosine monophosphate; CoA, coenzyme A; COX-1, cyclooxygenase-1; HMG, hydroxymethylglutarate; LDL, low-density lipoprotein.

SUMMARY BOX: PERIPHERAL VASCULAR DISEASE • Peripheral vascular disease (PVD) is caused by atherosclerotic narrowing of peripheral arteries, usually of the lower extremities (LEs). • Atherosclerotic lesions form preferentially at branch points and sites of tethering, as a result of turbulent blood flow and endothelial shear stress. • Blood flow to the LE: aorta → common iliac → external iliac → common femoral → profunda femoris (ends in thigh) and superficial femoral → popliteal → anterior tibial (supplies anterior compartment and ends as dorsalis pedis in the dorsum of the foot) and tibiofibular trunk → fibular (supplies lateral compartment) and posterior tibial (supplies posterior compartment) → lateral and medial plantar arteries. • From lateral to medial, the femoral triangle contains the femoral Nerve, Artery, Vein, canal (Empty space), and the Lymph nodes. Remember the acronym NAVEL.

Case 26.2, Part A A 22-year-old college student presents to the emergency department (ED) after rear-ending a car while driving his new motorcycle. He claims that the major site of impact was his left shoulder, but his head and neck were wrenched to the right as well. He is clearly intoxicated and has managed to sit up, though his left upper extremity (UE) hangs by his side in medial rotation. His forearm is extended and pronated and his wrist is frozen in flexion. The intern on call claims to be able to diagnose his injury from across the room.   

1. What structure has been injured, and how has it led to his upper extremity position?   He has damaged nerve roots C5 and C6 (or the upper trunk) of the brachial plexus, resulting in a condition known as Erb-Duchenne palsy. His left UE hangs by his side in medial rotation because the C5 component of the axillary nerve is necessary for shoulder flexion and abduction (via the deltoid) and lateral rotation (via the teres minor). His forearm is extended and pronated because C5 and C6 are the main components of the musculocutaneous nerve, which supplies the two major forearm flexors (brachialis and biceps) and the major forearm supinator (biceps). His wrist is flexed because the C6 component of the radial nerve is necessary for wrist extension (via the extensor muscles of the posterior compartment of the forearm). Remember that an upper brachial plexus injury (C5-6) gives you the “waiter’s tip position” (Fig. 26.4).

592  Clinical Anatomy

Figure 26.4.  “Waiter’s tip position” caused by upper brachial plexus injury.

2. If he had forced his upper extremity above his head by grabbing the handlebars of the motorcycle to prevent his fall, he may have presented with a loss of sensation and impaired flexion in digits 4 and 5, impaired wrist flexion, hyperextension of the metacarpophalangeal joints, and an inability to abduct and adduct digits 2 to 5. What would be the diagnosis in this situation?   This pattern of injury is characteristic of a tear of nerve roots C8 and T1 (or a lower trunk tear) of the brachial plexus, a condition known as Klumpke’s paralysis. The ulnar nerve is exclusively supplied by C8 and T1 and is responsible for sensory innervation to the fifth digit, the medial half of the fourth digit, and the corresponding palmar surface of the hand. It controls the majority of medial digit flexion (via the medial heads of the flexor digitorum profundus and the flexor digiti minimi muscles) as well as abduction (via the dorsal interossei) and adduction (via the palmar interossei) of digits 2 to 5. It is partially responsible for metacarpophalangeal joint flexion (via the lumbricals of digits 4 and 5) and wrist flexion (via the flexor carpi ulnaris) (Fig. 26.5).

Roots C5

Trunks er Sup

Musculocutaneous nerve

C7

dle

Mid

Cords Branches

C6

ior

Divisions

r

te

La

al

C8

er

ior

Inf

r rio

T1

ste

Po

Axillary nerve Radial nerve Median nerve Ulnar nerve

ial

Med

Medial antebrachial cutaneous nerve Medial brachial cutaneous nerve

Figure 26.5.  Roots, trunks, divisions, cords, and branches of the brachial plexus. (From Miller RD et al. Miller’s Anesthesia. 8th ed. Philadelphia: Elsevier; 2015.)

Clinical Anatomy  593

Case 26.2, Part A continued: One year later, his wild ways have continued, and he admits to twisting his ankle during an episode of extreme intoxication last weekend. He did not go to the doctor then, as he was a bit embarrassed, and his ankle feels much better. He presents today with an inability to extend his right elbow and right wrist drop since he started using crutches that a friend let him borrow.   

3. What structure has he injured this time, and how has it led to his upper extremity position?   A compression injury to the posterior cord of the brachial plexus can occur if underarm crutches are used incorrectly. The inability to extend his wrist is due to a lack of radial nerve (a branch of the posterior cord) input to the posterior compartment of the forearm. The radial nerve also innervates the triceps brachii and anconeus muscles, which are necessary for elbow extension. Pure radial nerve palsies (which often result from injury to the nerve at the spiral groove of the humerus) generally do not affect the triceps, because the branch of radial nerve to the triceps is very close to the origin at the posterior cord, proximal to the spiral groove.

Case 26.2, Part B One month later, his father, who happens to be a writer, presents to your clinic with paresthesias and pain involving his lateral palm, digits 1 to 3, and the lateral half of his fourth digit. He tells you that he has been typing long hours over the past few months and now has almost finished his latest masterpiece. On physical examination, you notice mild atrophy of the thenar eminence and tapping the middle of the wrist crease elicits paresthesias of the lateral aspect of the hand (positive Tinel sign). Palmar flexion of the wrist for longer than 1 minute results in paresthesias in the lateral aspect of the hand (positive Phalen test).   

4. What structure has been injured and how has this happened?   Repetitive use of the hands—typing, in this case—can lead to inflammation, swelling, and subsequent compression of the structures within the carpal tunnel. This condition is known as carpal tunnel syndrome, and the pain, paresthesias, and muscle wasting are due to median nerve injury within the tunnel. Pain and paresthesias have arisen in the distribution of the median nerve, the lateral palm, and the lateral 3½ digits. Because the median nerve also innervates the intrinsic muscles of the thumb, early thenar wasting has occurred. 5. Outline the contents of the carpal tunnel.   The carpal tunnel is formed by the eight wrist (carpal) bones and the transverse carpal ligament (flexor retinaculum), which spans from the tubercles of the scaphoid and trapezium to the pisiform and the hook of the hamate. The contents include the four flexor digitorum profundus tendons, the four flexor digitorum superficialis tendons, the flexor pollicis longus tendon, and the median nerve (Fig. 26.6).

Flexor tendons Median nerve Ulnar artery

Radial artery Flexor retinaculum

Tra

pez

ium

Ulnar nerve

Ha

ma

te

Capitate Ext

ens

or t

end

ons

d

ezoi

Trap

Figure 26.6.  Cross-sectional anatomy of wrist. Tendons and median nerve may be compressed by inflammation or infection because they are encompassed by synovial sheath and flexor retinaculum. (From Weiss J, Weiss L, Silver, J. Easy EMG, 2nd ed. Philadelphia: Elsevier; 2016.)

594  Clinical Anatomy 6. Describe the sensory and motor innervations of the median, ulnar, and radial nerves.   See Table 26.2 and Fig. 26.7.

Median nerve (Palm cut. br.) Musculocutaneous nerve Radial nerve

Ulnar nerve (Palm cut. br.) Ulnar nerve (Volar dig. br.)

Table 26.2.   S  ensory and Motor Innervations of the Median, Ulnar, and Radial Nerves SENSORY NERVE INNERVATION

MOTOR INNERVATION

Median

Lateral palm, thumb, digits 2 and 3, and lateral half of digit 4

LOAF muscles (Lateral 2 lumbricals, Opponens pollicis, Abductor pollicis brevis, Flexor pollicis brevis)

Medial palm, medial dorsal hand, digit 5, and medial half of digit 4

All the intrinsic muscles of the hand except the LOAF muscles

Ulnar

Radial

Lateral dorsal hand, ex- No motor innervation to tending distally to the the hand proximal interphalangeal (PIP) joint of digits 1 to 3 and the lateral half of digit 4

A

Median nerve (Volar dig. br.)

Ulnar nerve (Dorsal br. & dorsal dig. br.)

Radial nerve (Dorsal antebr. cut.) Radial nerve (Superficial br. & dorsal dig. br.)

Median nerve (Volar dig. br.)

B Figure 26.7.  Nerves of the hand. A, Volar aspect. B, Dorsal aspect. antebr. cut., antebrachial cutaneous; cut. br., cutaneous branch; dig. br., digital branch. (From Noble J. Textbook of Primary Care Medicine. 3rd ed. St. Louis: Mosby; 2001.)

STEP 1 SECRET Innervation of the hand is a particularly high-yield topic. If you know nothing else about the anatomy of the hand, know the nerve innervation.

Case 26.2, Part B continued: As the patient walks out of the office, he slips on icy steps and falls forward, breaking the fall with his right hand. He immediately feels severe pain in his wrist, and as his distal forearm is very obviously deformed, he carefully walks back into the clinic. You order an x-ray film, and the radiologist remarks that she sees the classic “dinner fork deformity” of a certain fracture commonly seen in patients over the age of 50, classically those with osteoporosis.   

7. What fracture has this patient suffered?   He has suffered a Colles fracture, a fracture of the distal radius in which the distal fragment is displaced posteriorly/ dorsally. Radiographically, the angle of the radius and the fragment in combination with the angle of the fragment and the hand resembles the curvature of a fork. This fracture is common after the age of 50 and most often occurs when one breaks a fall with an outstretched hand.

Clinical Anatomy  595

Case 26.2, Part C Two months later, you are working in the ED again, and you come across another member of the family. He is a 20-year-old former high school starting pitcher who was getting a lesson from his brother on how to ride a motorcycle. Unfortunately, he too has taken a nasty fall. The attending at the ED says she is worried about a humerus fracture.   

8. What are the three most common sites of humerus fracture, and which nerve and artery are at risk at each of these sites?   See Table 26.3. Table 26.3.   Three Most Common Sites of Humerus Fracture HUMERUS FRACTURE SITE

NERVE

ARTERY

Surgical neck

Axillary

Anterior and posterior circumflex humeral (branches of the axillary artery)

Midshaft

Radial

Profunda brachii (branch of the brachial artery)

Supracondylar

Median

Brachial

9. On reviewing his past medical history, you note that his baseball career was marred by a partially torn rotator cuff. Describe why the rotator cuff makes the glenohumeral joint different from other joints, and name its four components.   Most joints are stabilized primarily by a ligamentous capsule, but the glenohumeral joint is stabilized primarily by the rotator cuff, which consists of the tendons of four muscles—supraspinatus, infraspinatus, teres minor, and subscapularis (Fig. 26.8). This design allows the glenohumeral joint to have the widest range of motion of all joints in the body, at the expense of stability and resistance to injury. The rotator cuff stabilizes the glenohumeral joint by pulling the head of the humerus toward the glenoid fossa of the scapula as other muscles flex, extend, abduct, or adduct the arm. Rapid, forceful, or repetitive movements (such as repeatedly throwing a baseball) can tear the tendons of the rotator cuff, leading to a lack of joint stability, restricted movement, and pain.

Supraspinatus

Infraspinatus

Teres minor

Subscapularis Figure 26.8.  Cross-sectional view of the shoulder with humeral head stabilized in shallow scapular glenoid by rotator cuff and capsule. (From Noble J. Textbook of Primary Care Medicine. 3rd ed. St. Louis: Mosby; 2001.)

STEP 1 SECRET Upper extremity injuries, particularly those that involve the brachial plexus, are popular subjects for Step 1 anatomy questions. You should learn all of the brachial plexus components and the muscles that they innervate. Remember, board questions usually have a clinical focus! When you study the brachial plexus, spend most of your time reasoning through the various injuries that result from lesions to the different nerves, branches, trunks, divisions, and so on. You are expected to know the most common ways in which these injuries can occur, because it is likely that you will have to deduce this from the stem of the question (e.g., associate frequent computer use with median nerve damage). Lower extremity nerve injuries are also tested on boards but not quite as commonly as upper extremity injuries.

596  Clinical Anatomy

SUMMARY BOX: UPPER EXTREMITY INJURIES • The musculocutaneous nerve innervates the flexors of the elbow. • The axillary nerve innervates the deltoid and teres minor muscles as well as the long head of the triceps brachii. • The radial nerve innervates the extensors in the arm and forearm. • The ulnar nerve innervates the medial heads of the flexor digitorum profundus, the flexor carpi ulnaris, medial lumbricals, interossei, and hypothenar muscles. • The median nerve innervates the anterior forearm muscles not innervated by the ulnar nerve, the lateral lumbricals, and the thenar muscles. • The blood supply to the upper extremity (UE) is from the brachial artery, the continuation of the axillary artery after it crosses the teres major. • The carpal tunnel consists of the eight carpal bones and the transverse carpal ligament. It contains the median nerve and the tendons of the long flexors of the digits. • The rotator cuff consists of the tendons of four muscles: supraspinatus, infraspinatus, teres minor, and subscapularis.

Case 26.3, Part A You are working in an outpatient pediatrics clinic, and your next patient is a 14-day-old infant who was born 2 weeks prematurely. The mother has no complaints, and you proceed to examine the child. The infant is mildly diaphoretic, and you notice a continuous (both systolic and diastolic) “machine-like murmur” auscultated best at the second left intercostal space.   

1. What is the most likely diagnosis?   Patent ductus arteriosus (PDA) is the most likely diagnosis. 2. Describe the fetal circulation pathway.   In fetal life, oxygenated blood from the placenta enters the fetal circulation through the umbilical vein. Half of this blood enters the inferior vena cava (IVC) through the hepatic veins, while the other half bypasses the hepatic vasculature through the ductus venosus. Since the IVC receives a mixture of oxygenated blood and deoxygenated blood (return from the fetal systemic veins), oxygen tension in the IVC is higher than that in the superior vena cava (SVC). As a result, the IVC and SVC bloodstreams follow two distinct paths upon entering the right atrium. A large percentage of IVC blood entering the right atrium is shunted to the left atrium (thus bypassing the lungs) through the foramen ovale, where it mixes with poorly oxygenated blood entering the left atrium through the pulmonary veins (recall that the lungs are not ventilated in the fetus and thus simply extract oxygen from the blood). This blood is directed through the left ventricle and into the ascending aorta, where it is largely distributed to the brain, upper body, and coronaries. Blood from the IVC that is not shunted to the left atrium mixes with poorly oxygenated SVC blood and passes in typical fashion to the right ventricle. This output enters the pulmonary artery and from there either enters the lungs or gets shunted through the ductus arteriosus into the descending aorta. Blood in the descending aorta supplies the lower body and eventually makes its way into the umbilical arteries, which lead back to the placenta for oxygenation (Fig. 26.9). 3. What is the utility of the ductus arteriosus?   In utero, this connection between the aorta and the pulmonary artery acts as a right-to-left shunt. It allows the majority of oxygenated blood (which has entered the right side of the heart through the route of placenta → umbilical vein → ductus venosus → inferior vena cava) to bypass the developing lungs and enter the systemic circulation. Recall that bypassing the amniotic fluid-filled fetal lungs is desirable because they are incapable of gas exchange. Just after birth, the ductus arteriosus normally closes and undergoes fibrotic degeneration to become the ligamentum arteriosum. 4. What causes the ductus arteriosus to close after birth?   During life as a fetus, circulating prostaglandins and a low blood Po2 keep the ductus arteriosus open. Blood Po2 rises when breathing is initiated, signaling the newborn’s ability to obtain oxygenated blood from the lungs rather than from the umbilical vein. Along with rising Po2, falling levels of prostaglandins act to close the ductus arteriosus. In the event that the ductus arteriosus remains patent after birth, it can often be closed by administering a drug that blocks the production of prostaglandin E2, such as the COX inhibitor indomethacin. Alternatively, in the event that a baby is born with a congenital defect such as transposition of the great vessels, it is necessary to keep the ductus arteriosus open until the transposition can be surgically fixed. This can be achieved by administering alprostadil, a prostaglandin E1 analog.

Case 26.3, Part B It is a slow morning on your cardiology rotation when a 32-year-old man with a history of deep venous thrombosis (DVT) secondary to factor V Leiden thrombophilia is brought by ambulance to the ED suffering from an apparent stroke. He has no history of atrial fibrillation, valvular disease, or coronary artery disease. On cardiac auscultation, you hear a mild systolic ejection murmur and wide, fixed splitting of S2. Neurology confirms that he has had an ischemic stroke, and a transesophageal echocardiogram (TEE) reveals interatrial blood flow.   

Clinical Anatomy  597 To head To arm

To arm Aorta

Ductus arteriosus

Superior vena cava Pulmonary artery

Left atrium

Foramen ovale Right atrium

Left lung Right lung

Right ventricle Hepatic vein Left ventricle Ductus venosus Liver Inferior vena cava Renal arteries and veins Umbilical vein Portal vein

Umbilicus

Aorta

Umbilical arteries

Hypogastric artery Umbilical cord

Bladder

To left leg

Placenta Arterial blood Venous blood

Mixed arterial and venous blood

Figure 26.9.  Course of fetal circulation in late gestation. Note the selective blood flow patterns across the foramen ovale and the ductus arteriosus. (From Miller RD, et al. Miller’s Anesthesia. 8th ed. Philadelphia: Elsevier; 2015.)

5. What is the most likely diagnosis?   Atrial septal defect (ASD) is most likely. A thromboembolic stroke is relatively rare in patients without atrial fibrillation, valvular disease, or a past MI. Although factor V Leiden produces a hypercoagulable state, it is much more likely to cause DVT than a left-sided heart or arterial thrombus. The physical examination and TEE study findings strongly support ASD, so his stroke was most likely caused by a paradoxical embolus (in which an embolus of venous origin traveled through the ASD and then to the cerebral vasculature). 6. Is atrial septal defect the most common congenital heart defect?   No. Ventricular septal defect (VSD) is the most common. ASD is the second most common congenital heart defect, and PDA is the third. 7. What are the three most common types of atrial septal defects? 1. Ostium secundum defect Ostium secundum defect is the most common type of ASD. The atrial septum is formed by the septum primum and the septum secundum. In ostium secundum defect, there is usually excessive absorption of the septum primum, inadequate

598  Clinical Anatomy





growth of the septum secundum, or enlargement of the foramen ovale (the opening at the inferior margin of the septum secundum). A subtype of ostium secundum defect is patent foramen ovale, in which the septum primum and septum secundum fail to fuse. This common defect may allow interatrial blood flow (note that this is physiologic in fetal life). 2. Ostium primum defect In ostium primum defect, the septum primum fails to fuse with the endocardial (atrioventricular [AV]) cushion. This is often due to an endocardial cushion defect, commonly associated with Down syndrome. Note that the endocardial cushion is the point of fusion for the atrial septum, ventricular septum, mitral valve, and tricuspid valve, and the magnitude of the defect determines the pathology. For instance, partial ostium primum defect causes an interatrial connection, but complete ostium primum defect causes an AV connection. 3. Sinus venosus defect Normally, the atrial septum develops completely to the left of the sinus venosus, the structure that is to become the superior and inferior venae cavae and part of the right atrium. In this rare defect, the septum develops anterior to the sinus venosus, allowing interatrial flow via the sinus venosus due vena cava entry straddling the septum (Fig. 26.10). SS OS LA

LA

SP

SP

RA

RA FO TV

A

MV

AVC

OP

TV

B

RV

MV LV

RA/LV septum

Figure 26.10.  Development of atrial septum. A, The septum primum (SP) grows down from the roof of the primitive common atrium to meet the atrioventricular cushions (AVCs) and divides the primitive common atrium into right atrium (RA) and left atrium (LA). The defect below the growing free lower edge of SP is called ostium primum (OP)—shown here as a dotted oval. The septum primum has reached the AVCs, which have developed into tricuspid (TV) and mitral (MV) valves. The drawing shows that the upper part of the SP has (normally) degenerated to leave the large ostium secundum (OS) or fossa ovale defect. B, A second interatrial septum—septum secundum (SS)—grows to the right of the SP. The upper portions of the two septa fuse, and a portion of the upper part degenerates to form the OS. The lower edge of the SS grows downward to partially cover the OS but does not reach the AVCs. A valve-like opening—foramen ovale (FO)—is thereby established, permitting a shunt from RA to LA but not in the reverse direction. The FO persists during fetal life (during which it transmits an essential right-to-left shunt), but after birth it usually seals off by fusion of the lower part of SP with SS. Note that the interventricular septum separates the right ventricle (RV) from the left ventricle (LV). The interventricular septum meets AVCs to the right of the atrial septum, so that one portion of the AVC separates RA from LV. This portion later forms the upper part of the interventricular septum, and it is through this portion that the Gerbode defect occurs. (From Grainger RG, Allison D, Adams A. Grainger & Allison’s Diagnostic Radiology: A Textbook of Medical Imaging. 4th ed. Philadelphia: Churchill Livingstone; 2001.)

8. How might an ASD cause right-sided heart failure?   Left-sided heart pressures are higher than right-sided heart pressures, so an ASD allows for left-to-right shunting of blood. This increases flow volumes through the right side of the heart, leading to right ventricular (and atrial) dilation, increased stroke work, decreased pumping ability, and eventually right-sided heart failure. 9. What is the dreaded late complication of ASD?   Eisenmenger’s complex refers to a situation in which pathologically increased right-sided heart flow and pressure damage the pulmonary vasculature, causing small vessel fibrosis to develop. The fibrotic vasculature exacerbates pulmonary hypertension and does not contribute to gas exchange, causing the right side of the heart to increase its output. Eventually, right-sided heart pressures become high enough to reverse the shunt of ASD to a right-to-left shunt, resulting in cyanosis and heart failure.

Case 26.3, Part C You are in the newborn nursery on the first day of your inpatient pediatrics rotation, and the neonatology fellow invites you in to see an infant. The infant is obviously cyanotic but seems to be having no respiratory difficulty other than mild tachypnea.   

0. What are the five cardiogenic causes of cyanosis in a newborn? 1   These five causes are known as the five Ts: Tetralogy of Fallot, Transposition of the great vessels, Truncus arteriosus, Tricuspid atresia, and Total anomalous pulmonary venous return.

Case 26.3, Part C continued: On cardiac auscultation, you hear a 3/6 systolic crescendo-decrescendo murmur at the second left intercostal space and a loud S2 at the fourth left intercostal space. The fellow shows you the infant’s chest radiograph, which shows a “boot-shaped” heart, signifying right ventricular hypertrophy.   

Clinical Anatomy  599

1. What is the most likely diagnosis? 1   The most likely diagnosis is tetralogy of Fallot, the congenital heart defect most likely to cause cyanosis in infants. The murmurs of tetralogy of Fallot vary depending on the degree of pulmonary stenosis and the extent of the VSD. In this case, the pulmonary stenosis murmur overshadows any VSD murmur that might be appreciable. The loud S2 is due to the closure of the aortic valve. Recall the four components of this disease: VSD, overriding aorta, pulmonary stenosis, and right ventricular hypertrophy. 2. How does tetralogy of Fallot cause cyanosis? 1   Pulmonary stenosis causes a high resistance to flow through the pulmonary trunk. Coupled with concentric right ventricular hypertrophy, this leads to high right-sided heart pressures, causing right-to-left flow through the VSD (Fig. 26.11). When a critical proportion of deoxygenated blood is shunted through the VSD and mixed with oxygenated blood, cyanosis appears. It is classically seen first in the fingers and lips.

AO PA LA

RA LV

RV Figure 26.11.  Tetralogy of Fallot. AO, aorta; LA, left atrium; LV, left ventricle; PA, pulmonary artery; RA, right atrium; RV, right ventricle. (From Marcdante KJ, Kliegman RM. Nelson Essentials of Pediatrics. 7th ed. Philadelphia: Elsevier; 2015)

3. What is the cause of tetralogy of Fallot? 1   Unequal partitioning of the primitive truncus arteriosus by the truncoconal ridges misplaces the infundibular septum, the structure that divides the two ventricular outflow tracts. The anterosuperior displacement of the infundibular septum causes pulmonary stenosis in addition to causing the aorta to override the VSD that results from failure of the truncoconal ridges to fuse with the muscular interventricular septum. Stenosis of the pulmonary trunk increases afterload, leading to concentric right ventricular hypertrophy, seen on a radiograph as a “boot-shaped” heart. 4. What is the characteristic behavior in an infant with a later presentation of tetralogy? 1   Cyanosis that occurs with agitation, crying, or a warm bath that is corrected with squatting or “knees-to-chest” positioning. Crying increases pulmonary vascular resistance, and warmth decreases systemic vascular resistance, shunting blood away from the lungs and into the periphery. Squatting relieves the shunt by increasing systemic vascular resistance and forcing blood flow leaving the left ventricle to preferentially enter the lung, allowing oxygenation to resume. These episodes are known as tet spells.

SUMMARY BOX: CONGENITAL HEART DEFECTS • The ductus arteriosus connects the pulmonary artery to the aorta and allows blood to bypass the developing lungs during fetal life. • The patency of the ductus arteriosus is maintained by the presence of prostaglandin E2 and low Po2. It can be pharmacologically closed with indomethacin or kept open with a prostaglandin. • During fetal life, blood also bypasses the developing lungs via the foramen ovale, which is appropriately patent at this time. • Atrial septal defect (ASD) can be recognized on cardiac auscultation by a mild systolic ejection murmur and wide, fixed splitting of S2. • Ventricular septal defect (VSD) is the most common congenital heart defect, characterized by a harsh holosystolic murmur on exam. • Eisenmenger’s complex refers to a right-to-left shunt (resulting in cyanosis) that occurred because a left-to-right shunt (VSD, ASD, or patent ductus arteriosus [PDA]) caused right-sided heart overload and pulmonary hypertension, leading to high right-sided heart pressures and shunt reversal. • Tetralogy of Fallot is characterized by VSD, overriding aorta, pulmonary stenosis, and right ventricular hypertrophy. It is the most common cause of right-to-left cyanotic shunt in infants.

600  Clinical Anatomy

Case 26.4, Part A A 19-year-old college freshman presents to the ED with severe neck stiffness and a headache. His temperature is 38.8°C, and he has been experiencing chills. According to friends, he has become increasingly confused in the past 24 hours.   

1. What is the differential diagnosis for his symptoms?   Meningitis, encephalitis, mass lesion of brain (abscess or tumor), and subarachnoid hemorrhage are possible.

Case 26.4, Part A continued: Upon further questioning of his friends, it is discovered that the patient was suffering from an upper respiratory tract infection during the 3 days before his current illness. On examination, flexion of the neck with the patient supine elicits pain and involuntary hip and knee flexion (Brudzinski sign), and when the hip is flexed, attempted extension of the knee elicits pain (Kernig sign).   

2. What is the most likely diagnosis, and what is the next step to confirm this suspicion?   He most likely has meningitis, which can be confirmed by lumbar puncture (LP). 3. At what spinal level should a lumbar puncture be performed? Why?   In adults, the needle should be inserted between the spinous processes of L4 and L5. The space between L3 and L4 is also an acceptable choice, because the conus medullaris (the terminal portion of the spinal cord) usually ends near the superior border of L2. Recall that the cauda equina continues below this level, but the free-floating nature of the nerve bundles in the cerebrospinal fluid (CSF) makes them less likely to sustain puncture damage. It should be noted that in children, an LP should be performed only between L4 and L5, because the spinal cord in children can extend to L3. 4. Through what major structures and spaces, from superficial to deep, should the needle pass in a lumbar puncture?   Skin → subcutaneous tissue → spinal ligaments (supraspinous ligament, interspinous ligament, and ligamentum flavum) → epidural space → dura mater → arachnoid mater → subarachnoid space (from which CSF can be drawn). 5. Describe the three layers of the meninges.   Dura mater: This outermost layer is fused to the inside of the skull via the periosteal dural layer. The dura is doublelayered in the skull, allowing cranial compartmentalization and investment of the venous sinuses. In contrast, it is singlelayered within the vertebral canal, where it is separated from the sides by the epidural space. The dural sac extends to S2 and is attached to the coccyx via the filum terminale externum.     Arachnoid mater: This layer is fused to the inner surface of the dura and sends trabeculae to the outer surface of the pia. Between the arachnoid and the pia (the subarachnoid space) lies the CSF.     Pia mater: This is essentially the outermost layer of the brain, spinal cord, and nerve roots because it cannot be separated from them. It invests the blood vessels of the brain and spinal cord. The pia continues after the conus medullaris as the filum terminale internum (which is not part of the cauda equina because it does not contain any axons) until the end of the dural sac at S2, where it is invested with dura to become the filum terminale externum, terminating at the coccyx (Fig. 26.12). 6. What cerebrospinal fluid findings would you expect to find with different causes of meningitis?   See Table 26.4. 7. What are the most common causes of meningitis by age group?   See Table 26.5.

STEP 1 SECRET The information in Tables 26.4 and 26.5 is high yield for Step 1, and this knowledge will earn easy points for you if you take the time to learn it well. The causes of meningitis in Table 26.5 are not listed in any particular order, although you should note that Haemophilus influenzae is becoming an increasingly rare cause of meningitis secondary to vaccination. The mnemonic for recalling the common causes of meningitis by age group is GEL MESH, MESH GeLS, where each word represents a separate age group (see Table 26.5).

Case 26.4, Part B A 1-month-old infant presents for a well-child checkup. She is developmentally normal for her age. On examination, a tuft of hair overlying a 1-cm darkly pigmented patch is found at the level of L5. Exam is otherwise entirely unrevealing.   

Clinical Anatomy  601

Spinal cord Pia mater

L1

Dura mater and arachnoid L2

Conus medullaris Internal filum terminale Cauda equina

L3

Ligamentum flavum Supraspinous ligament

L4

Interspinous ligament

L5

Distal dural sac S2 External filum terminale Sacrum

Figure 26.12.  Spinal cord anatomy. Notice the termination of the spinal cord (i.e., conus medullaris) at L1-L2. (From Miller RD, et al. Miller’s Anesthesia. 8th ed. Philadelphia: Elsevier; 2015.)

Table 26.4.   Cerebrospinal Fluid Findings in Meningitis PATHOGENIC CATEGORY

WHITE CELL PREDOMINANCE

PRESSURE

PROTEIN

GLUCOSE

Bacterial

PMN





Tuberculosis

Lymphocyte





Fungal

Lymphocyte

↑↑





Viral

Lymphocyte

Normal/↑

Normal/↑

Normal





PMN, polymorphonuclear neutrophil (leukocyte).

Table 26.5.   Common Causes of Meningitis by Age Group 0–6 MONTHS

Group B streptococci Escherichia coli Listeria monocytogenes

6 MONTHS–6 YEARS

Neisseria meningitidis Enteroviruses Streptococcus pneumoniae Haemophilus influenzae type b

6 YEARS–60 YEARS

N. meningitidis Enteroviruses S. pneumoniae Herpes simplex virus (HSV)

60+ YEARS

Gram-negative rods S. pneumoniae L. monocytogenes

602  Clinical Anatomy 8. What disorder of neurologic development can be characterized by these findings?   Spina bifida occulta is a disorder in which the posterior neural tube fails to close, resulting in a failure of midline vertebral arch closure with intact dura. There are no associated neurologic deficits. 9. What are the other significant disorders related to a failure of posterior neural tube closure?   Spina bifida cystica—meningocele: Failure of posterior midline closure of both the vertebral arch and the dura mater. The arachnoid mater herniates through the defect, creating a cyst. Neurologic deficits may or may not occur.     Spina bifida cystica—meningomyelocele: Failure of posterior midline closure of both the vertebral arch and the dura mater, but the defect is wide enough to allow spinal cord herniation with the arachnoid. Neurologic deficits are level-dependent but usually include paralysis of some degree. This disorder can be associated with Arnold-Chiari malformation type II.

Case 26.4, Part C A 64-year-old carpenter presents with severe lower back pain. Most of the pain is localized to a single spot in his lower back, but he has ill-defined pain and tingling that begins in the left gluteal region and courses down the lateral side of his left LE to his foot. He also has experienced left LE weakness. He reports that all of these symptoms started 3 days ago, coming on suddenly while he was working in his yard. On physical examination, passive right straight leg raise causes moderate pain in the distribution (on the left) as described here.   

0. What is the most likely diagnosis? 1   He has suffered an intervertebral disk herniation. This is supported by his severe, acute-onset lumbar back pain (upward of 90% of disk herniations involve the L4-L5 or L5-S1 disks), unilateral motor deficit, and pain and tingling that follow a spinal nerve distribution. 1. Describe intervertebral disk anatomy and how herniation usually occurs. 1   The nucleus pulposus is the central elastic cartilaginous portion of the disk. It is surrounded by the annulus fibrosus, which consists of concentric rings of fibrocartilage. With age, the nuclei pulposi become thin and lose their elasticity, and the annuli fibrosi degenerate. This makes the disk more likely to herniate, an action characterized by the nucleus pulposus breaking through a localized weakness in the annulus fibrosus. Herniations usually occur in the posterolateral direction, because this is a site of relative annulus fibrosus weakness, and there is no support from the anterior or posterior longitudinal ligaments of the vertebral column. Posterolateral disk herniations occur proximal to the intervertebral foramina through which the spinal nerves pass, potentially compressing the spinal nerves and leading to radiculopathy (Fig. 26.13). S1 L5 L4

L4

L5

Figure 26.13.  Lumbosacral disk herniation. The most common posterolateral herniation compresses the nerve root traveling downward to emerge one level below the level of the exiting root. Hence, L5-S1 herniation most commonly compresses the descending S1 root (horizontal hatching). More lateral herniation may compress the root exiting at the level of herniation (diagonal hatching). A large central herniation may compress multiple bilateral descending roots of the cauda equina (vertical hatching). (From Goetz CG. Textbook of Clinical Neurology. 2nd ed. Philadelphia: WB Saunders; 2003.)

Clinical Anatomy  603

2. Describe the pattern of nerve compression seen in intervertebral disk herniations 1   Recall the scheme for numbering spinal nerves: The cervical spinal nerves C1 to C7 exit the spinal canal superior to the vertebra with the same number. The naming scheme changes at C8, which exits inferior to C7. Thereafter, the spinal nerve roots exit below the vertebra with the same number (the L1 spinal nerve exits below the L1 vertebra). Despite the change within this numbering scheme, disk herniations tend to compress the nerve root with the same number as the vertebra below the intervertebral disk. For example, C4-C5 disk herniations result in compression of the C5 nerve root, and L4-L5 disk herniations result in compression of the L5 nerve root. This relationship is maintained because of the increasingly acute angles at which the spinal nerves come off the spinal cord as it descends (Fig. 26.14). However, this change in angle and the presence of the cauda equina allow for multiple nerve compressions to occur with a relatively medial herniation in the lower lumbar region (e.g., an L5-S1 herniation can compress both the L5 and S1 nerves).

L4 C4 C5 root

L4 root

C5 L5 Figure 26.14.  Comparison of points at which nerve roots emerge from cervical and lumbar spine. (From Kikuchi S, Macnab I, Moreau P. Localisation of the level of symptomatic cervical disc degeneration. J Bone Joint Surg. 1981;63B:272.)

SUMMARY BOX: SPINAL CORD AND VERTEBRAL COLUMN • The best site for a lumbar puncture (LP) is the L4-L5 interspace, well below the end of the spinal cord at L2. • Spina bifida is a disorder in which the posterior neural tube fails to close. • Spina bifida occulta is the least severe subtype, followed by meningocele, and then by meningomyelocele. • Intervertebral disk herniation • Occurs when the central nucleus pulposus herniates through the annulus fibrosus • Usually occurs in the posterolateral direction, where support from the anterior and posterior longitudinal ligaments is lacking • Usually occurs in the lower lumbar region • Often results in compression of the nerve root named for the vertebra below the intervertebral disk

Case 26.5 An 18-year-old college student presents with a 1-month history of an intermittent bulge in the right side of his scrotum. He has recently started bodybuilding and states that the bulge is more likely to appear during workouts and less likely to appear while he is lying down. There is no associated pain or scrotal erythema or edema. On physical examination, the scrotum does not transilluminate, and a soft structure can be reduced through the superficial inguinal ring.   

1. Describe the characteristics of the most likely diagnosis   Indirect inguinal hernia, which is the most common type of hernia in both sexes, is characterized by a protrusion of parietal peritoneum and viscera through a part of the abdominal wall lateral to the inferior epigastric vessels. The viscera exit the abdominal cavity via the deep inguinal ring and enter the scrotum via the superficial inguinal ring, passing through the entirety of the inguinal canal. Recall that parietal peritoneum envelops the testicles during their descent out of the abdomen into the scrotum, eventually forming the tunica vaginalis. During the descent, the cavity of the tunica vaginalis is connected to the peritoneal cavity by the processus vaginalis, which is normally obliterated in the perinatal

604  Clinical Anatomy period. In certain cases, a persistent processus vaginalis remains, forming a potential space within the spermatic cord through which indirect inguinal hernias can protrude. Note that this etiology makes indirect inguinal hernias congenital (Fig. 26.15).

Deep inguinal ring Inguinal ligament Superficial inguinal ring Small intestine Hernial sac

Figure 26.15.  Indirect inguinal hernia. (From Roberts JR. Roberts and Hedges’ Clinical Procedures in Emergency Medicine. 6th ed. Philadelphia: Elsevier; 2014.)

2. What differentiates a direct from an indirect inguinal hernia?   A direct inguinal hernia is due to muscular weakness in the abdominal wall. It is characterized by protrusion of parietal peritoneum and viscera through the Hesselbach triangle, bordered laterally by the inferior epigastric artery, medially by the lateral margin of the rectus abdominis, and inferiorly by the inguinal ligament. The hernia sac is usually composed of transversalis fascia, and although it may pass through a portion of the inguinal canal, it rarely enters the scrotum and is not within the spermatic cord (Fig. 26.16). Most direct inguinal hernias are acquired.

Direct hernia

Inguinal ligament Superficial inguinal ring

Figure 26.16.  Direct inguinal hernia. (From Roberts JR. Roberts and Hedges’ Clinical Procedures in Emergency Medicine. 6th ed. Philadelphia: Elsevier; 2014.)

STEP 1 SECRET Differentiating among the various classes of hernias is commonly tested on Step 1. You should understand the relationship of the different hernia types to their respective anatomic borders. To quickly differentiate between an indirect and a direct inguinal hernia, remember that “MD’s do LIe” (Medial to the epigastric vessels is associated with a Direct hernia and Lateral to the epigastric vessels is associated with an Indirect hernia). That being said, also remember that if the femoral pulse is located lateral to the bulge, the patient has a direct hernia, while a femoral pulse medial to the bulge indicates an indirect hernia.

Clinical Anatomy  605

3. Describe the structure of the inguinal canal.   The inguinal canal is an inferomedially directed passageway that connects two openings, the deep and the superficial inguinal rings. The deep inguinal ring is in the transversalis fascia, just lateral to the inferior epigastric vessels. The superficial inguinal ring is in the external oblique aponeurosis and lies just superolateral to the pubic tubercle. Two walls, a roof, and a floor delineate the canal formed between these two rings. Its major constituents are as follows: the transversalis fascia forms the posterior wall, the external oblique aponeurosis forms the anterior wall, the internal oblique and transversus abdominis muscles form the roof, and the inguinal ligament forms the floor. It should be noted that the layers of the spermatic cord are applied via passage of the testicles through the inguinal canal. Consequently, the internal spermatic fascia is continuous with the transversalis fascia, the cremasteric fascia is continuous with the internal oblique, and the external spermatic fascia is continuous with the external oblique aponeurosis. 4. Discuss the major contents of the spermatic cord. • External spermatic fascia: Continuation of the external oblique aponeurosis. • Cremasteric muscle and fascia: Continuation of the internal oblique muscle and fascia; draws testes superiorly, often in response to cold temperatures. • Internal spermatic fascia: Continuation of the transversalis fascia. • Vas deferens: Transports sperm from the epididymis to the ejaculatory duct. • Testicular artery: Supplies testes and epididymis (testicular torsion is a medical emergency because twisting of the spermatic cord leads to occlusion of this artery). • Pampiniform plexus: A venous network that drains into the right and left testicular veins. The venous blood of the pampiniform plexus is cooler than the adjacent blood from the testicular artery. This countercurrent flow cools the blood destined for the testes, maintaining an intratesticular temperature just below core body temperature. • Genital branch of the genitofemoral nerve: Supplies sensory innervation to the anterior aspect of the scrotum and supplies motor innervation to the cremaster muscle. • Ilioinguinal nerve: This nerve pierces the internal oblique muscle to enter the inguinal canal, thereafter traveling on the surface of the spermatic cord, rather than within it, to supply some sensory innervation to the superior aspect of the scrotum and root of the penis. • Other: Autonomic nerve fibers and lymphatic vessels that drain to the para-aortic (lumbar) and preaortic lymph nodes are also present. 5. Which lymph nodes are the most likely site of first metastasis in testicular cancer? Why is this the case?   Testicular lymphatic fluid drains directly to the preaortic and para-aortic (lumbar) lymph nodes. Recall that during embryogenesis, each developing gonad arises from a combination of mesoderm and mesothelium called the gonadal ridge, which lies just medial to the mesonephros (itself, lying medial to the metanephros, which develops into the kidney). Thus, the testes (and ovaries) develop markedly superior to their position in adult life. Consequently, the blood supply and lymphatic drainage of the testes are located closer to the kidneys than to any structures of the pelvis. For example, the two testicular arteries branch directly from the aorta just inferior to the origin of the renal arteries. It should be noted that, in contrast with testicular cancer, cancer of the scrotum initially metastasizes to the superficial inguinal lymph nodes.

STEP 1 SECRET You should expect to get a question regarding the sites of local metastasis for various types of cancers. The USMLE is especially fond of the fact that testicular and ovarian cancers metastasize to the para-aortic lymph nodes.

6. After a vasectomy, by what means does a male produce an ejaculate that does not include sperm? Include a summary of the path of sperm from spermatogenesis to exit from the urethra.   Normally, sperm pass from their point of origin in the seminiferous tubules to the epididymis and onward into the vas deferens. The two vasa deferentia merge with the outlets of the two seminal vesicles to form the ejaculatory duct. The ejaculatory duct feeds into the prostatic urethra, where the prostate gland deposits its secretions. The prostatic urethra leads to the penile urethra, where the bulbourethral glands deposit their secretions. From the penile urethra, the ejaculate exits the body (Fig. 26.17). In a vasectomy, the vasa deferentia are ligated bilaterally, so sperm cannot pass into the ejaculatory duct, and most of them degenerate in the proximal vas deferens and epididymis. The secretions of the seminal vesicles, prostate, and bulbourethral glands enter the system distal to the ligation points at the vas deferens, and these secretions are ejaculated without sperm, which nominally contributes very little to the volume of normal ejaculate. Thus, the volume of ejaculate is not noticeably changed by vasectomy. Remember “SEVEN UP” for the pathway of the sperm: Seminiferous tubules → Epididymis → Vas Deferens → Ejaculatory Duct → Nothing → Urethra → Penis

606  Clinical Anatomy Head of epididymis Vas deferens

Body Seminiferous tubule

Tail

Figure 26.17.  Testis and epididymis. One to three seminiferous tubules fill each compartment and drain in the rete testis in the mediastinum. Twelve to 20 efferent ductules become convoluted in the head of the epididymis and drain into a single coiled duct of the epididymis. The vas is convoluted in its first portion. (From Wein AJ, Kavoussi LR, Partin AW, Peters CA. Campbell-Walsh Urology. 11th ed. Philadelphia: Elsevier; 2016.)

7. Describe the neurologic basis for erection, emission, and ejaculation.   In the unaroused state, arteriovenous anastomoses allow most of the blood from the deep artery of the penis to bypass the helicine arteries within the corpora cavernosa. Upon sexual stimulation, parasympathetic input to the helicine arteries causes vasodilation and vessel straightening, greatly increasing blood flow to the corpora cavernosa, which become engorged. As the corpora cavernosa increase in volume, they compress the obliquely exiting veins against the tunica albuginea, blocking outflow of blood. Blood drainage is also restricted by contraction of the ischiocavernosus and bulbospongiosus muscles, causing a complete erection to occur. Emission occurs via sympathetic input, which causes contraction of the smooth muscle of the epididymis, vas deferens, seminal vesicles, and prostate (effectively delivering sperm and secretions to the prostatic urethra). Ejaculation is a mixed autonomic and somatic response. As the sympathetic system closes the internal urethral sphincter to guard against backflow, the parasympathetic system causes peristalsis of the urethral muscle while the pudendal nerve causes contraction of the bulbospongiosus muscle to propel the semen forward. Remember “Point and Shoot”: Parasympathetic nerves are responsible for erection (Point) and Sympathetic nerves are responsible for emission (Shoot)! 8. Name the most common drugs used for treatment of erectile dysfunction, and outline their mechanism of action.   Sildenafil (Viagra), vardenafil, and tadalafil are the drugs most commonly used to treat erectile dysfunction.     Sexual stimulation normally results in parasympathetic-mediated endothelial cell nitric oxide (NO) release within the helicine arteries of the corpora cavernosa. NO diffuses to the adjacent vascular smooth muscle, where it causes vasodilation through a multistep pathway. NO directly activates guanylyl cyclase to produce cyclic guanosine monophosphate (cGMP). This activates protein kinase G (PKG), which then activates myosin light-chain phosphatase (MLCP), which dephosphorylates myosin light chains, leading to arterial smooth muscle relaxation and increased blood flow to the corpora cavernosa.     Sildenafil, vardenafil, and tadalafil inhibit cGMP-specific phosphodiesterase-5 (PDE-5), which breaks down cGMP. Note that these drugs do not act in the absence of sexual stimulation, which is the initial event that causes helicine NO to be produced. Given their mechanism of action, it should be noted that these drugs should not be administered with nitrates because hypotension may result. Note that the commercial warnings of priapism represent an exceedingly rare side effect. In fact, the most common cause of drug-induced priapism is trazodone, a selective serotonin reuptake inhibitor used to treat depression.

SUMMARY BOX: HERNIAS AND MALE REPRODUCTIVE FUNCTION • Indirect inguinal hernia: Protrusion begins lateral to the epigastric vessels, runs through deep inguinal ring into inguinal canal, and often enters scrotum. It is congenital. • Direct inguinal hernia: Protrusion begins medial to the epigastric vessels, bypasses deep inguinal ring into inguinal canal, and rarely enters scrotum. It is acquired.

Case 26.6 A 24-year-old fishing guide presents with intermittent scrotal enlargement, which he first noticed a few months ago. He relates that the left scrotum is larger than the right and that the swelling decreases significantly when he lies down. Occasionally, he experiences an aching scrotal pain and “heaviness.” On review of systems, you discover that he and his wife have been seen in the fertility clinic because they have failed to conceive after 15 months of trying.   

Clinical Anatomy  607

1. What is the differential diagnosis for his symptoms?   Indirect inguinal hernia, varicocele, hydrocele, hematocele, testicular cancer, and infection (epididymitis or infection of the scrotal skin) are all considerations. Testicular torsion and trauma should be ruled out but are much less likely, given that the onset is not acute.

Case 26.6 continued: He is found to be afebrile and in no acute distress. He is not currently having testicular pain. Upon scrotal palpation, the left side feels like there is a “bundle of worms” superior to the testicle. The scrotum becomes less tensely swollen when he moves from the upright to the supine position. The scrotum does not transilluminate. You are unable to appreciate a hernia sac or any focal testicular masses.   

2. Which of the possibilities is now the most likely diagnosis?   Varicocele is most likely. The infertility, aching scrotal pain and heaviness, and “bag of worms” on testicular palpation suggest this diagnosis. 3. Outline varicocele pathophysiology. Be sure to explain why varicocele is more likely to occur on the left than on the right and how this condition may lead to difficulty in having children.   A varicocele refers to a varicosity (dilated and tortuous veins) of the pampiniform plexus. The exact cause is still debated, but there are at least three important factors. Recall that the left testicular vein drains into the left renal vein, which then crosses between the superior mesenteric artery and aorta to drain into the inferior vena cava (IVC). The angle of the testicular-renal vein junction is large enough to disturb flow, which may result in back pressure down into the pampiniform plexus. Likewise, because it runs between two arteries, the left renal vein is subject to compression (nutcracker syndrome), which results in back pressure into the distal renal vein, left testicular artery, and left pampiniform plexus (Fig. 26.18). However, though more rare, varicocele can occur on the right side as well (the right testicular vein drains directly into the IVC at an acute angle and has no major compression points), so it is likely that defective valves in the testicular veins play a role in the development of varicocele on both sides. This condition results in some degree of venous stasis in the pampiniform plexus, thus decreasing its ability to cool the arterial blood en route to the testes. The high intratesticular temperatures decrease sperm production and quality.

m

nu

Hepatic flexure

e od

Du

I V C

SMA Pancreas LRV

IMV L. gonadal v. L. ureter IMA Descending colon

Figure 26.18.  The retroperitoneal space has been exposed. The duodenum has been kocherized; its second, third, and fourth portions have been reflected superiorly, along with the pancreas and the superior mesenteric artery (SMA). The entire right colon has been mobilized and exteriorized. IMA, inferior mesenteric artery; IMV, inferior mesenteric vein; IVC, inferior vena cava; LRV, left renal vein. (From Wein AJ, Kavoussi LR, Novick AC, et al. Campbell-Walsh Urology. 10th ed. Philadelphia: Saunders; 2012.)

608  Clinical Anatomy 4. Describe the difference between hydrocele and hematocele.   Hydrocele is a collection of excess nonsanguineous fluid within the tunica vaginalis. It can be caused by a persistent processus vaginalis that communicates between the cavity of the tunica vaginalis and peritoneal cavity, orchitis, epididymitis, and corditis, or it can be idiopathic. Scrotal transillumination is often seen on physical examination.     Hematocele occurs when injury to the spermatic vessels leads to hemorrhage into the cavity of the tunica vaginalis.

Case 26.6 continued: He undergoes surgical correction of his varicocele. He goes on to have three children and is so pleased with how you treated him that his entire family has transferred to your care. His father, a 65-year-old bartender, first presents to your office with apparent cirrhosis and portal hypertension. He has been feeling extremely fatigued for the past 3 days. In the office, he is jaundiced and breathing rapidly (tachypnea) and has a 2/6 systolic flow murmur. You quickly send him to the ED, where he is admitted for melena and anemia. Upper endoscopy reveals bleeding esophageal varices.   

5. What are esophageal varices?   Esophageal varices are dilated esophageal veins. The esophageal venous system is one of the sites of portacaval (portosystemic) anastomosis. Esophageal vein dilation occurs because of high portal pressures that force venous flow into the systemic circuit in higher volumes than normal and in the reverse direction of physiologic venous flow, to bypass the liver. 6. How do the esophageal veins connect the portal and systemic venous systems?   To reach portal circulation, the esophageal veins drain into the left gastric vein. The left gastric vein feeds directly into the portal vein. To reach systemic circulation, the esophageal veins drain into the veins of the azygous system. Other portacaval anastomoses include: • Superior rectal veins (portal) with inferior and middle rectal veins (systemic) (dilation can lead to hemorrhoids) • Paraumbilical veins (portal) with superficial epigastric veins (systemic) (dilation can lead to caput medusae) • Various branches of the colic veins (portal) with the retroperitoneal veins of Retzius (systemic) • Branches of the splenic vein (portal) with the left renal vein (systemic) 7. Describe the function of the azygos system.   The azygos system primarily drains the posterior walls of the thorax (via intercostal and vertebral veins) and the abdomen (via ascending lumbar and vertebral veins). It also receives the mediastinal, bronchial, and esophageal veins. The azygos system is infamous for variability, but in general, the primary vein is the azygos vein, which runs vertically along the right anterolateral aspect of the vertebral column within the thorax.

SUMMARY BOX: ASYMMETRIES OF THE VENA CAVA • A varicocele is characterized by varicose veins of the pampiniform plexus. • Hydrocele and hematocele differ in that these result from fluid and blood, respectively, in the cavity of the tunica vaginalis. • The right gonadal (testicular or ovarian) vein drains directly into the inferior vena cava (IVC), and the left gonadal vein drains into the left renal vein. • The left renal vein runs between the superior mesenteric artery and the abdominal aorta. • The esophageal veins are one of the five major anastomoses that connect the systemic (via the azygos system) and portal (via the left gastric vein) systems. • The azygos system is the main venous drainage of the posterior walls of the thorax and abdomen.

Case 26.7, Part A You are volunteering as a team doctor for a local high school football team. You watch in horror as a player from the opposing team puts a vicious hit on your team’s star running back. The primary point of impact is the lateral aspect of the right knee, which was the leg he had planted in an attempt to change direction. He is unwilling to put any weight on his right leg because of the extreme pain. On examination of the knee, you note that an abnormal degree of passive tibial valgus deviation is achievable, and there is a positive McMurray test, as well as a positive anterior drawer sign.   

1. List the structures that he has injured.   He is suffering from the “unhappy triad” of knee injuries: he has torn his tibial (medial) collateral ligament (MCL), allowing tibial valgus deviation; lateral meniscus with a positive result on a McMurray test; and anterior cruciate ligament (ACL), associated with an anterior drawer sign. Note that the McMurray test can be used to check for both medial and lateral meniscus tears, depending on whether the medial or the lateral meniscus is stabilized by the examiner. 2. How does the posterior cruciate ligament differ from the anterior cruciate ligament?   The ACL runs from the posteromedial aspect of the lateral condyle of the femur to the anterior intercondylar area of the tibia. It is weaker than the posterior cruciate ligament (PCL), and it prevents anterior displacement of the tibia. Thus, a

Clinical Anatomy  609

torn ACL yields a positive anterior drawer sign: flexing the knee to 90 degrees and pulling the tibia anteriorly under a fixed femur results in the tibia’s being pulled out a short distance like a drawer.     The PCL runs from the medial condyle of the femur to the posterior intercondylar area of the tibia, crossing posterior to the ACL. The PCL prevents posterior displacement of the tibia. The most common way to suffer a torn PCL is an impact to the superior tibia with a flexed knee. Consequently, a torn PCL allows the tibia to be displaced posteriorly under a fixed femur, a maneuver known as the posterior drawer sign (Fig. 26.19).

ACL

PCL

MCL

Valgus/rotational stress

Dashboard injury

ACL

Hyperextension

Figure 26.19.  Common mechanisms of knee injury. ACL, anterior cruciate ligament; MCL, medial collateral ligament; PCL, posterior cruciate ligament. (From Browner BD, Jupiter JB, Levine AM, et al. Skeletal Trauma: Basic Science, Management, and Reconstruction. 3rd ed. Philadelphia: WB Saunders; 2003.)

3. Explain why his medial collateral ligament and anterior cruciate ligament tears led to the tear in his lateral meniscus.   In this case, he was hit in the knee from the lateral aspect with his foot planted. The force of the hit abducted his knee joint, tearing the MCL, and propelled his tibia forward in relation to the femur, tearing his ACL. Because of the knee abduction, the medial condyle of his femur was essentially lifted off of the medial meniscus, putting the burden of the impact onto the lateral meniscus, which tore in response to shear forces within the joint allowed by his torn ACL. It

610  Clinical Anatomy should be noted that the MCL fibers intertwine with those of the medial meniscus, making it possible for chronic damage to the MCL to extend to the medial meniscus. However, in acute injuries such as this one, medial meniscus injuries occur only in combination with lateral meniscus injuries.

Case 26.7, Part A continued: He returns to you a year and a half later with issues resulting from another football injury. This past season, he suffered a right fibular neck fracture, and the leg was immobilized for several weeks. Since his cast was removed, he has noticed that his right foot “hangs” and that he must step higher than he did before in order to prevent his toes from dragging on the ground. On physical examination, testing his foot dorsiflexion reveals 5/5 strength on the left and 2/5 strength on the right. In addition, he has reduced sensation over the dorsum of his right foot.   

4. This clinical picture suggests injury to what structure?   The common fibular (or peroneal) nerve is injured. This nerve runs lateral to the fibular neck, coming from just posterior to the fibular head and coursing anterior to the fibular neck, where it divides into the deep and superficial fibular nerves. Owing to its close proximity to the fibular neck, fractures of this structure often injure the common fibular nerve. 5. How does injury to the common fibular nerve result in footdrop, as seen in this patient?   Footdrop is characterized by difficulty with or an inability to perform dorsiflexion and eversion of the foot, leading to passive plantar flexion and inversion of the foot, especially when walking. Injury to the common fibular nerve is responsible for this dysfunction because the deep fibular nerve innervates the anterior compartment muscles (tibialis anterior, extensor hallucis longus, extensor digitorum longus, and fibularis tertius) that dorsiflex the foot, and the superficial fibular nerve innervates the lateral compartment muscles (fibularis longus and brevis) that evert the foot. Note that the superficial fibular nerve has sensory branches distributed upon the dorsum of the foot and the distal third of the anterior leg. Also note that the fibularis tertius acts to evert the foot and that the actions of all of these muscles have been simplified for this discussion (Fig. 26.20).

Sciatic nerve Common peroneal nerve Fibular tunnel Peroneus longus muscle Superficial peroneal nerve To peroneus brevis To extensor digitorum brevis

Tibial nerve Deep peroneal nerve To tibialis anterior To extensor digitorum longus To extensor hallucis longus Figure 26.20.  Four compartments of the leg: transverse section through middle portion of left leg. (Redrawn from Mubarak SJ, Owen CA: Double incision fasciotomy of the leg. J Bone Joint Surg. 1977;59A:184.)

STEP 1 SECRET Damage to the common peroneal nerve is a favorite on boards and is commonly posed as a mononeuropathic sequela of uncontrolled diabetes. Another favorite is damage to the tibial nerve, which innervates muscles that invert and plantar flex the foot. The tibial nerve is sensory to the sole of the foot.

6. List the muscles of the posterior compartment of the leg, and describe their innervation.   The superficial muscle group consists of the gastrocnemius, soleus, and plantaris. This is separated from the deep muscle group by the fibrous transverse intermuscular septum. The deep muscle group includes the popliteus, flexor hallucis longus, flexor digitorum longus, and tibialis posterior. The primary function of the posterior

Clinical Anatomy  611

compartment muscles is plantar flexion. They are all innervated by the tibial nerve, which arises just superior to the lateral femoral condyle from the divergence of the two nerve roots (L4-L5) that compose the sciatic nerve (the other being the common fibular nerve). It should be noted that the sciatic nerve is made up of five nerve roots (L4-L5 and S1-S3), but S1-3 are only involved in sensory innervation whereas L4-L5 are involved in skeletal muscle innervation.

Case 26.7, Part A continued: One year later, he is involved in an automobile accident and suffers a distal left femur fracture, which shears his popliteal artery. He undergoes emergent vascular surgery, and his popliteal artery is successfully repaired. However, 4 hours after surgery, Nikolai begins complaining of severe calf pain. On examination, his leg appears pale and his calf is firm. The leg is painful with passive movement and has markedly decreased sensation and posterior tibial (PT)/dorsalis pedis (DP) pulses.   

7. What dreaded vascular surgery complication has he suffered?   This patient has suffered compartment syndrome. 8. What divides the compartments of the leg?   The crural fascia envelops all of the muscles and bones of the leg. The four compartments (anterior, lateral, deep posterior, and superficial posterior) are separated by the anterior, posterior, and transverse intermuscular septa, as well as the interosseous membrane (which connects the tibia and fibula) (Fig. 26.21).

Deep fascia of leg Anterior intermuscular septum of leg 1

Posterior intermuscular septum of leg

Tibia

2

Interosseous membrane of leg

Fibula

Great saphenous vein

3

Saphenous nerve

Deep fascia of leg

4 ∗

Medial sural cutaneous nerve Small saphenous vein 1 Anterior compartment of leg: Anterior tibialis artery; vein Deep fibular nerve Tibialis anterior Extensor digitorum longus Extensor hallucis longus Fibularis [peroneus] tertius

2 Lateral compartment of leg: Superficial fibular nerve Fibularis [peroneus] longus Fibularis [peroneus] brevis

3 Posterior compartment of leg, deep part: Posterior tibial artery; vein Fibular artery; vein Tibial nerve Flexor digitorum longus Tibialis posterior Flexor hallucis longus

4 Posterior compartment of leg, superficial part: Triceps surae Plantaris

Figure 26.21.  Leg, right side; transverse section at the mid-leg level with illustration of the osteofibrous compartments; distal view. The deep fascia of leg is attached to the bones of the leg by dense connective tissue septa. (From Paulsen F, Waschke J. Sobotta Atlas of Human Anatomy. Vol. 1. Urban & Fischer: Elsevier; 2013, Fig. 4.210.)

612  Clinical Anatomy 9. Describe the major pathophysiologic characteristics of compartment syndrome.   The four compartments of the leg are invested with fascia that is highly resistant to stretching. Thus, relatively small volume increases, as would be seen in swelling, result in rapid increases in pressure. Because a compartment represents a closed system, an increase in intracompartmental pressure is directly transmitted to the vasculature, leading first to compression of small, thin-walled vessels. Higher pressures result in compression of progressively larger, thicker walled vessels. If prolonged, this leads to ischemia and necrosis. In this case, his intracompartmental pressure rose because of inflammation and swelling resulting from reperfusion injury. The most common symptoms of compartment syndrome are summed up in the 6 P’s: Pain, Paresthesia, Pallor, Paralysis, Pulselessness, and Poikilothermia.

Case 26.7, Part B A female patient sustains a gunshot wound to the right pelvis. She undergoes emergency surgery to remove the bullet. The surgery is successful, but the attending surgeon admits that some nerve damage was unavoidable during the procedure. Two months later, she presents to your clinic claiming that the surgery made her right leg shorter than her left leg. To prove this to you, she demonstrates that to keep the foot of her “longer leg” off the ground while walking, she must consciously lift it higher, or she must lean to the right while walking, in effect, using a waddling gait. You note that while her left foot is in the air, her left pelvis sags.   

0. What structure has been injured, and how has this led to her awkward gait? 1   Injury to the right superior gluteal nerve has led to paralysis of the gluteus medius and gluteus minimus muscles. These muscles abduct the thigh, and when they are not functional, the pelvis cannot be stabilized while stepping. While the patient is standing on one foot, the contralateral pelvis sags, a characteristic known as the Trendelenburg sign (Fig. 26.22). This is definitely a clinical test to know for boards.

B A Figure 26.22.  Trendelenburg test. A, Position of the hips when standing on the normal left leg. Note that the hip elevates as a result of contraction of the left hip musculature. B, Position of the hips when standing on the abnormal right leg. Note that the left hip falls as a result of lack of adequate contraction of the right hip muscles. (From Swartz MH: Textbook of Physical Diagnosis. 4th ed. Philadelphia: WB Saunders; 2001.)

Clinical Anatomy  613

SUMMARY BOX: LOWER EXTREMITY INJURIES • Note the lower extremity terminology: thigh = hip joint to knee, leg = knee to ankle, foot = ankle to digits. • The unhappy triad consists of injury to the medial collateral ligament (MCL), anterior cruciate ligament (ACL), and lateral meniscus. • The ACL prevents anterior displacement of the tibia, while the posterior cruciate ligament (PCL) prevents posterior displacement of the tibia. • The common fibular nerve is often injured in knee injuries or fibular neck fractures and results in footdrop. • A functional gluteus medius and gluteus minimus (innervated by the superior gluteal nerve) are necessary for maintaining pelvic stability while walking.

Case 26.8, Part A A 42-year-old librarian suffers from Graves disease and requires a thyroidectomy. She presents to your office 1 month after her surgery with a chief complaint of hoarseness, which she first noticed after her surgery.   

1. What is the differential diagnosis for her hoarseness?   Vocal fold paralysis, laryngitis (infectious or as a result of gastroesophageal reflux disease [GERD]), carcinoma of the vocal folds, nodule of the vocal folds, laryngeal muscle spasm, and idiopathic origin are considerations. 2. Given the surgical history, which of these diagnoses is most likely and why?   Vocal fold paralysis results from injury to the recurrent laryngeal nerve. The two recurrent laryngeal nerves run just posterior to the thyroid gland and are prone to injury during thyroidectomy. 3. How does injury to the recurrent laryngeal nerve result in hoarseness?   The recurrent laryngeal nerve gives rise to the inferior laryngeal nerve, which innervates all of the intrinsic laryngeal muscles except for one, the cricothyroid muscle. An injury to either of these nerves results in nearly complete vocal cord paralysis on the side of the affected nerve, causing hoarseness. Note that dysfunction of the posterior cricoarytenoid muscle is key in the development of hoarseness because this is the only muscle that can abduct the vocal folds. Thus, bilateral injury to the recurrent laryngeal nerve can result in dyspnea and stridor, caused by an inability to abduct either vocal fold, which obstructs the airway at the larynx.

STEP 1 SECRET Be on the lookout for damage to the recurrent laryngeal nerve that results from thyroid surgery. This is a favorite scenario on boards.

4. Describe the path of the recurrent laryngeal nerve, noting any asymmetries.   In the developing embryo, the right and left recurrent laryngeal nerves branch off of the right and left vagus nerves and loop around the fourth aortic arches on their way back into the neck, eventually ending as the inferior laryngeal nerves. The right fourth aortic arch becomes the right subclavian artery, so this is the artery that the right recurrent laryngeal nerve loops around when development is complete. In contrast, the left fourth aortic arch becomes the arch of the aorta, so this structure is what the left recurrent laryngeal nerve loops around when development is complete (Fig. 26.23). 5. Describe the innervation of the lone intrinsic laryngeal muscle not innervated by the recurrent laryngeal nerve: the cricothyroid muscle.   The cricothyroid muscle is a tensor of the vocal cords that allows high-pitched phonation. It is innervated by the external laryngeal nerve. Sometimes this nerve can be injured during thyroid surgery as well. This is one of two branches of the superior laryngeal nerve. The other branch is the internal laryngeal nerve, which supplies sensory innervation to the mucous membranes superior to the vocal folds. The superior laryngeal nerve is a direct branch from the vagus nerve.

Case 26.8, Part B You are at a fancy restaurant having dinner with your date. Unfortunately, the man at the table next to you begins choking on a piece of steak. You attempt the Heimlich maneuver with no success. You theorize that the piece of steak has entered the larynx and sent the intrinsic laryngeal muscles into spasm, thus tensing the vocal folds and obstructing the airway. As the man loses consciousness, you request a sharp knife from the nearest waitress.   

614  Clinical Anatomy

Accessory n.

Epiglottis

Left jugular v.

Superior laryngeal n.

Right vagus n.

Left vagus n.

Middle cervical ganglion

Superior cardiac br. vagus n.

Left common carotid a. 1st thoracic ganglion

Left subclavian a. and v.

Right recurrent laryngeal n.

Left recurrent laryngeal n.

Right vagus n. Trachea SVC

AO Figure 26.23.  Diagram of the vagus nerve (cranial nerve X), specifically, its branch, the recurrent laryngeal nerve, and its relationship to the large vessels of the neck. The right and left nerves are not identical, and the recurrent laryngeal nerve branches at the base of the neck on the right and in the thorax on the left. (From Goetz CG. Textbook of Clinical Neurology. 3rd ed. Philadelphia: Saunders; 2007.)

6. To save this man’s life, which structure must you incise? Why?   The cricothyroid membrane must be cut. This fibrous membrane lies inferior to the thyroid cartilage and superior to the cricoid cartilage, connecting the two structures. Note that the thyroid gland does not overlie the thyroid cartilage, as the bulk of the gland is much more caudal, lying inferior to the cricoid cartilage. Creating an opening in the cricothyroid membrane allows the passage of lifesaving air to bypass the obstruction, because the incision site is inferior to the vocal folds, which are deep to the thyroid cartilage. 7. Describe the surface anatomy of the neck that allows one to find the cricothyroid membrane.   The laryngeal prominence (Adam’s apple) is the median protrusion of the thyroid cartilage. This lies inferior to the hyoid bone at about the level of C5. By running the fingers inferior to the laryngeal prominence, down to about the level of C6, the arch of the cricoid cartilage can be palpated. The cricothyroid membrane lies just superior to the cricoid cartilage. The incision should be made here, with care not to move too far superiorly, because the thyroid cartilage lies just above. Disruption of the thyroid cartilage could damage the intrinsic laryngeal muscles (Fig. 26.24).

Clinical Anatomy  615

Thyroid cartilage

Hyoid bone

Cricoid cartilage

Cricothyroid membrane Tracheal rings

A

Thyroid cartilage Cricoid cartilage

Cricothyroid membrane Tracheal rings

B

Base of tongue Vallecula Epiglottis Hyoid bone

Aryepiglottic fold

Vocal cords

C

Figure 26.24.  Anatomy of the neck. A, Surface anatomy of the neck, showing important external landmarks. B, Anterior view of the neck, showing various internal structures (overlying superficial skin and structures removed to show cricothyroid membrane). C, Lateral view of the neck, showing various structures. (From Roberts JR. Clinical Procedures in Emergency Medicine. 4th ed. Philadelphia: WB Saunders; 2004.)

Case 26.8, Part C It is Saturday night on your pediatrics rotation, and you are covering the obstetrics floor. You enter the room of the next newborn you have prepared to see and find a mother very upset about the fact that her child has two fissures running to his mouth, one from each nostril. On physical examination, you note that the fissures, just lateral to each side of the philtrum, extend into the mouth and meet at the incisive foramen, in essence creating a U-shape from the two nostrils to the incisive foramen.   

8. With what defect(s) has this child been born?   The child has bilateral cleft lip and cleft primary palate. 9. Describe the embryologic basis for cleft lip.   Cleft lip, one of the more common developmental defects, occurs when one or both of the two maxillary processes fail to completely fuse with the corresponding medial nasal process. Note that the two medial nasal processes together make up the intermaxillary segment, the superior portion of which becomes the philtrum. 0. Describe the embryologic basis for cleft palate. 1   Clefts of the primary palate occur when the palatal shelves of the maxillary processes fail to fuse with the primary palate, itself formed by the fusion of the maxillary and median nasal processes. If it is bilateral, it results in a U-shaped

616  Clinical Anatomy fissure, with the apex of the U at the incisive foramen (this lies near the three-way fusion point of the primary palate and the two palatal shelves).     Clefts of the secondary palate occur when the two palatal shelves fail to fuse at the midline. The nasal septum can be visualized in the middle of the cleft secondary palate (Fig. 26.25).

Intermaxillary segment

A

Maxillary process

Philtrum Maxilla with 4 of lip incisor teeth Primary palate Fused palatal plates

B

Figure 26.25.  A, Schematic drawing of the intermaxillary segment and maxillary processes. B, The intermaxillary segment gives rise to the philtrum of the upper lip, the median part of the maxillary bone and its four incisor teeth, and the triangular primary palate. (From Sadler TW. Head and neck embryology. In Sadler TW, Langman J, eds. Langman’s Medical Embryology. 6th ed. Baltimore: Williams & Wilkins; 1990.)

OTHER IMPORTANT CONCEPTS IN EMBRYOLOGY OF THE FACE AND NECK 1. Discuss the difference between pharyngeal (branchial) pouches, arches, and clefts. 1   The six pharyngeal (branchial) arches consist of a combination of neural crest cells and mesoderm and play a role in the formation of many structures of the face and neck.     The four pharyngeal pouches lie internally, between the pharyngeal arches, and are lined with foregut endoderm.     The four pharyngeal clefts lie externally, between the pharyngeal arches, and are lined with ectoderm. 12. In Table 26.6, cover the right column, and name the derivatives of each structure listed in the left column. Table 26.6.   Embryology of the Face and Neck STRUCTURE

DERIVATIVE(S)

First pouch

Auditory tube and middle ear

Second pouch

Palatine tonsil

Third pouch

Inferior parathyroids and thymus

Fourth pouch

Superior parathyroids and ultimobranchial body (forms thyroid parafollicular C cells)

First arch

Malleus, incus, mandible, maxilla, zygomatic and squamous portion of the temporal bones; muscles of mastication; anterior belly of digastric, mylohyoid, tensor tympani, and tensor veli palatini muscles; innervated by cranial nerves V2 and V3

Second arch

Stapes, styloid, most of hyoid bone; muscles of facial expression; stapedius, stylohyoid, and posterior belly of digastric muscle; innervated by cranial nerve VII

Third arch

Greater cornu of hyoid bone, stylopharyngeus muscle; innervated by cranial nerve IX

Fourth and sixth arches

Laryngeal and upper tracheal cartilage; muscles of the soft palate, pharynx, and larynx; striated muscle of esophagus; innervated by cranial nerve X

First cleft

External acoustic meatus

Second/third/fourth cleft

Cervical sinus (eventually becomes obliterated)

STEP 1 SECRET Embryology in itself is a relatively low-yield subject on boards, but Table 26.6 presents extremely high-yield content. You should expect at least one question on this material. 3. From where do the parts of the thyroid gland, other than the parafollicular C cells, originate? 1   The thyroid gland develops from a proliferation of foregut endoderm at the base of the tongue. From here, the thyroid gland descends through the thyroglossal duct to just inferior to the cricoid cartilage. It remains connected to the foramen cecum via the thyroglossal duct during development. The thyroglossal duct normally degenerates before birth, but the foramen cecum persists, marking the location of the original epithelial proliferation that formed the thyroid.

Clinical Anatomy  617

SUMMARY BOX: NECK ANATOMY AND EMBRYOLOGY • The recurrent laryngeal nerves lie just deep to the thyroid and are susceptible to injury during thyroid surgery. • These nerves innervate the intrinsic laryngeal muscles except the cricothyroids. • The right recurrent laryngeal ascends from beneath the right subclavian artery. • The left recurrent laryngeal ascends from beneath the aortic arch. • The external laryngeal nerves, direct branches off the right and left vagus nerves, innervate the cricothyroid muscles. • The cricothyroid membrane lies inferior to the vocal cords (which are deep to the thyroid cartilage); an airway formed in this membrane can bypass a laryngeal obstruction. • Cleft lip: Maxillary process fails to fuse with medial nasal process. • Cleft primary palate: Palatal shelves fail to fuse with the primary palate. • Cleft secondary palate: Palatal shelves fail to fuse at the midline.

Case 26.9 A 20-year-old waitress presents to the urgent care clinic with severe, sharp, right upper quadrant (RUQ) pain of 6 hours’ duration. The pain radiates to her right shoulder, and breathing is moderately painful. She states that she had a mild RUQ ache for a few days, but it didn’t bother her and she saw no reason to see a doctor. A urine pregnancy test is negative.   

1. What is the differential diagnosis for this patient’s right upper quadrant pain?   Cholecystitis, choledocholithiasis, cholangitis, peptic ulcer disease, hepatitis, perihepatitis, hepatic abscess or tumor, pyelonephritis, nephrolithiasis, appendicitis, right lower lobe pneumonia, ovarian cysts or tumors, and acute enteritis are possibilities.

Case 26.9 continued: Her past medical history is unremarkable. However, her sexual history is notable for unprotected sexual intercourse with about 40 different partners over the past 2 years. On physical examination, you note that on palpation, she is experiencing right lower quadrant (RLQ) pain of slightly less severity than her RUQ pain. On pelvic examination, she has exquisite cervical motion tenderness and bilateral adnexal tenderness.   

2. What is the most likely diagnosis?   Pelvic inflammatory disease (PID) is most likely. 3. Name the two most common organisms implicated in pelvic inflammatory disease.   Chlamydia trachomatis and Neisseria gonorrhoeae are most commonly implicated in PID. Note that rarely PID can be caused by normal vaginal bacterial flora as well as viruses, fungi, and parasites. 4. How can pelvic inflammatory disease lead to right upper quadrant pain?   PID is classically characterized by ascent of bacteria that have infected the vagina and cervix, leading to endometritis, salpingitis, and peritonitis. Peritonitis is possible because the infundibulum of the uterine tubes opens directly into the peritoneal cavity. This means that there is a direct route from the vagina to the peritoneal cavity (via the uterus and uterine tubes) by which bacteria can ascend. In rare cases, this peritonitis can lead to RUQ pain when the offending bacteria reach the liver capsule and cause perihepatitis and inflammation of the right hemidiaphragm. This condition is known as Fitz-Hugh–Curtis syndrome. 5. Describe how the uterus and ovaries are supported.   The cervix is supported anteriorly by the pubocervical ligaments, laterally by the transverse cervical ligaments, and posteriorly by the uterosacral ligaments. The body of an anteverted uterus gains much of its support by resting on the bladder (note that this is not the case in patients with a retroverted uterus). The broad ligament, a double layer of peritoneum, extends laterally from the uterus and functions to support the uterus and all associated structures (note that the mesosalpinx portions of the broad ligament support the uterine tubes), as well as to carry the uterine vasculature. The round ligament, analogous to the spermatic cord, supports the uterine fundus. Each ovary is attached to the uterus via the ovarian ligament, is enveloped by the mesovarium portion of the broad ligament, and is supported laterally by the suspensory ligament of the ovary, which attaches to the lateral pelvic wall and carries the ovarian vasculature (Fig. 26.26). 6. Can Fitz-Hugh–Curtis syndrome be seen in males?   Although PID and Fitz-Hugh–Curtis syndrome classically develop in females through the open connection between the vagina → uterus → uterine tubes → peritoneal cavity, infection can occur through lymphatic, hematogenous, or direct spread from intraperitoneal infections. In males, infections can spread through these routes to the peritoneum and liver capsule, so they also can develop Fitz-Hugh–Curtis syndrome. Note that the lumina of the urethra, ejaculatory duct, vas deferens, epididymis, and seminiferous tubules are not continuous with the peritoneal cavity at any time.

618  Clinical Anatomy

Rectum

Appendices Fundus of uterus

Ureter Infundibulum and fimbriae of uterine tube

Ampulla of uterine tube

Suspensory ligament of ovary

Ovary, medial surface

Mesosalpinx

Mesovarian border

Isthmus of uterine tube

Ovarian ligament

Round ligament of uterus

Anterior surface of uterus

Vesicouterine pouch Bladder

Median umbilical ligament Figure 26.26.  The organs of the female pelvis. The uterus is surrounded by the bladder anteriorly, the rectum posteriorly, and the folds of the broad ligaments laterally. (Redrawn from Clemente CD. Anatomy: A Regional Atlas of the Human Body. Baltimore-Munich: Urban & Schwarzenberg; 1987.)

7. Outline the common drugs used in antimicrobial pharmacotherapy for pelvic inflammatory disease.   C. trachomatis and N. gonorrhoeae are the most common organisms, but the normal flora of the vagina (e.g., Gardnerella vaginalis, Streptococcus agalactiae) or gastrointestinal (GI) tract (e.g., Bacteroides fragilis, Peptostreptococcus, Escherichia coli) can play a role in the infection. Infection with multiple organisms is common, so broad-spectrum antibiotics are recommended. For mild to moderately severe infections, the recommended regimen is a single intramuscular (IM) dose of a third-generation cephalosporin (such as ceftriaxone or cefoxitin) plus a 14-day course of oral doxycycline and metronidazole. An alternative and equivalent treatment is a single IM dose of ceftriaxone plus high-dose oral azithromycin weekly for 2 weeks. Severe infections (i.e., those associated with tubo-ovarian abscesses) require inpatient treatment with parenteral antibiotics. The two preferred regimens are cefotetan or cefoxitin plus oral or intravenous doxycycline and clindamycin plus gentamicin. 8. In Table 26.7, cover the column on the right, and name the abdominal organs in each location. Table 26.7.   Abdominal Organs LOCATION

ABDOMINAL ORGANS

Within the peritoneal cavity

None note: The ovaries are exposed to the peritoneal cavity.

Intraperitoneal

Stomach and first part of duodenum Liver and gallbladder Spleen Tail of pancreas Jejunum and ileum Cecum and appendix Transverse and sigmoid colon

Secondarily retroperitoneala

Duodenum: first, second, and third parts Ascending and descending colon Rectum Pancreas: head, neck, and body

Retroperitoneal

Kidneys (plus ureters and adrenal glands) Abdominal aorta Inferior vena cava

aNote

that secondarily retroperitoneal organs develop intraperitoneally (covered by visceral peritoneum) but later move toward the posterior body wall and the retroperitoneal space, leaving only their anterior aspect covered by peritoneum.

Clinical Anatomy  619

SUMMARY BOX: STR UCTURE OF THE FEMALE REPRODUCTIVE SYSTEM AND THE PERITONEUM • In most cases of pelvic inflammatory disease (PID), bacteria (usually Chlamydia trachomatis or Neisseria gonorrhoeae) from the vagina pass through the cervix into the uterus and uterine tubes, causing inflammation. • Peritonitis and inflammation in the liver capsule (Fitz-Hugh–Curtis syndrome) can result because the uterine tubes open directly into the peritoneal cavity. • In males, there is no direct connection between the lumen of the genitourinary tract and the peritoneal cavity, so FitzHugh–Curtis syndrome is much more rare. • The uterus is supported by the broad ligament (composed of peritoneum), the round ligaments, and by resting anteriorly on the bladder. • The ovary is supported by the ovarian ligament, the mesovarium, and the suspensory ligament of the ovary.

Case 26.10, Part A A 64-year-old nurse suffered an MI 1 month ago. He now presents to the ED with new-onset chest pain that is intermittent and not related to exertion. The pain is severe and is located in the left precordial and retrosternal regions, and it radiates to the neck and back.   

1. What is the differential diagnosis for his chest pain?   Unstable angina, variant (Prinzmetal) angina, MI, pulmonary embolus, aortic dissection, pericarditis, pleuritis, pneumothorax, pneumonia, costochondritis, rib fracture, anxiety/panic attack, GERD, diffuse esophageal spasm, and peptic ulcer disease are possibilities. 2. A myocardial infarction in what distribution would be most concerning for damage to the sinoatrial and atrioventricular nodes?   Occlusion of the right coronary artery (RCA). The RCA supplies the AV node in nearly 100% of the population via the AV nodal branch, which originates near the origin of the posterior interventricular artery. However, it should be noted that the posterior interventricular artery is a branch of the RCA in 80% of the population and a branch of the circumflex artery in 15% of the population. The remaining 5% have other variations. The sinoatrial (SA) node is supplied by the RCA in 60% of the population via the SA nodal branch, which lies near the origin of the RCA. The SA nodal branch originates from the circumflex artery in the remaining 40%. 3. Which coronary arteries supply the left ventricle? • Anterior interventricular: Shortly after originating from the ascending aorta, the left coronary artery bifurcates. One branch, the anterior interventricular artery, also known as the left anterior descending (LAD) artery, descends in the anterior interventricular groove to the apex. This artery supplies nearly the entire interventricular septum and much of the right ventricle as well. • Circumflex: The second branch of the bifurcation of the left coronary artery, this artery runs in the AV groove to the posterior side of the heart. It also supplies the left atrium. • Left marginal: This branch of the circumflex artery descends along the left heart border. • Posterior interventricular: This artery branches from the RCA, circumflex artery, or both. It descends in the posterior interventricular groove to the apex. It also supplies a small portion of the interventricular septum and part of the right ventricle (Fig. 26.27). RIGHT ANTERIOR OBLIQUE VIEW Ascending aorta Left atrial branch SA node A.

Left main stem Diagonal branch Anterior descending Obtuse marginal

Conus branch

Circumflex

Right coronary

Septal branch

Ventricular branch

Diagonal branch

Atrial branch

Left posterolateral branch

Acute marginal AV node artery Posterior descending

Septal branch Septal perforators

Figure 26.27.  Coronary circulation. (From McCance K, Huether S. Pathophysiology. 5th ed. St. Louis: Mosby; 2006.)

620  Clinical Anatomy 4. If his chest pain were caused by pleuritis (also known as pleurisy), from which pleural layer would he be sensing pain?   He would feel pain in the parietal pleura only. In pleuritis, the inflammation involves both pleural layers. Although the visceral pleura lacks sensory innervation and therefore cannot transmit pain sensation, the parietal pleura is exquisitely sensitive to pain because it is abundantly supplied by somatic branches of the intercostal nerves (in the areas bordering the body wall) and the phrenic nerves (in the areas bordering the mediastinum and diaphragm). Because of its differential innervation, referred pain is localized to the body wall for pleuritis in the distribution of the intercostal nerves and to the shoulder and neck for pleuritis in the distribution of the phrenic nerves.

Case 26.10, Part A continued: He goes on to say that this chest pain does not feel like the pain that was associated with his heart attack. He has noticed that the pain waxes and wanes with breathing and it is position-dependent: he notes that the pain is least while he sits up and leans forward. On cardiac auscultation, a friction rub is heard.   

5. What is the most likely diagnosis?   Acute pericarditis is most likely. Weeks to months after an MI, fibrinous pericarditis can occur. This phenomenon is called Dressler syndrome and is thought to be an autoimmune reaction to novel antigens resulting from cardiac damage, most often in the setting of MI. Preferred treatment is high-dose aspirin or ibuprofen. 6. If one were to pass a needle from outside the pericardium to the lumen of the left ventricle, through which layers would it pass, in sequence? 1. Fibrous pericardium (unyielding, protects heart from acute volume overload; fused with the diaphragm, tunica adventitia of the great vessels, and the posterior sternal surface) 2. Parietal layer of serous pericardium (fused to the fibrous pericardium) 3. Pericardial fluid (normally a thin lubricating film; allows the heart to move within the pericardial sac) 4. Visceral layer of serous pericardium (synonymous with the epicardium; continuous with the parietal layer at the base of the heart) 5. Myocardium (composed of cardiac muscle) 6. Endocardium (an endothelial lining; the Purkinje fibers run between this layer and the myocardium) 7. Enlargement of which chamber of the heart is most likely to cause dysphagia?   The left atrium, located at the base of the heart, is directly anterior to the esophagus. Marked enlargement of this chamber, as can occur with mitral stenosis, can lead to compression of the esophagus around the level of T6 through T9. 8. Enlargement of which chamber of the heart is most likely to cause a parasternal lift?   A parasternal lift occurs when the right ventricle, which composes the anterior-sternocostal surface of the heart, is enlarged. The elevation is usually seen or felt just to the left of the sternum (i.e., parasternally).

Case 26.10, Part B At the end of a long week, you leave the clinic and head to a local movie theater with friends. Soon after settling into your seat, a group of teenagers clamber into the seats directly in front of you. To entertain his buddies before the show, one of the group starts tossing his chewy fruit candies high into the air, catching them in his mouth. Before you can point out the inevitable, one of the candies lands in the boy’s mouth just as he is taking a breath, and the gummy giraffe is rapidly out of sight. As the boy coughs, you recall the restaurant fiasco and prepare to start the Heimlich maneuver. However, he immediately begins to breathe, albeit with quite a bit of wheezing and dyspnea.   

9. If the candy passed into the bronchial tree, on which side would it most likely be found?   The right main bronchus is wider and oriented more vertically as compared with the left main bronchus, so it is the more likely site of aspiration. In supine patients, aspirated contents are likely to end up in the posterior segment of the right upper lobe and/or the superior segment of the lower lobe. 0. Describe the other major asymmetry of the bronchial tree. 1   The right main bronchus divides into three lobar bronchi, one for each lobe, whereas the left main bronchus divides into only two, again one for each lobe. The right lung has superior, middle, and inferior lobes. The oblique (major) fissure separates the inferior lobe from the other two lobes, and the horizontal (minor) fissure separates the superior lobe from the middle lobe. The left lung has only superior and inferior lobes, separated by the oblique fissure. The inferior portion of the superior lobe of the left lung is called the lingula. It lies adjacent to the heart and is the counterpart of the right middle lobe (Fig. 26.28).

Clinical Anatomy  621

1 2

1 3

2

1 3

6

4 8

9

4

5

5

8

8

Right 1. Apical 2. Posterior 3. Anterior 4. Lateral 5. Medial

1+

3 6

2

7+8

Key:

6. 7. 8. 9. 10.

Middle lobe

5 10 9

10

Right ant.

Upper lobe

4 6

7 9

Right lat.

1+2

2 3 4 5

Superior Medial (basal) RLL Anterior basal Lateral basal Posterior basal

Lower lobe

1+2

3

3 4

4 5

Left ant.

6

7 + 8

5

7+8

9

Left 1,2. 3. 4. 5. 6. 7,8. 9. 10.

Apical posterior Anterior Superior of lingula Inferior of lingula Superior Anterior-medial basal Lateral basal Posterior basal

Left lat.

Upper lobe

Lower lobe

Figure 26.28.  Segments of the pulmonary lobes. (Modified from Jackson CL, Huber JF. Correlated applied anatomy of the bronchial tree and lungs with a system of nomenclature. Dis Chest. 1943;9:319.)

SUMMARY BOX: CARDIOTHORACIC ANATOMY • The right coronary artery supplies the atrioventricular (AV) node. • The right coronary artery supplies the sinoatrial (SA) node in 60% of the population, with the circumflex artery supplying the SA node in the other 40%. • The anterior interventricular, circumflex, left marginal, and posterior interventricular arteries supply the left ventricle. • The left coronary artery bifurcates into the anterior interventricular and circumflex arteries. • The left marginal artery is a branch of the circumflex artery. • The posterior interventricular artery is a branch of the right coronary in 80% of the population and the circumflex in 15% of the population. • In pleuritis, although both the visceral and parietal pleurae become inflamed, pain is transmitted only from the parietal pleura because the visceral pleura lacks sensory innervation. • The outermost fibrous pericardium is fused to the parietal layer of the serous pericardium. • The parietal and visceral layers of the pericardium are continuous near the base of the heart and contain the pericardial fluid between them. • The left atrium is the most posterior portion of the heart, lying just anterior to the esophagus. • The right ventricle is the anterior-most portion of the heart, and enlargement can lead to a parasternal lift. • The right main bronchus is wider and more vertical than the left main bronchus, so an aspirated foreign body is more likely to enter it. • The three-lobed right lung possesses a middle lobe that corresponds to the lingula of the superior lobe of the left lung, which has only two lobes. • Because of inadequate respiratory development, approximately 50% of births before 24 weeks of gestation are nonviable.

CHAPTER 27

DERMATOLOGY Melissa Argraves, MD, Gillian Weston, MD, Andrew Kelsey, MD, Thomas A. Brown, MD, and Sonali J. Bracken

Insider’s Guide to Dermatology on the USMLE Step 1 You may be surprised to know that dermatology is extensively covered on the USMLE Step 1, but the reason for this is because it relates to so many other high-yield topics, particularly infectious disease and oncology. You will likely be given photographs of various lesions and rashes, and it is expected that you will be able to interpret them in the context of the question stem. Having a solid understanding of the terminology reviewed in this chapter will lead you to the correct answer in many instances. With dermatology, the rules of real estate apply: location, location, location! Be sure to pay attention to buzzwords within the question stem that will help you quickly eliminate answer choices (e.g., a rash that appears on the palms and soles, a rash that crosses midline). Skin cancer is a high-yield Step 1 topic, as are infection-associated rashes (reviewed here and in Chapters 21 and 22), eczema, bullous pemphigoid, pemphigus vulgaris, psoriasis, herpes zoster, dermatitis herpetiformis, and impetigo.

BASIC CONCEPTS 1. What are the major functions of the skin?   The skin, which comprises the largest organ in the body, acts as a semipermeable barrier from the external environment and protects individuals from mechanical stress, fluid loss, and harmful exposures (e.g., pathogens, toxins, ultraviolet [UV] radiation). It is involved in regulating body temperature and contains a variety of nerve endings that react to heat, cold, pain, and pressure. The skin also plays an important role in the synthesis of vitamin D. 2. Describe the individual layers of the skin.   See Table 27.1 and Fig. 27.1.

Epidermis

Dermis (corium)

Stratum corneum (keratin or horny layer) Stratum granulosum (granular layer) Stratum spinosum (squamous or prickle layer) Rete peg Stratum germinativum (basal layer) Sebaceous gland Hair shaft Apocrine sweat gland

Subcutaneous tissue

Adnexal epithelium Eccrine sweat gland

Figure 27.1.  Normal layers of skin. (From Yanoff, Sassani JW. Ocular Pathology. 7th ed. Philadelphia: Saunders; 2015.)

STEP 1 SECRET Students often use the mnemonic “Come, Let’s Get Sun Burned!” to recall the layers of the epidermis (Corneum, Lucidum, Granulosum, Spinosum, Basale). 3. Can you describe the dermatologic terms listed in Table 27.2?   You should have a thorough understanding of the terms mentioned in Table 27.2 because they may be used to describe lesions in a question stem. 622

Dermatology  623

Table 27.1.   Skin Layers from Superficial to Deep COMPOSITION

CLINICAL CORRELATIONS

Epidermis

1. Stratum corneum, stratum lucidum, stratum granulosum, stratum spinosum, and stratum basale, which contain: 2. Keratinocytes: Predominant cell type of the epidermis that produces keratin, which gives strength to the skin and acts as a barrier against the outside world 3. Merkel cells: Involved in sensory discrimination of textures and shapes 4. Langerhans cells: Initiate processing of foreign antigens 5. Melanocytes: Found in the stratum basale and produce melanin, which gives skin its natural pigmentation

Disruptions in the proteins involved in barrier function of the skin are responsible for diseases such as eczema. Calluses occur as a result of excess keratinocyte accumulation secondary to repeated friction Basal cell carcinoma arises from melanocytes in the stratum basale. Vitiligo, a disease of skin depigmentation, occurs secondary to melanocyte destruction

Basement membrane

Basal lamina, collagen, and connective tissue

Desmosomes connecting the basement membrane to the stratum basale are the target of autoimmune destruction in bullous pemphigoid, resulting in subepidermal blisters

Dermis

Connective tissue, hair follicles, sweat glands, sebaceous glands, apocrine glands, lymphatics, blood vessels, mechanoreceptors

Site of intradermal injections (e.g., purified protein derivative [PPD] for tuberculosis [TB] screening)

Hypodermis

Primarily composed of fat. Contains pacinian and Ruffini corpuscles responsible for sensitivity to vibration/pressure and stretch/ grip, respectively

Site of subcutaneous injections (e.g., insulin)

Table 27.2.   Terms Used to Describe the Appearance and Progression of Skin Lesions Primary Morphology Use these words to describe the appearance of the lesion. Macule

Flat area of color change; up to 1 cm in diameter

Patch

Flat area of color change; larger than 1 cm in diameter

Papule

Raised, round, lesion above the surface of the skin; up to 1 cm in diameter

Plaque

Raised, flat-topped, lesion greater than 1 cm in diameter; often formed by coalescence of papules

Nodule

Raised or flat lesion that involves the underlying dermis and often the subcutaneous tissue; larger than 1 cm in diameter

Cyst

A papule or nodule that contains fluid or semisolid material; gives resilient fluctuation upon palpation

Vesicle

Blister containing clear fluid; less than 1 cm in diameter

Bulla

Blister containing clear fluid; greater than 1 cm in diameter

Pustule

Blister filled with pus

Wheal/Hive

Evanescent (rapidly fading) papule or plaque that is typically edematous and pruritic (itchy)

Verruca

Wart

Secondary Morphology Use these words to describe the progression of the lesion over time. Scale

Hyperkeratosis (excess keratin or keratinocytes) within the stratum corneum

Crust

Rough exudate that dries on the lesion (can be from blood, plasma, or pus)

Excoriation

Superficial skin loss secondary to scratching or rubbing

Lichenification

Thickening caused by continuous rubbing (seen in untreated eczema)

Erosion

Break in the skin that involves the epidermis only Continued

624  Dermatology Table 27.2.   Terms Used to Describe the Appearance and Progression of Skin Lesions—cont’d Ulcer

Break in the skin that extends to the dermis or subcutaneous tissue

Atrophy

Thinning of skin secondary to a decrease in the underlying tissue

Hypertrophy

Excessive growth of an area of skin

Configuration Use these words to describe the pattern or shape of lesion formation. Linear

Lesion forms a line

Targetoid

Lesion has a bull’s-eye appearance

Annular

Lesion forms a ring

Nummular (Discoid)

Lesion is coin-shaped

Guttate

Individual, drop-like lesion

Confluent

Lesions are joined together

Generalized

Lesions are distributed diffusely over the whole body

Grouped

Lesions are clustered together

Solitary

A single lesion

Case 27.1 A 72-year-old Caucasian man presents to the dermatologist for a lesion on his nose that his wife has been badgering him about for the past year (Fig. 27.2). He has been an avid golfer for 35 years and has never seen a dermatologist before. The dermatologist tells the patient that he is concerned about this lesion and would like to biopsy it.   

Figure 27.2.  Appearance of lesion on patient in case 27.1. (From Lebwohl M, Heymann W, Berth-Jones J, Coulson I. Treatment of Skin Disease: Comprehensive Therapeutic Strategies. 4th ed. Philadelphia: Saunders; 2014.)

1. What is the most likely diagnosis?   This is a classic presentation for a basal cell carcinoma. Basal cell carcinoma is the most common type of skin cancer. This cancer can be locally invasive but has very low metastatic potential. The lesion is most often described as a pearly, pink papule, but it may also be described as having raised borders with an area of central ulceration. 2. What are common locations for basal cell carcinomas?   Basal cell carcinomas commonly are found on sun-exposed areas of the body, especially the face.

STEP 1 SECRET As a rule of thumb, basal cell carcinoma will generally be found at the level of the upper lip or above because these tend to be the most sun-exposed areas of the body.

Dermatology  625

3. How should this patient be treated?   This patient should be treated via excisional biopsy with clear margins. This is typically sufficient because of the low metastatic potential of basal cell carcinomas. Often, a Mohs surgeon will perform the biopsy because the tumors are frequently located on the face where cosmetically pleasing outcomes are highly desired.

Case 27.1 continued: The dermatologist also notes several small, scaly, erythematous plaques over the dorsal surface of the patient’s hands, forehead, and scalp (Fig. 27.3). He states that they are precancerous lesions and recommends treating them to prevent a cancer from developing.   

Figure 27.3.  Lesions on patient in case 27.1. (From Habif TP. Clinical Dermatology. 6th ed. Philadelphia: Saunders; 2016.)

4. What is the most likely cause of these lesions?   Excessive sun exposure caused these scaly plaques known as actinic keratoses. If untreated, these may progress into squamous cell carcinoma, which is the second most common type of skin cancer. Like basal cell carcinoma, squamous cell carcinoma has a predilection for the face, although is also found below the lower lip, on the ears, and on the hands. The lesions are often erythematous, ulcerative, and scaly. Like basal cell carcinoma, squamous cell carcinoma has a low metastatic potential but may become severely disfiguring or metastasize to local lymph nodes and other tissue sites if left untreated. Diagnosis is typically performed using biopsy (note that this skin cancer can also be histopathologically identified by the presence of keratin pearls), and like basal cell carcinoma, treatment is primarily surgical.

Case 27.1 continued: Two years later, the man returns with concern about a “mole” on his left forearm that he claims has been evolving in appearance. On examination, an 8-mm, asymmetric, brown, tan, and black papule with a scalloped border is appreciated on the extensor surface of the patient’s left forearm (Fig. 27.4). The dermatologist performs an excisional biopsy. The biopsy sample stains positive for S-100, a tumor marker for melanoma.   

5. What are the ABCDE characteristics suggestive of melanoma? • A—Asymmetry of lesion • B—Border irregularity (irregular or poorly defined) • C—Color variation from one area to another • D—Diameter greater than 6 mm (larger than a pencil eraser) • E—Evolution (changing in size, shape, or color) 6. Who is at highest risk for melanoma?   People with UV-damaged skin are at highest risk for melanoma. UV damage may result from excessive sun exposure or tanning bed exposure without use of sunscreen. Other risk factors include fair skin and personal or family history of melanoma. While fair-skinned individuals are at greater risk than dark-skinned individuals, people with darker skin are at higher risk for acral lentiginous melanoma, which typically appears on the palms and soles, under the nails, and in the oral mucosa.

626  Dermatology

Figure 27.4.  Lesion on patient in case 27.1. (From Bolognia J, et al. Dermatology. 3rd ed. Philadelphia: Saunders; 2012.)

7. To which locations does melanoma most commonly metastasize?   Melanoma most commonly metastasizes to distal skin, subcutaneous tissue, lymph nodes, brain, bone, lung, and liver. S100 positivity of a tumor found in any of these locations is a useful way to confirm that it metastasized from a primary melanoma. 8. What is the important factor in determining this patient’s prognosis?   Likelihood of metastasis, which is dependent on tumor depth, is important in determining prognosis. The deeper a melanoma extends into the dermis, the more likely it is that it will have metastasized. 9. How should this patient be managed?   Primary treatment includes excisional biopsy with clean, wide margins. Lymph node biopsy should also be performed for medium- to high-risk lesions, and imaging may be necessary for the purposes of staging. Unfortunately, metastatic melanoma is devastating and often deadly. Although chemotherapy and radiation are traditionally used for metastatic melanoma, they have not been incredibly successful. In recent years, several biologic therapies have been developed for melanoma including ipilimumab, an anti-CTLA4 antibody, and vemurafenib, a BRAF kinase inhibitor that may be used in patients positive for the BRAFV600E mutation. Note that although melanoma only accounts for about 1% of all skin cancer cases, it is responsible for the most skin cancer deaths.

SUMMARY BOX: BASAL CELL CARCINOMA • Epidemiology: Most common form of skin cancer • Presentation: Typically found on the sun-exposed portions of the body, especially the upper face • Diagnosis: Skin biopsy • Treatment: Excisional biopsy with clear margins • Prognosis: Excellent, as this tumor rarely metastasizes

SUMMARY BOX: SQUAMOUS CELL CARCINOMA • Epidemiology: Second most common type of skin cancer • Presentation: Typically found on the sun-exposed portions of the body, especially the face, lower lip, ears, and hands. Actinic keratosis is a precursor lesion. • Diagnosis: Skin biopsy. Note that this tumor can be identified from other skin cancers by the presence of keratin pearls. • Treatment: Surgical (e.g., excisional biopsy) • Prognosis: Very good if detected at earlier stages due to low metastatic potential, although does have capacity to become disfiguring and metastasize if left untreated

SUMMARY BOX: MELANOMA • Epidemiology: Most common cause of skin cancer death but accounts for only 1% of skin cancers. Often seen in patients with excessive ultraviolet (UV) damage or fair-skinned individuals. • Presentation: Positive for ABCDE characteristics • Diagnosis: Skin biopsy • Treatment: Surgical, radiation, chemotherapy, and biologics • Prognosis: Varied although does result in a significant number of fatalities. Tumor depth is typically inversely related to prognosis.

Dermatology  627

Case 27.2 A 73-year-old man with a history of Parkinson’s disease presents with several large blisters on his extremities and trunk (Fig. 27.5). He complains of severely itchy skin, which he states has been present since 2 weeks before the onset of his blistering. On physical exam, several tense bullae are appreciated. Thorough examination of the oral cavity is negative for any lesions.   

Figure 27.5.  Appearance of vesicles and bullae in patient in case 27.2. (From Goldman L, Schafer A. Goldman-Cecil Medicine. 25th ed. Philadelphia: Elsevier; 2016.)

1. What is the most likely diagnosis in this patient?   The most likely diagnosis is bullous pemphigoid, an autoimmune disorder caused by the development of immunoglobulin G (IgG) antibodies against hemidesmosomes, which anchor basal epithelial cells to the basement membrane. As a result, this condition leads to the development of tense, subepithelial blisters as opposed to the flaccid blisters seen in patients with pemphigus vulgaris. This condition most commonly develops in patients over the age of 70 and has been associated with several neurologic disorders including Parkinson’s disease, stroke, and dementia. A prodromal phase marked by development of a skin rash and/or a feeling of pruritus may precede the development of bullae. 2. How is bullous pemphigoid differentiated from pemphigus vulgaris?   Pemphigus vulgaris is an autoimmune skin disorder marked by the formation of IgG antibodies against desmoglein, a component of desmosomes, which attack adjacent epidermal cells to one another. As a result of autoantibodies attacking desmoglein, cells become separated from one another, leading to significant acantholysis (loss of intracellular connections between keratinocytes) and subsequent formation of intraepithelial, flaccid blisters. Unlike bullous pemphigoid, pemphigus vulgaris typically involves both mucosal and cutaneous sites and may make eating uncomfortable. While the incidence of pemphigus vulgaris is very low (approximately 0.5 per 100,000 people annually), Ashkenazi Jews and people of Indian, southeast European, and Middle Eastern descent are at higher risk for this condition.     For more information to help differentiate blistering disorders, see Table 27.3, which reviews presentations of the most common blistering skin disorders.

Table 27.3.   Blistering Skin Disorders BLISTERING GROSS DISORDER PRESENTATION

HISTOLOGY

PATHOPHYSIOLOGY

DIRECT IMMUNOFLUORESCENCE

Bullous pemphi- Tense blisters, usual- Subepidermal goid ly spares mucous separation membranes

IgG autoantibodies against Linear pattern hemidesmosomes

Pemphigus vulgaris

Flaccid blisters, mu- Intradermal cous membranes separation often involved

IgG autoantibodies against Reticular pattern desmoglein

Dermatitis herpetiformis

Small vesicles Subepidermal often on extensor separation surface

IgA autoantibodies deposit Granular pattern in papillary dermis; typically associated with celiac disease Continued

628  Dermatology Table 27.3.   Blistering Skin Disorders—cont’d BLISTERING GROSS DISORDER PRESENTATION

HISTOLOGY

Erythema multi- Multiple types of Varies based on forme lesions including lesion type macules, papules, vesicles, targetoid lesions

StevensJohnson syndrome

PATHOPHYSIOLOGY

Delayed type IV hypersensitivity reaction to infections (typically HSV, Mycoplasma) or drugs (typically sulfa drugs and β-lactam antibiotics)

DIRECT IMMUNOFLUORESCENCE

Nonspecific

Prodrome followed Necrotic epithe- Delayed type IV hypersen- Nonspecific by macules that lium sitivity reaction usually become flaccid to drugs (typically sulfa bullae that slough drugs, antibiotics, and off antiepileptics)

HSV, herpes simplex virus; IgG, immunoglobulin G.

Case 27.2 continued: The dermatologist performs a skin biopsy from the perilesional area of an intact bulla and direct immunofluorescence is performed. A linear staining pattern at the dermoepidermal junction is seen (Fig. 27.6).   

Figure 27.6.  Direct immunofluorescence showing linear immunoglobulin G (IgG) along the dermoepidermal junction. (From Brinster NK, Liu V, Diwan AH, McKee PH. Dermatopathology: High-Yield Pathology. Philadelphia: Elsevier; 2011.)

3. How do these results affect your diagnosis?   They confirm the diagnosis of bullous pemphigoid, which is associated with a linear band of IgG autoantibodies that deposit at the dermoepidermal junction. Similar to bullous pemphigoid, the definitive diagnosis of pemphigus vulgaris is also made by direct immunofluorescence of a skin biopsy sample. However, since pemphigus vulgaris involves deposits of IgG antibodies along the connections between cells, the immunofluorescence pattern throughout the epidermis is said to be reticular or resemble “chicken wire” (Fig. 27.7). 4. How should this patient be treated?   Treatment of bullous pemphigoid involves either class I topical or oral corticosteroids, the latter of which is typically reserved for more difficult-to-manage cases. Immunomodulatory drugs including rituximab (anti-CD20 antibody) have also been used for refractory cases. Although this condition is chronic and may wax and wane over the course of several months or years, it is usually self-limited in nature. Clinical Pearl If not treated, pemphigus vulgaris carries a high risk of secondary infection. Patients are typically administered high-dose oral steroids or immunomodulatory agents.

Dermatology  629

Figure 27.7.  Direct immunofluorescence of pemphigus vulgaris. (From High WA. Dermatopathology. 2nd ed. Philadelphia: Elsevier; 2014.)

SUMMARY BOX: BULLOUS PEMPHIGOID • Epidemiology: More commonly seen in elderly individuals and in patients with certain neurologic disorders • Presentation: Tense, subepithelial blisters; typically spares mucosal sites • Pathophysiology: Immunoglobulin G (IgG) autoantibodies against hemidesmosomes, resulting in separation of the dermoepidermal junction • Diagnosis: Direct immunofluorescence demonstrating a linear pattern • Treatment: Topical or oral steroids, immunomodulatory drugs

SUMMARY BOX: PEMPHIGUS VULGARIS • Epidemiology: Very rare; increased incidence in Ashkenazi Jews and individuals of Indian, southeast European, and Middle Eastern descent • Presentation: Flaccid blisters involving mucosal and cutaneous sites • Pathophysiology: Immunoglobulin G (IgG) autoantibodies against desmoglein (a component of desmosomes), leading to acantholysis • Diagnosis: Direct immunofluorescence demonstrating a reticular pattern • Treatment: Oral steroids, immunomodulatory drugs

STEP 1 SECRET HIGH-YIELD DERMATOLOGY The rashes discussed in subsequent cases are known to appear very frequently on Step 1. Learn their appearance, patterns, and epidemiologic presentations well!

5. A 34-year-old male presents with itchy plaques on his elbows, knees, and buttock region. He states that his mother had a similar condition.   Psoriasis is a genetically inherited autoimmune disorder. Patients commonly complain of pruritus. On examination, look out for the erythematous, silvery-scaled appearance of these plaques that may wax and wane over time. Note that these plaques are classically found on the extensor surfaces of the skin (e.g., elbows, knees) as seen in Fig. 27.8. 6. A 6-year-old asthmatic child presents with an itchy, irritated rash on his arms, inner surface of his elbows, and stomach (Fig. 27.9). His mother asks you why his rash is so much worse in the winter.   Atopic dermatitis (eczema) is a condition marked by poor skin barrier function that results in transepidermal water loss. Eczema is often found in conjunction with other allergic disorders and tends to be worse during times of the year when humidity is low. Eczema is classically found on the flexural regions of the skin but commonly presents on the cheeks in infants. Treat with moisturizing lotions and topical steroids. Bleach baths are also recommended to prevent secondary infection.

630  Dermatology

Figure 27.8.  Psoriasis on the knees. (From Gawkrodger, DJ, Ardern-Jones MR. Dermatology: An Illustrated Colour Text. 5th ed. Philadelphia: Churchill Livingstone; 2012.)

Figure 27.9.  Appearance of rash on patient in question 6. (From Lyons JJ, Milner JD, Stone KD. Atopic dermatitis in children: clinical features, pathophysiology, and treatment. Immunol Allergy Clin North Am. 2015;35(1):161-83.)

7. A 42-year-old gardener presents with a 2-day history of an itchy rash marked by erythema and blisters on his arms. He states that this has happened to him twice before.

Figure 27.10.  Note the acute eczematous rash with vesicle formation. (Courtesy of the Honickman Collection of Medical Images in memory of Elaine Garfinkel and the Jefferson Clinical Images Collection [through the generosity of JMB, AKR, LKB, and DA].)

  The patient has a poison ivy rash, an example of contact dermatitis. As the name implies, contact dermatitis results from direct skin contact with an irritating substance, leading to a type IV hypersensitivity response that occurs 24 to 72 hours after exposure. Poison ivy and similar plants such as poison oak and poison sumac precipitate acute contact dermatitis reactions, which (as opposed to chronic dermatitis) are commonly associated with erythematous vesicles and extreme pruritus. Precipitating factors for chronic contact dermatitis include metals (e.g., jewelry), soaps, detergents, and cosmetics.

Dermatology  631

STEP 1 SECRET Contact dermatitis may also be presented in the context of a patient who presents with a rash that perfectly underlies the button on their jeans. This button often contains nickel, a classic skin irritant.

8. A 7-year-old child presents to the emergency department (ED) with a whole body rash (Fig. 27.11) that developed 15 minutes after consuming shellfish for the first time. His parents deny any known allergies to foods or medications.

Figure 27.11.  Rash of child in question 8. (From Amar SM, Dreskin SC. Urticaria. Prim Care. 2008; 35(1):141-57.)

  Urticaria (hives) is most frequently triggered by a systemic allergic reaction to environmental or food substances that results in histamine release from mast cells. However, there are many nonallergic causes for urticaria that you should be aware of, including viral infections, drugs, cold temperature, and autoimmune conditions. Pay attention to time intervals, as chronic urticaria is not likely to be allergic in nature. Although this patient did not present with other symptoms of anaphylaxis, he should be given epinephrine (EpiPen) for any future emergency situations. 9. A 3-year-old female presents with 2 days of rash that her mom states has spread from her head down to her trunk (Fig. 27.12B). Before the development of this rash, the girl had spiked a 104°F fever and developed a cough; runny nose; and red, watery eyes. Her mom also noticed some tiny, white spots inside her daughter’s mouth that she swears were not there 1 week ago (Fig. 27.12A).   This presentation is very suspicious for measles, a highly contagious virus that presents with a prodromal phase consisting of high fever and the 4 “C’s” (cough, coryza, conjunctivitis, and Koplik spots). This is followed by the development of a characteristic maculopapular rash that spreads from head to toe. Koplik spots (see Fig. 27.12A) are white lesions that appear on the buccal mucosa before the development of the rash; they are pathognomonic for measles infection but are not required to make the diagnosis. 10. A 4-year-old male presents with 7 days of high fever and refusal to walk. On exam, you note that the child’s extremities are swollen and covered by an erythematous, maculopapular rash that is also present on his trunk. His tongue and oral mucosa are bright red in color (Fig. 27.13), and his lips are cracked. You note the presence of large lymph nodes on the right side of his neck and bilateral conjunctivitis that seems to spare the limbic region.   Kawasaki disease is a medium-vessel vasculitis that commonly affects children under the age of 5. In order to make this diagnosis, the patient must have fever for a minimum of 5 days along with the presence of at least four

632  Dermatology

A

B

Figure 27.12.  Rash of patient in question 9. (From Dockrell DH, Sundar S, Angus, BJ, Hobson RP. Davidson’s Principles and Practice of Medicine. 22nd ed. Philadelphia: Elsevier; 2014.)

Figure 27.13.  Tongue of patient in question 10. (From Crawford M, DiMarco J, Paulus W. Cardiology. 3rd ed. Philadelphia: Mosby; 2010.)

out of five of the following CRASH symptoms: limbic-sparing Conjunctivitis, Rash, unilateral cervical Adenopathy, Strawberry tongue, and Hand and/or foot swelling. This is one of the rare occasions where children should be treated with high-dose aspirin to prevent thrombosis. Recall that the most severe consequence of this disease is coronary artery aneurysm, for which these patients must be monitored extensively during the course of their illness.

STEP 1 SECRET A popular mnemonic for recalling the criteria for Kawasaki disease is CRASH and Burn, the latter of which will remind you of the need for at least 5 days of fever to make this diagnosis.

11. A 2-year-old male presents with a crusty-looking golden rash on his face for 2 days prior (Fig. 27.14).   Impetigo is a superficial skin infection of early childhood that is most commonly caused by Staphylococcus aureus and Streptococcus pyogenes. As a result, this rash is often seen below the nares (secondary to colonization from rubbing) and around the mouth but can be found anywhere on the body. You can recognize this rash by its yellow, crusted appearance. Impetigo is typically treated with topical antibiotics such as mupirocin but may require systemic antibiotics if invasive complications (e.g., abscess formation from methicillin-resistant S. aureus) occur. Be on the lookout for poststreptococcal glomerulonephritis and rheumatic fever, which are also potential complications that can occur following development of this rash.

Dermatology  633

Figure 27.14.  Rash of patient in question 11. (From Bolognia J, Schaffer J, Duncan K, Ko C. Dermatology Essentials. Philadelphia: Saunders; 2014.)

12. A 2-year-old male is brought in for a rash by his concerned babysitter (Fig. 27.15). You note that he has a low-grade fever and clear discharge from his nose and are reassured that this rash is not secondary to abuse despite appearing as though the child has been hit across the cheeks.

Figure 27.15.  Rash of patient in question 12. (From Cherry J, DemmlerHarrison G, et al. Feigin and Cherry’s Textbook of Pediatric Infectious Diseases. 7th ed. Philadelphia: Saunders; 2014.)

  This patient is infected with parvovirus B19, resulting in erythema infectiosum, or fifth disease, a common viral exanthem of childhood. The bright red rash that appears following the onset of fever and upper respiratory infection (URI) symptoms often occurs on the cheeks, giving the child a slapped appearance. The rash is also commonly seen on the torso, arms, and legs but is generally more reticular in appearance in these locations. 13. A 6-month-old infant is found to have a 5-day rash on her genitals, upper thighs, and buttocks (Fig. 27.16).   Diaper dermatitis (diaper rash) is an extremely common condition that is found in the skin surfaces that are in direct contact with a child’s diaper (note that the skinfolds, which do not typically come in contact with the diaper, are often spared). Excessive moisture in the diaper area secondary to the constricting effects of the diaper itself along with the presence of fecal and urinary contents disturbs the upper layers of the skin, making it increasingly susceptible to frictional damage. This damage then permits the secondary growth of bacterial and fungal organisms such as Candida albicans, which may then spread to the skinfolds. Treatment involves keeping the skin dry and use of topical steroids and barrier creams, as well as topical antifungals in more severe cases.

634  Dermatology

Figure 27.16.  Rash of patient in question 13. (From Lebwohl M, Heymann W, Berth-Jones J, Coulson I. Treatment of Skin Disease: Comprehensive Therapeutic Strategies. 4th ed. Philadelphia: Saunders; 2014.)

14. A 65-year-old, poorly controlled diabetic female presents to the ED with fever and severe pain over her right foot, which is extremely tender to palpation. Range of motion is limited secondary to swelling. Several hours later, she develops deep reddish-purple discoloration of the skin overlying her foot as her pain continues to increase (Fig. 27.17). She denies any known trauma or injuries but admits that her sensation in her feet is not well preserved.

Figure 27.17.  Foot of patient in question 14. (From Torok ME. Skin and soft tissue infections. Conlon CP. Medicine. 2009;37(11):603-609.)

  Necrotizing fasciitis is a severe soft tissue infection that results in destruction of underlying subcutaneous fat and fascial layers. Incidence is higher in diabetic individuals versus the general population and is usually the result of a mixed aerobic and anaerobic infection. Since inflammation is typically deep within the tissue, patients often present with pain and fever before the onset of skin changes. Crepitus is present in approximately half of all patients with necrotizing fasciitis and is secondary to bacterial gas formation. Necrotizing fasciitis is an emergency that must be treated with broad-spectrum antibiotics and aggressive surgical debridement.

Dermatology  635

STEP 1 SECRET Although necrotizing fasciitis is not associated with a “classic” rash that you may easily recognize from an image, this vignette is a good example of one type of question you should expect to see on the USMLE Step 1. In these questions, an image may be provided to you in order to support the ultimate diagnosis, but the definitive clues will come from history and physical findings provided in the question stem. In other words, the image itself (which you should know often projects poorly on your test screen) may not matter at all because it is frequently nonessential for answering the question correctly.

15. An 8-year-old child presents to your office for a well-child check. You note the presence of several shiny, dome-shaped papules on his trunk that his mother states have been present for 5 weeks (Fig. 27.18). Your patient affirms that they are not bothersome to him. You find out that the patient’s younger brother has a similar rash on his abdomen that began 3 months ago.

Figure 27.18.  Rash of patient in question 15. (From Habif TP: Clinical Dermatology. 6th ed. Philadelphia: Saunders; 2016.)

  Molluscum contagiosum is a highly contagious skin rash that results from infection by poxvirus. Although this rash is most common in young children, it can be transmitted sexually in adolescents and adults. This condition is typically diagnosed by its classic appearance of flesh-colored, waxy, dome-shaped papules that may persist for months before resolving. Be on the lookout for the central indentation/umbilication of these papules, which can be key to identifying molluscum. 16. An 8-year-old male presents with a 2-day history of diffuse, erythematous rash. His mother tells you that the rash first appeared in his armpits and then spread down his trunk and to his extremities. On physical exam, you note a blanching rash with numerous small papules that gives the skin a rough textural appearance (Fig. 27.19). The patient states that he had had a fever, chills, and sore throat 3 days before the onset of the rash.   Scarlet fever is a classic sequela of streptococcal A pharyngitis that typically occurs in children from ages 5 to 15. As a result, patients classically complain of fever and sore throat before the onset of the rash a few days later. Scarlet fever can be distinguished by its fine, erythematous, maculopapular appearance. Because these papules are so small and clustered together, the skin tends to adopt a rough “sandpaper-like” texture that is characteristic of this rash. Four to 7 days following its onset, the rash will begin to desquamate. Patients should be treated with antibiotics (e.g., amoxicillin) as soon as possible. 17. A 75-year-old man presents with a 3-day history of burning sensation over his left shoulder. On physical exam, you note a vesicular rash arranged in a band-like pattern over the left shoulder blade that does not extend past his neck (Fig. 27.20).   Shingles/herpes zoster is a condition that results from reactivation of latent varicella-zoster (VZV) virus within the sensory ganglia, leading to the onset of a painful, vesicular rash that follows a dermatomal distribution. This rash is easily recognizable by the fact that it does not cross the midline. It typically begins to crust 1 to 2 weeks before its onset. Treatment for herpes zoster consists of oral acyclovir or valacyclovir. The most common complication of this condition is postherpetic neuralgia, which leads to chronic and often debilitating pain as well as altered sensation over the involved areas. 18. A 34-year-old woman with a history of systemic lupus erythematosus (SLE) is prescribed trimethoprim/sulfamethoxazole (TMP-SMX) for a urinary tract infection. Two days later, she develops a fever of 103°F, malaise, joint pain, sore throat, itchy eyes, and targetoid lesions on her chest and face (Fig. 27.21). The following day, she develops painful vesicles and bullae on her chest and neck, and her lips become cracked and ulcerated. Her mouth hurts so badly that she cannot eat or drink. You notice that the surrounding mucosa and distinct areas of her skin have begun to slough off. You advise her to discontinue the medication immediately.   Stevens-Johnson syndrome (SJS) is a life-threatening rash that is thought to result from a hypersensitivity reaction that causes apoptosis of keratinocytes and separation of the epidermis from the dermis. Although SJS resembles toxic

636  Dermatology

Figure 27.19.  Rash for patient in question 16. (Courtesy Dr. Franklin H. Top, Professor and Head of the Department of Hygiene and Preventive Medicine, State University of Iowa, College of Medicine, Iowa City, IA; and Parke, Davis & Company’s Therapeutic Notes. From Gershon AA, Hotez PJ, Katz SL. Krugman’s Infectious Diseases of Children. 11th ed. Philadelphia: Mosby; 2004.)

Figure 27.20.  Shoulder of patient in question 17. (From High WA. General Dermatology. Philadelphia: Saunders; 2009.)

Figure 27.21.  Appearance of patient in question 18. (From Ferri F, Studdiford J, Tully A. Ferri’s Fast Facts in Dermatology: A Practical Guide to Skin Disease and Disorders. Philadelphia: Saunders; 2010.)

Dermatology  637

epidermal necrosis (TEN) in disease pathogenesis, the latter is considered to be a more severe form of the illness. Mucous membrane involvement and targetoid lesions are hallmarks of the disease. The most common causes of SJS include infections, medications (particularly sulfa drugs, penicillin, analgesics, and anticonvulsant agents), and malignancy. Patients with SLE are at increased risk for this condition. Clinical Pearl Erythema multiforme (EM) is a self-limited condition that occurs as a hypersensitivity reaction to drugs and infections. Although it typically presents with localized papules and targetoid lesions, unlike Stevens-Johnson syndrome (SJS), the mucosa is seldom involved. However, even EM can present with a wide spectrum of severity.

19. A 54-year-old woman presents for her annual preventative care visit. Your preceptor tells you to examine her before he comes into the room. You note a morbidly obese but otherwise well-appearing female with thick, dark brown plaques around her neck (Fig. 27.22) and in her axillary folds.

Figure 27.22.  Neck of patient in question 19. (From MurphyChutorian B, Han G, Cohen SR. Dermatologic manifestations of diabetes mellitus: a review. Endocrinol Metab Clin North Am. 2013;42(4):869-898.)

  Acanthosis nigricans is a common condition marked by hyperpigmentation of intertriginous areas of the skin (e.g., neck, axillae, groin, skinfolds of the breasts). It is linked to conditions that result in insulin resistance, including obesity, type 2 diabetes mellitus, and polycystic ovarian syndrome. This rash can easily be recognized by its velvety, hyperpigmented appearance in patients with comorbid conditions. 20. A 65-year-old man presents for his annual preventative care visit. His wife is concerned about the red spots on his back (Fig. 27.23), though he states that they have been present for years and have not changed in appearance or caused him any pain.

Figure 27.23.  Appearance of skin lesion for patient in question 20. (From Brinster NK, Liu V, Diwan AH, McKee PH. Dermatopathology: High-Yield Pathology. Philadelphia: Elsevier; 2011.)

  Campbell de Morgan spots are benign skin growths that result from abnormal proliferation of endothelial cells within capillary beds. As they are easily recognizable by their bright red papular appearance, they are more commonly referred to as cherry angiomas. Although they may bleed if scratched or ruptured, they are generally not concerning so long as they remain stable in appearance.

638  Dermatology 21. A 70-year-old Caucasian man presents to your office for a rash on his back. He recently went to the beach with his brother, who noticed the lesions on his back and told him that he may have skin cancer. On physical examination, you note several well-demarcated, scaly, and hyperpigmented lesions with a “stuck-on” appearance (Fig. 27.24).

Figure 27.24.  Appearance of skin lesion for patient in question 21. (From Brinster NK, Liu V, Diwan AH, McKee PH. Dermatopathology: High-Yield Pathology. Philadelphia: Elsevier; 2011.)

  Seborrheic keratoses are common growths of middle-aged and elderly individuals that result from benign proliferation of immature keratinocytes. This condition is typically marked by the presence of several, well-demarcated round or oval hyperpigmented lesions on the back, trunk, arms, or face, although singular lesions may also be present. As these are slow-growing, noncancerous lesions, treatment is not generally indicated unless they cause pain or cosmetic issues. However, biopsy should be performed if diagnosis is uncertain or there is suspicion of malignancy. 22. A 20-year-old woman presents with an intensely itchy rash on her knees, ankles, and feet (Fig. 27.25). When you ask her about associated symptoms, she tells you that over the past 6 months she has been experiencing significant abdominal pain, bloating, and diarrhea with meals.

Figure 27.25.  Appearance of rash for patient in question 22. (From Bolognia J, Jorizzo J, Schaffer J. Dermatology. 3rd ed. Philadelphia: Elsevier; 2012.)

  Dermatitis herpetiformis is a highly pruritic, symmetrically distributed blistering condition of the skin that results from deposition of IgA autoantibodies in the subepidermal layers of the skin. The vast majority of patients with this condition have an associated history or symptoms of celiac disease (this clue will absolutely be given to you on Step 1!). This condition responds extremely well to a gluten-free diet.

Kyle T. Wright, MD, PhD, Thomas J.S. Durant, MPT, MD, Rebecca Flugrad, Thomas A. Brown, MD, and Sonali J. Bracken

CHAPTER 28

PATHOLOGY

Insider’s Guide to Pathology for the USMLE Step 1 The USMLE loves gross images, histopathologic images, blood smears, and radiographs, and they will show up on your examination. Study the high-yield images in this chapter (yes, they are all high yield) and in First Aid for the USMLE Step 1 carefully. Go over them more than once. Pattern recognition is important because you will most likely see slightly different images on your exam. You will find that the images in this chapter are divided into five major sections: H–Hematopathology O–Oncology G–General Pathology R–Radiology M–Microbiology The vignettes in this chapter are commonly used on the USMLE Step 1. Be sure to pay close attention to associations and “buzzwords.”

1. A 17-year-old Caucasian female is seen by her gynecologist for her yearly exam. She reports regular but heavy menstrual periods that last 7 to 8 days. She also reports feeling “tired a lot.” When discussing her diet, she states that she eats “a normal amount” but does not like meat. She takes no medications or supplements. Physical exam is unremarkable. The following image was obtained from a smear of her peripheral blood (Fig. 28.1). What laboratory tests would confirm the likely diagnosis in this patient?

Figure 28.1.  Peripheral blood smear of patient in question 1. (From Aster J, Pozdnyakova O, Kutok J. Hematopathology: A Volume in the High Yield Pathology Series. Philadelphia: Saunders; 2013:19.)

  Low hemoglobin, low ferritin, low mean corpuscular volume (MCV), and high total iron-binding capacity (TIBC) would likely confirm the diagnosis because this patient suffers from iron-deficiency anemia (IDA). The peripheral blood smear in Fig. 28.1 shows the classic picture of IDA consisting of red blood cells (RBCs) that are hypochromic (increased central pallor) and microcytic (small). Inadequate intake of dietary iron or increased blood loss can lead to IDA. Young females with heavy menses are at particularly increased risk because of ongoing losses as well as lower overall stores compared to men. This patient also reports a diet that includes little to no meat, and although vegetables do contain free ferric iron, it is inefficiently absorbed compared to the heme-bound iron found in meat products. Often, patients with IDA have vague symptoms such as fatigue and weakness, or they can be completely asymptomatic. An iron supplement may be of benefit to this patient. 639

640  Pathology 2. A 23-year-old African-American male is brought to the emergency department with intense chest pain. He reports the chest pain is constant, 10/10 in severity, and has only been partially relieved by opioid analgesics given in the emergency department. He states that the pain began this morning and has progressively gotten worse throughout the day. Recently, he was treated for an upper respiratory tract infection. On presentation, his only other symptoms include a cough and a low-grade fever. He states that he has been to the hospital before with similar symptoms. His chest x-ray is remarkable for bilateral pulmonary infiltrates. During his workup, the peripheral blood smear shown in Fig. 28.2 was obtained. What complication is this patient most likely experiencing?

Figure 28.2.  Peripheral blood smear of patient in question 2. (From Barth D. Approach to peripheral blood film assessment for pathologists. Semin Diagn Pathol. 2012;29:31-48.)

  This patient is most likely experiencing acute chest syndrome, a vaso-occlusive crisis of the lungs typically seen in the setting of respiratory infections in patients with sickle cell anemia. The peripheral smear shows classic sickle-shaped RBCs (solid arrow), frank anemia, and Howell-Jolly bodies (nuclear RBCs remnants due to hyposplenism; dashed arrow). Tissue hypoxia is the main cause of sickle cell formation in patients with the HbS variant of hemoglobin beta chain. Precipitators of sickle cell pain crises are often difficult to ascertain; however, in this case the patient is recently getting over an upper respiratory tract infection, which can promote sickling through hypoxemia. The mainstay of treatment is pain management; aggressive hydration; broad-spectrum antibiotics; and in many cases, exchange transfusion. 3. A 45-year-old Caucasian female is seen by her primary care physician for complaints of general fatigue and weakness. She also is concerned that her tongue “looks different.” Her past medical history includes a sleeve gastrectomy 15 years ago for morbid obesity. With this procedure, and a strict post-op dietary regimen, she was able to significantly decrease her body mass index. Since that time, she has had intermittent follow-up because of problems keeping insurance. On physical exam, she is noted to have a slightly enlarged, “beefy,” magenta-colored tongue. Fig. 28.3 shows a peripheral blood smear from this patient. What is the most likely etiology of her fatigue?

Figure 28.3.  Peripheral blood smear of patient in question 3. (From Shah DR, Daver N, Borthakur G, et al. Pernicious anemia with spuriously normal vitamin B12 level might be misdiagnosed as myelodysplastic syndrome. Clin Lymphoma Myeloma Leuk. 2014;14:e141-e143.)

Pathology  641

  Vitamin B12 deficiency is the most likely cause of her fatigue. The cellular abnormality in question is a hypersegmented neutrophil (polymorphonuclear leukocyte; see Fig. 28.3, arrow), which is commonly seen in megaloblastic anemias. Megaloblastic changes are seen in situations that interfere with DNA synthesis, and thus a dyssynchrony between cytoplasmic and nuclear development exists. Common causes of megaloblastic anemias include vitamin B12 and folate deficiencies as well as treatment with medications that interrupt folate synthesis, such as methotrexate. In this case, the patient underwent sleeve gastrectomy, which has reduced her overall levels of intrinsic factor production, a glycoprotein necessary for vitamin B12 absorption. The patient’s glossitis is also a hallmark of vitamin B12 deficiency, although the exact mechanism of its development is incompletely understood. 4. A 75-year-old Caucasian female is evaluated for increasing fatigue, unintentional weight loss, night sweats, and a sense of fullness in her left upper abdomen. Her exam is benign except for some dullness to percussion in the left upper quadrant. During her workup, a peripheral blood smear shows teardrop-shaped red blood cells (dacrocytes) and many immature granulocytes and red blood cells (leukoerythroblastosis). Subsequently, the patient was sent for a bone marrow biopsy, which resulted in the image shown in Fig. 28.4 when stained for reticulin. What is the likely diagnosis in this patient?

A

B

Figure 28.4.  A, Peripheral blood smear from patient in question 4. B, Bone marrow biopsy stained for reticulin (Part A adapted from Goldman L and Schafer AI. Cecil Textbook of Medicine, 24 ed. Philadelphia: Elsevier; 2011. Part B from S. Hudnall. Hematology: A Pathophysiologic Approach. Philadelphia: Mosby; 2012:209, Figure 12-30.)

  A marrow-infiltrating disorder such as myelofibrosis is the likely diagnosis. The marked increase in reticulin-stained marrow is consistent with the diagnosis of myelofibrosis. The peripheral smear will often show dacrocytes (arrows in Fig. 28.4A, due to RBC damage in the marrow) as well as the premature release of immature forms (leukoerythroblastosis). Myelofibrosis is classified as a myeloproliferative disease (MPD), which includes conditions such as chronic myelogenous leukemia (CML), essential thrombocytosis, and polycythemia vera. The disease is most likely caused by the inappropriate release of fibrogenic factors (transforming growth factor β [TGF-β]/platelet–derived growth factor [PDGF]) from megakaryocytes, which are often seen as large and dysplastic cells within the marrow. Like other MPDs, activating JAK2 mutations have been implicated as a causative derangement and provide a unique therapeutic target for ongoing clinical trials. The presentation of classic “B symptoms” (fever, night sweats, weight loss) is likely due to the vast amount of extramedullary hematopoiesis that occurs in the spleen due to an obliterated bone marrow. This process is ultimately disordered for unknown reasons, eventually lagging behind and resulting in pancytopenia as the disease progresses. 5. An 86-year-old female with Alzheimer’s dementia lives in a long-term nursing facility. Because she is unable to care for herself, she has a urine catheter in place. Today, she is rushed to the emergency department with a temperature of 102.5°F and rigors. In the emergency department, she has a white blood cell count of 20,000, her blood pressure is 80/60 mm Hg, heart rate is 122 bpm, and she has an elevated serum D-dimer. Fig. 28.5 shows the image of the peripheral smear taken during her workup. What process is occurring in this patient?   Disseminated intravascular coagulation (DIC), exemplified by schistocytes on the peripheral smear (arrows) and elevated D-dimer in the context of evolving urosepsis, is occurring in this patient. DIC can be initiated by many factors, but the most common mechanisms involve widespread endothelial cell injury leading to microangiopathic hemolytic anemia or systemic release of tissue factor and/or thromboplastic substrates (e.g., in placental products, retained dead fetus, APML, and some adenocarcinomas).

642  Pathology

Figure 28.5.  Peripheral blood smear of patient in question 5. (From Carey, WD. Cleveland Clinic: Current Clinical Medicine. 2nd ed. Philadelphia: Saunders; 2011:579, Figure 1.)

6. A 66-year-old black male is evaluated for increased weakness, lethargy, constipation, body aches, and polyuria. His physical exam is completely unremarkable. Laboratory workup reveals an elevated serum calcium, elevated blood urea nitrogen and creatinine, and a mild anemia. A urinalysis is unremarkable except for 3+ proteinuria, and a skeletal survey is done (Fig. 28.6). A bone marrow biopsy and serum protein electrophoresis (SPEP) are ordered to confirm the diagnosis. A peripheral blood smear from the patient is shown in Fig. 28.7. What is the likely diagnosis for this patient?

RT

Figure 28.6.  X-ray of right femur of patient in question 6. (From Punja M, McWey RP, Heller M. Lytic lesion. J Emerg Med. 2013;44:179-180.)

  Multiple myeloma (MM) is the likely diagnosis. MM is a plasma cell neoplasm that causes clinical features due to occupying lesions in bone marrow, excessive production of immunoglobulin, and interference with normal humoral immunity. The important clinical sequelae are compiled in the “CRAB” criteria: elevated serum Calcium, Renal disease (secondary to glomerular damage from excessive light chain aggregates sometimes referred to as Bence-Jones proteins), Anemia, and lytic Bone lesions. The “punched-out” bone lesions shown in Fig. 28.6 are a result of upregulation of receptor activator of nuclear transcription factor-kappa B ligand (RANKL) by myeloma-derived CCL3, which in turn activates osteoclasts and leads to bone resorption and hypercalcemia. 7. A 32-year-old Caucasian male is seen in the emergency department for right upper quadrant pain, which he describes as constant and sharp. He reports a recent history of similar pain that was transient and was especially noticeable after eating at fast food restaurants. His only other pertinent medical history is long-standing anemia. His physical exam is remarkable for

Pathology  643

Figure 28.7.  Peripheral blood smear of patient in question 6. (From Jaffe E. Hematopathology. Philadelphia: Saunders; 2011, Figure 25-4B.)

tenderness to palpation in the right upper quadrant, splenomegaly, and yellow discoloration under his tongue. A right upper quadrant ultrasound demonstrates stones, one of which is occluding the opening of the cystic duct. His complete blood count shows a low hemoglobin and hematocrit and a high mean corpuscular hemoglobin concentration. An image from his peripheral smear is shown in Fig. 28.8. How can his current condition be explained?

Figure 28.8.  Peripheral blood smear of patient in question 7. (From Klatt E. Robbins and Cotran Atlas of Pathology. 3rd ed. Philadelphia: Saunders; 2015, Figure 4-7.)

  This patient suffers from hereditary spherocytosis (HS). HS is due to an inherited defect in proteins important for RBC skeleton structure—namely spectrin, and ankyrin. Seventy-five percent of cases are autosomal dominant with the remaining 25% being compound heterozygotes with much more severe disease. The loss or dysfunction of these proteins lowers the deformability of the RBC, causing these cells to adopt the shape with the smallest diameter for any given volume: a sphere. These spherocytes (arrows) are hyperchromic and have a high mean corpuscular hemoglobin concentration (MCHC). In HS, the normal lifespan of the RBC is decreased from 120 days to 10 to 20 days because these

644  Pathology cells are more susceptible to both rupture and destruction in the spleen, resulting in anemia. Three important clinical sequelae are aplastic crisis in patients with Parvovirus infections (which halts normal erythropoiesis by infecting RBC progenitors), hemolytic crisis with any inciting event that increases hemolysis of RBCs, and biliary colic or frank obstructive cholecystitis from indirect hyperbilirubinemia and subsequent formation of bilirubin pigment stones (as is seen in this patient). Patients may benefit from therapeutic splenectomy. 8. A 21-year-old female presents to her primary care physician for a lump on her neck. She has had a low-grade fever for several weeks, night sweats, and unintentional weight loss. The patient has large, painless cervical lymphadenopathy. Her past medical history includes infectious mononucleosis but is otherwise unremarkable. A biopsy of her lymph node is taken and the slide appears as shown in Fig. 28.9. A minority of cells appear to be large and bilobed and are shown by immunohistochemical analysis to be CD15+. What is the most likely diagnosis in this patient?

Figure 28.9.  Biopsy of lymph node of patient in question 8. This high-power photomicrograph shows the characteristic Reed-Sternberg cell (arrow) of Hodgkin’s lymphoma, identified by its “owl-eye” nucleus. (From Neville BW, et al. Oral and Maxillofacial Pathology, 3rd ed. Philadelphia: Saunders; 2009.)

  Hodgkin lymphoma is the most likely diagnosis. Hodgkin lymphoma has a bimodal distribution, presenting in young adults or the elderly, and progresses in a predictable fashion. B symptoms, such as night sweats, weight loss, and low-grade fevers are the first to appear, and as the disease continues, lymph nodes, spleen, liver, bone marrow, and other organs become involved. Because of the predictable progression, staging is useful because it will inform decisions about treatment plans. Hodgkin lymphoma is associated with EBV infections because EBV-infected tumor cells express a protein from the EBV genome that allows for upregulation of NF-κB and other genes that prompt lymphocyte proliferation. Additionally, Reed-Sternberg (RS) cells (arrow) release factors that lead to an accumulation of reactive lymphocytes, macrophages, and granulocytes, which account for most of the mass in Hodgkin tumors. RS cells are positive for the marker CD15 and can be seen on histology as bilobed or binucleated cells. While they are critical for diagnosis, they alone are not pathognomonic for Hodgkin lymphoma, as they can be seen in infectious mononucleosis, other cancers, and large-cell non-Hodgkin lymphoma. Several subtypes of Hodgkin lymphoma exist, including nodular sclerosis type (most common), mixed cellularity type, lymphocyte-rich type, and lymphocyte-deplete type. Another related disease moiety, referred to as lymphocyte-predominant Hodgkin, also exists, but it acts very differently from classic Hodgkin and has a different immunophenotype. Each of these different types has characteristic features that are beyond the scope of USMLE Step 1. 9. A 58-year-old man is seeing his primary care physician for a nonhealing lesion on the lateral surface of his tongue (Fig. 28.10) and some unusual oral bleeding. He is otherwise asymptomatic. The patient has no significant past medical or family history of malignancy, but he consumes 3 to 4 beers daily and has smoked 2 packs of cigarettes per day for the past 25 years. The patient is referred to a dermatologist who performs a biopsy, which yields the image shown in Fig. 28.11. What is the likely diagnosis?

Pathology  645

Figure 28.10.  Tongue lesions of patient in question 9. (From Sook-Bin W, Lin D. Morsicatio mucosae oris—a chronic oral frictional keratosis, not a leukoplakia. J Oral Maxillofac Surg. 2009;67:140-146.)

Figure 28.11.  Biopsy sample of the patient in question 9. (From Ojha J, Kossak E, Mangat S, et al. Recurrent pain and swelling associated with impacted maxillary third molar. J Am Dent Assoc. 2015;146:840-844.)

  Oral squamous cell carcinoma (SCC) is the likely diagnosis. SCC is a cancer of stratified squamous epithelium and is usually a disease found in older people that is also more commonly seen in men. Although SCC is often found on sunexposed areas such as the face and outer lip, the most significant contributors to the risk of developing oral SCC are excessive smoking and alcohol consumption. Persistent red or white patches, ulcers, and overlying swelling as shown in Fig. 28.10 are classic patterns for oral SCC. Oral SCC is often asymptomatic or associated with oral bleeding at the time of presentation but may progress to cause pain with movement of the tongue, tooth pain, dysarthria, and dysphagia. On biopsy, circular eosinophilic keratin pearls (see Fig. 28.11, arrow) and intercellular bridging (unable to be appreciated at low magnification) are two histologic features that may be helpful for diagnosis. If localized (i.e., no lymph node involvement), 5-year survival rates typically exceed 75%.

646  Pathology 10. A 54-year-old male with a history of chronic obstructive pulmonary disease is evaluated for several weeks of cough, dyspnea, unintentional weight loss, occasional hemoptysis, and significant muscle weakness. Exam is significant for decreased breath sounds in the right lung field and muscle weakness that seems to improve with serial measurement. Specialized testing reveals high levels of an antibody to presynaptic calcium channels. Imaging shows a central lung mass obstructing the right main bronchus. The mass is biopsied and stains positive for synaptophysin and chromogranin. The biopsy is seen in Fig. 28.12. What is the likely diagnosis?

Figure 28.12.  Biopsy sample of the patient in question 10. (From Skarin A. Atlas of Diagnostic Oncology. 4th ed. Philadelphia: Mosby; 2010.)

  Small cell carcinoma is the likely diagnosis. Small cell carcinoma of the lungs (also referred to as oat cell carcinoma) is primarily seen in males with a history of smoking. It is found centrally within the lungs (e.g., in the main bronchus as in this case). On biopsy, round to oval cells with little cytoplasm can be visualized. Cells are arranged in clusters without glandular or squamous organization, and necrosis is common and substantial. Small cell carcinoma originates from neuroendocrine progenitor cells lining the bronchial epithelium, and some have the ability to produce neurosecretory granules containing adrenocorticotropic hormone (ACTH) or antidiuretic hormone (ADH), causing paraneoplastic syndromes of Cushing syndrome and syndrome of inappropriate secretion of antidiuretic hormone (SIADH). Tumors are often positive for chromogranin and synaptophysin as a result of their neuroendocrine origins. Recall that patients with small cell carcinoma of the lungs may also present with Lambert-Eaton syndrome as in this case because of the production of antibodies against presynaptic calcium channels. This leads to symptoms of muscle weakness that improve with use. 11. A 32-year-old woman is evaluated for a lump on her throat. Past medical history is remarkable only for undergoing successful treatment for head and neck cancer as a child. She feels well otherwise and has no other complaints. On exam, there is a small nodule protruding from the thyroid that moves freely during swallowing. Scintigraphy reveals poor uptake of radioactive iodine by the nodule. A fine needle aspirate and biopsy are taken. The aspirate shows emptyappearing nuclei, and the biopsy is shown in Fig. 28.13. What environmental exposures may have contributed to her current condition?   Previous exposure to radiation may have contributed to this patient’s presenting with papillary thyroid carcinoma. Papillary carcinomas are the most common type of thyroid cancer found in the United States, and they are often associated with a mutation in the RET proto-oncogene or ionizing radiation (this patient was presumably treated with ionizing radiation for her head and neck cancer as a child). On histology, the hallmarks of this disease are psammoma bodies (thin arrow) and orphan Annie eye nuclei (thick arrow). Psammoma bodies are circular calcifications, and are rarely found in other thyroid cancers, so they are a good indication for papillary carcinoma. Orphan Annie eyes (arrow) are empty-appearing nuclei that are created by dispersed chromatin. They may also be evident as intranuclear grooves, which are thin, hypochromatic lines traversing the nucleus. Papillary thyroid cancer has an excellent prognosis in the large majority of patients.

Pathology  647

Figure 28.13.  Biopsy sample of patient in question 11. (From Klatt. Robbins and Cotran Atlas of Pathology, 3rd ed. Philadelphia: Elsevier; 2015.)

12. A 28-year-old woman presents to her gynecologist for infertility. The patient has had spotting that began a few years after menarche, and she has been trying to conceive for nearly 2 years with no success. On exam, the physician palpates a pelvic mass. Ultrasound reveals several round masses in the uterus (see Fig. 9.9 for reference), and the patient is sent for surgery. The surgeon discovers multiple round, well-circumscribed masses of various sizes within the myometrium and excises them. The masses are biopsied, and a representative image is shown in Fig. 28.14. What is the most likely diagnosis in this patient?

Figure 28.14.  Biopsy sample of patient in question 12. (From Oliva E, Baker PM. Endometrial/ioid stromal tumors and related neoplasms of the female genital tract. Surgical Pathol Clin. 2009;2:679-705.)

  Leiomyoma is the most likely diagnosis. Leiomyomas, or fibroids, are a very common neoplasm in women, consisting of a benign growth of well-differentiated smooth muscle cells and a capsule. It is not uncommon to find multiple leiomyomata within the myometrium of a patient’s uterus. The masses have a characteristic whorled pattern that can be seen both grossly on bisection and microscopically. Cells have long nuclei and cytoplasmic processes with a low mitotic index, making them nearly indistinguishable from normal smooth muscle. Leiomyomata are often asymptomatic; however,

648  Pathology they can present with abnormal vaginal bleeding and impaired fertility since implantation is less likely to occur at a site containing a fibroid mass. Growth can change over time because these tumors are sensitive to estrogen, causing them to shrink after menopause. Fibroids are generally not considered precancerous and very rarely convert to a malignant neoplasm (leiomyosarcoma). 13. A 32-year-old woman presents to her gynecologist for infertility. She has pain with intercourse as well as dysmenorrhea. On exam, her cervix and uterus are within normal limits; however, her ovaries are somewhat enlarged on palpation. Ultrasound shows cystic dilations in both ovaries. Surgery is performed; a depiction of one of her ovaries if additional dissection and examination were to be performed is as shown in Fig. 28.15. What is the likely etiology of this patient’s infertility?

Figure 28.15.  Gross pathology of ovary for patient in question 13. (From Rosai J. Rosai & Ackerman’s Surgical Pathology. 10th ed. Philadelphia: Mosby; 2011, Figure 19.225.)

  Endometriosis is the likely etiology of the infertility. When endometrial tissue is found outside of the uterus, it is referred to as endometriosis. The ovary and fallopian tube are commonly affected sites. There are several hypotheses for the development of endometriosis, including retrograde menstruation through the fallopian tubes, lymphatic or vascular spread of endometrial tissue, and development of multiprogenitor cells into endometrial cells that seed in abnormal locations. The tissue responds to estrogen the same way that endometrium in the uterus does by entering a proliferative state. During menstruation, foci of endometriosis also bleed and cause painful menstrual cycles by inducing a strong inflammatory response. Chocolate cysts, or endometriomas, of the ovaries represent sites of previous hemorrhage. Scarring may also occur in these locations, leading to infertility. Since the ectopic tissue responds to estrogen, foci enlarge during pregnancy and decrease in size after menopause. 14. A 42-year-old male with a history of gastroesophageal reflux disease is evaluated for persistent heartburn. Upper endoscopy reveals red patches extending from the gastroesophageal junction. A biopsy from the lower esophageal segment is shown in Fig. 28.16. For what condition is this patient at increased risk?   This patient is at increased risk for esophageal adenocarcinoma secondary to Barrett’s esophagus. Long-term acid stress from chronic gastroesophageal reflux disease (GERD) leads to intestinal metaplasia, in which the nonkeratinized stratified squamous epithelium in the lower esophagus becomes nonciliated columnar with interspersed goblet cells. Intestinal metaplasia in the esophagus is called Barrett’s esophagus. The risk for dysplasia and adenocarcinoma is increased in patients with Barrett’s, and periodic endoscopy with biopsy is recommended for this population. 15. A 3-year-old boy, accompanied by his mother, is seen by his pediatrician for an abdominal lump. The mother explains that as she was holding her son, she noticed a hard lump on the right side of his abdomen. She also reports that she has noticed dark urine over the past week. On exam, the patient has elevated blood pressure and a large, smooth, right-sided mass that extends across the midline. Urinalysis reveals hematuria. The boy is sent for imaging, and a photo of the radiologic findings is presented in Fig. 28.17. What is the most likely diagnosis in this patient?   Wilms tumor is the most likely diagnosis. The most common primary renal tumor found in childhood is Wilms tumor. Most cases occur before the age of 10. Symptoms include hematuria, hypertension, and a large palpable abdominal mass caused by a well-circumscribed tumor that can become so large that it crosses the midline. These masses are often accompanied by deletions or mutations in the WT1 gene, which is important for both kidney and genital formation. WT1 insults may also result in WAGR complex, which causes Wilms tumor, aniridia, genital malformations, and mental retardation.

Pathology  649

Figure 28.16.  A biopsy from the lower esophageal segment of patient in question 14. (From Mikhail L, Amitabh S. Barrett esophagus: evolving concepts in diagnosis and neoplastic progression. Surg Pathol Clin. 2013;6:475-496.)

Figure 28.17.  Abdominal CT scan of patient in question 15. (From Ehrlich PF. Wilms tumor: progress and considerations for the surgeon. Surg Oncol. 2007;16:157-171.)

16. A 16-year-old female is evaluated for bloody stool. The patient has had several episodes of hematochezia, abdominal pain, and diarrhea in the past few weeks. Her father has a history of colorectal carcinoma, and her grandfather had a history of colonic polyps. On imaging, there are multiple nodular hypodensities in her colon. Endoscopy confirms the diagnosis, and resection of the colon is performed. The gross appearance of the resected colon appears in the photo shown in Fig. 28.18. What gene mutation is most likely responsible for this patient’s condition?   The APC gene, which leads to this patient’s condition of familial adenomatous polyposis (FAP) is most likely responsible for this patient’s condition. FAP is an autosomal dominant condition leading to the generation of hundreds to thousands of colonic polyps or adenomas. The risk of colorectal carcinoma is 100% in untreated patients and warrants resection of the colon. Common presenting symptoms are hematochezia, abdominal pain, and diarrhea. Symptoms appear early in life, usually in the teens, and colorectal carcinoma develops before the age of 40. Intestinal polyps can also be seen in Gardner syndrome, along with osteomas of the mandible or skull. Turcot syndrome is similar, presenting with intestinal polyps and central nervous system (CNS) tumors. 17. A 43-year-old female is evaluated for several weeks of fatigue, unintentional weight loss, and diarrhea. She is also concerned about an itchy rash on her elbows. On exam, there are grouped

650  Pathology

Figure 28.18.  Adenomatous polyposis coli. (From Skarin AT. Atlas of Diagnostic Oncology. 4th ed. St. Louis: Mosby; 2010.)

vesicles on the extensor surfaces of her arms. Laboratory tests indicate anemia, and serum is positive for anti-gliadin antibodies. The patient is sent for a duodenal biopsy, the results of which are shown below (Fig. 28.19). What is the most likely diagnosis in this patient?

Figure 28.19.  Gastrointestinal tract. (From Rosai J. Rosai & Ackerman’s Surgical Pathology. 10th ed. St. Louis: Mosby; 2011:585-816.)

  Celiac disease is the most likely diagnosis. The biopsy shows villous atrophy and intraepithelial lymphocytes, indicating a diagnosis of celiac disease. Gluten, a component of wheat, barley, and rye, is the inciting factor in celiac disease. Gluten is digested into gliadin and subsequently to component peptides that are recognized by CD4+ T cells, resulting in their activation in susceptible individuals. Gliadin can also cause T cell–mediated enterocyte damage and produce the characteristic flattened appearance of villi. The damaged villi are unable to properly absorb nutrients in the proximal small intestine,

Pathology  651

leading to weight loss, fatigue, and deficiencies in iron and fat-soluble vitamins. Laboratory findings will often show the presence of immunoglobulin A (IgA) antibodies against gliadin, tissue transglutaminase, or endomysium. IgA is also deposited at the tips of dermal papillae, causing a pruritic rash on extensor surfaces called dermatitis herpetiformis (see Fig. 27.26). 18. A 38-year-old male with a history of type 2 diabetes mellitus is evaluated for several weeks of fatigue, abdominal pain, and joint pain. Exam is significant for nontender hepatomegaly, testicular atrophy, and skin bronzing. On cardiac exam, the patient has a new, irregular heart rhythm. Labs show high iron, high ferritin, and low total iron-binding capacity. A liver biopsy is performed and stained with Prussian blue as shown in Fig. 28.20. What is the most likely diagnosis in this patient?

Figure 28.20.  Liver biopsy of patient in question 18. (From Kumar V, Abbas A. Robbins and Cotran Pathologic Basis of Disease. 7th ed. Philadelphia: Saunders; 2004, Figure 18-28.)

  Hemochromatosis is the most likely diagnosis. The patient presents with a triad of hepatomegaly, skin pigmentation, and diabetes mellitus, which are consistent with hemochromatosis, also known as bronze diabetes. Prussian blue stains iron, which accumulates in the liver as illustrated in the biopsy image. Hemochromatosis can be caused by repeated blood transfusions or a genetic defect, most commonly in the HFE gene. Because there is no physiologic mechanism for iron to leave the body, iron uptake must be well controlled. Hepcidin is the main regulator of iron homeostasis and works by causing ferroportin to be internalized and cleaved, preventing iron release into serum. In hereditary hemochromatosis, hepcidin is deficient and ferroportin is free to release iron into the serum. Iron accumulates in tissues and organs, leading to free radical damage, particularly in the liver. The heart is also susceptible to iron deposition, leading to arrhythmias, restrictive cardiomyopathy, and congestive heart failure. Patients are also at an increased risk of hepatocellular carcinoma. Hemochromatosis is diagnosed more frequently in men because menstruation is protective against iron overload in women. 19. An obese 35-year-old female with a history of diabetes is seen for a routine physical. She has no complaints. On exam, there is mild hepatomegaly, and lab workup reveals elevated serum bilirubin and alkaline phosphatase. A liver biopsy is taken and is shown in Fig. 28.21. What process is likely taking place in this patient?   Hepatic steatosis is likely occurring. The patient’s biopsy demonstrates microvesicular droplets and macrovesicular lipid globules within hepatocytes, indicating hepatic steatosis. The disease mechanism is incompletely understood, but it is believed that oxidative stress on the hepatocytes leads to lipid peroxidation and reactive oxygen species that over time can lead to cirrhosis. Hepatic steatosis can be seen in alcoholic steatohepatitis, nonalcoholic fatty liver disease (NAFLD), or nonalcoholic steatohepatitis (NASH). NAFLD is associated with obesity, insulin resistance, and diabetes, as seen in this case. However, patients are often asymptomatic. The histology in NASH is similar, but also includes lobular inflammation. 20. A 45-year-old male presents in the emergency department after a minor motor vehicle accident. The patient admits to drinking five alcoholic drinks that evening, and he has up to three cans of beer each night after work. When an intravenous (IV) line is started, the IV site bleeds profusely. Exam is significant for jaundice, scleral icterus, gynecomastia, and ascites. Lab workup reveals elevated alanine transaminase, aspartate transaminase, bilirubin, prothrombin time, and anemia. An ultrasound reveals nodular hepatic architecture. A liver biopsy is ordered, and the trichrome stain is presented in Fig. 28.22. What is the most likely diagnosis in this patient?   Cirrhosis is the most likely diagnosis. The patient’s history of alcohol use and presenting symptoms are consistent with hepatic cirrhosis. Alcohol metabolism creates excess NADH, the reduced form of nicotinamide adenine dinucleotide

652  Pathology

Figure 28.21.  Liver. (From Rosai J: Rosai & Ackerman’s Surgical Pathology. 10th ed. St. Louis: Mosby, 2011:857-980.)

Figure 28.22.  Alcoholic liver disease. (From Carey WD. Cleveland Clinic: Current Clinical Medicine. 2nd ed. Philadelphia: Saunders; 2011:508, Figure 3.)

(NAD), increasing the NADH/NAD+ ratio, which is a positive regulator for lipid synthesis. Additionally, creation and secretion of lipoproteins are impaired, trapping lipids inside hepatocytes (dashed arrow). Lipid peroxidation leads to production of reactive oxygen species, promoting hepatocyte damage and eventually fibrosis (solid arrow). Parenchymal hepatocytes entrapped in the fibrosis create regenerative nodules, which may be visible on imaging. On trichrome stain, biopsy demonstrates these nodules with surrounding blue collagen bands (circle), which is the hallmark of chronic liver insult. The patient’s symptoms can all be explained by hepatocyte damage. Loss of albumin and other proteins leads to loss of oncotic pressure, which is demonstrated by his ascites. Jaundice and scleral icterus are products of elevated serum bilirubin, which cannot be taken out of circulation by the liver. Gynecomastia is a result of increased estrogen, which would normally be metabolized by hepatocytes. Clotting factors are also in short supply, which accounts for the elevated prothrombin time (PT). 21. A 51-year-old male presents to his primary care physician complaining of a skin rash. The patient explains that he has had several blisters on the inside of his mouth and that recently his chest and head have developed scabbing and blisters as well. The physician notes vesicles distributed on the patient’s trunk, scalp, and oral mucosa. The vesicles rupture when gentle pressure is applied. A skin biopsy is taken and shown in Fig. 28.23. What laboratory test will confirm his suspected diagnosis?   Elevated anti-desmoglein levels would confirm that this patient likely has pemphigus vulgaris. Flaccid vesicles that rupture easily and are found on the oral mucosa indicate pemphigus vulgaris. Crusting erosions may also be observed on exam. In pemphigus vulgaris, IgG autoantibodies against desmogleins cause damage to desmosomes, forcing the epithelial cells to separate from the basal cell layer. This is called acantholysis. The row of basal cells left behind are described as having a “tombstone” appearance. In addition to the hematoxylin and eosin (H&E) stain, direct

Pathology  653

Figure 28.23.  The vesiculobullous reaction pattern. (From Patterson JW. Weedon’s Skin Pathology. 4th ed. Philadelphia: Elsevier; 2015.)

immunofluorescence will demonstrate a reticular, or fish-net pattern where the IgG has deposited on the surface of epithelial cells (see Fig. 27.7). 22. A 64-year-old female, accompanied by her son, is being seen by her primary care physician. The son explains that a year ago she began repeatedly forgetting where she placed her jewelry and keys. Beginning a few months ago she would get lost on her way home and is having difficulty paying for groceries and bills. Physical exam is unremarkable. On mental status exam, the patient is oriented to person but not place or time and cannot recall three unrelated words after a 5-minute interval. If a postmortem biopsy were taken, it would appear as shown in Fig. 28.24. What is the most likely diagnosis in this patient?

NP

NP

NP

Figure 28.24.  Biopsy sample of patient in question 22. (From Stern T, Fava M, Wilens T, Rosenbaum J. Massachusetts General Hospital Psychopharmacology and Neurotherapeutics. London: Elsevier; 2016, Figure 14-1.)

  Alzheimer’s disease (AD) is the most likely diagnosis. AD is the most common cause of dementia in elderly patients and is usually a diagnosis of exclusion. Although imaging can aid in determining the cause of dementia, a definitive diagnosis can only be made using a biopsy showing neurofibrillary tangles and neuritic plaques. Neurofibrillary tangles are formed when tau protein, normally involved in microtubule assembly, becomes abnormally hyperphosphorylated, resulting in an insoluble protein that is resistant to clearance in the extracellular space. The tangles display a characteristic flame shape around pyramidal neurons (see Fig. 28.24). Even after neuron death, the protein can leave a “ghost” image visible on microscopy. Neuritic plaques are derived from amyloid precursor protein (APP) that is broken down into various components, including Aβ amyloid. Aβ amyloid accumulates extracellularly and forms characteristic rounded structures [see neuritic plaques (NP) in Fig. 28.24]. 23. A 27-year-old female is evaluated for impaired vision in her left eye. Several months ago she experienced a sudden visual deficit in her right eye along with an episode of lower extremity

654  Pathology weakness, both of which resolved on their own. On exam, the patient has a left-sided visual field deficit, but vision in her right eye remains intact. A radiograph of the brain is taken and is shown in Fig. 28.25. What is the most likely diagnosis in this patient?

Figure 28.25.  Brain imaging of patient in question 23. (From Perkin G, Miller D, Lane R, Patel M, Hochberg F. Atlas of Clinical Neurology. 3rd ed. Philadelphia: Saunders; 2011:343-359.)

  Multiple sclerosis (MS) is the most likely diagnosis. MS usually presents in women between the ages of 20 and 40 years of age. Lesions are separated by time and space, meaning symptoms recur and resolve on their own, and the lesions are not found in a single location in the CNS. Common symptoms include visual disturbances caused by optic neuritis and motor or sensory deficits in the limbs. Symptoms occur as a result of T cells reacting to antigens found in myelin. Plaques can be viewed on imaging where white matter has been demyelinated. Histologically, these regions will display low oligodendrocyte numbers and reactive astrocytosis. The radiograph shows periventricular plaques, which is a classic location for demyelination in multiple sclerosis. Cerebrospinal fluid (CSF) analysis will typically reveal elevated IgG levels. 24. A 56-year-old woman is evaluated for several months of fatigue, weakness, unintentional weight gain, and constipation. Exam reveals bradycardia, dry skin, diminished (1+) patellar reflexes, and an enlarged thyroid. The patient is sent for a thyroid biopsy, the results of which are shown in Fig. 28.26. Laboratory workup is performed prior to biopsy. Thyroid testing is most likely to reveal what findings?   Testing would reveal a high thyroid-stimulating hormone (TSH), low T3, and low T4 secondary to hypothyroidism due to Hashimoto thyroiditis. Weight gain, bradycardia, cold intolerance, and hyporeflexia are indicative of hypothyroidism. In parts of the world where iodine levels are adequate, Hashimoto thyroiditis is the most common cause of hypothyroidism. Hashimoto thyroiditis is an autoimmune condition that destroys the thyroid gland and is associated with circulating antibodies against thyroid antigens, thyroglobulin, and thyroid peroxidase. Continuous injury to the cuboidal epithelium causes metaplasia, producing Hürthle cells, which are characterized by profuse eosinophilic cytoplasm (arrow). Biopsies will also demonstrate well-developed germinal centers (GC). 25. A 2-year-old boy accompanied by his father is evaluated for fever and dark, cloudy urine. The father notes that his son had a sore throat about 2 weeks ago, but otherwise his history is unremarkable. On exam the child has a temperature of 101°F and periorbital edema. His urinalysis shows proteinuria, hematuria, and red cell casts. A kidney biopsy is taken, and an electron microscopy image is presented in Fig. 28.27. What is the likely etiology of this patient’s condition?   Poststreptococcal glomerulonephritis (PSGN) is the likely etiology. The patient’s lab values and presentation 1 to 2 weeks after a sore throat are consistent with PSGN. During a streptococcal infection, antibodies form immune complexes with bacterial antigens that are deposited beneath the basement membrane—that is, subepithelially. The depositions appear as dense “humps” or “meatballs” on electron microscopy (arrows). On immunofluorescence, the

Pathology  655

GC

Figure 28.26.  Thyroid gland. (From Gray W, Kocjan G. Diagnostic Cytopathology. 3rd ed. Edinburgh: Churchill Livingstone; 2011, Figure 17-14.)

Figure 28.27.  Kidney biopsy sample showing an electron microscopy image of patient in question 25.

immune complex accumulations appear granular, and on H&E the glomerulus will appear larger and hypercellular because of leukocyte infiltration. Alterations in the filtering ability of the kidney cause protein to leak into the urine, accounting for the patient’s proteinuria and periorbital edema. Since this is a nephritic process, hematuria and red blood cell casts will also be seen. 26. A 61-year-old man is evaluated for a 1-month history of fatigue, fever, and unintentional weight loss and a new painful “rash” on his fingers and toes. The patient’s history includes rheumatic fever as a child. Three weeks ago, he underwent an uncomplicated dental procedure. On exam, the patient has red streaks on his nail bed (Fig. 28.28) and tender, raised, erythematous lesions on the pads of his fingers and toes (Fig. 28.29). The physician notes circular, hemorrhaged spots on the retina and a new cardiac murmur. Blood cultures demonstrate gram-positive cocci. If the patient’s heart were to be dissected on autopsy, it would appear as the photo provided in Fig. 28.30. What is the most likely diagnosis in this patient?   Bacterial endocarditis is the most likely diagnosis. Most cases of bacterial endocarditis present with fever, fatigue, weight loss, and a new murmur. Additional symptoms include red streaks on the nail bed called splinter hemorrhages (Fig. 28.28; arrow), tender raised red lesions on the finger and toe pads (Osler nodes), erythematous nontender lesions on the palms and soles (Janeway lesions; see Fig. 28.29), and white spots on the retina surrounded by erythema (Roth spots), all of which are the result of bacterial emboli. The patient’s dental procedure caused Streptococcus viridans to enter the blood and infect a cardiac valve, leading to deposition of fibrin and additional bacteria. The depositions are evident as the friable vegetations

656  Pathology

Figure 28.28.  Red streaks on nail bed of patient in question 26. (From Swartz M. Textbook of Physical Diagnosis. 7th ed. Philadelphia: Saunders; 2014, Figure 5-10.)

Figure 28.29.  Janeway lesions of patient in question 26. (From James W, Berger T, Elston D. Andrews’ Diseases of the Skin. 12th ed. London: Elsevier; 2015.)

Figure 28.30.  Patient’s dissected heart on autopsy showing friable vegetations of patient in question 26.

Pathology  657

seen in Fig. 28.30 (arrows) and are the reason for the murmur. The insidious development of symptoms indicates that the infection is subacute. Subacute bacterial endocarditis causes infections on previously damaged or abnormal valves, such as in rheumatic fever, by low-virulence organisms. Acute bacterial endocarditis occurs more rapidly, damages previously healthy cardiac valves, and is caused by high-virulence organisms like Staphylococcus aureus. Given its normal placement on skin, patients who abuse IV drugs are more susceptible to acute bacterial endocarditis. 27. A 23-year-old woman with a body mass index of 19 is evaluated for fatigue, periodic fever, abdominal pain, and bloody diarrhea, which she has had for nearly 1 year. She has also recently developed tingling in her feet. Exam is positive for right lower quadrant tenderness, conjunctival pallor, and 1+ ankle reflexes. Barium swallow (Fig. 28.31) and small bowel biopsy are performed; results are shown in Fig. 28.32. What is the most likely diagnosis in this patient?   Crohn’s disease is the likely diagnosis. Crohn’s disease can occur anywhere in the gastrointestinal (GI) tract but is commonly found in the terminal ileum, ileocecal valve, and cecum. Transmural inflammation is a distinguishing feature that helps dif-

Figure 28.31.  Barium swallow procedure for patient in question 27. (From Slovis TL, Caffey J. Caffey’s Pediatric Diagnosic Imagine, 11th ed. Philadelphia: Elsevier; 2008.)

ferentiate this disease from ulcerative colitis (UC), which involves only the mucosa and submucosa. Fistulas and strictures are common complications that occur in patients with Crohn’s disease as a result of full-thickness wall involvement. In addition to leukocyte infiltration, another histologic feature of Crohn’s disease is the presence of noncaseating granulomas (arrow in Fig. 28.32) in the involved areas of the intestine. The inflammation in the intestinal wall damages villi and leads to symptoms of malabsorption, weight loss, and vitamin B12 deficiency in cases that involve the terminal ileum (as suggested by this patient’s neurologic symptoms). Anemia is also a concern in patients with Crohn’s disease as a result of blood loss in stool and/or chronic inflammation (as suggested by this patient’s fatigue and conjunctival pallor). On gross examination of the GI tract, skip lesions may be present, which are characterized by multiple sharply demarcated lesions with normal areas in between, leading to a “cobblestoned” appearance that classically spares the rectum. This can be contrasted with UC, in which lesions always involve the colon/rectum and are continuous in the caudal to cranial direction. Note that the bowel wall may be thickened in Crohn’s disease, which narrows the intestinal lumen, leading to a string sign on barium swallow (arrows in Fig. 28.31). 28. A 46-year-old male with a history of hypertension is evaluated for a painful, inflamed toe. The patient’s symptoms began the previous night after he consumed a large meal and several alcoholic drinks at a dinner celebration with his family. On exam, the patient’s right great toe is erythematous, swollen, and tender around the first metatarsophalangeal joint. Blood is drawn, and laboratory analysis shows hyperuricemia. Joint aspiration is performed at the affected joint, and a sample of the synovial fluid is shown in Fig. 28.33 under polarized light. How would you explain these findings?   This patient has gout, as is demonstrated by needle-shaped monosodium urate (MSU) crystals, which are negatively birefringent under polarized light (thus causing the crystals to appear yellow). The crystals are formed under conditions

658  Pathology

Figure 28.32.  Small bowel biopsy sample of patient in question 27.

Figure 28.33.  Synovial fluid under polarized light in patient in question 28. (From Young B, William S, O’Dowd G. Wheater’s Basic Pathology: A Text, Atlas and Review of Histopathology. 5th ed. Philadelphia: Churchill Livingstone; 2011.)

that lead to hyperuricemia, such as overproduction of uric acid (e.g., cancer or tumor lysis syndrome), or when excretion is reduced. MSU has low solubility at low temperatures and in synovial fluid, which allows supersaturation to occur more readily in joints that are accustomed to colder temperatures, including toes, ankles, knees, and wrists. The crystals develop in the synovium and cartilage around the joint, and a precipitating event releases them into the synovial fluid. The MSU crystals initiate an inflammatory response via activation of the NLRP3 inflammasome, causing pain and swelling at the affected joint. Acute attacks are often precipitated by large meals, purine-rich foods (e.g., shellfish), and/or alcohol consumption. 29. A 37-year-old male is evaluated for several months of weakness, unintentional weight loss, and tongue swelling. On exam, there is hepatomegaly, several bruises on the arms and legs, and the sides of the tongue have indentations. Lab findings include proteinuria and elevated serum immunoglobulin light chains. A biopsy sample is stained with Congo red and examined under polarized light (Fig. 28.34). What is the most likely diagnosis in this patient?   Amyloidosis (AL) is the most likely diagnosis. Congo red staining and the apple green birefringence of extracellular hyaline material indicates amyloidosis. Amyloidosis occurs when there are excessive, abnormally folded proteins that form beta-pleated sheets that deposit extracellularly. Accumulation of the abnormal proteins causes crowding and pressure atrophy of nearby cells, leading to damage in a variety of organs including the kidneys, GI tract, and heart, which accounts for the multisystem symptoms seen in this disease. The most common form of AL is due to the depositions of free immunoglobulin light chains from monoclonal plasma cells; however, less common forms of the disease can also be seen that involve other proteins such as serum amyloid A and transthyretin. 30. A 15-year-old male is evaluated for dyspnea, hemoptysis, and dark urine. On chest imaging, the patient has several focal consolidations. Urinalysis reveals hematuria, and a kidney biopsy

Pathology  659

U

GC

V

V Figure 28.34.  Biopsy sample of patient in question 29. (From Neville B, Damm DD, Allen C, Chi A. Oral and Maxillofacial Pathology. 4th ed. St. Louis: Saunders; 2015, Figure 15-95.)

stained with hematoxylin and eosin shows crescentic lesions (Fig. 28.35). Immunofluorescence of the biopsy is also performed and seen in Fig. 28.36. What is the most likely diagnosis in this patient?

Figure 28.35.  Kidney biopsy sample showing crescentic lesions of patient in question 30.

  Goodpasture syndrome is the most likely diagnosis. Goodpasture syndrome is caused by accumulations of antibodies against type IV collagen in alveolar and glomerular basement membranes (GBMs), leading to hematuria, edema, hemoptysis, dyspnea, and/or dry cough. Recall that deposition of antibodies along the GBM leads to the linear pattern seen on immunofluorescence. Goodpasture syndrome is a rapidly progressive glomerulonephritis that creates crescent-shaped lesions around the glomeruli on H&E staining. 31. A 59-year-old female with a history of smoking is evaluated for several weeks of worsening dyspnea. On a previous visit, her body mass index was 24; however, today her body mass index is 20. She is barrel-chested and she is breathing through nearly closed lips. Total lung capacity and reserve volume are both increased. Forced expiratory volume/forced vital capacity ratio and diffusing capacity of the lung for carbon monoxide (DLCO) are markedly decreased. A chest radiograph (Fig. 28.37) and lung biopsy (Fig. 28.38) are shown. What is the most likely diagnosis?   Emphysema is the most likely diagnosis. Smoking increases the number of neutrophils in the alveolar spaces and stimulates release of protease-containing granules. α1-Antitrypsin is the protein responsible for inhibiting proteases, and when the ratio of α1-antitrypsin to protease is low, elastin begins to degrade and alveolar spaces become enlarged,

660  Pathology

Figure 28.36.  Immunofluorescence of the biopsy sample of patient in question 30.

Figure 28.37.  Chest radiograph of patient in question 31. (From Sebire NJ, et al. Diagnostic Pediatric Surgical Pathology. Edinburgh: Churchill Livingstone; 2009.)

leading to emphysema. Smokers typically demonstrate a centrilobular pattern of destruction, as alveoli in these areas are most frequently contacted by particles from cigarette smoke (note the enlargement of alveoli secondary to destruction of alveolar walls in Fig. 28.38). In cases of α1-antitrypsin deficiency, destruction is more evenly distributed in a panacinar pattern. However these cases present at a younger age than the patient in this vignette. As damage to elastin accumulates and elastic recoil diminishes, the patient loses the ability to fully exhale, leading to air trapping. This

Pathology  661

Figure 28.38.  Lung biopsy sample of patient in question 31. (From Klatt E. Robbins and Cotran Atlas of Pathology. 3rd ed. Philadelphia: Saunders; 2015, Figure 5-23.)

increases the patient’s reserve volume and total lung capacity. The patient’s vital capacity has also decreased, causing her to take quick, short breaths that increase the work of breathing and leads to weight loss. Air trapping can be visualized as an expanded barrel chest (see Fig. 28.37) and diaphragm flattening, as seen in the radiograph. 32. A 54-year-old man with a history of diabetes is evaluated for leg pain. The patient explains that he takes his dog for a walk every morning, but the past few weeks he has experienced leg pain while walking that is relieved with rest. His father had two myocardial infarctions before the age of 55. On exam, the patient has a body mass index of 27, and his blood pressure is 135/70 mm Hg. The sample lipid profile reveals elevated low-density lipoprotein and low high-density lipoprotein. If an artery biopsy were to be taken, it may appear as the slide seen in Fig. 28.39. What process is occurring in this patient?

E

H

P Figure 28.39.  Artery biopsy sample of patient in question 32. (From Young B, William S, O’Dowd G. Wheater’s Basic Pathology: A Text, Atlas and Review of Histopathology. 5th ed. Philadelphia: Churchill Livingstone; 2011.)

  Atherosclerosis is likely occurring in this patient. This patient’s history of diabetes, hypertension, and dyslipidemia, along with his first-degree family history of premature heart disease, points to the development of atherosclerosis. The inciting event in atherosclerosis is endothelial injury (E), which allows inflammatory cells into the vessel lumen and lipoprotein

662  Pathology accumulations on the wall. Macrophages accumulate with these lipoproteins and become foam cells (P). Platelet adhesion occurs on the lesion, and along with the foam cells, release factors that allow for smooth muscle proliferation and extracellular matrix production in the lumen, creating a cap (H) over the lipid accumulations. This structure is called a stable plaque. When atherosclerotic plaques occur in the popliteal artery, they can cause leg pain on exertion, also known as claudication. Note that myocardial perfusion abnormalities due to stable plaques can cause stable angina, whereas perfusion abnormalities associated with unstable plaques can cause unstable angina. 33. A 58-year-old man is evaluated in the emergency department for a 2-hour history of dyspnea associated with severe left-sided chest pain radiating to the left arm. He has a history of hypertension and diabetes. On exam, the patient is obese, diaphoretic, and has a pulse of 120. Electrocardiogram shows ST-segment elevation in the anterior leads. Blood work reveals elevated CK-MB and troponin I. An image from cardiac catheterization is shown in Fig. 28.40 along with an image of how the patient’s heart would appear on autopsy (Fig. 28.41). What is the most likely diagnosis in this patient?   Acute ST-elevation myocardial infarction (STEMI) is the most likely diagnosis. Hypertension, diabetes, and obesity are associated with coronary artery disease, which can lead to a myocardial infarction (MI). The patient’s current

Figure 28.40.  Cardiac catheterization image of patient in question 33. (From Zheng Y, Mao JY. Typical coronary artery aneurysm exactly within drug-eluting stent implantation region in a patient with rheumatoid arthritis. J Cardiovasc Dis Res. 2012;3:329-331.)

Figure 28.41.  Image of patient’s heart during autopsy discussed in question 33. (From Burke AP: Pathology of Acute Myocardial Infarction, http://emedicine.medscape.com/article/1960472-overview.)

Pathology  663

presentation (radiating chest pain, diaphoresis, and rapid heart rate) is consistent with an MI. MIs occur when a coronary artery becomes 100% occluded by an embolus or thrombus. Unstable atherosclerotic plaques can rupture, causing platelet adhesion and degranulation. Production of microthrombi allows the plaque to completely block the flow of blood, causing the underlying myocytes to become ischemic. An area of ischemia appears in the gross image as a region of dark and damaged tissue (see Fig. 28.41, arrows), and the catheterization image depicts an occluded coronary artery. In this case, ST elevations in the anterior leads, plus the location of the ischemic area and the blockage of flow in the catheterization, point to total occlusion of the left anterior descending (LAD) artery. Damage to the myocytes allows for release of cytoplasmic proteins, allowing proteins such as CK-MB and troponins to be released into the blood. Troponin I is the most sensitive and specific biomarker for MI because it is specifically found in cardiac cells. When cardiac myocyte damage extends through the entire thickness of the heart wall, ST elevations appear on ECG, causing STEMI. If damage occurs only to the subendocardium, the ECG will show ST depressions instead and is known as a non-STelevation myocardial infarction (NSTEMI). 34. A 48-year-old woman is evaluated for 2 to 3 months of fatigue and joint pain. She has joint stiffness in the morning, which improves as the day progresses. On exam, the joints in her hands, feet, and wrists are swollen, warm, and tender, with limited range of motion. There is radial deviation of the wrists and ulnar deviation of the fingers. On plain films, there is narrowing of the joint space. Labs reveal elevated anti-cyclic citrullinated protein (CCP) antibodies. A photo of the affected finger joints is shown in Fig. 28.42. What do these findings indicate?

Figure 28.42.  Patient’s affected finger joint discussed in question 34. (From Cifu D. Braddom’s Physical Medicine and Rehabilitation. 5th ed. Philadelphia: Elsevier; 2015, Figure 31-1B.)

  Rheumatoid arthritis (RA) is indicated. RA is an autoimmune disorder in which T lymphocytes react against an unknown antigen in the joint synovium. The T cells release cytokines that cause inflammation and macrophage activation, leading to destruction of collagen and bone in the joint space. Additionally, certain subsets of T lymphocytes can be directly cytotoxic to components of the joints. The diagnosis of rheumatoid arthritis can be supported by laboratory tests through the presence of anti-CCP and rheumatoid factor (IgM to the Fc portion of IgG). Although the sensitivity of these two tests are nearly identical, the specificity of anti-CCP antibodies exceeds that of rheumatoid factor, making it a better diagnostic test for RA. Radiographic tests may show joint effusions, erosions, and narrow joint spaces in RA patients. Unlike osteoarthritis, in which symptoms improve with rest, RA presents with joint stiffness after rest that improves with use as the inflammatory infiltrate in the joints is mechanically cleared. Other symptoms of RA include ulnar deviation of fingers and symptoms of synovitis (joint tenderness, swelling, and hard lumps). Note the finger adduction and “swan-neck” deformation (in which the proximal finger joints are extended and the distal finger joints are flexed) in Fig. 28.42. 35. A 56-year-old male with a history of hypertension presents to the emergency department with intense chest pain. The pain occurred without warning and radiates to his back, between the scapulae. On exam, he is noticeably tall and slender, with very long extremities and digits. His femoral pulses are diminished bilaterally. The electrocardiogram is unremarkable. Chest x-ray reveals mediastinal widening. A histologic image of the aortic wall in a patient with this condition is as shown in Fig. 28.43. What is the most likely diagnosis?   Aortic dissection is the most likely diagnosis. Aortic dissections in older patients with high blood pressure occur when there is hypertrophy of the vasa vasorum and loss of blood flow to the myocytes in the tunica media, leading to atrophy. The atrophic smooth muscle layer is prone to tearing and can progress distally or proximally toward the heart, leading to cardiac tamponade. Often the damage extends through the adventitia, causing massive hemorrhage. In young patients, aortic dissections can often be attributed to diseases that affect normal collagen homeostasis and produce abnormal vasculature, such as Marfan syndrome or Ehlers-Danlos syndrome.

664  Pathology

Figure 28.43.  Photomicrograph of a portion of the wall of the aorta, demonstrating pools of mucoid material. (H&E, × 4 obj.). (From Bullough PG. Orthopaedic Pathology. 5th ed. Philadelphia: Mosby; 2010, Figure 6-27.)

36. A 28-year-old female ballet dancer accidentally falls and hits her head on the stage floor when her dance partner fails to catch her. Outside of a headache, she states that she feels fine but decides to leave early for the day. The following afternoon, her husband returns home from work to find her sleeping on the couch and very difficult to rouse. He quickly calls emergency response, and she is rushed to the hospital where she is put on mechanical ventilation and other life-supportive measures. The patient expires several hours later. If a computed tomography scan had been performed near the time of her injury, the image shown in Fig. 28.44 would have

CRANEO SIMPLE

Figure 28.44.  Computed Tomography (CT) scan image of patient in question 36. (From Ferri F. Ferri’s Clinical Advisor 2016. Philadelphia: Elsevier; 2017.)

Pathology  665

been obtained), led to the diagnosis, and could have saved her life. What is the most likely cause of this patient’s death?   An epidural hematoma caused this patient’s death. This unfortunate patient has the classic presentation of an epidural hematoma. This includes a lucid interval where the patient experiences little to no symptoms for several hours after the initial injury. If the pathology goes unrecognized, it can quickly lead to altered mental status, coma, and death secondary to herniation of the brainstem. Epidural hematomas are a result of injury to meningeal arteries, which will bleed into the space between the dura mater and the skull (the epidural space), causing the dura mater to separate off of the calvarium and impinge upon brain parenchyma (see arrows). The middle meningeal artery is the most likely to be injured because the overlying pterion region is relatively weak. Note that while epidural hematomas appear as bright, almond-shaped regions on computed tomography (CT), subdural hematomas, which are typically associated with “the worst headache of one’s life,” form crescent-shaped lesions. 37. A 66-year-old male is brought to the emergency department after a fall at home. His wife reports that he slipped on the kitchen floor and fell and hit the side of his head. She reports he experienced a brief loss of consciousness, and upon waking he complained of a headache but was following commands. In the emergency department, he has somewhat altered mental status and right-sided upper extremity weakness. His wife reports no other medical problems except for atrial fibrillation for which he receives warfarin and a daily “baby aspirin” for heart health. She also states that his dose of warfarin was recently increased because his last international normalized ratio value was subtherapeutic. His computed tomography scan is shown in Fig. 28.45. What is the most likely diagnosis?

Figure 28.45.  Computed tomography (CT) scan image of patient in question 37. (From Dharsono F, Constantine CP. Arterial origin subdural hematoma and associated pial pseudoaneurysm following minor head trauma. Clin Imag 2013;37:750-752.)

  Subdural hematoma is the most likely diagnosis. The patient’s presentation of loss of consciousness, headache, altered mental status, and focal neurologic symptoms in the context of anticoagulation is strongly suggestive of an intracranial hemorrhage. The CT scan shows a left-sided, crescent-shaped hemorrhage that crosses suture lines (arrow), which classifies this as a subdural hematoma. These lesions are a result of shear force on bridging veins after head trauma or rapid accelerating/decelerating force. Because the bleed is venous rather than arterial, the signs and symptoms tend to come on more slowly than an epidural hematoma. Subdural hemorrhages do not typically have an associated lucid interval. 38. A 71-year-old male with a past history significant for atrial fibrillation, hypertension, and hyperlipidemia is rushed to the emergency department after his wife noticed a facial droop. Once there, the patient is awake, alert, and oriented to person, place, and time; however, he has a noticeable left-sided facial droop with forehead sparing, loss of sharp and dull sensation

666  Pathology in both his upper and lower extremities on the left, as well as substantial loss of motor strength (2/5). An emergent computed tomography scan is negative for an acute bleed but shows a loss of gray-white differentiation on the right cortex, and because his last seen normal time was determined to be less than 3 hours ago, he receives tissue plasminogen activator. A magnetic resonance image taken the following day demonstrates the lesion shown in Fig. 28.46. What is the most likely diagnosis?

A

B

Figure 28.46.  Dual source energy CT of a patient after intra-arterial thrombolysis for a left middle cerebral artery (MCA) stroke: on the left one can see some blood (A), but on the right there is more contrast extravasation visible (B), thus allowing differentiation between acute blood and contrast on the same examination. (From Lövblad KO, et al. Imaging of acute stroke: CT and or MRI. J Neuroradiol. 2015;42:55-64.)

  Middle cerebral artery (MCA) ischemic stroke is the likely diagnosis. This patient is exhibiting focal neurologic signs consistent with a stroke of middle cerebral artery distribution of the right cortical hemisphere. The most common imaging modalities used to define an ischemic stroke’s distribution are T2-FLAIR (which looks like the T2 image in Fig. 28-46B except CSF appears black, which enhances intraparenchymal edema from an injury) and DWI (diffusion-weighted imaging). However, in the emergent setting, a CT is always ordered to rule out the possibility of intracranial hemorrhage and to look for early signs of ischemic stroke. Ischemic strokes are much more common than hemorrhagic strokes and can be either thrombotic or embolic. Although this patient’s stroke is ischemic in nature (negative for acute bleed on CT), the precise etiology is unclear because he has risk factors for both subtypes: hypertension and dyslipidemia (thrombotic secondary to atherosclerotic disease) and atrial fibrillation (embolic). Thrombolysis with tissue plasminogen activator (tPA) is the appropriate treatment, but it has a time-sensitive window of use. 39. A 48-year-old male, former marine, is seen by his primary care physician for joint pain. He states that the pain is located in both knees without radiation, started several years ago, and has progressively worsened with the most excruciating pain occurring in the evenings. He now finds himself using 8 to 10 200-mg tablets of ibuprofen per day, which helps with the pain. He denies any fever, chills, recent infection, or other symptoms. He has an unremarkable past medical history. On exam, his knees are slightly swollen but otherwise appear grossly normal. An x-ray of the patient’s knee is shown in Fig. 28.47. What is the most likely diagnosis?   Osteoarthritis (OA) is the most likely diagnosis. The patient’s plain film x-ray shows classic hallmarks of osteoarthritis—namely, marked joint space narrowing due to loss of fibrocartilage. Bony outgrowths called osteophytes may also be present. OA is a very common condition that results from chronic wear and tear on a joint leading to bone-on-bone contact, inflammation, and associated pain. Pain is worsened by exertion/use and is often most pronounced later in the day. Patients can usually be managed conservatively with antiinflammatory agents; however, sometimes intra-articular injections of corticosteroids or hyaluronic acid are needed to achieve pain relief. In advanced cases, total joint replacement may be required. 40. A 67-year-old male with a past medical history significant for congestive heart failure is seen in the emergency department for a 1-day history of increasing dyspnea on exertion, pleuritic chest pain, leg swelling, orthopnea, and paroxysmal nocturnal dyspnea. On exam, he has dullness to percussion and decreased breath sounds over the lung bases bilaterally. A chest x-ray (Fig. 28.48A) and chest computed tomography scan (Fig. 28.48B) are performed to confirm the

Pathology  667

Figure 28.47.  X-ray of patient’s knee discussed in question 39. (From Ralston SH, McInnes IB. Davidson’s Principles and Practice of Medicine. 22nd ed. Philadelphia: Elsevier; 2015:1057-1135.)

diagnosis. The patients’ at-home dose of furosemide is increased, and he is admitted to the hospital to monitor his respiratory status. What is the most likely diagnosis?   Congestion heart failure causing a transudative plural effusion. The chest x-ray (CXR) in Fig. 28.48A shows blunting of the left costophrenic angle (arrow) with bilateral opacities in both lung fields. The normal cardiac outline is obscured (silhouette sign), and the presence of fluid within the lung fissures is confirmed on the CT scan in Fig. 28.48B (black arrow). Effusions can be either transudative of exudative depending on the etiology. Situations that increase intracapillary pressure or decrease plasma oncotic pressure will lead to a transudative process (e.g., congestive heart failure [CHF], cirrhosis). Exudative effusions will be seen in states of increased vascular permeability (e.g., infection, malignancy, collagen vascular diseases). Evaluation of the pleural fluid by thoracentesis can determine if the process is transudative or exudative using Light’s criteria (pleural fluid-serum protein ratio > 0.5, lactate dehydrogenase [LDH] ratio > 0.6, or LDH > two-thirds the upper limit of normal favoring the latter diagnosis), although this concept is most likely beyond the scope of the USMLE Step 1. 41. A 70-year-old male is being seen for an annual Medicare wellness visit. He has a known history of hypertension and hyperlipidemia and is a past smoker (45 pack-years). He also has a notable family history, with multiple family members having myocardial infarction and stroke at relatively young ages. Examination is significant for a small pulsatile nontender mass near the umbilicus. His exam is otherwise normal, and he has no symptoms to report. As part of a preoperative

A

B

Figure 28.48.  A. Chest X-ray, B. Computed Tomography (CT) scan of patient in question 40. (From Muller N. Imaging of the Chest. Philadelphia: Saunders; 2008:1336-1371.)

668  Pathology workup, the patient is sent for an abdominal computed tomography scan with angiography, which is shown in Fig. 28.49. What is the most likely diagnosis?

Figure 28.49.  Abdominal Computed Tomography (CT) scan with angiography image of patient in question 41. (From Skow G. Abdominal aortic aneurysm. J Mens Health. 2011;8:306-312.)

  Abdominal aortic aneurysm (AAA) is the most likely diagnosis. AAAs are usually asymptomatic unless ruptured and often present as a pulsatile nontender abdominal mass or abdominal “fullness” on routine examination. Risk factors for AAA development are similar for that of atherosclerosis with the addition of hereditary conditions that affect the integrity of vessel walls (e.g., vasculitis, diseases of collagen, etc.). If rupture occurs, the patient can destabilize quickly secondary to massive hemorrhage and can present with severe low back pain, hypotension, syncope, or cardiovascular collapse. Important diagnostic signs that may indicate aortic rupture include Grey Turner sign (ecchymosis on the flank) or Cullen sign (ecchymosis near the umbilicus). Diagnosis in the acute setting is made with ultrasound, which is quick and has a near 100% sensitivity for identifying AAAs. Otherwise, the preoperative planning imaging of choice involves abdominal CT with or without angiography. AAAs over 5 cm in diameter are repaired either openly or endovascularly with graft placement. 42. A 44-year-old male is brought to the emergency department by his wife for an acute onset of severe lower back pain, nausea, and vomiting. He reports that the pain is located on his left side and has been intermittent for the past several days. He states that when it comes on “I can’t get comfortable.” Today, the pain became sharp, constant, and severe and is radiating into his groin. His exam is benign, except for some tenderness at the left costovertebral angle. He is started on IV fluids and analgesia for his pain. A computed tomography scan is ordered and is shown in Fig. 28.50. What process is most likely to have occurred in this patient?   Nephrolithiasis is likely. This patient presents with classic renal colic secondary to a kidney stone (sudden, severe, writhing pain) that progressed from his flank to his groin. Patients may also experience nausea and vomiting, urinary tract infection (UTI), or frank hematuria. Kidney stones occur most commonly at the ureterovesicular junction, renal calyx, and ureteropelvic junction. Calcium (oxalate/phosphate) stones make up about 80% to 85% of cases and are radiopaque (obstruct the passage of radiation, thereby appearing white/opaque on plain film). Other less common stones include uric acid stones, struvite stones, and cystine stones. Risk factors for stone development include dehydration, family history, medications, male gender, UTI, and a low-calcium–high-oxalate diet. If sufficiently large, the stone can cause urinary obstruction leading to hydronephrosis. 43. A 50-year-old female is evaluated for several weeks of headache. Review of systems is positive for slight bilateral nipple discharge and several months of irregular menses. She also notes being in three car accidents in recent months but denies head trauma. Past medical history is

Pathology  669

Figure 28.50.  Computed Tomography (CT) scan of abdominal cavity for patient in question 42. (Courtesy of Marc Brown, MD, and Lawrence H. Schwartz, MD, Department of Radiology, Columbia University Medical Center. From Goldman L, Schafer A. Goldman-Cecil Medicine. 25th ed. Philadelphia: Elsevier; 2016, Figure 126-2.)

unremarkable. Exam is significant for bilateral outer visual field deficits and the ability to express a white discharge from her nipples. A magnetic resonance image of the brain is ordered and is shown in Fig. 28.51. What is the most likely diagnosis?   Pituitary adenoma (prolactinoma) is the likely diagnosis. The magnetic resonance image (MRI) demonstrates a mass expanding from within the sella turcica and compression of the underlying optic chiasm, most consistent with a pituitary tumor. Pituitary adenomas account for 10% of intracranial neoplasms, most of which are benign. Symptomatology is driven by excessive hormone secretion (which varies depending on the cell type of origin), hypopituitarism (if the infundibular stalk is compressed), and bitemporal hemianopsia, due to compression of the optic chiasm that lies just above the pituitary fossa. In this vignette, the patient presents with signs and symptoms of a prolactinoma (galactorrhea, amenorrhea), the most common type of hypersecreting pituitary adenoma. Other manifestations of hypersecreting pituitary adenomas include acromegaly (excess growth hormone [GH]), Cushing disease (excess ACTH), and hyperthyroidism (excess TSH). MRI and hormone levels are included in the standard diagnostic workup of pituitary tumors. Transsphenoidal excision is the surgical option for pituitary adenomas, although prolactinomas can often be treated medically with dopamine agonists (e.g., bromocriptine). 44. A 20-year-old female from Bosnia presents to the emergency department following a motor vehicle accident. The patient undergoes an urgent chest x-ray and computed tomography scan of the head and neck. The incidental findings, noted in the apices of the lung, are shown in Fig. 28.52A and B. The trauma physician immediately puts the patient on airborne precautions and orders a sputum sample as seen in Fig. 28.53. Why was this done?   The patient’s immigrant status and incidental radiographic findings are suspicious for active Mycobacterium tuberculosis (TB) infection. This diagnosis is confirmed by the presence of bacteria on the acid-fast stain, which resist decolorization by acid due to the high mycolic acid content of their cell walls. Primary TB infection is acquired through inhalation of droplets containing the active bacilli, which then deposit in the lungs and are ingested by alveolar macrophages. The surviving bacteria, which as obligate aerobes prefer the oxygen-rich areas of the lung, are sequestered and kept dormant by the formation of granulomas within the lung. However, the bacteria can be reactivated by any circumstance that weakens the immune system, leading to the clinical manifestations of TB (e.g., fever, night sweats, weight loss, hemoptysis) and potential for hematogenous or lymphatic dissemination. Risk factors for TB include an immunocompromised state, recent immigrant status, and employment within the health care system. Note that while the tuberculin skin test (PPD) is used to screen for TB exposure, CXR is required to distinguish active TB from latent infection and will classically demonstrate upper lobe infiltrates with cavitations if the former is present (Fig. 28.53). It is important to realize that cavitary pulmonary lesions do not confirm the diagnosis of pulmonary TB as these lesions can be seen in association with other species of Mycobacterium; however, the presence of cavitary lesions in a patient with likely exposure should elicit isolation precautions until the patient has undergone appropriate screening for Mycobacterium tuberculosis and the results are sufficiently negative.

Figure 28.51.  Magnetic Resonance Image (MRI) of the brain of patient in question 43. (From Oh MC, Kunwar S, Blevins L, et al. Medical versus surgical management of prolactinomas. Neurosurg Clin N Am. 2012;23:669-678.)

A

B

Figure 28.52.  CT slice with cavitary lesions in lung apex. (From Gadkowski LB, Stout JE. Cavitary pulmonary disease. Clin Microbiol Rev. 2008;21:305-333.)

Pathology  671

Figure 28.53.  Acid-fast stain of sputum for patient in question 44. (From Walter N, Daley CL. Clinical Respiratory Medicine. 4th ed. Philadelphia: Elsevier; 2012, Figure 31-9.)

45. An 8-year-old male whose status is 4 months post bone marrow transplant for acute myeloid leukemia presents to the emergency department with 3 hours of altered mental status and a fever of 103.4°F. A workup for a fever of unknown origin is initiated, which includes a spinal tap. The cerebrospinal fluid is stained with India ink, and the histologic findings are shown in Fig. 28.54. What is the most likely diagnosis?

Figure 28.54.  Cerebrospinal fluid of patient in question 45. (From Haley L, CDC. Centers for Disease Control and Prevention’s Public Health Image Library (PHIL), with identification number #3771: Nov 24, 2016. Available at: http://phil.cdc.gov/phil/home.asp. Accessed Nov 9, 2016.)

  Cryptococcus neoformans is the likely diagnosis. This dimorphic fungus can cause an asymptomatic pulmonary infection, which is later followed by the development of meningitis, as is seen in this case. Cryptococcus is an opportunistic infection that preferentially infects individuals who are immunocompromised, such as those with AIDS, leukemia, or lymphoma, and in transplant recipients treated with immunosuppressive agents to prevent tissue rejection. Cryptococcus is best visualized by India ink stain, which outlines the cell wall of the yeast and allows for visualization of its polysaccharide capsular halo as shown in Fig. 28.54. 46. A 22-year-old female presents to the OB/GYN with symptoms of increased vaginal discharge and a vaginal odor that is predominant after intercourse. The physician performs a vaginal swab and prepares a wet mount to perform a “bedside” examination (Fig. 28.55). What is the most likely diagnosis?   Bacterial vaginosis is the most likely diagnosis. This infection is most commonly associated with Gardnerella vaginalis but represents multimicrobial overgrowth in the vaginal environment. The diagnosis is made by the presence of “clue cells” on the vaginal swab, which represent vaginal squamous epithelial cells that are covered by coccobacilli. These are a pathognomonic feature of bacterial vaginosis and warrant treatment with metronidazole in symptomatic women and their sexual partners. If this is detected incidentally on a Pap smear, and the woman is asymptomatic, decision to treat is not as clear and should not simply be reflexive. 47. A 66-year-old patient presents to the emergency department with a “cut” on his penis (Fig. 28.56). He asks whether this may have resulted from the fact that he sometimes catches his zipper on his foreskin without noticing because his sensation has been blunted by his long-standing history of poorly controlled diabetes. What is the appropriate next step in diagnosis?   Laboratory testing for syphilis is the appropriate next step because this lesion is not likely to have resulted from a zipper laceration. The penile chancre demonstrated in Fig. 28.56 is associated with primary syphilis and is a painless, hard,

672  Pathology

Figure 28.55.  Vaginal swab sample of patient in question 46. (From Rein M, CDC. Centers for Disease Control and Prevention’s Public Health Image Library (PHIL), with identification number #3720: Nov 24, 2016. Available at: http://phil.cdc.gov/phil/home.asp. Accessed Nov 9, 2016.)

Figure 28.56.  Patient’s penis showing a cut as discussed in question 47. (From Wikipedia contributors. Chancre. Wikipedia, The Free Encyclopedia. October 26, 2016, 16:23 UTC. Available at: https://en.wikipedia.org/w/index.php?title=Chancre&oldid=746309541. Accessed October 26, 2016.)

and indurated (punched-out base) lesion that results from infection with the Treponema pallidum spirochete. Symptoms in the secondary stage include a maculopapular rash on the palms and soles and condyloma lata (painless, wart-like lesions) on the genitals. The tertiary stage is defined by the formation of soft growths with necrotic centers on other areas of the body, aortic aneurysms, and Argyll-Robertson pupils, which do not react to light. Traditional algorithms suggest two-step testing for definitive diagnosis of syphilis and include rapid plasma reagin (RPR) followed by fluorescent treponemal antibody absorption (FTA-Abs) for confirmation of results. However, you should note that syphilis polymerase chain reaction (PCR) is the preferred testing method in most clinical practices. Penicillin G is the treatment of choice for all stages of syphilis. 48. A 45-year-old man comes to the emergency department after returning from central Africa 2 weeks ago to volunteer at a non-profit medical clinic. He admits that he did not take “those pills they prescribed me before I left” because he was told they may cause lucid dreams. He has

Pathology  673

been having paroxysmal fevers up to 104°F but is otherwise asymptomatic. Laboratory testing reveals anemia; blood smear is shown in Fig. 28.57. What is the most likely diagnosis?

Figure 28.57.  Blood smear of patient in question 48. (From Wikipedia contributors. Plasmodium falciparum. Wikipedia, The Free Encyclopedia. November 2, 2016, 17:03 UTC. Available at: https://en.wikipedia.org/w/index.php?title=Plasmodium_falciparum&oldid=747483880. Accessed November 2, 2016.)

  Plasmodium falciparum is the most likely diagnosis. The incubation period for malaria can be between 2 and 5 weeks, which explains this patient’s delayed presentation. A blood smear with a Giemsa stain is the gold standard for laboratory diagnosis of malaria. As shown in Fig. 28-57, trophozoite “rings” can be visualized within red blood cells (solid arrows). The trophozoites age into multinucleated schizonts (dashed arrow) and eventually become individual merozoites, which cause red cell lysis as they are released into the vasculature and account for this patient’s anemia. Rapid antigen testing is often combined with smear results for a final clinical interpretation. Treatment for malaria is with artemisinin-combination therapy (ACT) using artemisinins and other antimalarial agents such as quinine derivatives.

STEP 1 SECRET A good rule of thumb to remember for your exam: If an infection is not caused by bacteria (i.e., it is viral or parasitic in etiology), confirmatory testing is usually done via PCR or antigen detection assays.

49. A 26-year-old male from Connecticut is evaluated for several days of fever, chills, and arthralgias. He normally enjoys hiking and rock climbing but has not felt well enough in recent days to perform these activities. An image of the patient’s peripheral smear is shown in Fig. 28.58. What is the most likely diagnosis?   Babesiosis, which is a malaria-like parasitic disease caused by infection with the protozoan Babesia is the most likely diagnosis. The new-onset anemia and a history of outdoor activity in an endemic area should lead you to suspect babesiosis in this patient. In a Babesia infection, ringed-shaped parasites can be seen both intracellularly within erythrocytes (arrow) as well as extracellularly. Maltese crosses, which are pathognomonic for babesiosis, are formed by multiple Babesia within a single erythrocyte. While it is rare to visualize Maltese crosses on smears, it is often possible to see multiple parasites within a single cell, which is uncommon for malaria infection. 50. A 9-year-old boy accompanied by his mother presents to his pediatrician for a routine physical examination. Upon questioning, the mother recalls seeing a rash on his face about 1 month ago (Fig. 28.59), but otherwise there is no new information to report. Exam at this time is entirely unrevealing. The infectious agent that causes the rash pictured in the figure is also responsible for which bone marrow abnormality?   Aplastic anemia (crisis) is caused by infection with parvovirus B19 (also known as fifth disease or erythema infectiosum). Aplastic crisis can occur in patients with a preexisting hemolytic anemia (sickle cell disease, thalassemia) because the

674  Pathology

Figure 28.58.  Peripheral smear of the patient in question 49. (From Aster J, Pozdnyakova O, Kutok J. Hematopathology: A Volume in the High Yield Pathology Series. Philadelphia: Saunders; 2013:44, Figure 1.)

Figure 28.59.  Facial rash of patient in question 50. (From Pride HB. Pediatric Dermatology. London: Saunders Ltd; 2008:43-75.)

virus preferentially destroys erythroid progenitor cells in bone marrow. Healthy, immunocompetent people with the infection are asymptomatic. Adults may experience arthralgias while children can display the “slapped cheek” rash shown in the figure. An intrauterine infection during pregnancy may result in spontaneous abortion, stillbirth, or hydrops fetalis because of red cell destruction in the fetus.

INDEX

Note: Page numbers followed by f indicate figures, t indicate tables, and b indicate boxes. A AAA. see Abdominal aortic aneurysm (AAA) ABCDE characteristics, of melanoma, 625 Abciximab, 325t Abdominal adhesions, small bowel obstruction and, 132 Abdominal aortic aneurysm (AAA), 668, 668f Abdominal organs, 618t Abetalipoproteinemia, 22 Abnormal uterine bleeding, 212 Abortion, 532 ABPA. see Allergic bronchopulmonary aspergillosis (ABPA) Absence seizures, 429 Absolute risk, 585 Absolute risk reduction (ARR), 586 Absolute risk reduction percent (ARR%), 586 Acantholysis, 652–653 Acanthosis nigricans, 637, 637f Acarbose, 186 Accommodation, 440 Accuracy, 574 Acetaminophen, 550b case study on, 550b for fever, in child, 539 hepatic damage in, 151 induced fulminant liver failure, 152b maximum daily dosage of, 151, 151b mechanism of action of, 550 for osteoarthritis, 448, 448t toxicity, 151 Acetazolamide, 79, 85, 91, 91b, 442 Acetylcholine myasthenia gravis and, 412 Parkinson’s disease and, 408 Acetylcholine agonists, and antagonists, 552t Achalasia, 113b–115b, 114f, 115 Achondroplasia, 166, 472t–473t Acid-base balance, 89–105, 89b bicarbonate/carbon dioxide buffering system in, 89–90 collecting duct and, 53 derangements in, preventing, 89 diuretics in, 87 hypertrophic pyloric stenosis and, 123 kidneys in, 90–91, 90t lungs in, 90, 90t net renal acid excretion in, 91 renal control of, 78, 79f–80f, 79b Acid-base compensatory mechanisms, time course of, 99f Acid-base disorder, diagnosis of, 94, 95f Acid-fast bacteria, 513 Acidosis diabetic ketoacidosis, 184 kidney in, 91 metabolic, 96, 96b–97b, 101b anion gap, 93, 93f, 104b compensation for, 97 diarrhea in, 96 non-anion gap, 94, 94b

Acidosis (Continued) in renal acid excretion, 78 respiratory compensation for, 90t type II to diarrhea, 97b pathophysiology of, 100 preventing, kidney in, 78 renal tubular, 91 classification of, 97t respiratory, 104, 104b–105b compensation for, 105 exacerbation in, 105, 105b Acinar cells, 106, 108 Acoustic neuroma, 428 Acquired immunity, 346, 346t Acquired immunodeficiency syndrome (AIDS), 367 opportunistic pathogens and malignancies in, 369t Acromegaly, 165b–166b, 166 ACTH. see Adrenocorticotropic hormone (ACTH) Acting out, 386 Actinic keratosis, 625, 625f, 625b Actinomyces israelii infection, 209 Action potential, 412 Activated partial thromboplastin time (aPTT), 317, 317f Acute abdominal pain, 150, 150b, 151t Acute bacterial endocarditis, 655–657 Acute chest syndrome, 640, 640f Acute coronary syndrome, 12 Acute gastritis, 119–120, 119f, 119b, 121b Acute glomerulonephritis, 68, 68f, 68b Acute interstitial nephritis, 58, 58b–59b, 74 Acute lymphoblastic leukemia (ALL), 334t age range for, 328b case study for, 338b–339b origin of, 334t–335t poor prognostic factors for, 339 Acute myelogenous leukemia (AML), 334t age range for, 328b case study for, 343b–344b, 344f diagnosis of, 331b features of, in marrow biopsy, 344 M3 subtype, 330f, 330b origin of, 329, 334t–335t Acute pancreatitis, 122, 123b Acute pulmonary edema, 104 Acute pyelonephritis, 73b–74b, 74 Acute respiratory acidosis, renal compensation for, 90t Acute respiratory alkalosis, renal compensation for, 90t Acute respiratory distress syndrome (ARDS), 43, 43b–44b Acute stress disorder, 394, 395b Acute tubular necrosis, 57, 57b–58b Acyclovir, for herpes zoster, 635 AD. see Alzheimer’s disease (AD) ADAMTS13, 322 Adaptive immune system, 328 Adaptive immunity, 346, 346t ADCC. see Antibody-dependent cellular cytotoxicity (ADCC) Addison disease, 170

675

676  INDEX Adenocarcinoma of colon, 245 endometrial cancer of, 216 esophageal, 112, 112b–113b, 113f, 648 gastric, 229, 230b gastric cancer and, 229 pancreatic, 233 Adenoma fibroadenoma, 217t, 231 gastric, 246, 246t pituitary, 174t, 177t, 669, 670f toxic, 174t Adenomyosis, 212 Adenosine, 12 Adenosine deaminase (ADA) deficiency, 359 Adenosquamous carcinoma, 235 Adenoviridae, 521t Adenylate cyclase, 158–160 ADH. see Alcohol dehydrogenase (ADH) ADHD. see Attention-deficit/hyperactivity disorder (ADHD) Adhesions, surgical, small bowel obstruction and, 132, 132b Adjustment disorder, 385 ADPKD. see Autosomal dominant (adult) polycystic kidney disease (ADPKD) Adrenal cortex, three layers of, 169, 169f Adrenal hyperplasia, 168 Adrenal insufficiency primary, 170, 171b secondary, in low ACTH secretion, 171 α-Adrenergic receptors, 554–555, 555t α1-Adrenergic receptors, 3, 173, 554 α2-Adrenergic receptors, 555 β-Adrenergic receptors, 555, 555t β1-Adrenergic receptors, 5, 555 β2-Adrenergic receptors, 555 Adrenocorticotropic hormone (ACTH), 163t, 646 adrenal insufficiency and, 170–171 in Cushing syndrome, 168t disorders, in deficiency or excess of, 164t, 170b Advance directives, 562, 562b–563b Adventitia, anatomy of, 106, 107f Aflatoxin B1, 150 Afterload, 2 Age-related macular degeneration, 443, 443b–444b Agoraphobia, 394 Agranulocytosis, 178 AIDS. see Acquired immunodeficiency syndrome (AIDS) AIHA. see Autoimmune hemolytic anemia (AIHA) Air trapping, 29 Airway resistance, 23, 23f Akathisia, antipsychotics and, 378 Alanine aminotransferase, in acute hepatitis B, 526 Alanine transaminase (ALT), 136t, 136b acute hepatitis A and, 142, 142b chronic hepatitis B and, 146b, 148 statins and, 21 Albumin, 651–652 in anion gap, 92 in heart failure, 84 hypoalbuminemia, 336 in liver biochemical tests, 136t unconjugated bilirubin and, 134 Albuterol, 30, 30t, 36t Alcohol, 550b benzodiazepines and, 549 consumption of, 140 metabolized in body, 549, 549f Alcohol abuse, 565b–566b case study, 390b–391b, 431b, 433b chronic, 549

Alcohol dehydrogenase (ADH), 140 Alcohol toxicity case study on, 548b symptoms of, 549 Alcohol withdrawal, 546t Alcoholic hepatitis, 137, 140b Alcoholics, acetaminophen and, 152 Aldosterone, 53, 168 Aldosterone antagonists, 86 Alkaline phosphatase (ALP), 136t, 136b acute hepatitis A and, 142 Alkalosis metabolic, 100, 101b in anion gap, 102 anorexia nervosa and, 396 differential diagnosis of, 96f hypochloremic, hypertrophic pyloric stenosis and, 123 respiratory compensation for, 90t respiratory, 91f, 99, 99b compensation in, 98–99 ALL. see Acute lymphoblastic leukemia (ALL) Allergic asthma, 29 Allergic bronchopulmonary aspergillosis (ABPA), 32, 32b Allopurinol, 339 for gout, 453 for hyperuricemia, 263 All-trans-retinoic acid (ATRA), 330f, 330b Alogliptin, 186 ALP. see Alkaline phosphatase (ALP) Alpha fetoprotein (AFP), 149–150, 149b Alport syndrome, 70 ALT. see Alanine transaminase (ALT) Altruism, 387t Alveolar dead space, 25 Alveolar minute ventilation, 25–26 Alveolar-arterial oxygen gradient, 27 Alzheimer’s disease (AD), 280b, 551, 550b, 653, 653f case study, 430b–431b Down syndrome and, 280 Amantadine, 409 Amenorrhea, 221 anorexia nervosa and, 395 progesterone and, 196 secondary, 200, 203b American trypanosomiasis, 115 Amiloride, 86, 381 Amino acid catabolism, disorders of, 283b γ-Aminobutyric acid (GABA), 391, 562 benzodiazepines and, 546, 546b Huntington disease and, 410 Aminocaproic acid, 14 Aminoglycosides, 492, 492b, 494t–497t Aminotransferases, 136t AML. see Acute myelogenous leukemia (AML) Amlodipine, 9t–10t Ammonia, liver failure and, 139t Ammonium, generating de novo, by kidney, 79, 79b, 80f, 91 Amoxicillin, 352b, 353–354 Amphetamine derivatives, 387 Amphetamines, 388, 392t, 542, 543b Amphiarthrodial joints, 445 Amphotericin B for cryptococcal meningitis, 530 mechanism of action, 364, 524 Amsler grid, 443 Amylase, 106 Amylin, 186 Aβ amyloid, 653, 653f Amyloid angiopathy, 430 Amyloid precursor protein (APP), 431, 653

INDEX  677 Amyloidosis (AL), 659, 659f diabetes and, 67 multiple myeloma and, 336–337 Amyotrophic lateral sclerosis, 405–406, 405f–406f, 407b Anaerobes, 490t Anaphylactic shock, 355b, 362 Anaplasia, defined, 226 Anatomic dead space, 25 Anatomic shunts, 26 Ancylostoma duodenale, 523t Androgen, testicular descent and, 197 Androgen insensitivity syndrome, 223b Androgen receptor, 223 Anemia(s), 284–313 angina and, 11 anorexia nervosa and, 396 bacterial endocarditis and, 505t case studies anemia of chronic disease, 299b autoimmune hemolytic anemia, 355, 355b–356b glucose-6-phosphate dehydrogenase (G6PD) deficiency, 307b–309b hemolytic disease of the newborn, 309b–311b iron deficiency anemia, 40, 40b–41b, 43b, 295b, 297b–298b lead poisoning, 311b–312b sickle-cell anemia, 288f, 288b–291b spherocytosis, hereditary, 304b–306b β-thalassemia major, 292b–293b vitamin B12 deficiency, 300b–301b, 303b classifications of, 287f Crohn’s disease and, 657–658 definition of, 284 gastric adenocarcinoma and, 228–229 hemolysis and, 290 hereditary spherocytosis, 643–644, 643f hypertensive nephrosclerosis and, 60 insider’s guide, 284b intestinal bleed and, 298 iron deficiency, 639, 639f vs. anemia of chronic disease, 298t jaundice and, 143 macrocytic, 120, 120f megaloblastic, 120, 120f, 416 microcytic, celiac disease and, 126 multiple myeloma and, 336–337 osteopetrosis and, 470 pathophysiologic mechanisms, 285, 285t–286t secrets for approaching, 286–313 systemic lupus erythematosus and, 464 Aneurysms berry, 420, 423–424, 424f Charcot-Bouchard microaneurysms, 421 coronary artery, 478 Kawasaki disease and, 478 mesenteric, 480–481 subarachnoid hemorrhage and, 420 Takayasu arteritis and, 482 Angelman syndrome, 281 maternal microdeletion in, 281f Angina, 660–661 Angina pectoris, 10b–12b, 11 Angiogenesis in solid tumor growth, 226 tumors and, 226 Angiokeratoma corporis diffusum, 261t Angiotensin II, 52, 76, 78b, 80–81, 81b Angiotensin receptor blockers (ARBs), 207 as antihypertensive drug, 9t–10t in bilateral renal artery stenosis, 82 in glomerular filtration rate, 76, 78b

Angiotensin receptor blockers (ARBs) (Continued) for hypertension in diabetics, 8 mechanism of action of, 8, 9f Angiotensin-converting enzyme (ACE), sarcoidosis and, 46 Angiotensin-converting enzyme (ACE) inhibitors as antihypertensive drug, 9t–10t in bilateral renal artery stenosis, 82 cough and, 9 in glomerular filtration rate, 76, 78b for hypertension in diabetics, 8 mechanism of action of, 8, 9f preeclampsia and, 207 Angle, glaucoma and, 442 Anion gap (AG) calculation of, 92, 101, 101b hypernatremia in, 92b, 93 metabolic acidosis, 93, 93f, 104b MUDPILES mnemonic in, 93–94, 94b metabolic alkalosis in, 102 Anisocytosis, 287 Ankylosing spondylitis, 129f, 130, 457, 457b, 458f Annuli fibrosi, 602 Anorexia nervosa, 395, 395b–396b Anosmia, 221 Kallmann syndrome and, 221 Anterior compartment of leg, 611f Anterior cruciate ligament (ACL), 608–609, 609f, 613b Anterior horn cells, 406 Anterior interventricular artery, 619 Antiarrhythmic agents, 6–7, 7t Antibiotics, 489b bactericidal, 485 bacteriostatic, 485 classes of, 493b, 494t–497t β-lactam, 486 mechanisms of, 486 for pelvic inflammatory disease, 618 Antibodies, 346–347 elimination of extracellular pathogens by, 348 monoclonal, and graft-versus-host disease, 373 structure of, 347, 347f Antibody-dependent cellular cytotoxicity (ADCC), 349 Anti-CCP (citrullinated cyclic peptide) antibody, 450 Anticholinergic agent, 553b intoxication from, 392t for Parkinson’s disease, 409 side effects of, 378 Anticipation, in genetic disease, 411 Anticonvulsants, 416, 429, 429t, 429b triggers acute porphyrias, 270 Antidepressants manic episodes and, 383 selective serotonin reuptake inhibitors advantages of, 383 for Alzheimer’s disease, 430 manic episodes and, 383 MAOI and, combination of, 384 for panic disorders, 394 for posttraumatic stress disorder, 395 premature ejaculation and, 383 in SIADH, 82 tricyclic, 383, 430 Anti-desmoglein, elevated levels of, 652–653 Antidiuretic hormone (ADH), 162t, 646 in collecting duct, 53 in extracellular fluid volume, 81 function of, 82–83, 84b inadequate level of, 83, 84b role of, 53, 54f–55f in SIADH, 83

678  INDEX Anti-dsDNA antibodies, 463 Antiepileptics, in SIADH, 82 Antifungal agents, 524 Antigen, 147, 346 serum sickness and, 356 Antihistamines, 354t Antihypertensive drugs, 9, 9t–10t Antilymphocyte globulin (ALG), 372t Antimitochondrial antibodies, 138 Antimuscarinic/atropine toxicity, 552b Antineutrophil cytoplasmic antibodies (ANCA), 69 Antinuclear antibodies, 463–464 Antiphospholipid syndrome, 327 Antipsychotics hyperprolactinemia and, 165 for Parkinson’s disease, 409 in SIADH, 82 side effects atypical, 379 high-potency, 378 low-potency, 378 typical and atypical, 379–380 Anti-Rh globulin (RhoGAM), 310–311 Anti-Smith antibodies, 463 Antisocial personality disorder, 397, 397b, 399b Antistreptolysin O (ASO) titer, 506 Antithrombin III (ATIII), 318 deficiency in, and deep venous thrombosis, 327, 327b Antithymocyte, 371–372 Antithymocyte globulin (ATG), 372t α1-Antitrypsin, 35, 660 deficiency, 660 Aortic arch baroreceptors, 3 Aortic dissection, 664, 664f Aortic stenosis, 3t, 18, 20b case study on, 18b pressure-volume loop changes in, 18–19, 18f, 19b APC gene, 246, 649 Apgar scores, 559 Aplastic anemia, 532, 674f, 675 Apolipoprotein E gene, 431 Apoptosis, p53 and, 225 APP. see Amyloid precursor protein (APP) Appendicitis, 130–131, 130f, 130b–131b Apple green birefringence, 659 AR. see Attributable risk (AR) Arachidonic acid, 550 Arachnoid granulations, 425 Arachnoid layer, 426 Arachnoid mater, 600, 601f, 602 ARBs. see Angiotensin receptor blockers (ARBs) ARDS. see Acute respiratory distress syndrome (ARDS) Argyll Robertson pupil, 417, 440, 672 Arnold-Chiari malformations, 414 ARPKD. see Autosomal recessive (infantile) polycystic kidney disease (ARPKD) ARR. see Absolute risk reduction (ARR) Arrhythmias basic concepts in, 5–17 cocaine and, 542 myocardial infarction and, 14 Artemisinin-combination therapy (ACT), 672–673 Artemisinins, 672–673 Arterial blood, to the femoral sheath, 588, 588f Arterial dissection, 436 Arterial oxygen content, 40, 40t, 41f Arterial oxygen saturation, 41, 41f Arterial oxygen-carrying capacity, 11 Arterial plaque formation, 590 Arteriolar vasoconstriction, 3 Arteriovenous fistulas, 4, 5f

Arteritis Takayasu, 475t temporal (giant cell), 455, 474, 475t, 476 Arthralgias, 675 Arthritis crystal and septic, 451–452 osteoarthritis and case study of, 446, 446f–447f, 446b, 448t, 449b pathophysiology of, 472t–473t parvovirus B19 infection and, 532 psoriatic, 458b, 459–460, 459f reactive, 374, 374b–375b, 458, 458b organisms associated with, 374 pathogenesis of, 374 rheumatoid case study of, 449–450, 449f–450f, 449b–451b osteoarthritis and, 447, 447f pathophysiology of, 472t–473t septic, 451–452 Asbestos bodies, 39f, 40 Asbestosis, 38, 38b, 40b Ascaris lumbricoides, 523t Ascending sensory pathways cross over, locations of, 402, 402f–403f, 402b overview, 401 syringomyelia, 413 Aschoff bodies, 506, 506f Ascites, liver disease and, 137 ASD. see Atrial septal defect (ASD) Aseptic meningitis, 530 Asherman syndrome, 200 Aspartate aminotransferase, in acute hepatitis B, 526 Aspartate transaminase (AST), 136t acute hepatitis A and, 142, 142b chronic hepatitis B and, 146b, 148 myocardial infarction and, 13, 13f statins and, 21 Asperger syndrome, 388, 389b Aspergillosis, allergic bronchopulmonary, 32, 32b Aspergillus, 524, 524f, 525t in tuberculosis, 514 Aspiration, rheumatoid arthritis and, 450 Aspiration pneumonia, 493 Aspirin on asthmatics, 29 case study on, 538b for cocaine-induced coronary vasospasm, 392 Kawasaki disease and, 478 mechanism of action of, and bleeding disorders, 318, 319t, 325t for osteoarthritis, 448t for peripheral vascular disease, 591t pharmacotherapeutic action of, 539 toxicity, 539b Asplenia, 290 AST. see Aspartate transaminase (AST) Asthma, 29 case study, 28b–29b pathophysiology of, 30 presentation, 31b SIADH and, 82 treatment of, 30, 30t Astrocytes, 243 Astrocytoma, 242, 433 Asymptomatic bacteriuria, 74 Ataxia-telangiectasia, 360t–361t Ataxic gait, 509 Atelectasis, 26 Atenolol, 9t–10t Atherosclerosis, 420, 660–661, 662f absence of, angina in, 11

INDEX  679 Athetosis, chorea with, 409–410 Atonic seizure, 429 Atopic dermatitis, 357–358, 629, 630f ATRA. see All-trans-retinoic acid (ATRA) Atrial fibrillation, 19–20, 420 “Atrial kick,” 20 Atrial natriuretic peptide (ANP), 81 Atrial septal defect (ASD), 597, 599b Atrial septum, 597–598, 598f Atrophic glossitis, 416b Atrophic vaginitis, 217 Atrophy, 664 Atropine, 551 for organophosphate poisoning, 556, 556b Attention-deficit/hyperactivity disorder (ADHD), 387, 388b Attributable risk (AR), 585, 585f, 586b Attributable risk percent (AR%), 586 Atypical pneumonia, 498 Auer rods, 330f, 330b, 343 Auerbach plexus, 107f Autism spectrum disorders, 388, 389b Autistic disorder, 388 Autoantibodies, 463, 464t–465t Autocrine secretions, 156, 156f Autoimmune conditions, autoantibody and, 180t–181t, 181b Autoimmune hemolytic anemia (AIHA), 355, 355b–356b cold, 356 Autonomy, 559 Autosomal dominant diseases, significance of, 254 Autosomal dominant (adult) polycystic kidney disease (ADPKD), 62, 62b, 63f, 64t Autosomal recessive (infantile) polycystic kidney disease (ARPKD), 63, 64t Avascular necrosis of bones, sickle cell anemia and, 291 Avoidant personality disorder, 397, 397b, 399b Axial artery, remnants of, 590f Azathioprine, 371–372, 372t Azithromycin, for chlamydial infections, 515 Azole antifungals, cytochrome P-450 enzymes and, 118t Azotemia, 62, 87 Azygous system, anatomy of, 608 B B lymphocytes, origins of, 328 B19 virion, 532 Babesia, 674 Babesiosis, 674, 674f Babinski sign, 400b, 417 Bacilli, gram-positive, 487t Bacillus anthracis, 484, 487t Back pain, differential diagnosis for, in older patient, 335 Bacteria anaerobes, 490t encapsulated, 484 gram-negative, 483 cocci, 488t rods, 488t–489t sepsis and, 483 structure of, 484f gram-positive, 483 bacilli, 487t cocci, 487t, 493b structure of, 484f spirochetes, 490t Bacterial diseases, 483–519, 483b antibacterial pharmacology of, 485–514, 494t–497t basic concepts of, 483–485 chlamydial infection in, 515b, 517b endocarditis in, 507b Lyme disease in, 510, 512b meningitis in, 519b

Bacterial diseases (Continued) pneumonia in, 501b syphilis in, 510b Traveler’s diarrhea in, 501, 502t, 504b tuberculosis in, 512, 515b Bacterial endocarditis, 655–657, 656f Bacterial vaginosis, 672, 672f Bactericidal antibiotics, 485 Bacteriostatic antibiotics, 485 Bamboo spine, 458 Barbiturates, 548b adverse effects of, 548 for alcoholic withdrawal, 432 benzodiazepines and, 548b case study on, 547b indications for using, 547 intoxication from, 392t mechanism of action of, 432, 547 pharmacokinetic effects of, 391 Bare lymphocyte syndrome, 360t–361t Barium swallow, 657–658 Baroreceptor reflex, 2f, 3 Baroreceptors, 3 Barr bodies, 254f Barrett’s esophagus, 112, 112b, 113f, 648, 649f Basal cell carcinoma, 624, 624f, 624b, 626b Basal ganglia, 408, 408f, 410 Basal metabolic rate (BMR), 174 Basement membrane, 623t underlying, of glomerular filter, 77 Basiliximab, 372t Basophil, 350t–351t B-cells, 350t–351t adaptive immunity and, 346–347 cell surface markers of, 352t defects, in babies, 359 deficiencies, 360t–361t tolerance, 375 Becker muscular dystrophy, 467 Beclomethasone, 30t Behavioral sciences, 559–569, 559b basic concepts in, 559–569 Bell curve distribution, 576f, 577b, 581 Bell’s palsy, 427, 428b Bence Jones proteins, 336, 642 monoclonal gammopathy and, 336 Beneficence, 559 Benign nephrosclerosis. see Hypertensive nephrosclerosis Benign paroxysmal positional vertigo (BPPV), 435 Benign tumor, 228 Benzocaine, 358 Benzodiazepines, 546t, 547b adverse effects of, 547 for alcoholic withdrawal, 432 case study on, 545b categories of, 546t clinical indications for, 546 for delirium tremens, 391 differential diagnosis of, 546 intoxication, 392t and withdrawal, 547, 547t mechanism of action of, 432, 546 for panic disorders, 394 pharmacokinetic effects of, 391 pharmacologic effect of, 562 for seizures, 429 sleep and, 562 Benztropine, 378, 409 Berkson’s bias, 578t–580t Bernard-Soulier syndrome, 315b, 324, 324b

680  INDEX Berry aneurysms, 420, 423–424, 424f rupture of, 273 Best interests standard, 563 Beta-blockers, 7, 555b for angina, mechanism of action of, 12 in asthmatics, 30 case study on, 554b cocaine-induced coronary vasospasm and, 392 contraindicated in diabetics, 184 glaucoma and, 442 for heart failure, 16 for hypoglycemia, 8 for panic disorders, 394 pheochromocytoma and, 171–172 physiologic rationale for administration of, 14 primary hypertension and, 7 Beta-endorphin, 163t Bias case-control study and, 577t cohort studies and, 577t, 584, 586b overview of, 577, 578t–580t, 580b Bicarbonate (HCO3−) buffering system in acid-base balance, 89–90 effectiveness of, 90 filtered, reabsorbing, 91 by kidneys, 79, 79f, 79b generating de novo, by kidney, 79, 79b, 80f, 91 Pco2, and pH, interrelationship among, 90, 91f stomach and, 108t BIG SHIFT, 380, 381b Bilateral breast lumps, 231 Bilateral breast masses, 231 Bilateral renal artery stenosis, angiotensin-converting enzyme inhibitors in, 82 Bile salts, 108–109, 109f, 128, 154 Bile-sequestering resins, 22 Biliary disease, 154b Biliary tree, anatomy of, 153f Bilirubin conjugated and unconjugated, 134 in diagnosing common causes of jaundice, 134 hepatitis A and, 142 jaundice and, 143 in liver biochemical tests, 136t liver failure and, 139t Bilirubin pigment stones, 643–644 Bimodal distribution, 581, 581f Binasal hemianopia, 438f Bioavailability, 535 Biomedical ethics, 559 Biopsy of chronic bronchitis, 37, 37f of emphysema, 35 renal, hypertensive nephrosclerosis and, 59–60 Biostatistics, 570–586, 570b basic concepts of, 570–575 case studies case-control studies, 577–578, 577t, 580b cohort studies, 577t, 584, 586b likelihood ratios, predictive values, and principles of screening, 582b normal distribution and standard deviation, 581, 581b sensitivity, specificity, likelihood ratios, and predictive values, 584b measures of spread in, 575–577 study designs in, 577–586, 577t Bipolar disorder, 380, 380b, 382b Birbeck granules, 332 “Bird’s beak” esophagus, 115 Bitemporal hemianopia, 437–438, 438f, 439b

Biventricular failure, 15 Blast crisis, 339 Blastomyces, 525t Bleeding disorders, 314–327 basic concepts, 314–327, 315f, 315t case studies for deep venous thrombosis, 326, 326b–327b disseminated intravascular coagulation, 321b, 322t, 323b hemophilia, 320b–321b immune thrombocytopenic purpura, 325, 325f, 325b–326b von Willebrand disease, 323, 323b Bleeding time, 317–318 Blindness, temporal (giant cell) arteritis and, 476 Blistering skin disorders, 627t–628t bullous pemphigoid, 627, 627f, 627t–628t, 627b, 629b dermatitis herpetiformis, 627t–628t, 638, 638f erythema multiforme, 627t–628t, 637b pemphigus vulgaris, 627, 627t–628t, 628b–629b, 629f Stevens-Johnson syndrome, 627t–628t, 635–637, 636f Blood pressure, 581b arterial, 1, 2f. see also Mean arterial pressure (MAP) Blood urea nitrogen (BUN), 139, 139t Blood urea nitrogen/creatinine (BUN/Cr) ratio, 58b Blood-brain barrier, 434 BMPR2 gene, 46 BMR. see Basal metabolic rate (BMR) Bone fractures, β-thalassemia major and, 293 Bone marrow transplant, for severe combined immunodeficiency, 359 Borderline hypocalcemia, 60 Borderline personality disorder, 396b, 397, 399b Bordetella pertussis, 489t Borrelia burgdorferi, 490t, 510 Borrelia recurrentis, 490t Botulinum toxin, achalasia and, 115, 115b Bouchard nodes, 446f Boutonnière deformities, 451 Bowman’s space, 75, 75f BPPV. see Benign paroxysmal positional vertigo (BPPV) Brachial plexus, 591–592, 592f, 595b case studies, 591b, 593b–595b Brain cancers, 243b case studies, metastatic, 247b–248b, 248 Brain tumors, 433, 433b–434b Branchial pouches, arches, and clefts, 616, 616t BRCA gene testing, 250 BRCA1 mutations, 231 BRCA2 mutations, 231 Breast anatomy of, 232f benign mass of, 217–218 Breast cancer, 220b, 230, 233b case studies, 218b–219b, 230b–232b metastatic brain cancer and, 248 physical examination findings indicated in, 230–231 presentation of, 230 risk factors for, 219 Breast mass characteristics of, 217t differential diagnosis for, 217 palpable, 218b Breastfeeding, as contraceptive, 206 Breathing, mechanics of, 23–26, 23f, 26b Brenner tumors, 251 Brief psychotic disorder, 377 Broad ligament, 617, 618f Broca’s aphasia, 421, 422f Bromocriptine, 165, 409 Bronchial tree, anatomy of, 620, 621f Bronchiectasis, cystic fibrosis and, 274

INDEX  681 Bronchitis acute, 235 chronic, 36, 36b–38b Bronchoalveolar carcinoma, 235 Bronchogenic carcinoma, 39 Bronze diabetes, 651 Brown-Séquard syndrome, 403, 404f Brucella spp., 489t Brudzinski sign, 529 Bruton agammaglobulinemia, 360t–361t Budd-Chiari syndrome, 136 Buerger disease, 475t “Buffalo hump,” 45 Bulimia nervosa, 396 Bullous pemphigoid, 627, 627f, 627t–628t, 627b, 629b immunofluorescence of, 628f, 628b BUN. see Blood urea nitrogen (BUN) Bupropion, 384–385 Burkitt lymphoma, 333t–334t Busulfan, mechanism of action of, 373 C C1 esterase inhibitor, 350t CAD. see Coronary artery disease (CAD) CAGE questionnaire, 566 CAH. see Congenital adrenal hyperplasia (CAH) CAIs. see Carbonic anhydrase inhibitors (CAIs) Calcineurin, 370, 370f Calcium (Ca), 92 cytosolic, source of during ventricular systole, 5 function of, in muscle contraction, 5 Calcium balance, diuretics in, 87 Calcium channel blockers, 545t, 544b–545b cardiac output and, 1 case study on, 544b cocaine-induced coronary vasospasm and, 392 of labor suppression, 207 mechanism of action of, 544 side effects of, 545 types of, 544–545 Calcium pyrophosphate dihydrate (CPPD) deposition, 453 Calcium stones, 71 Calcium (oxalate/phosphate) stones, 669 Calcium-induced calcium release, 5 Calf muscles, pseudohypertrophy of, 467, 468f cAMP receptors, classes of, 158t Campbell de Morgan spots, 637, 637f Campylobacter, 503t Campylobacter jejuni, 488t c-ANCA, 479 Cancer(s), 224–226, 224b case studies, 228b–229b adenocarcinoma, gastric, 228b brain cancer, 242b, 247b–248b, 248 cervical cancer, 242b colon cancer, 245 lung cancer, 235b–237b ovarian cancer, 249b–251b pancreatic cancer, 233b–234b prostate cancer, 238b–239b, 241b thyroid cancer, 243b–244b classification of, 227–252, 228b, 241b, 245b–247b, 249b–250b epidemiology of, 226, 226b epithelial ovarian, 249 esophageal, 112 naming of, 228 types by gender, 226t Cancer antigen 125 (CA-125), 215, 250 Candida albicans, 524, 525t, 633 Candidiasis, oropharyngeal, 30

Capillary oncotic pressure, capillary, in filtration fraction, 76 Capsule, 484 Captopril, 17t Caput medusae, 137, 137b Carbachol, glaucoma and, 442–443 Carbamazepine, 382, 416 Carbapenems, 494t–497t Carbidopa, 408 Carbohydrates, 106, 106t Carbon dioxide alveolar concentration of, 27 buffering system, in acid-base balance, 89–90 Carbon monoxide, shift of, 4 Carbonic anhydrase, 79, 91b Carbonic anhydrase agents, 442 Carbonic anhydrase inhibitors (CAIs), 85 Carcinoid syndrome, 131, 131b, 248t, 394 Carcinoid tumors, 131, 235 Carcinoma adenosquamous, 235 basal cell, 624, 624f, 624b, 626b bronchoalveolar, 235 bronchogenic, 39 of cervix, 238 choriocarcinoma, 204 defined, 226, 227f ductal carcinoma in situ, 230 epithelial ovarian, 250 gastric, 116–117 hepatocellular, 149b–150b, 150 inflammatory breast, 219 lobular, 232 lobular carcinoma in situ, 232 medullary, 232, 243 pancreatic, 233 papillary, invasive, 218 renal cell, 248 of salivary gland, 235 serous cystadenocarcinoma, 215 in situ, 232 small cell lung cancer, 235 squamous cell, 625, 626b esophageal, 112 Cardiac contractility, 5 digitalis and, 17 Cardiac enzymes, myocardial infarction and, 13 Cardiac glycosides, 543 Cardiac output (CO) exercise and, 4 heart failure and, 15 mean arterial pressure and, 1 stenotic aortic valves and, 19 venous return curves and, 5f Cardiac rhabdomyoma, 434 Cardiology, 1–22, 1b arrhythmia in, 5–17 case studies in angina pectoris, 10b–12b, 11 heart failure, 15b hypertension, 7b metabolic syndrome, 20b–22b myocardial infarction, 12b–13b type 2 diabetes mellitus, 8b–9b excitation-contraction coupling in, 5 hemodynamics in, 1–4 Cardiomyocytes, 18, 543 Cardiomyopathy, sarcoidosis and, 46 Cardiothoracic anatomy, 621b Carmustine, 434 Carotid bruits, 18

682  INDEX Carotid sinus baroreceptors, 3 Carpal tunnel, 593, 596b Carpal tunnel syndrome, 456–457, 456b–457b, 593 Carvedilol, 9t–10t Case-control studies, 577–578, 577t, 580b Caseous necrosis, 514 Catalase, 362 Cataracts, galactosemia, infants with, 271–272 CATCH 22 mnemonic, for DiGeorge syndrome, 364 Catecholamines, 157, 172, 172b, 542 Causal relationship, 575 Cavities, 396 CCK. see Cholecystokinin (CCK) CD3+ cells, lack of, in severe combined immunodeficiency, 359 CD4+ T cells, 369 CD8 molecule, 347 CD55 protein, 350t CD59 protein, 350t Celecoxib, for osteoarthritis, 448t Celiac disease, 125b, 126–127, 126f, 127b, 650–651, 650f Cell cycle, 224, 225f, 225t Cell cycle arrests, 225 Cell surface markers, 352, 352t Cell-mediated immunity, 346–347 for tuberculosis, 514 Central pontine myelinolysis (CPM), 83 Central tendency, 575 Centriacinar emphysema, 35, 39f Cephalosporins, 494t–497t antibacterial spectrum of, 490–491 patients allergic to, 490 Cerebellum, 401 Cerebral aqueduct, 425, 425f Cerebral artery, 421, 421f–422f Cerebral circulation, infarction and, 422 Cerebral edema, 83 Cerebral infarction, 392 Cerebrospinal fluid flow, pathway of, 425, 425f meningitis and, 530, 530t, 601t production and function of, 425 reabsorbed, 425 Cerebrovascular accidents, 420, 421f, 422b Ceruloplasmin, 138, 141, 411 Cervical cancer, 216, 238, 241b–242b risk factors for, 239 screening, 239 Cervical carcinoma, 241 Cervical dysplasia, 241 Cervical intraepithelial neoplasia (CIN), 239, 240t cytologic appearance of, 241f Cervical sympathetic chain, compression of, 45 Cervical transformation zone, 240f Cervix, anatomy of, 617 Cesarean section, 207 Cestodes, 523, 523t CFTR. see Cystic fibrosis transmembrane regulator (CFTR) CGD. see Chronic granulomatous disease (CGD) cGMP receptors, classes of, 158t Chagas disease, achalasia and, 115 Charcot-Bouchard microaneurysms, 421 Charcot-Leyden crystals, 30 Charcot’s triad, 146, 146b Checkpoints, 225 Chédiak-Higashi syndrome, 364, 360t–361t Chemoreceptors, 3 Chemotherapy, 150b Cherry-red spot, 259 Chest, expanded barrel, 660, 661f Chest syndrome, acute, sickled RBCs, 290–291, 291b

Chest x-ray films, 499b of lobar pneumonia, 33, 33f CHF. see Congestive heart failure (CHF) Chickenpox, 533t Childhood disorders, 388b–389b Childhood exanthems, 533t Chlamydia, 209f Chlamydia pneumoniae, 491t, 517t pneumonia and, 500t–501t Chlamydia psittaci, 491t, 517t Chlamydia trachomatis, 209, 491t, 515b, 517t pelvic inflammatory disease and, 208, 515, 617 serotypes of, 516, 517t Chlamydial infection, 515b, 517b life cycle of, 516, 517f Chloramphenicol, 492, 494t–497t Chlorpromazine, 378 Chocolate cysts, 211, 648 Cholangitis, 146, 154b sclerosing, 129 Cholecystitis, 152, 152b, 154b spherocytosis and, 306 Cholecystokinin (CCK), 108t, 152, 154b Choledocholithiasis, 154b Cholelithiasis, 154b sickle cell anemia and, 291 Cholera, 577b–578b, 578, 580b Cholera toxin, 502f Cholestasis, 153 Cholesterol, 108 Cholesterol stones, 153 Cholesterol-lowering drugs, 21 Cholestyramine, diarrhea and, 128–129 Cholinergic poisoning, 392t Cholinergics, 553b Cholinesterase inhibitors, 412–413, 430 Cholinomimetics, glaucoma and, 442–443 Chondrocalcinosis, 447 Chondrocytes, 447 Chorda tympani, 428 Chorea, 409–410 Choreoathetosis, 411b Choriocarcinoma, 204 Choroid plexus, 425 Chromaffin cells, 172, 172b Chromosomal origin, 205f Chronic bronchitis, 36, 36b–38b Chronic disease, anemia of, 299 cause of, 299 iron deficiency anemia and, 298t, 300b summary, 300b total iron-binding capacity, reduced, 299 transferrin saturation, reduced, 299 Chronic gastritis, 121, 121b Chronic granulomatous disease (CGD), 360t–361t, 362, 362f–363f, 362b–364b staphylococcal infections and, 363–364 Chronic inflammation, anemia of. see Chronic disease, anemia of Chronic inflammatory disorders, 299 Chronic lymphocytic leukemia (CLL) age range for, 328b origin of, 334t–335t vs. chronic myelogenous leukemia, 340t Chronic mucocutaneous candidiasis, 360t–361t Chronic myelogenous leukemia (CML), 334t, 641 age range for, 328b case study for, 339, 339b–340b pathogenesis of, 339 vs. chronic lymphocytic leukemia, 340t

INDEX  683 Chronic obstructive pulmonary disease (COPD), 36b acid-base abnormality in, 37 acute exacerbation of, 37 airway obstruction in, 26 case study on, 33b–34b, 34f functional residual capacity in, 24, 24f pursed lips breathing, 35–36 SIADH and, 82 treatment of, 36t on ventilation-perfusion ratio, 26 wheezing in, 29 Chronic pancreatitis, 122f, 123, 123b, 390 Chronic renal failure, 59b, 62b Chronic respiratory acidosis, 100 renal compensation for, 90t Chronic respiratory alkalosis, renal compensation for, 90t Churg-Strauss syndrome, 474, 475t Chylomicrons, 20–21 Cigarette smoking, triggers acute porphyrias, 270 Cilostazol, for peripheral vascular disease, 591t Cimetidine cytochrome P-450 enzymes and, 118t for gastroesophageal reflux disease, 112t CIN. see Cervical intraepithelial neoplasia (CIN) Ciprofloxacin, cytochrome P-450 enzymes and, 118t Circle of Willis, 423 Circumflex artery, 619, 619f Cirrhosis, 81, 135, 150, 651–652, 652f Cisplatin, 249 Claudication, 660–661 Clavulanic acid, 486 Cleft lip, 615, 615b, 617b Cleft palate, 615–616, 615b, 616f, 617b Clindamycin, 494t–497t Clinical anatomy, 587–621, 587b cardiothoracic, 621b of congenital heart defects, 597, 599, 599b embryology of face and neck, 616–621, 616t, 616b–617b of hernias and male reproductive function, 603–604, 604f, 604b, 606b of lower extremity injuries, 613b of peripheral vascular disease, 587, 591t, 591b of peritoneum and female reproductive system, 617, 619b of spinal cord and vertebral column, 600, 601f, 603b of upper extremity injuries, 591–593, 595b–596b of vena cava, 607, 608b Clonidine, 9t–10t, 172, 555 Clonorchis sinensis, 154, 523t Clopidogrel, 325t for peripheral vascular disease, 591t Closed-angle glaucoma, 442, 442f Clostridium botulinum, 490t Clostridium difficile, 490t colitis, 502 diarrhea and, 503t Clostridium perfringens, 490t Clostridium tetani, 490t “Clot-busting” drugs, 318b Clotting factors, 138 Clozapine, 380 “Clue cells,” 672 CML. see Chronic myelogenous leukemia (CML) CO. see Cardiac output (CO) Coagulation cascade, 314, 316f fibrin clot and, 316, 316f, 316b Coarctation of the aorta, 221 Cobalamin. see also Vitamin B12 biochemical role of, 302f Cocaine, 543b angina and, 11 case study on, 541b

Cocaine (Continued) development of tolerance to, 542 effects of, 541 adverse, 542 in fetal exposure, 202t intoxication, 392, 393b mechanism of action of, 541 “Cocaine bugs,” 392 Cocci gram-negative, 488t gram-positive, 487t, 493b Coccidioides spp., 525t Coccobacilli, 672 Cockroft-Gault equation, 78 Cognitive development, stages of, 389 Cognitive disorders, 393b Cohort studies, 577t, 584, 586b limitations of, 585 Colchicine, gout and, 453 Cold agglutinins, 498b Cold autoimmune hemolytic anemia, 356 Collagen, osteogenesis imperfecta and, 472 Collagen bands, 651–652, 652f Collapsing pressure, 25 Collecting duct, 53 Colles fracture, 594 Colon, 245 Colon cancer, 245, 247, 247b metastasize, 247 Colorectal adenocarcinoma, 246, 246f Coma, epidural hematoma and, 664 Combined deficiencies, 360t–361t Common variable immunodeficiency, 360t–361t Communicating hydrocephalus, 425 Community-acquired pneumonia, 498 Compartment syndrome, 611–612, 611b Compensated heart failure, 4, 5f Competence, 563 Competitive inhibitors, 537 Complement, decreased serum levels of, 357 Complement cascade, 349f, 349b Complement proteins, 349, 349f, 349t, 349b deficiencies in, 350, 350t Complex seizures, simple seizures and, 429 Compliance, 567–568 curve, of lungs, 24, 24f pulmonary, 24 resistance, 23f, 24 Concentration, 535 Concentric hypertrophy, 18 Concrete operational, 389 Conduct disorder, 387, 388b, 397 Conducting airways, dead space and, 25 Condylomata lata, 508, 672 Confounding bias, 578t–580t Confounding variables, 577t, 580, 585 Congenital adrenal hyperplasia (CAH), 192, 192b, 193f, 194b Congenital heart defects, 597, 599, 599b Congenital hypothyroidism, 182 Congestive heart failure (CHF), 15, 236 acute respiratory distress syndrome and, 44 antidiuretic hormone in, 81 drugs for, 17t pulmonary edema in, 44 Congo red staining, 659 Conn syndrome, 100, 100b Consent, for minors, 563, 563b Consumptive coagulopathy, 321 Contact dermatitis, 357–358, 357b–358b, 631b Continuous positive airway pressure (CPAP), 560

684  INDEX Contraceptives, 150 oral, 199b Contractility, 2 Contraction alkalosis, pathophysiology of, 100 Convergence, 441 Cooley anemia. see β-thalassemia major Coombs test, 355 autoimmune hemolytic anemias and, 306 spherocytosis, hereditary and, 306 COPD. see Chronic obstructive pulmonary disease (COPD) Copper, in Wilson’s disease, 141, 141b Cor pulmonale, sarcoidosis and, 46 Core antigen (HBcAg), 526 Cori disease. see Glycogen storage disease, type III Corneal reflex, 428 Coronary artery aneurysms, 478 Coronary artery disease (CAD), 15b, 582t, 582b risk factors for, 13 Coronary circulation, anatomy of, 619f Coronary vasospasm, 392 Corpus luteum, 196 Corticospinal tract, 402, 402b, 417 Corticosteroids, 354t, 372t for anaphylaxis, 354 for bullous pemphigoid, 628 for immune thrombocytopenic purpura, 326 monitoring response of, 477 for osteoarthritis, 667 for polymyalgia rheumatica, 455 for rheumatoid arthritis, 450 for Takayasu arteritis, 482 for temporal (giant cell) arteritis, 477 Cortisol, 167, 168f Corynebacterium spp., 487t Cosyntropin, 170b, 171 Countercurrent exchange mechanism, 53, 53f Courvoisier sign, 233 CPAP. see Continuous positive airway pressure (CPAP) C-peptide, 184 CPK. see Creatine phosphokinase (CPK) CPM. see Central pontine myelinolysis (CPM) “CRAB” criteria, 642 Cranial nerve III, 242 Cranial nerve VII, 427 Cranial nerves, 412 CRASH symptoms, 631–632 C-reactive protein (CRP), 447, 476 Creatine kinase MB fraction (CK-MB), 13, 13f Creatine phosphokinase (CPK), 21 Creatinine, 336 clearance, 51, 51f, 78, 78b Cremasteric muscle and fascia, 605 Crepitus, 634 CREST syndrome, 461, 462t Cretinism, 182 “Crew cut” appearance, 293f, 293b Cricothyroid membrane, 614, 615f, 617b Cricothyroid muscle, 613 Crigler-Najjar syndrome, 144, 145t Crohn’s disease, 657–658, 657f–658f case study of, 127–128, 127b–128b, 130b cholesterol stones and, 154 colon cancer and, 247 kidney stones and, 73 ulcerative colitis and, 128t Cromolyn sodium, 30, 30t CRP. see C-reactive protein (CRP) Crural fascia, 611 Cryoglobulinemic vasculitis, 474

Cryptococcal meningitis, 530 Cryptococcus neoformans, 524, 525t, 671–672, 671f Cryptorchidism, 197 Cryptosporidium spp., 503t, 523t Crystal arthritis, septic arthritis and, 451–452 Cullen sign, 668 Curare, 413, 552 Curschmann spirals, 30 Cushing disease, 167–168 pathophysiology of, 100 Cushing syndrome, 167, 167b, 168f, 168t, 170b, 646 paraneoplastic syndrome and, 45, 248t Cutaneous T-cell lymphoma, 333t Cyanosis, 41, 598b, 599 Cyclic neutropenia, 360t–361t Cyclin(s), 225 Cyclin-dependent kinase(s) (CDKs), 225 Cyclooxygenase (COX) aspirin in, 29 pathway, 539 Cyclooxygenase-2 (COX-2) inhibitors danger of, 448 for osteoarthritis, 448, 448t peptic ulcer disease and, 116, 118b–119b Cyclophosphamide, 372t for systemic lupus erythematosus, 464 Wegener granulomatosis and, 480 Cyclosporine, 372t mechanism of action of, 370, 370f Cyclothymia, 381 CYP-450 drug metabolism, 558b case study on, 556b–557b CYP-450 substrates, 557, 557t, 557b Cystic fibrosis, 274b bronchiectasis and, 274 carrier frequency for, 258–259 chronic pancreatitis and, 122 etiology of, 274 N-acetylcysteine for, 274 pancreatitis and, 274 probability of having, 258 respiratory infection in child, 274 screening methods, 258 summary, 275b variants of, 258 Cystic fibrosis transmembrane regulator (CFTR), 274 Cystic hygroma, 221 Cystic kidney disease, 64t Cystic tumors, 234 Cystine stones, 71, 72f, 73 Cystinuria, 72f, 73, 91 Cystosarcoma phyllodes, 218 Cytochrome P-450 enzymes, 118, 118b hepatic, 199 Cytokines, functions of, by secreting class of immune cells, 351–352 Cytotoxic hypersensitivity, 355 Cytotoxic T cell, 350t–351t Cytotoxic T lymphocytes (CTLs), 346 D Daclizumab, 372t Dactylitis, 459, 459f DAF. see Decay-accelerating factor (DAF) Danazol, 212 Daptomycin, 494t–497t Dashboard injury, 609f De Quervain thyroiditis, 174t, 181 Dead space, 25 Deamination, of glutamine, 91

INDEX  685 Death, in United States, causes of, 226 Decay-accelerating factor (DAF), 350t Decision-making capacity, 559, 563, 563b Decompensated heart failure, 15 Deep venous thrombosis (DVT), 234, 326, 326b–327b protein C, protein S, and antithrombin III deficiency, 327 pulmonary embolism and, 327 Virchow’s triad and, 327 Defense mechanisms, 387t Deferoxamine, 294 Dehydroepiandrosterone (DHEA), 223 Dehydroepiandrosterone sulfate (DHEA-S), 223 Delayed-type hypersensitivity (DTH), 353t, 357–358 Delirium, 393, 393b Delirium tremens, 391 “Delta-delta” concept, 102 Delta-delta protein gap, myeloma gammopathy in, 92b Delusions, schizophrenia and, 376 Dementia, 393, 393b, 410, 430, 653 Dendritic cell, 350t–351t Denial, 386 Dependence, substance abuse and, 391 Dependent personality disorder, 397, 397b, 399b Depolarizing neuromuscular blockers, 552–553, 553f Depression, 382, 385b, 395 Dermatitis contact, 357–358, 357b–358b diaper, 633, 634f herpetiformis, 627t–628t, 638, 638f Dermatology, 622–638, 622b basic concepts in, 622–638 high-yield, 629b Dermatomyositis, case study of, 466, 466f Dermis, 623t Desmopressin acetate (DDAVP), 323–324 Developmental age, 200–201 Developmental milestones, 566t–567t, 566b–567b Dexamethasone, 167b Dexamethasone suppression test, 168 Diabetes, amyloidosis and, 67 Diabetes insipidus (DI), 83, 83b, 381 ADH levels in, 84, 84t Diabetes mellitus case studies on, type 2, 8b–9b chronic complications of, 183f chronic pancreatitis and, 123, 123b maturity-onset diabetes of youth, 183 type 1, 182–183, 182b, 184b–185b type 2, 183, 185, 185b Diabetic ketoacidosis (DKA), 102, 102b, 184 Diabetic neuropathy, 342b Diaper dermatitis (diaper rash), 633, 634f Diaphragmatic paralysis, 236 Diarrhea, 96, 96b anaphylaxis and, 353 bile salts and, 128 bloody, 503t case study on, 501b diagnosis of, 501 hypertrophic pyloric stenosis and, 124 in hyponatremia, 83–84 metabolic acidosis type II to, 97b traveler’s, 501, 502t, 504b types of, 501 watery, 503t Diarthrodial joint, 445, 445f Diastolic blood pressure, primary determinant of, 1 Diastolic dysfunction, 14 Diastolic heart failure, 3t, 15–16 Diazepam, 392

DIC. see Disseminated intravascular coagulation (DIC) Diethylstilbestrol (DES), 201t Diffuse large B-cell lymphoma, 333t Diffuse scleroderma, 461, 462t Diffusing capacity, 27 Diffusion equation, 44 DiGeorge syndrome, 359, 360t–361t case study for, 364, 364b–365b, 365f Digitalis, 17, 17t, 17b case study on, 543b mechanism of action of, 543 toxicity, 544, 544b Digitalis lanata plant, 543 Dilated cardiomyopathy, 3t Diltiazem, 6, 9t–10t, 545, 545t Dimorphic fungi, 524 Diphenhydramine, 354, 378 Diphyllobothrium latum, 523t Diplopia, 412, 418 Dipyridamole, 325t Direct inguinal hernia, 604, 604f, 606b Disclosure, of patient information, 568b Discoid lupus, 463 Disease-modifying antirheumatic drugs (DMARDs), for rheumatoid arthritis, 450 Disseminated intravascular coagulation (DIC), 321, 321b, 322t, 323b, 641, 642f pathogenesis of, 321 schistocytes in, 321, 321f thrombotic thrombocytopenic purpura and, 322 Distal tubule, 53 Disulfiram, for alcoholism, 549 Diuretic acetazolamide, in acid-base balance, 79 Diuretics for hypoglycemia, 8–9 for hyponatremia, 83–84 pharmacology of, 85–88, 85t, 86f, 87b–88b side effects of, 87, 87b DKA. see Diabetic ketoacidosis (DKA) DNA synthesis, 416–417 DNA viruses, 520, 520b, 521f, 521t Dobutamine, 12 Donepezil, 430 Dopamine amphetamines and, 542 etiology of depression, 383 Parkinson’s disease and, 408 schizophrenia and, 378t, 379, 379b Sheehan syndrome and, 162 Dopamine receptor agonists, 409 Dorsal column-medial lemniscus, 401, 402b, 413 Dorsal columns, 402, 417 “Double-blind” placebo-controlled trials, 586 Down syndrome, 202, 202b, 279 Alzheimer’s disease and, 280 diagnosis, prenatal, 280 duodenal atresia and, 280 leukemia, acute, and, 280 mechanism of mosaic, 280 summary, 281b Doxazosin, 9t–10t Doxycycline, for Lyme disease, 511 DPP-4 inhibitors, 186 Dressler syndrome, 14, 620 Drug intoxication, 391b, 392t, 393b Drug-induced lupus, 464 Dubin-Johnson syndrome, 144, 145t Duchenne muscular dystrophy, 467, 468f Ductus arteriosus, 596, 597f, 599b DUMBBELSS symptoms, 556 Dumping syndrome, 124, 124b

686  INDEX Duodenal atresia, 123, 124b, 125f Down syndrome and, 280 Duodenal ulcers, 115, 117 Duodenum, gastrointestinal hormones and, 108t Dura mater, 426, 600, 601f DVT. see Deep venous thrombosis (DVT) Dwarfism, 166 Dyskinesia, 410 Dyslipidemia, 20–21, 22b ischemic stroke and, 665–666 Dyspepsia, 228 Dysphagia, 113, 620 Dysplasia, 226, 226b Dyspnea in hypersensitivity pneumonitis, 31 iron deficiency anemia and, 40–41 Dysthymic disorder, 385 Dystonia, acute, 378 Dystrophin, 467 E E antigen (HBeAg), 526 Eating disorders, 396b Ebola infection, 534b case study on, 532b, 534b diagnosis of, 534 differential diagnosis of, 532 managed, 534 symptoms of, 534 transmitted, 534 Ebstein anomaly, 381 Eccentric hypertrophy, 18 Ecchymosis, 668 Echinococcus granulosus, 523t Eclampsia, 206–207 Ecstasy, 384, 392t Ectasia, 274 Ectasis, 274 Ectopic endometrial tissue, 211 Ectopic pregnancy, 200 Eczema, 629 ED. see Erectile dysfunction (ED) Edinger-Westphal nucleus, 437, 439 Edrophonium, 412 Edrophonium chloride. see Tensilon test Edward’s syndrome, 280 Effective circulating volume (ECV), 80, 82b Ego, 389 Ego defenses, 385b–386b, 387t Ehlers-Danlos syndrome, 273–274, 664 Eighth Joint National Committee on Detection, Evaluation, and Treatment of High Blood Pressure, 7, 8t Eisenmenger’s complex, 598, 599b Ejaculation, 220 neurologic basis for, 606 Ejection fraction, 15 Elastance, pulmonary, 24 Elasticity, 24 Elastin, 660 Electrocardiogram, of emphysema, 35 Electrocardiography (ECG), for myocardial infarction, 13 Electroconvulsive therapy (ECT), 383 EM. see Erythema multiforme (EM) Emboli, in bacterial endocarditis, 505t Embryology, of face and neck, 616–621, 616t, 616b–617b Emission, neurologic basis for, 606 Emphysema, 34, 660, 661f. see also Chronic obstructive pulmonary disease (COPD) cause of, 34–35 pulmonary changes in, 35f

En coup de sabre, 461, 461f Endocarditis, 507b acute rheumatic, 505f acute vs. subacute, 507t bacterial, 505, 505b, 507, 655–657, 656f case study on, 504f, 504b diagnosis of, 504 drugs for, 507 Libman-Sacks, 464, 507 risk factors for developing, 505 systemic lupus erythematosus and, 464 Endocervical polyp, 238 Endocrine secretions, 156, 156f Endocrinology basic concepts of, 156–171, 156b case studies, gestational hyperthyroidism, 205b cases acromegaly, 165b–166b, 166 adrenal insufficiency, 170, 170b–171b congenital adrenal hyperplasia, 192, 192b, 193f, 194b Cushing syndrome, 167, 167b, 168t, 170b familial hypocalciuric hypercalcemia, 191, 191b–192b gestational diabetes, 187t, 187b–188b, 188 hyperthyroidism, 173, 173b, 174t, 177t, 179b, 180t, 182t hypothyroidism, 170b, 179–180, 179b, 180t, 182t, 182b multiple endocrine neoplasia, 189b–190b, 190 pheochromocytoma, 171b, 172, 173b prolactinoma and hyperprolactinemia, 163, 163b, 164t, 165b Sheehan syndrome, 160, 160b, 163b type 1 diabetes mellitus, 182–183, 182b, 184b–185b type 2 diabetes mellitus, 183, 185, 185b, 187b of pregnancy, 206b differential diagnosis, 205 Endometrial cancer, 199, 216, 217b diagnosis of, 216 Endometrial hyperplasia, 216 Endometriomas, 648 Endometriosis, 212b, 648, 648f Endometrium, 196, 197f Endothelial cells, of glomerular capillaries, in glomerular filter, 77 Endothelial injury, 660–661 Endothelium-derived relaxation factor (EDRF), 12 Endotoxin, 483 Entamoeba histolytica, 503t, 523t Enterobius vermicularis, 523t Enterococcus spp., 487t Enterohemorrhagic E. coli (EHEC), 501, 501b, 502t–503t Enterohepatic circulation, 109 Enteroinvasive E. coli (EIEC), 501, 502t–503t Enteropathogenic E. coli (EPEC), 501, 502t Enterotoxigenic E. coli (ETEC), 501, 502t–503t Enterotoxins, 483 Enzyme-linked immunosorbent assay (ELISA), 366 Enzymopathy definition of, 253 mechanisms of, 253f proteins in, 255t Eosinophil, 350t–351t Eosinophilic granuloma, 332 Eosinophilic granulomatosis with polyangiitis, 474 Eosinophilic pneumonias, 32 Ependymoma, 424 Epidemiology, of cancer, 226, 226b Epidermis, 622b, 623t Epidermophyton, 525t Epididymis, anatomy of, 606f Epidural hematoma, 426, 427f, 427b, 664, 665f Epidural hemorrhage, 420

INDEX  687 Epilepsy, 430, 430b Epinephrine, 554 for anaphylaxis, 354 conversion from phenylalanine, 256f, 256b glaucoma and, 443 pheochromocytoma and, 171 for urticaria, 631 Epithelial cells, 226 of glomerular filter, 77 Epithelial ovarian cancer, 249, 251 Eplerenone, 86 Eptifibatide, 324, 324b, 325t Equal pressure point (EPP), emphysema and, 35–36 Erb-Duchenne palsy, 591 Erectile dysfunction (ED), 220b, 606 case studies, 220b diagnosis of, 220 Erection, 220 neurologic basis for, 606 Ergot alkaloids, 203 Erythema infectiosum, 531–532, 531b, 533t, 633, 633f, 675 case study on, 531b Erythema multiforme (EM), 627t–628t, 637b Erythrocyte cast, in acute glomerulonephritis, 68f Erythrocyte sedimentation rate (ESR), 447, 476 Erythrocytes, 328 in babesiosis, 674, 674f Erythrocytosis, differential diagnosis of, 342, 342b Erythromelalgia, 342–343 Erythropoiesis, 336 Erythropoietin (EPO) polycythemia vera and, 343 role of, 285 Escherichia coli, 488t diarrhea and, 501, 502t pneumonia and, 499t Esmolol, 9t–10t Esophageal adenocarcinoma, 648 Esophageal atresia, 123 Esophageal cancer, 112 Esophageal stricture, 112 Esophageal varices, 137, 608 Essential hypertension, 7 Essential tremor, 407 Estrogen containing contraceptives, 198, 199 follicular phase of menstrual cycle and, 196 liver failure and, 139t luteal phase of menstrual cycle and, 196 ovulation and, 196 for Turner syndrome, 221 Estrogen receptor (ER), 219, 232 Ethacrynic acid, 87 Ethambutol, 494t–497t, 514 Ethanol, 140 Ethosuximide, 429 Ethylene glycol intoxication, 103, 103b Ethylene glycol poisoning, 140 Etoposide, 249 Euvolemia, 83 Excessive daytime sleepiness, 560b Exchange transfusion, 640 Excitation-contraction coupling, 5 Exenatide, 186 Exercise, 4, 5f Exercise stress test, 12, 12b Exercise-induced asthma, 29 Exotoxins, 483

Expiratory airflow driving forces for, 23 forces of resistance for, 24 obstructive lung disease in, 28 Expressive aphasia, 421, 422f External respiration, 42 Extracellular fluid (ECF), buffers in, 89 Extracellular fluid balance, renal control of, 80–82, 81b–82b Extracellular fluid compartments, 80 Extracellular fluid osmolarity, renal control of, 82–84, 82b–84b, 84t Extracellular fluid volume, lowering, diuretics in, 85 Extramedullary hematopoiesis, 288 Extraocular muscles, 441t Extrapyramidal side effects of high-potency typical antipsychotics, 378 of low-potency typical antipsychotics, 378 of typical antipsychotics, 379 Extravascular hemolysis, 287 Extrinsic allergic alveolitis, 31 Extrinsic asthma, 29 Exudates, 138–139, 139t, 139b Exudative effusions, 668 Eye muscles, 440b F Fabry disease, 261t Face, embryology of, 616–621, 616t Facial swelling, 357 Factor IX, 320 Factor VIII, 320 deficiency in, 320 False negative, 571t False positive, 571t, 582 Familial adenomatous polyposis (FAP), 247, 649, 650f Familial cancer syndromes, 247 Familial hypercholesterolemia (FH), 21, 276 heterozygous/homozygous, 276 summary, 277b treatment of, 276–277 Familial hypocalciuric hypercalcemia, 191, 191b–192b Famotidine, for gastroesophageal reflux disease, 112t FAP. see Familial adenomatous polyposis (FAP) Fasciculations, 400b, 406 Fasciculus cuneatus, 401, 402f–403f Fasciculus gracilis, 401, 402f–403f Fasciitis, necrotizing, 634, 634f, 635b Fats, 106, 106t Fatty acid classifications, 268 metabolism of, 268f synthesis and breakdown of, by glucagon, 265–266, 266f Febuxostat, for gout, 453 Feedback loops, 160, 161f Felty syndrome, 451 Femoral triangle, 589, 589f Fenestration, 77 Ferritin, 639 polymyalgia rheumatica and, 455 Ferroportin, 651 Ferruginous bodies, 39f, 40 Festinating gait, 407 Fetal alcohol syndrome, 201, 202f Fetal amniotic fluid, 25 Fetal circulation, course of, 597f Fetal macrosomia, 188 Fetor hepaticus, 137 FEV1/FVC ratio, 28b asthma and, 28 chronic bronchitis and, 37 chronic obstructive pulmonary disease and, 29

688  INDEX Fever, rheumatic, 374 FHF. see Fulminant hepatic failure (FHF) Fibrillation potentials, 406 Fibrillations, 406 Fibrillin, 272–273 Fibrin, 314 Fibrin clot, coagulation cascade and, 316, 316f, 316b Fibrinous pericarditis, 620 Fibroadenoma, 218, 231 Fibroblast growth factor receptor gene 3 (FGFR3), 166 Fibrocystic breast disease, 218 Fibroids, 647–648 uterine, 212, 213f Fibromuscular dysplasia, in renovascular hypertension, 81 Fibromyalgia, 454–455, 454f, 454b–455b Fibrosis, 651–652, 652f Fibular (peroneal) nerve, common, 610 Fick’s law of diffusion, 27 Fifth disease, 633, 633f, 675 Filamentous mold, 524 Filtered bicarbonate, reabsorbing, by kidneys, 79, 79f, 79b, 91 Filtered sodium, reabsorbing, under normal conditions, 85, 85t Filtration forces, 49, 50f, 52f Filtration fraction (FF), 76 regulation of, 51, 52f, 52t First-order kinetics, 537 First-pass metabolism, 535 First-trimester bleeding, differential diagnosis for, 203 Fistulas, Crohn’s disease with, 657–658 Fitz-Hugh–Curtis syndrome, 209, 617 FLAT P(i)G mnemonic, 160 Flatworms, 523t Flow-through system, 53f Fluid retention, 16 Fluids and electrolytes, 75–88, 75b acid-base balance in, renal control of, 78–79, 79f–80f, 79b diuretics in, pharmacology of, 85–88, 85t, 86f, 87b–88b extracellular fluid balance in, renal control of, 80–82, 81b–82b extracellular fluid osmolarity in, renal control of, 82–84, 82b–84b, 84t renal filtration and transport processes in, 75–78, 75f, 76b, 77f, 78b Flukes, 523t Flumazenil, 547 Fluorescent treponemal antibody absorption (FTA-ABS) test, 509, 509t, 672 Fluoroquinolones, 491, 494t–497t Fluphenazine, extrapyramidal side effects of, 378 Fluticasone, 30t FMR1 gene, 277 Foam cells (P), 660–661 Foamy histiocytes, 259 Focal segmental glomerular sclerosis (FSGS), 65 Folate, role of, 302f Folate deficiency, 304t, 416–417 causes of, 303 hypersegmented neutrophil, marker for, 300f, 301b summary, 304b total RBS folate level, 304b vitamin B12 deficiency and, 301 Folic acid supplements, 202 Follicle-stimulating hormone (FSH), 162f, 163t–164t, 196 Follicular lymphoma, 333t–334t Follicular phase, of menstrual cycle, 196 Fomepizole, 103, 103b, 140 Foot drop, 610 Foramen ovale (FO), 597–598, 597f–598f Forced expiratory volume, 28 Forced vital capacity (FVC), 28 pulmonary fibrosis in, 40 Foregut, anatomy of, 109, 110f Formal operations, 389 Formication, 392

Fractional excretion, of sodium, 57–58 Fragile X syndrome, 277 pathogenesis of, 277 summary, 279b Francisella tularensis, 489t Frank-Starling mechanism, 1 Freud, Sigmund, 389 Fulminant hepatic failure (FHF), 151, 151b–152b Functional residual capacity, in chronic obstructive pulmonary disease, 24, 24f Functional ventilation, 25–26 Fungal diseases, 520–534 basic concepts in, 524–534 Fungal infections clinical manifestations of, 525t map of, 524b, 525f Fungi dimorphic, 524 morphologic types of, 524, 524b Fungus balls, 524 Furosemide, 86 Fusion inhibitors, 368t G G protein–coupled receptors, 157, 158f–159f, 158t GABA. see γ-Aminobutyric acid (GABA) Galactokinase deficiency, galactosemia comparison to, 272 Galactorrhea, 163b, 164 Galactose-1-phosphate uridyltransferase deficiency, jaundice and, 271 Galactosemia, 271 galactokinase deficiency comparison to, 272 infants develop cataracts, 271–272 pathophysiology of, 271f summary, 272b symptoms of, 272b treatment of, 272 Gallbladder, cholecystokinin and, 108t Gallstones, 153 acute pancreatitis and, 122 sickle cell anemia and, 291 Gamma-glutamyl transferase (GGT), 136t, 142 Ganglionic blockers, 552 Gangrene, 480–481 Gardner syndrome, 649 Gardnerella vaginalis, 672 Gas diffusion, 27 Gas exchange, basic concepts of, 27–41, 28b Gastric cancer, 229 biopsy of, 229 suspicious for, 230 Gastric carcinoma, 116–117, 121 diffuse type of, 229f Gastric emptying, cholecystokinin and, 108t Gastric outlet obstruction, 116 Gastric reflux, 460 Gastric secretions, loss of, pathophysiology of, 100 Gastric ulcers, 116–117, 117f, 118b–119b Gastrin, 108t Gastritis acute, 119–120, 119f, 119b, 121b chronic, 121, 121b Gastroenterology, 106–132, 106b basic concepts of, 106–132 case studies of achalasia, 113b–115b, 114f, 115 acute/chronic gastritis, 119–121, 119f, 119b–121b acute/chronic pancreatitis, 121b–123b, 122f appendicitis, 130–131, 130f, 130b–131b celiac disease, 125b, 126–127, 126f, 127b

INDEX  689 Gastroenterology (Continued) gastroesophageal reflux disease, 110–112, 110b–113b, 111f, 112t hypertrophic pyloric stenosis, 123b–124b, 124, 124f inflammatory bowel disease, 127b, 129–130, 129f, 129b–130b peptic ulcer disease, 115b–119b, 116–118, 117f small bowel obstruction, 132b Gastroesophageal reflux disease (GERD), 110–112, 110b–113b, 111f, 112t, 648 Gastrointestinal bleeding, 116b, 136, 136b differential diagnosis of, 228 Gaucher cells, 260f Gaucher disease, 260, 261t Generalized seizures, 429, 429t Genetic and metabolic disease, 253–283 basic concepts, 253–257 case studies cystic fibrosis, 258b, 274b–275b Down syndrome, 279b familial hypercholesterolemia, 275t, 275b fragile X syndrome, 277b galactosemia, 271f, 271b glycogen storage diseases, 263b, 267t hypervitaminosis A, 281b Lesch-Nyhan syndrome, 262b Marfan syndrome, 272b medium-chain fatty acyl-CoA dehydrogenase, 268f, 268b, 269t ornithine transcarbamoylase deficiency, 282b phenylketonuria, 255b porphyrias, 269b Tay-Sachs disease, 259f, 259b insider’s guide, 253b Genetic anticipation fragile X syndrome, 277–278, 278f Huntington disease, 278–279 Genetic probabilities, 258b Genitalia, development of, 198t Genitofemoral nerve, 605 Genomic imprinting, 281 GERD. see Gastroesophageal reflux disease (GERD) Germ cell neoplasms, 251 Germ cell tumors, 215 German measles, 533t Gestational age, 200–201, 204 Gestational diabetes, 187t, 187b–188b, 188 Gestational trophoblastic diseases (GTDs), 204, 204t GFR. see Glomerular filtration rate (GFR) Ghon complex, 513 Gi receptors, 158–160, 159f, 160b Giant cell (temporal) arteritis, 474, 475t, 476 polymyalgia rheumatica and, 455 Giardia, 503t Giardia lamblia, 523t Gigantism, 166 Gilbert syndrome, 144, 145t Glanzmann’s thrombasthenia, 324, 324b Glaucoma, 442, 442b–443b Gleason grading system, 238 Glenohumeral joint, 595, 595f Gliadin, 650–651 celiac disease and, 126 Glial fibrillary acidic protein (GFAP), 243 Glioblastoma multiforme, 242, 433 Glioblastomas, 242 Glipizide, 186 Globoid cell leukodystrophy, 261t Glomerular basement membranes (GBMs), 659 Glomerular capillary oncotic pressure, in filtration fraction, 76 Glomerular crescents, 69–70, 70f Glomerular epithelial cells, of glomerular filter, 77 Glomerular filter, three layers of, 77, 78b

Glomerular filtration rate (GFR) angiotensin receptor blockers in, 76, 78b angiotensin-converting enzyme inhibitors in, 76, 78b filtration forces and, 49 forces governing, at level of glomerulus, 75, 75f regulation of, 51, 52f, 52t renal blood flow and, 76, 76b renal clearance and, 49, 50f Glomerulonephritis, 357 differential diagnosis for, 71t poststreptococcal, 654–655, 655f rapidly progressive, 659 Glomerulotubular balance, 81 Glomerulus, 654–655 level of, glomerular filtration rate at, forces governing, 75, 75f renal filtration at, 77, 77f GLP-1 analogs, 186 Glucagon, 555 fatty acid synthesis and breakdown by, 265–266, 266f Glucocorticoids, 36t, 167 conditions associated with, 99 for gout, 453 for systemic lupus erythematosus, 464 Glucose intolerance, 234 Glucose-6-phosphate dehydrogenase (G6PD) deficiency, 307, 309b peripheral blood smear, 308f summary, 309b treatment, 309 Glucose-dependent insulinotropic peptide (GIP), 108t α-Glucosidase, 186 Glutamine deamination, 91 Glutathione, 550 Gluteal nerves, injury to, 612, 612b Gluten, 650–651 Glyburide, 186 Glycemic control, 184 Glycogen degradation, 264, 264f–265f regulation of, 265, 265f synthesis, 264, 264f Glycogen storage disease enzymatic deficiency, 263–264 metabolic pathways in, 266t type I, 263, 267t, 267b type II, 267, 267t type III, 267t type V, 267t, 267b Glycosaminoglycans (GAGS), 177 Glycosides, 543, 544b Gonadotropin-releasing hormone (GnRH), 196 Gonadotropin-releasing hormone (GnRH) agonists, 211–212 Goodpasture syndrome, 69–70, 70b, 659, 660f Gottron papules, 466, 466f Gout, 451b–454b, 452–453, 452f–453f, 472t–473t, 658–659, 658f Gouty interstitial nephropathy, 452 Gower sign, for muscular dystrophies, 468, 468f Gq receptors, 158–160, 159f, 160b Graft-versus-host (GVH) disease, 373, 373b–374b pathogenesis of, 373 skin and intestinal tract, 373 Gram stain, 485, 485t–486t Grand mal seizures, 428 Granulocytes, origins of, 328 Granuloma, 362 contents of, 363 formation of, 363, 363f noncaseating, 657–658, 658f Granulomatosis, 69, 70b with polyangiitis, 474, 475t Granulosa cell tumor, 217, 251

690  INDEX Grapefruit juice, cytochrome P-450 enzymes and, 118t Graves disease, 173, 174t, 177f–178f, 177, 177t, 177b, 179b, 180t–181t Grey Turner sign, 668 Grief, stages of, 386 Group atrophy, 406 Group B streptococci, 500t–501t Growth hormone (GH), 163t, 166, 166b, 167f Gs receptors, 158–160, 159f, 160b Guillain-Barré syndrome, 419 Gut wall, anatomy of, 106, 107f Gynecologic infections, 209–219 Gynecomastia, 137, 137b, 222, 651–652 peptic ulcer disease and, 118 H Helicobacter pylori acute/chronic gastritis and, 119, 121b peptic ulcer disease and, 117–118, 118b–119b HAART. see Highly active antiretroviral therapy (HAART) Haemophilus ducreyi, 508b Haemophilus influenzae, 489t Hairy cell leukemia, 328f, 328b, 330, 330f, 334t origin of, 334t–335t Half-life, 537 Hallucinations, schizophrenia and, 376 Haloperidol, 407–408, 410 extrapyramidal side effects of, 378 Hamartomas, of CNS, 434 Hand innervation of, 594b osteoarthritis and, 446f Hand-Schüller-Christian disease, 332 Haptoglobin, 288 G6PD and, 308 Hardy-Weinberg equilibrium, 253b Hardy-Weinberg law, 258 Hashimoto (autoimmune) thyroiditis, 174t, 179–180, 179b, 654, 655f Hawthorne effect, overview of, 578t–580t HbAS, 291 HbF, β-thalassemia major and, 292–293 Headache, autosomal dominant polycystic kidney disease and, 63 Heart failure, 15, 17b case study on, 15b etiology of, 16 Heberden nodes, 446f Heinz bodies, 307f G6PD and, 307 Helicobacter pylori, 488t gastric adenocarcinoma and, 229 HELLP syndrome, 206–207 Helminths, 523t Helper T cells, 346, 350t–351t Hematemesis, 116, 137 Hematocele, 608 Hematologic malignancies, 328–344 age ranges for, 328b basic concepts of, 328–337 case studies for acute lymphoproliferative leukemia, 338b–339b acute myelogenous leukemia, 343b–344b, 344f chronic myelogenous leukemia, 339, 339b–340b Hodgkin’s lymphoma, 340b–342b, 341f, 341t multiple myeloma, 335f–336f, 335b–337b polycythemia vera, 342, 342b–343b from lymphoid lineage, 329, 334t–335t from myeloid lineage, 329, 334t–335t suspicion of, 329 Hematology, 322b Hematoma, 143, 143b epidural, 664, 665f

Hematopoiesis, 345, 345f extramedullary, 288 Hematospermia, 238 Hematoxylin-eosin, staining, 406 Hematuria, 514 Heme biosynthesis pathway enzymatic defects in, 270f lead poisoning affect, 270 Hemochromatosis, 651, 651f pseudogout and, 453 β-thalassemia major and, 293, 294t Hemodynamics, basic concepts of, 1–4, 2f, 3t, 4f–5f Hemoglobin, 40, 41f, 284 Hemoglobin A1c, 184 Hemoglobin C (HbC), 291 Hemoglobin dissociation curve, 42, 42f Hemoglobin S (HbS), hemolysis/microvascular occlusions, 290 Hemoglobin S sickle cell (HbSS), 289f Hemoglobinemia, intravascular hemolysis and, 287–288 Hemoglobinuria G6PD and, 308–309 intravascular hemolysis and, 287–288 Hemolysis, 356 characteristics of, 288t Hemolytic anemia, 355 Coombs tests and, 306 Hemolytic disease of newborn, 310 ABO incompatibility, 311 summary, 311b Hemolytic jaundice, 134t, 135 Hemolytic uremic syndrome, 322t, 323b pentad of thrombotic thrombocytopenic purpura and, 322 Hemophilia, 320, 320f, 320b–321b measure of coagulation in, 320 von Willebrand disease and, 323, 323b Hemoptysis causes of, 235 lung cancer and, 235 Hemorrhage epidural, 420 intracerebral, 430 intracranial hemorrhage, 420 peptic ulcer disease and, 116, 118b–119b postpartum, 208 splinter, 480–481 subarachnoid, 420, 423, 423b, 424f subdural, 420 Hemorrhagic stroke, 420 Hemosiderin, 296f Hemostasis, 314 endothelial injury and, 314 primary and secondary, 314–315, 315f, 315t Henderson-Hasselbalch equation, 536–537 Henoch-Schönlein purpura (HSP), 67, 68b, 475, 475t Hepadnaviridae, 521t Heparin, 326 mechanism of action of, and bleeding disorders, 318, 319t Heparin-induced thrombocytopenia (HIT), 326, 326b Hepatic cirrhosis, 142 Hepatic encephalopathy, 140b Hepatic lobule, anatomy of, 133, 133f Hepatic steatosis, 651, 652f Hepatitis, 527b alcoholic, 137, 140b case study on, 526b viral, 527 Hepatitis A, 142, 142t, 143b, 527, 527t Hepatitis B, 146b, 527t antigens in, 526 characteristics of, 142t, 142b

INDEX  691 Hepatitis B (Continued) chronic, 147–155, 148t, 149b diagram of, 147f hepatocellular carcinoma and, 149 polyarteritis nodosa and, 480, 480b serologic markers in, 526t Hepatitis B e antigen (HBeAg), 147, 147f, 149b Hepatitis C, 142t, 527, 527t Hepatitis D, 142t, 527, 527t Hepatitis E, 142t, 527, 527t Hepatitis serology, 138 Hepatitis viruses causes of, 149t characteristics of, 142t, 143b vaccine for, 143 Hepatocellular carcinoma, 149b–150b, 150 viral hepatitis and, 527 Hepatocellular jaundice, 134t, 135 Hepatocytes, 133, 133f Hepatology, 133–155, 133b basic concepts of, 133–134, 133f, 134t–135t, 134b case studies fulminant hepatic failure, 151, 151b–152b hepatitis A, acute, 141b–143b, 142, 142t hepatitis B, chronic, 147–155, 148t, 149b hepatocellular carcinoma, 150, 150b physiologic jaundice of the newborn, 143, 143b–144b Wilson’s disease in, 140, 140f, 140b Hepatomegaly, 265 Hepatosplenomegaly, β-thalassemia major and, 293 Hepcidin, 296, 651 Hereditary nonpolyposis colorectal cancer (HNPCC), 233 Hereditary nonpolyposis colorectal cancer (HNPCC)–Lynch II syndrome, 215 Hereditary spherocytosis (HS), 643–644, 643f Hermaphroditism, defined, 197–198 Hernias, 603–604, 604f, 604b, 606b Herniation, 602, 602f, 602b–603b Heroin, effect of fetal exposure, 202t Herpes simplex virus 1 (HSV-1), 522b Herpesviridae, 521t Hexose monophosphate, G6PD and, 307–308 HFE gene, 651 HGPRT. see Hypoxanthine-guanine phosphoribosyltransferase (HGPRT) Hiatal hernia, sliding, 111, 111f, 113b High-density lipoprotein (HDL), 21 Highly active antiretroviral therapy (HAART), 366b, 367 suboptimal compliance with, 367 High-output heart failure, 17 Hindgut, anatomy of, 109, 110f Hippocampus, 430 Hirschsprung disease, 128, 129f Hirsutism, 168, 170b Histiocytoses, 332, 332f origin of, 334t–335t Histoplasma capsulatum, 524, 525t Histrionic personality disorder, 397, 397b HIT. see Heparin-induced thrombocytopenia (HIT) HIV. see Human immunodeficiency virus (HIV) Hives, 357 HLA-B27 allele, 458 HMG-CoA reductase, for low-density lipoprotein, 277 HNPCC. see Hereditary nonpolyposis colorectal cancer (HNPCC) Hoarseness, 236 Hodgkin lymphoma, 332, 334t, 644, 644f case study for, 340b–342b, 341f, 341t origin of, 334t–335t vs. non-Hodgkin lymphoma, 341–342 Homer-Wright rosettes, 172 Homunculus, 421f

Hormonal contraceptives, 198, 217 case studies, 198b Hormones, gastrointestinal, 108, 108t Horner syndrome, 236, 436 Howell-Jolly bodies, 640 HPV. see Human papillomavirus (HPV) HS. see Hereditary spherocytosis (HS) 11β-HSD deficiency, pathophysiology of, 100 HSP. see Henoch-Schönlein purpura (HSP) β-Human chorionic gonadotropin (β-hCG) amenorrhea and, 200 molar pregnancy and, 204 normal function of, 200, 201f in normal vs. ectopic pregnancy, 200 pregnancy and, 200 Human epidermal growth factor receptor-2 (HER2), 219 Human immunodeficiency virus (HIV), 146b, 366–367 life cycle of, 366–367, 366b, 367f Human immunodeficiency virus (HIV) infection case study for, 366, 366b–367b, 369b drugs in treatment for, 368t major stages of, 367, 368f opportunistic pathogens and malignancies in, 369t progression of, determinants of, 369 Human leukocyte antigens, 371 Human papillomavirus (HPV), 239 associated oncogenes, 241 vaccination, 239 Humerus fracture, 595, 595t, 595b Humoral immunity, 346–347 for tuberculosis, 514 Hunter syndrome, 261t Huntington disease, 279b, 410, 410f, 411b genetic anticipation, 278–279 Hurler syndrome, 261t Hürthle cell metaplasia, 180b Hürthle cells, 654 Hyaline arteriolar nephrosclerosis. see Hypertensive nephrosclerosis Hyaline arteriosclerosis, 59, 60f Hyaline membranes, 43 Hyaluronic acid, for osteoarthritis, 667 Hydatidiform mole, 204, 204b Hydralazine, 9t–10t Hydrocele, 608 Hydrocephalus, 424, 426b Hydrochloric acid, 108t, 112t Hydrochlorothiazide, 9t–10t, 86 Hydrogen ion bicarbonate and, 91 extracellular concentration of, 89, 89f Hydrogen peroxide, 362 Hydronephrosis, 55–56, 56f, 71, 669 Hydrops fetalis, 294 Hydrostatic pressure, 75, 75f, 78b glomerular, 76 Hydroxyapatite crystal deposition, 453 γ-Hydroxybutyrate (GHB), 546 5-Hydroxyindoleacetic acid (5-HIAA), 131 Hydroxyurea, 291 Hygroma, cystic, 221 Hyper IgE syndrome, 360t–361t Hyperacusis, 428 Hyperaldosteronism, 168 Hyperandrogenism, 214 Hyperbilirubinemia, 134, 134t, 134b, 139, 142–143 G6PD and, 308 Hypercalcemia in anion gap, 92 causes of, 72, 72b multiple myeloma and, 336

692  INDEX Hypercalcemia (Continued) as paraneoplastic syndrome, 248t renal calculi and, 72 in squamous cell carcinoma, 45 Hypercapnia, 35 Hypercholesterolemia, familial, 21 Hypergammaglobulinemia, 336 Hypergastrinemia, 111–112 Hyperglycemia, 166, 168, 185 Hyperkalemia in anion gap, 92 hypertensive nephrosclerosis and, 60 Hyperlipidemia, in nephrotic syndrome, 65 Hypernatremia, 83 in anion gap, 92b, 93 Hyperparathyroidism hypercalcemia and, 72 hypocalcemia and, 61, 61b pseudogout and, 453 renal failure and, 62 thyroid cancer and, 244 Hyperprolactinemia, 163t, 164–165, 165b Hyperreactivity, of tracheobronchial tree, 29 Hypersegmented neutrophils, 120, 120f, 120b, 300f, 301b megaloblastic changes of, 297f Hypersensitivity pneumonitis, 31, 31b–32b Hypersensitivity reactions, 352–353, 353t Hypersplenism, 326 Hypertension, 3t acid-base disturbance and, 99, 99b case study on, 7b diabetes mellitus in, 8b–9b complication of, 7 Cushing syndrome and, 170b definition of, 7, 8t diffuse scleroderma and, 461 ischemic stroke and, 665–666 pharmacotherapy for, 10b pheochromocytoma and, 171 polyarteritis nodosa and, 480–481 portal, 135, 135t preeclampsia, eclampsia and, 206–207 pulmonary, 46 renovascular, 81, 81b stroke and, 420 subarachnoid hemorrhage and, 423 treatment of, 8t types of, 7 Hypertensive nephrosclerosis, 59 Hyperthyroidism, 173, 173b, 174t, 176f–177f, 177t, 179b, 180t gestational, 205 pregnant state predispose to, 205 Hypertriglyceridemia, 21, 265–266 Hypertrophic pyloric stenosis, 123b–124b, 124, 124f Hyperuricemia, 658–659 cause of, 266 gout and, 452 pharmacologic treatment, 263 purine salvage pathway and, 263 Hyperventilation, in pulmonary embolism, 98 Hypervitaminosis A, 282 summary, 282b Hypoalbuminemia, 84, 336 Hypocalcemia, 181, 191, 191b in chronic renal failure, 60, 60b Hypochromic red blood cells, 639, 639f Hypodermis, 623t Hypoglycemia, 188, 188b–189b, 265 beta-blockers for, 8 hyperuricemia and, 266

Hypoglycemia (Continued) liver failure and, 139t muscle glycogen phosphorylase and, 267 thiazide diuretic for, 8–9 Hypokalemia, 8, 86, 169 pathophysiology of, 100 Hyponatremia adrenal insufficiency and, 170 cause of, 84b in central nervous system symptoms, 83 in diuretics, 87 Hypoperfusion, 57 Hypophosphatemia, 244 Hyporeflexia, 414 Hypotension, 3t, 422, 668 orthostatic, 378 Hypothalamic dopamine secretion, 162 Hypothalamic releasing hormone, 162, 163t Hypothalamic-pituitary-adrenal axis, 169f Hypothalamic-pituitary-end organ (HPO) axes, 160, 161f Hypothyroidism, 170b, 179–180, 179b, 180t, 182t, 182b depression and, 383 Hashimoto thyroiditis and, 654 lithium and, 381 panic attack and, 394 Hypovolemic hyponatremia, 83 Hypoxanthine-guanine phosphoribosyltransferase (HGPRT), 262, 452 Hypoxemia, 44, 44t Hypoxia, in pulmonary embolism, 98 Hypoxia-induced vasoconstriction, for pneumothorax, 47 Hysterectomy, 212–213 I IBD. see Inflammatory bowel disease (IBD) Id, 389 IDA. see Iron deficiency anemia (IDA) Identification, 385 Idiopathic inflammatory myopathies, 466 If Sodium channel, 6 IFN-γ receptor deficiency, 360t–361t IgA deficiency, 360, 360t–361t IgG autoantibodies, 652–653 bullous pemphigoid with, 628 IL-12 receptor deficiency, 360t–361t Ilioinguinal nerve, 605 Imipramine, for bed wetting, 384 Immature defenses, 386 Immotile cilia syndrome. see Primary ciliary dyskinesia (PCD) Immune complex deposition, 357 Immune complex-mediated glomerulonephritis, 70 Immune thrombocytopenic purpura, 325, 325f, 325b–326b etiology of, 325 pregnancy and, 326 treatment for, 325b, 326 Immunoglobulin A (IgA) antibodies, 650–651 Immunoglobulin A (IgA) nephropathy, 70, 70b Immunoglobulin G (IgG) autoantibodies, 325 Immunoglobulins five classes of, 347–348, 348t gene orders of, 348, 348b structure of, 347f T cell–independent response, 347–348 Immunology, 345–375, 345b basic concepts of, 345–354 case studies for anaphylactic shock, 352, 352b, 355b autoimmune hemolytic anemia, 355, 355b–356b chronic granulomatous disease, 360t–361t, 362, 362f–363f, 362b–364b contact dermatitis, 357, 357b–358b

INDEX  693 Immunology (Continued) DiGeorge syndrome, 364, 364b–365b, 365f graft-versus-host disease, 373, 373b–374b human immunodeficiency virus infection, 366, 366b–367b, 367f–368f, 369b reactive arthritis, 374, 374b–375b serum sickness, 356b–357b severe combined immunodeficiency, 359, 359b, 361b transplant rejection, 370, 370b–372b, 372t Impetigo, 632, 633f In situ carcinoma, 232 Incidence, 575, 572, 572t Inclusion body myositis, 466 India ink stain, 484 Indirect bilirubin, 308 Indirect inguinal hernia, 603–604, 603b, 604f, 606b Infertility, 648 Infiltrating (invasive) ductal carcinoma, 219 Inflammatory bowel disease (IBD), 127b, 129–130, 129f, 129b–130b, 247 Inflammatory myopathies, 465b–467b, 466f idiopathic, 466 Influenza, 531, 531b Informed consent, 559 Inguinal canal, anatomy of, 605 Inguinal hernias, 603–604, 604f, 606b Inheritance patterns, 305b Inheritance risk, 253b Inhibin, 196 Initiation phase, of acute tubular necrosis, 57 Innate immune system, 328, 345–346, 346t Inspiratory airflow driving forces for, 23 forces of resistance for, 23–24 Insulin, 21 in diabetic ketoacidosis, 102 renal clearance and, 49 Insulin-dependent glucose transporter, 185 Insulin-induced hypoglycemia, 185 Insulin-like growth factors, 167f Insulinoma, 184, 188–189, 189b Integrase inhibitors, 368t Intellectual disability, 388 Intention tremor, 407 “Intention-to-treat” (ITT) approach, 586 Intercostal space, chest tube in, 47 Interferon alpha, 148, 149b Interferon beta, 419 Interleukin 1 (IL-1), 483 Interleukin receptors, and severe combined immunodeficiency, 359 Internal respiration, 42 Internuclear ophthalmoplegia, 418 Interstitial osmolality, 53 Intervertebral disk herniation, 602–603, 602b–603b Interviewer bias, 578 Intestinal bleed, anemia and, 298 Intestinal metaplasia, in esophagus, 648 Intestinal tract, graft-versus-host (GVH) disease and, 373 Intracellular fluid compartment, extracellular fluid compartments and, 80 Intracerebral hemorrhage, 430 Intracranial (berry) aneurysms, 63, 63b Intracranial bleeding, 420, 426 Intracranial hemorrhage, 420 Intraductal papilloma, 230, 218 Intrapleural pressure, in types of airflows, 23 Intrauterine device, 209 Intravascular hemolysis, 287 hemoglobinemia and, 287–288 hemoglobinuria and, 287–288 Intravenous hydration, in diabetic ketoacidosis, 102

Intravenous immunoglobulin (IVIG), for immune thrombocytopenic purpura, 326 Intrinsic factor, 108t Inulin, 49, 51f Invasive moles, 204t Iodine, hyperthyroidism and, 174t Ionized calcium level, pneumothorax and, 47 IP3 receptor, classes of, 158t Ipratropium, 30t, 36t, 551–552 Ipsilateral blindness, 438f Iron, dietary absorption, 296–297 circulation, 296 Iron deficiency anemia (IDA), 295, 639, 639f case study, 40, 40b–41b, 43b cause of, 295–296, 298 chronic disease, anemia of, and, 298t, 300b Plummer-Vinson syndrome and, 296 significance of iron study results, 297–298 summary, 298b Ischemia, 392, 662–663, 663f Takayasu arteritis and, 482 Ischemic bowel, perforation of, 480–481 Ischemic stroke, 420, 665–666, 666f Isometric (isovolumic) contraction, 19 Isoniazid, 494t–497t, 513–514 cytochrome P-450 enzymes and, 118t Isotonic contraction, 19 Isotretinoin, 282 Ixodes ticks, 510–511, 511b, 512f J Janeway lesions, 655–657, 656f Jarisch-Herxheimer reaction, 510 Jaundice, 651–652 causes of, 134, 134t, 134b in neonate, 310 Clonorchis sinensis and, infection with, 154 diagnosing common causes of, 134–155 galactose-1-phosphate uridyltransferase deficiency and, 271 kernicterus and, 310 neoplasm causing, 153 in newborn, 143, 143b–144b physiologic, 310 Jaw claudication, 476 Job syndrome, 360t–361t Jod-Basedow effect, 174t Justice, 559 K K+-sparing diuretics, 86 Kallmann syndrome, 221, 221b Kawasaki disease, 474, 477f, 477b–479b, 478, 539, 631–632, 632f, 632b Kayser-Fleischer rings, 140–141, 140f, 141b, 411b Keratin pearls, 645, 645f Kernicterus, 144 jaundice and, 310 Kernig sign, 529 Ketamine, 542 Ketoacidosis, diabetic, 184 Ketoaciduria, branched chain, 283, 283b Ketoconazole, 524 Kidney stones, 669 Kidneys in acid-base balance, 78, 90–91, 90t blood supply of, 49f diffuse scleroderma and, 461 in extracellular fluid volume, 80 gross anatomic features of, 48, 48f major functions of, 48 reabsorbing filtered bicarbonate, 79, 79f, 79b, 91

694  INDEX Kimmelstein-Wilson lesion, 187, 187f Klebsiella spp., 500t–501t Klinefelter syndrome, 222, 222b Klumpke’s paralysis, 592 Knee, 608–609, 609f arthritis and, 447f osteoarthritis and, 446f Knee injuries, 608, 608b, 609f, 610b Knudson’s “two-hit hypothesis,” 224 Koilocytes, 241, 241f Koplik spots, 631, 632f Korsakoff psychosis, 431 Krabbe disease, 261t KRAS gene, pancreatic cancer and, 234 Krukenberg tumor, 116–117, 214, 251 L Labetalol, 9t–10t Labor and delivery, 207–209, 208b, 211b, 215b β-lactam antibiotics, 486 β-lactamase, 486 Lactation, 205 Lactic acidosis, 265 Lactobacillus acidophilus, 209 Lambert-Eaton syndrome, 45, 248t, 413, 646 Langerhans cell histiocytosis, 332, 332f Lansoprazole, for gastroesophageal reflux disease, 112t Lanugo, 395 Laparoscopic cholecystectomy, 153b Laplace’s law, 25 Large cell carcinoma, 235 Laryngeal nerve, recurrent, 613, 613b, 614f, 617b Late-look bias, 578t–580t Latent tuberculosis, 513 Lateral compartment of leg, 611f Lateral geniculate body, 438f, 439b Lateral geniculate nucleus (LGN), 437 Latex agglutination assay, 484 Law of Laplace, 2, 16 l-dopa, 410 Lead, 313b Lead lines, 312–313, 312f Lead poisoning, 311 mechanism of, 312 signs of, 313b source of, 311–312 summary, 313b treatment, 313 Lead-time bias, 578t–580t Learning disorder, 388 Lectin complement pathway, 349f Left anterior descending (LAD) artery, 619, 662–663 Left eye, pupillary response to light and, 439 Left marginal artery, 619 Left oculomotor nerve, pupillary response and, 440 Left superior homonymous quadrantanopia, 439 Left ventricular end-diastolic volume, 5f Left ventricular hypertrophy (LVH), 3t Left-sided colon carcinomas, 245 Left-sided heart failure, 15 Legionella pneumophila, 489t Leiomyomata, 212–213, 214b, 647–648, 647f Leiomyosarcoma, 213, 647–648 Leishmania spp., 523t Leptospira interrogans, 490t LES. see Lower esophageal sphincter (LES) Lesch-Nyhan syndrome (LNS), 262, 452 pharmacologic treatment, 263 renal dysfunction and, 263 summary, 263b

Letterer-Siwe disease, 332 Leukemia, 228 acute Down syndrome and, 280 vs. chronic, 329–330 acute lymphoblastic, 334t age range for, 328b case study for, 338b–339b origin of, 334t–335t acute myelogenous, 334t age range for, 328b case study for, 343b–344b, 344f diagnosis of, 331b features of, in marrow biopsy, 344 M3 subtype, 330f, 330b origin of, 329, 334t–335t chronic lymphocytic age range for, 328b origin of, 334t–335t vs. chronic myelogenous, 340t chronic myelogenous, 334t age range for, 328b case study for, 339, 339b–340b pathogenesis of, 339 vs. chronic lymphocytic leukemia, 340t hairy cell, 328f, 328b, 330, 330f, 334t origin of, 334t–335t promyelocytic, 344 vs. lymphoma, 329, 329f Leukocyte adhesion deficiency, 360t–361t Leukocyte alkaline phosphatase (LAP), 332 Leukocytes, 328, 328f, 350, 350t–351t Leukocytosis, reactive, 332 Leukoerythroblastosis, 641 Leukotriene pathway inhibitors, 354t Leukotrienes, 357 pulmonary, 29 Leuprolide, 211–212 Levodopa, 408 Levothyroxine, 181 Lewy bodies, 409 Lewy body dementia (LBD), 393 Leydig cells, 197 LH. see Luteinizing hormone (LH) Libman-Sacks endocarditis, 464, 507 Licorice, 169 Ligamentum teres hepatica, 137 Ligand-gated ion channel, 157–158, 158f, 158t Light’s criteria, 668 Likelihood ratios, 573, 582b Limb-girdle muscular dystrophy, 469, 469b Linagliptin, 186 Linezolid, 494t–497t Linitis plastica, 229 Lipase, 106 Lipid A, 483 Lipid-lowering fibrates/fibric acid derivatives, 22 Lipids, inside hepatocytes, 651–652, 652f Lipoid nephrosis. see Minimal change disease Lipoprotein lipase, 21 Lipoproteins, 20–21 Liposomal amphotericin B, 362b–364b Lipoxygenase, 350 Lipoxygenase (LOX) pathway, 539 Lisinopril, 9 Listeria monocytogenes, 487t Lithium, 179, 179b, 381 Livedo reticularis, 480–481 Liver, 109, 110b Liver biochemical tests, 136t, 136b

INDEX  695 Liver cirrhosis, 138b Liver failure, 137b–138b, 138, 139t, 151, 152b Liver function test, for acute hepatitis A, 136b, 142 Liver transplantation, 150b LNS. see Lesch-Nyhan syndrome (LNS) Loading dose, 537 Lobar pneumonia, 498, 499f Lobar (typical) pneumonia, 32b, 33, 33f, 33t Lobular carcinoma, 232 Lobular carcinoma in situ (LCIS), 232 Localized scleroderma, 461, 462t Locked-in syndrome, 83 Lomustine, 434 Long-term potentiation, 430 Loop diuretics, 9t–10t, 86 Losartan, 9t–10t, 17t Lou Gehrig disease, 405, 405f–406f, 407b Low HDL-C. see Low high-density lipoprotein cholesterol Low high-density lipoprotein cholesterol, 22 Low-density lipoprotein (LDL), 21–22, 276 HMG-CoA reductase for, 277 pathologic consequence of elevated, 276 Lower esophageal sphincter (LES), 107, 111, 113b, 115, 115b Lower extremity injuries, 613b Lower motor neuron, 400, 400b, 401f, 405 LSD, 392t Lucid interval, 426 Lumbar puncture (LP), 600, 603b Lumbar spine, cervical and, nerve roots from, 603f Lumbosacral disk herniation, 602f Lung cancer, 236b–237b case studies, 235b–237b classification of, 45 metastasize, 237 non–small cell, 235 paraneoplastic syndrome and, 248–249 Lungs in acid-base balance, 90, 90t respiratory compensation, 102 small cell carcinoma of, 646, 646f Lupus anticoagulant, 464 Lupus nephritis, 464 Luteal phase, of menstrual cycle, 195 Luteinizing hormone (LH), 163t disorders, in deficiency or excess of, 164t follicular phase of menstrual cycle and, 195f gestational hypertension and, 205 Klinefelter syndrome and, 222 polycystic ovary syndrome and, 214 Lyme disease, 428, 510, 512b case study on, 510f, 510b diagnosis of, 511 stages of, 511, 511t treatment for, 511 Lymph nodes, 605, 605b bone and pelvic, 238 enlarged, location, 340, 340b Lymphadenopathy, with infection vs. due to malignancy, 338 Lymphocyte-predominant Hodgkin, 644 Lymphocytic leukemias, 329 Lymphoid leukocyte, lineage of, 328 hematologic malignancy from, 329, 334t–335t Lymphoid system, 345 Lymphoma, 228, 329, 340, 340b Burkitt, 333t–334t classification of, 332–334, 333f cutaneous T-cell, 333t diffuse large B-cell, 333t follicular, 333t–334t Hodgkin, 332, 334t, 644, 644f

Lymphoma (Continued) case study for, 340b–342b, 341f, 341t origin of, 334t–335t vs. non-Hodgkin lymphoma, 341–342 lymphoplasmacytic, 337 mantle cell, 333t–334t marginal zone, 333t non-Hodgkin, 332 description of, 333t origin of, 334t–335t vs. Hodgkin lymphoma, 341–342 vs. leukemia, 329, 329f Lymphopenia, in severe combined immunodeficiency, 359 Lymphoplasmacytic lymphoma, 337 Lyonization, 254, 320 Lysergic acid diethylamide (LSD), 542 Lysosomal storage diseases, 261t Lysozyme, 362 M M protein, 337, 374 Macrocytic anemias, 286 acute/chronic gastritis and, 120, 120f differential diagnosis, 292 normocytic and, 287 vitamin B12 deficiency, 300 Macrocytosis neutrophils, megaloblastic changes of, 297f Macroglobulinemia, Waldenström, 334t–335t, 337 Macrolides, 492, 494t–497t cytochrome P-450 enzymes and, 118t Macrophages, 350t–351t atherosclerosis and, 660–661 cell surface markers of, 352t cell-mediated immunity and, 346 cytokines secreted by, 351–352 mechanisms for killing bacteria after phagocytosis, 362 Macula, 443 Macular degeneration, age-related, 443, 443b–444b Magnesium (Mg), 92 Magnesium (Mg2+) balance, diuretics in, 87 Magnesium sulfate toxicity, 207 Maintenance dose, 537 Maintenance phase, of acute tubular necrosis, 57 Major congenital metabolic disorders, 194t Major depressive disorder, 382–383, 382b, 385 Major histocompatibility complex, 371 molecules, 347 Malaria, 672–673, 673b Malignant mesothelioma, 235 Malignant tumor, 228 Mallory-Weiss tears, 137 Maltese crosses, 65, 65f, 674 Mammogram, 218 Mannitol, 85–86 Mantle cell lymphoma, 333t–334t Mantoux test, 514 Maple syrup urine disease, 283, 283b Marble bone disease, 470 Marcus Gunn pupil, 439b Marfan syndrome, 272, 664 causes of death, 273 summary, 273b Marginal zone lymphoma, 333t Marijuana, 542 intoxication, 392t Mast cell stabilizers, 354t Mast cells, 350t–351t, 353 Mature defenses, 386 Maturity-onset diabetes of youth (MODY), 183, 183f Maximum velocity, of enzyme, 538

696  INDEX McArdle disease. see Glycogen storage disease; type V McMurray test, 608 Mean, 575, 576f Mean arterial pressure (MAP), 1, 2f, 4 Measles, 533t, 631, 632f Mechanical ventilation, respiratory alkalosis in, 38 Meckel diverticulitis, 131 Medial collateral ligament (MCL), injuries to, 609–610, 609f Medial longitudinal fasciculus (MLF), 418, 440–441, 441f Median, 575, 576f Median nerve, 593, 593f–594f, 594t, 596b carpal tunnel syndrome and, 457, 457f Medical error, 568b Medium-chain fatty acyl-CoA dehydrogenase (MCAD) deficiency, 268 clinical findings, 268 pathways affected by, 269t summary, 269b treatment of, 268–269 Medullary carcinoma, 232 thyroid, 243 Medullary cystic disease, 64, 64t Medullary sponge kidney, 64 Medulloblastoma, 434 Megaloblastic anemia, 120, 120f, 416, 641 Meglitinides, 186 Meissner’s plexus, 106, 107f Melanocyte-stimulating hormone (MSH), 163t Melanoma, 248, 625b–626b, 626f Melena, 116, 118b–119b Membrane-spanning receptors, primary classes of, 157–158, 158f, 158t Membranous nephropathy, 67, 67f, 67b Meniere’s disease, 435 Meninges, layers of, 426, 600, 601f Meningiomas, 242, 433 Meningitis, 519b, 531b antibiotic therapy for, 519 aseptic, 530 vs. bacterial, 530 case studies, 518f, 518b, 529b–530b, 530f, 600, 600b, 601t causes of, 529t, 600, 600b, 601t by age, 518, 518t cerebrospinal fluid findings in, 530, 530t complications of, 518 cryptococcal, 530 diagnosis of, 529 in human immunodeficiency virus, 519 symptoms of, 518 Waterhouse-Friderichsen syndrome and, 171, 171b Meningocele, 602 Meningomyelocele, 602 Meniscus, lateral, 609–610 Menopause, 219 Menorrhagia, 295 Menstrual cycle, 195, 195f hormonal contraceptives and, 199 Mental retardation diagnosis, 388 hypothyroidism and, 182 Mental status abnormalities, 139–140 Merozoites, 672–673 Mesangium, 77 Mesenchymal cells, 228 Mesenteric aneurysms, 480–481 Mesenteric ischemia, G6PD and, 309 Mesocortical pathway, 379 Mesolimbic pathway, 379 Mesothelioma, 39 Metabolic acid-base disturbances, respiratory compensation for, 90t

Metabolic acidosis, 96, 96b–97b, 101b anion gap, 93, 93f, 104b MUDPILES mnemonic in, 93–94, 94b compensation for, 97 diarrhea in, 96 hypertensive nephrosclerosis and, 60 non-anion gap, 94, 94b in renal acid excretion, 78 respiratory compensation for, 90t type II to diarrhea, 97b Metabolic alkalosis, 100, 101b in anion gap, 102 anorexia nervosa and, 396 differential diagnosis of, 96f hypochloremic, hypertrophic pyloric stenosis and, 123 respiratory compensation for, 90t Metabolic syndrome, 20, 20b–22b Metachromatic leukodystrophy, 261t Metastasis in melanoma, 626 in testicular cancer, 605, 605b Metazoa, 522 Metformin (glucophage), 186 Methacholine, on FEV1/FVC ratio, 29 Methadone, 541 Methanol intoxication, 103, 103b Methanol poisoning, 140 Methemoglobinemia, in cyanosis, 41 Methimazole, 178 Methotrexate, 641 mechanism of action of, 373 Methyldopa, 9t–10t, 555 Methylmalonic acid level, 417 Methylprednisolone, 354, 371–372 Methylxanthines, 354t Metoclopramide, 407–408 for gastroesophageal reflux disease, 112t Metoprolol, 9t–10t, 17t Metronidazole, 494t–497t, 672 MgSO4, as tocolytic, 207 MI. see Myocardial infarction (MI) Micelle, 108, 109f Microangiopathic hemolytic anemia, 321, 641 Microcytic anemias, 286 celiac disease and, 126 iron deficiency, 295 lead poisoning, 311 b-thalassemia major, 292 Microcytic red blood cells, 639, 639f Microdeletion syndromes, 281 Microscopic polyangiitis (MPA), 475t Microscopic pyuria, 514 Microsporum, 525t Midbrain, visual fields and, 438f Middle cerebral artery (MCA) ischemic stroke, 665–666, 666f Midgut, anatomy of, 109, 110f Milwaukee shoulder syndrome, 453, 453b Mineralocorticoids, excess conditions associated with, 99 pathophysiology of, 100 Minimal change disease, 65, 66f Minute ventilation, 25 Mirtazapine, 396, 555 Misoprostol, 203 peptic ulcer disease and, 116 Missing WBCs mnemonic, 359b Mixed disorder, in acid-base balance, 97, 103b, 104 MM. see Multiple myeloma (MM) Modafinil, for narcolepsy, 561 Mode, 575, 576f

INDEX  697 Modification of diet in renal disease (MDRD) equation, 78 Mohs surgeon, 625 Molar pregnancy, 204 Molecular mimicry, 374 Molluscum contagiosum, 635, 635f Monoamine deficiency theory, 383 Monoamine oxidase, 409 Monoamine oxidase inhibitors, 383 Monobactam, 494t–497t Monoclonal antibodies, graft-versus-host disease and, 373 Monoclonal anti-IgE antibody, 354t Monoclonal gammopathy, 336 Bence Jones proteinuria and, 336 Monocytes, 328, 350t–351t Mononeuritis multiplex, 474 Monosodium urate (MSU) crystals, 658–659 Montelukast, 30t Morning periorbital edema, 54 Morphea, 461, 462t Motor neuron, 400, 400b, 401f Motor unit, 400 MPTP (1-methyl-4-phenyl-1,2,3,6-tetrahydropyridine), 409 MS. see Multiple sclerosis (MS) Mucopolysaccharidosis, 261 type I H, 261t type II, 261t Mucosa, anatomy of, 106, 107f Mucosa-associated lymphoid tissue (MALT), 345 Mucosal edema, 30 Mucous hypersecretion, 37 Mucus, stomach and, 108t MUDPILES mnemonic, in anion gap metabolic acidosis, 93–94, 94b Müllerian-inhibiting factor (MIF), 197 Multi-infarct dementia, 430 Multiple endocrine neoplasia syndrome, 244 Multiple endocrine neoplasia type I, 244 Multiple endocrine neoplasia type IIA (MEN IIA), 244 Multiple endocrine neoplasia type IIB, 244 Multiple myeloma (MM), 334t, 642, 642f case study for, 335f–336f, 335b–337b diagnosis of, 336 origin of, 334t–335t Multiple sclerosis (MS), 416, 418, 419b, 654, 654f Mumps, 428 Mupirocin, for impetigo, 632 Murmur in bacterial endocarditis, 505t causes of, 18, 20b myocardial infarction and, 14 Murphy’s sign, 152–153, 152b Muscarinic receptors, 552, 552t Muscle glycogen phosphorylase, hypoglycemia and, 267 Muscular dystrophies, 467b, 468f, 469–473, 469b Muscularis propria, anatomy of, 106, 107f Myasthenia gravis, 412, 412b–413b cholinergic drug for, 553, 553b Mycobacterium avium-intracellulare, 512 Mycobacterium leprae, 491t Mycobacterium tuberculosis, 491t, 512 infection, 670f–671f, 671 Mycolic acids, 513, 671 Mycology, 524–534 Mycophenolate mofetil (MMF), 372t Mycoplasma pneumoniae, 491t Mycoplasma spp., 500t–501t Myelinolysis, central pontine, 83 Myelodysplastic syndromes, 329 distinctive features of, 330–331 origin of, 334t–335t Myelofibrosis, 331–332, 332f, 332b, 641, 641f

Myeloid leukocyte lineage, 328 hematologic neoplasms from, 329, 334t Myeloma, multiple, 334t origin of, 334t–335t Myeloma gammopathy, in delta-delta protein gap, 92b Myelophthisic anemia, 470 Myelophthisis, 337 Myeloproliferative disorders, 329, 331 distinct features of, 331 myelofibrosis and, 331–332, 641 origin of, 334t–335t Myenteric plexus, 106, 107f, 115, 115b Myocardial hypertrophy, 16 Myocardial infarction (MI), 12, 662–663 abnormalities in, 14 case studies in, 12b–13b complications of, 14b serum tests for, 13, 13f stroke and, 420 Myocardial oxygen demand, 11, 19, 19f Myocardial oxygen supply, 11, 19, 19f Myocardial perfusion, 11 Myocyte action potential, ventricular, 5, 6f Myocytes, 19, 662–663 Myoglobin, 13, 13f Myomectomy, 213 Myopathies, idiopathic inflammatory, 466 Myosin adenosine triphosphatase staining, 406 Myotonic dystrophy, 469, 469b Myotonin protein kinase, 469 Myxedema, 177 Myxedema coma, 181, 181b N N-acetylcysteine, 151 acetaminophen and, 550 treatment for cystic fibrosis, 274 NADPH oxidase, 362 Nail bed hemorrhages, 505t Naloxone, 541 Naltrexone, 541 Narcissistic personality disorder, 396, 396b, 399b Narcolepsy, 560b, 561, 562b Nateglinide, 186 2001 National Cholesterol Education Program (ATP III), on metabolic syndrome, 20 Natriuresis, 81 Natural immunity, 345–346, 346t Natural killer (NK) cells, 328, 350t–351t cell surface markers of, 352t NAVEL mnemonic, 589, 589f Nebivolol, 9t–10t Necator americanus, 523t Neck, embryology of, 615f, 616–621, 616t, 617b Necrosis, 514 postpartum, 160, 160b, 163b Necrotizing fasciitis, 634, 634f, 635b Nedocromil, 30 NeD’s DOWN, 383b Nefazodone, 384 Negative birefringence, 452 Negative feedback, 160 Negative likelihood ratio (NLR), 573 Negative predictive value (NPV), 573, 573t, 582–583, 582b Negative symptoms, 377 Neisseria bacteria, 350t Neisseria gonorrhoeae, 208–209, 515, 516f, 617 Neisseria meningitidis, 171, 518 Nelson syndrome, 169–170 Nematodes, 523, 523t

698  INDEX Neonatal abstinence syndrome (NAS), 202b Neonatal respiratory distress syndrome, 25, 207 Neoplasm, “grade” and “stage” of, 227–228 Neostigmine, 412 Nephritic syndrome, 54, 54b, 55t, 68b Nephritis, 54 Nephrogenic diabetes insipidus, 381 Nephrolithiasis, 71, 71f–72f, 71b, 73b, 669, 669f Nephrology, 48–74, 48b case study acute glomerulonephritis, 68, 68f, 68b acute interstitial nephritis, 58, 58b–59b acute pyelonephritis, 73b–74b, 74 acute tubular necrosis, 57, 57b–58b chronic renal failure, 59, 59b, 62b cystic kidney disease, 62b, 64b nephrolithiasis, 71, 71f–72f, 71b, 73b nephrotic syndrome, 65, 65f, 65b, 68b obstructive acute renal failure, 55b–56b Nephron, 48–49, 49f diuretics in, 85–86, 86f renal handling and, 49, 50f segments of, 52–53 “Nephrotic range proteinuria,” 54 Nephrotic syndrome, 54, 54b, 55t, 81 acute glomerulonephritis vs., 68–69 case study, 65, 65f, 65b, 68b causes of, 67 idiopathic, 66t Nephrotoxic agents, 57b Nervous system, organization of, 551, 551t Net renal acid excretion in acid-base balance, 91 in acid-base homeostasis, 78 Neural crest cells, 172b Neural tube, disorders in, 602 Neuritic plaques, 653 Neuritic senile plaques, 430 Neuroblastoma, 172 Neurocrine secretion, 156, 156f Neurocutaneous disorder, 434, 434b Neurocysticercosis, 529b case study on, 527b–528b, 528f diagnosis of, 528 differential diagnosis of, 528 epidemiology of, 529 management of, 528 pathology of, 528 taeniasis and, 528 Neuroendocrine tumors, 234 Neurofibrillary tangles, 430, 653 Neurofibromatosis, 434 Neuroleptic malignant syndrome, 378, 379b Neurology basic concepts of, 400–403, 400b case studies alcohol and related drugs, abuse of, 431b, 433b Alzheimer’s disease, 430b–431b amyotrophic lateral sclerosis, 404b–405b, 405f–406f, 407b arterial dissection, 434b–436b Bell’s palsy, 427b–428b brain tumors, 433b–434b cerebrovascular accidents, 419b–420b, 421f–422f, 422b epidural and subdural hematomas, 426b–427b, 427f epilepsy, 428b, 430b Huntington disease, 409b–411b, 410f hydrocephalus, 424f, 424b, 426b multiple sclerosis, 418b–419b myasthenia gravis, 411b–413b neurocutaneous disorder, 434b

Neurology (Continued) Parkinson’s disease, 407b–409b subarachnoid hemorrhage, 423b, 424f syringomyelia, 413b, 414f, 415b trigeminal neuralgia, 415b–416b vitamin B12 deficiency, 416b–418b Neuromuscular blocking agents, 413, 552 Neuronal firing, anticonvulsants and, 429 Neurotic defenses, 386 Neurotoxins, 483 Neurotransmitters, 377, 377b, 378t, 551 synthesis of, 554f Neutrophil chemotaxis, 350 Neutrophils, 350t–351t hypersegmented, 120, 120f, 120b Newborn, hemolytic disease of. see Hemolytic disease of newborn Niacin, 22 rush, 22 Nicotine, effect of fetal exposure, 202t Nicotinic receptors, 552, 552t Niemann-Pick disease, 259, 261t macrophages in, 260f Nifedipine, 9t–10t, 545, 545t Night terrors, 561 Nightmares, 561 Nigrostriatal pathway, 379 Nil disease. see Minimal change disease Nitroglycerin, 12 Nitrosoureas, 434 Nizatidine, for gastroesophageal reflux disease, 112t NLR. see Negative likelihood ratio (NLR) NLRP3 inflammasome, 658–659 NNRTIs. see Non-nucleoside reverse transcriptase inhibitors (NNRTIs) Nodal cell action potential, 6, 6f Nodal cells, 6 Nodular sclerosis, 644 Nonalcoholic fatty liver disease (NAFLD), 651 Nonalcoholic steatohepatitis (NASH), 651 Non-anion gap metabolic acidoses, 94, 94b, 96, 96f Noncommunicating hydrocephalus, 425 Noncompetitive inhibitors, 537 Noncompliant patient, 567b–568b Nondepolarizing neuromuscular blockers, 552, 553f Non-Hodgkin lymphoma, 332, 644 description of, 333t origin of, 334t–335t vs. Hodgkin lymphoma, 341–342 Nonmaleficence, 559 Non-nucleoside reverse transcriptase inhibitors (NNRTIs), 367, 368t Nonreactive final crossmatch, 371 Nonseminomatous germ cell tumors, 150 Non-small cell lung cancer (NSCLC), 45, 235 Non-ST elevation myocardial infarction (NSTEMI), 13, 662–663 Nonsteroidal antiinflammatory drugs (NSAIDs) acquired platelet deficiency and, 324 acute/chronic gastritis and, 119 anemia of chronic disease, 299 for gout, 453 induced renal toxicity, 59, 59b for labor suppression, 207, 211 mechanism of action of, 539, 539f for osteoarthritis, 447–448, 448t peptic ulcer disease and, 116, 118b–119b pharmacotherapeutic action of, 539 in renal blood flow, 76, 78b Reye syndrome and, 540b for rheumatoid arthritis, 450 side effects of, 539 Nontender lymphadenopathy, 229 Nonvolatile acids, 78, 91

INDEX  699 Norepinephrine, 6–7, 172–173, 383 Normal distribution, 576, 576f, 577b, 581, 581b Normal reference ranges, 284b Normocytic anemias, 286 G6PD, 307 macrocytic and, 287 sickle anemia, 288 Northern blotting, 255 Norwalk virus, 503t Nosocomial pneumonia, 498 NPV. see Negative predictive value (NPV) NREM sleep, 561 NRTIs. see Nucleoside analog reverse transcriptase inhibitors (NRTIs) NSAIDs. see Nonsteroidal antiinflammatory drugs (NSAIDs) NSCLC. see Non–small cell lung cancer (NSCLC) Nuclear factor of activated T cells (NFAT), 370 Nuclei pulposi, 602 Nucleoside analog, 149b Nucleoside analog reverse transcriptase inhibitors (NRTIs), 367, 368t Nucleus ambiguus, 435 Nucleus solitarius, 435 Null hypothesis (H0), 574 Number needed to harm, 586 Number needed to treat (NNT), 586 Nutcracker syndrome, 607 Nutrition, 282b Nystagmus, pathologic, 442 Nystatin, mechanism of action of, 524 O OA. see Osteoarthritis (OA) Oat cell carcinoma, 646 Observer bias, overview of, 578t–580t Obsessive-compulsive disorder, 398 Obsessive-compulsive personality disorder, 398, 398b–399b Obstructive communicating hydrocephalus, 425 Obstructive jaundice, 134t, 135 Obstructive lung disease, 28, 28t Occipital cortex, damage to, 439b Occipital lobe, visual information and, 437 Octreotide, for acromegaly, 166 Oculomotor nerve palsy, 440 Odds, defined, 573 Odds ratio (OR), 572, 572t, 580 OKT3/muromonab, 371–372, 372t Olanzapine, 380 Oligodendrocytes, 418, 654 Oligodendrogliomas, 433 Oligohydramnios, 202 Olive-like mass, palpable, 124b Omeprazole, for gastroesophageal reflux disease, 111–112, 112t Oncogenes, 224 Oncology. see Cancer(s) Open-angle glaucoma, 442, 442f Ophthalmology, 437–444, 437b Opioid, 541b case study on, 540b mechanism of action of, 540 members of, 540 Opioid dependence, 541 Opioid intoxication, 392t, 540, 540t Opioid overdose, 541 Opioid receptors, 540 Opioid tolerance, 541 Opioid withdrawal, 541 Oppositional defiant disorder (ODD), 387, 388b Opsoclonus-myoclonus syndrome, 172 Opsonizing antibodies, 348 Optic chiasm, 437–439, 438f, 439b Optic nerve, 438f, 439, 439b

Optic tract, 438f OR. see Odds ratio (OR) Oral contraceptives, 199b case study, 198b hepatocellular adenomas, 150 uterine fibroids and, 213 Oral glucose tolerance test (OGTT), 184b Oral squamous cell carcinoma, 645, 645f Organophosphate poisoning, 413, 556, 556b Organophosphates, 556b Ornithine transcarbamoylase (OTC) deficiency, 282, 283b treatment of, 282 Oropharyngeal candidiasis, 30 Orotic aciduria, 282–283 Orphan Annie eye nuclei, 646, 647f Orthostatic hypotension, 3–4, 378 Osler’s nodes, 505, 505t, 655–657 Osmotic demyelination syndrome, 83 Osmotic diarrhea, 502 Osmotic diuretics, 85–86 Osmotic fragility test, spherocytosis, hereditary, and, 306 Osteitis fibrosa cystica, 62 Osteoarthritis (OA), 446–447, 446f–447f, 446b, 448t, 449b, 472t–473t, 667, 667f rheumatoid arthritis and, 663–664 Osteoclasts, 470 Osteodystrophy, renal, 471, 471f Osteogenesis imperfecta (OI), 273, 471b, 472, 472t–473t, 472f, 473b Osteomalacia, 62, 471, 471f, 471b, 472t–473t Osteomyelitis, 471 sickle cell anemia and, 291 Osteopetrosis, 469b, 470, 470f, 472t–473t, 473b Osteophytes, 667 Osteoporosis, 169, 170b Ostium primum defect, 598 Ostium secundum defect, 597–598 Ovarian cancer, 216b, 249, 252b BRCA1 and, 231 case studies, 214b–215b hereditary, 215 markers, 215t prognosis of, 216 risk factors of, 215 Ovarian cycle, 195 Ovarian cysts, 199 Ovarian ligament, 617, 618f Ovarian mass, 214 Ovary anatomy of, 617, 618f, 619b menstrual cycle and, 196 Ovulation, hormonal trigger for, 196 Oxidative stress, 308b Oxygen capacity, 40t Oxygen dissociation curves, 41f, 42b Oxygen tension, 40, 41f Oxytocin, 162t, 208 P P values, 574, 574b p16 gene, 234 p53 gene, 225 P-450 enzymes, 270 Paget disease, 219, 230f, 472t–473t Pain, anterolateral system and, 415 Palpable breast mass, 218b Palpable nonblanching purpura, 474 Palpable purpura, 68b Pampiniform plexus, 605, 607, 607f PAN. see Polyarteritis nodosa (PAN) Panacinar emphysema, 35, 39f

700  INDEX Pancoast tumors, 45 Pancreas, 109, 110b Pancreatic cancer, 233–234, 234b Pancreatic carcinoma, 233 Pancreatic enzymes, 108 Pancreatic mass, 233 Pancreatitis acute, 122 alcohol abuse and, 390 case study of, 121b–123b, 122f chronic, 123 cystic fibrosis and, 274 Pancytopenia, 641 Panic attacks, 394 Panic disorder, 394, 394b Pannus formation, rheumatoid arthritis and, 450 Pap smears cervical cancer and, 239–241 sensitivity of, 582 Papillary carcinoma, 646, 647f Papillary necrosis, sickle-cell anemia, 291 Papilledema, 433 Papilloma, intraductal, 230, 218 Papovaviridae, 521t Para-aortic lymph nodes, 216 Paracrine secretion, 156, 156f Paradoxical anticoagulation, 327b Paradoxical sleep, 561 Paraesophageal hiatal hernia, 111 Parafollicular cells, 243 Paragonimus westermani, 523t Paramedian pontine reticular formation (PPRF), 440 Paraneoplastic syndrome, 248, 249b Cushing syndrome and, 45, 248t with hepatocellular carcinoma, 150 Lambert-Eaton syndrome and, 248t, 413 with small cell carcinomas, 45, 45b tumors associated with, 248t Paranoid personality disorder, 398, 399b Parasitic diseases, 520–534 basic concepts in, 522–523 Parasitology, 522–523 Parasternal lift, 620 Parasympathetic nervous system, 551–552 Parathyroid hormone (PTH) hypocalcemia and, 61 regulation of serum calcium by, 61, 61f thyroid cancer and, 244 vitamin D and, 61, 62b Parathyroid hormone–related peptide (PTHrP), 45 Parenchymal hematoma, 420 Parenchymal hepatocytes, 651–652 Parental consent, for minors, 563, 563b Parenteral administration, 535 Parietal lobe, visual information and, 437 Parietal pleura, 620 Parkinsonism, 378–379, 407–409 Parkinson’s disease, 407–408, 409b Partial agonist, 538 Partial pressure of alveolar oxygen, 40t of carbon dioxide, HCO3−, pH and, interrelationship among, 90, 91f of dissolved oxygen in arterial blood, 40t Partial seizures, 429, 429t Parvoviridae, 521t Parvovirus, sickle cell anemia and, 290 Parvovirus B19, 633, 675 infection, 532, 532b Passive immunity, for parvovirus B19 infection, 532 Patau syndrome, 280

Patent ductus arteriosus (PDA), 596, 599b case studies, 596b Pathogen-associated molecular patterns (PAMPs), 345–346 Pathologic nystagmus, 442 Pathology, 639–675, 639b Pathways, 266b Patient confidentiality, 564, 564b Patient-requested prescriptions, 565b Pattern recognition receptors (PRRs), 345–346 Pauci-immune glomerulonephritis, 69 PCOS. see Polycystic ovary syndrome (PCOS) PCP, intoxication, 392t PDA. see Patent ductus arteriosus (PDA) PE. see Pulmonary embolism (PE) Peau d’orange appearance, 231, 231f Pelger-Huët cells, 330–331, 331f Pelvic inflammatory disease (PID), 515, 617, 617b birth control and, 516 case studies, 208b, 515b risk factors of, 516 Pelvic pain, differential diagnosis for, 208, 211 Pemphigus vulgaris, 627, 627t–628t, 628b–629b, 629f, 652–653, 653f Penetrance, 411 Penicillamine, 411b d-penicillamine, scleroderma and, 461–462 Penicillin G for pneumonia, 499 for syphilis, 672 Penicillins, 486, 494t–497t anaphylaxis and, 353 antibacterial spectrum of, 490–491 autoimmune hemolytic anemia and, 355 patients allergic to, 490 syphilis and, 510 Pentobarbital, 547–548 Pentoxifylline, for peripheral vascular disease, 591t Pepsinogen, 108t Peptic ulcer disease, 115b–119b, 116–118, 117f Peptide hormones, 157, 157f, 157t Pergolide, 409 Pericarditis, 14, 619b–620b, 620 Pericardium, layers of, 620 Periosteum, osteoarthritis and, 447 Peripheral nerves, 419 Peripheral resistance, primary determinants of, 2–3 Peripheral vascular disease (PVD), 342b, 587, 591t, 591b case studies, 587b Peripheral zone, 238b Peritoneum, anatomy of, 617, 619b Peritonitis, 131 Pernicious anemia, 120–121, 121b treatment of, 303 Peroneal nerve, common, 610f, 610b Peroxisome proliferator-activated receptor-alpha (PPAR-alpha), 22 Personality disorders, 399b pH Pco2 arterial, 27 HCO3− and, interrelationship among, 90, 91f vaginal, 209 Phagocyte deficiencies, 360t–361t Phagocyte disorders, 364–365 Phagocytosis, 362 Phalen sign, 456, 456f, 456b Pharmacology and toxicology, 535–558, 535b basic concepts of, 535–538 maximal effect and substrate concentration for, 538–542 Pharyngeal pouches, arches, and clefts, 616, 616t Pharyngitis, 635

INDEX  701 Phase I drug metabolism, 536 Phase II drug metabolism, 536 Phencyclidine (PCP), 542, 543b Phenobarbital, 199, 382, 547–548 triggers acute porphyrias, 270 Phenoxybenzamine, 173, 173b Phenylalanine, conversion to epinephrine, 256f, 256b Phenylephrine, 554 Phenylketonuria (PKU), 255 dietary therapy, 257 pathologic mechanisms of, 257f pathophysiology, 256 screening test of, 257 summary, 257b treatment of, 256–257 Phenytoin, 199, 382 triggers acute porphyrias, 270 Pheochromocytoma, 171b–173b, 172, 394 Philadelphia chromosome, 331, 339 Phosphodiesterase-5 (PDE-5), 606 Phospholamban, 5 Physician-patient relationship, 564b Physiologic dead space, 25 Physiologic jaundice of the newborn, 143, 143b–144b Physiologic shunt, 26 Pia mater, 426, 600, 601f Pica, 296b Pick bodies, 430 Pick disease, 430 PID. see Pelvic inflammatory disease (PID) Pilocarpine, glaucoma and, 442–443 Pioglitazone, 186 Pitocin, 208 Pituitary adenoma, 174t, 177t, 669, 670f Pituitary gland anterior, 160, 161f–162f, 163t posterior, 160, 162t Sheehan syndrome and, 160, 160b, 163b Pituitary tumors, bitemporal hemianopia and, 438–439 PKU. see Phenylketonuria (PKU) Placebo-controlled trials, 586 Placenta accreta, 207 Placenta previa, 207 Placental abruption, 207 Plasma anions, 92 Plasma cations, 92 Plasma cells, in multiple myeloma, 336 Plasma concentrations, 89 Plasmin, 318 Plasminogen, 318 Plasmodium falciparum, 523t, 672–673, 673f Plasmodium malariae, 523t Plasmodium ovale, 523t Plasmodium vivax, 523t Platelets, 329 cell surface markers of, 352t Pleural pressure, pneumothorax and, 47 Pleuritis, 620, 621b PLR. see Positive likelihood ratio (PLR) Plummer disease, 174t Plummer-Vinson syndrome, iron deficiency anemia and, 296 Pneumoconioses, 38–39 Pneumocystis carinii pneumonia, 369 Pneumocystis jiroveci, 525t Pneumonia, 501b atypical (walking), 498 case study on, 493b, 498f, 498b causes of, 33t by age, 499t organisms, 500t–501t

Pneumonia (Continued) community-acquired, 498 diagnosis of, 498–499 eosinophilic, 32 lobar (typical), case study of, 32b, 33, 33f nosocomial, 498 pneumococcal, 498f Pneumocystis carinii, 369 SIADH and, 82 Pneumothorax, 46b–47b, 47 SIADH and, 82 Podagra, 452 Podocytes, 77 Poison ivy, 630, 630f Polyarteritis nodosa (PAN), 475t, 480, 480b–481b, 481f case study for, 480, 480b–481b hepatitis virus and, 148, 148b Polycystic kidney disease, 423 Polycystic ovary syndrome (PCOS), 214–215, 214f Polycythemia, 248t Polycythemia vera (PV), 641 case study for, 342, 342b–343b erythropoietin levels and, 343 management of, 343 Polyhydramnios, 202 Polymerase chain reaction (PCR), 255 Polymorphonuclear leukocyte, 640f, 641 Polymorphonuclear neutrophils, 416 Polymyalgia rheumatica, 455, 455b–456b Polymyositis, 466 Pompe disease. see Glycogen storage disease; type II Pons, 403b Popliteal artery, atherosclerosis in, 660–661 Porphyrias acute anticonvulsants trigger, 270 hemin/glucose for, 270 summary, 271b definition of, 269 types of, 269 Portacaval anastomosis, 135t Portal circulation, 138f Portal hypertension, 135, 135t, 136b Portal triads, 133, 133f Portal vein, 135 Positive likelihood ratio (PLR), 573, 583 Positive predictive value (PPV), 573, 573t, 582b–583b, 583, 584t Positive symptoms, 377 Posterior compartment of leg, 610–611, 611f Posterior cruciate ligament (PCL), 609, 609f Posterior fossa, 416 Posterior interventricular artery, 619, 619f Postmenopausal bleeding, 216 Postpartum necrosis, 160, 160b, 163b Postpartum psychosis, 377 Postprandial epigastric pain, differential diagnosis of, 119 Poststreptococcal glomerulonephritis (PSGN), 69, 69f, 70b–71b, 654–655, 655f Posttraumatic stress disorder (PTSD), 394, 394b–395b Postural tremor, 407 Potassium (K+), 92 repletion of, in diabetic ketoacidosis, 102 Pott’s disease, 514 Poxviridae, 521t Poxvirus, 635 PPV. see Positive predictive value (PPV) Prader-Willi syndrome, 281 Prazosin, 9t–10t, 172, 172b, 554 Precision, 573

702  INDEX Prednisone for dermatomyositis, 466 Wegener granulomatosis and, 480 Preeclampsia, 206, 207b Pregnancy bacteriuria and, 74 breast cancer risk and, 218 case studies amenorrhea (secondary), 199b–200b, 201f gestational hyperthyroidism, 205b gestational trophoblastic diseases, 203f, 203b preeclampsia, eclampsia, HELLP syndrome, 206b corpus luteum in, 196 endocrinology of, 206b labor, delivery and, 207–209, 208b, 211b, 215b lithium and, 381 molar, 204 prolactin and, 160 thrombocytopenia, 326 Preload, 1, 12 Premature delivery, 207 Premature ejaculation, 383 Preoperational, stage of cognitive development, 389 Prerenal azotemia, 57–58, 87 Pressure-volume loop changes, in aortic stenosis, 18–19, 18f, 19b Prevalence, incidence vs., 575 Priapism, 384, 606 Primary acid-base disorder, determining, 90, 91f Primary ciliary dyskinesia (PCD), 275 Primary hyperaldosteronism, 100, 100b Primary hypertension, first-line treatments for, 7 Primary immunodeficiencies, 360t–361t Primary pulmonary hypertension, 46 Primary sclerosing cholangitis, 129 Primary tuberculosis, 513 Prinzmetal angina, 11 Probability, 573 Probenecid, for gout, 453 Procedure bias, overview of, 578t–580t Progesterone, 196 Progesterone receptor (PR), 219, 232 Progesterone-only challenge test, 200 Prokinetic drugs, for gastroesophageal reflux disease, 112t Prolactin, 160, 163t disorders of, in deficiency or excess of, 164t elevated, in infertility, 165 normal physiologic functions of, 163–164 physiologic actions of, 164f tuberoinfundibular tract disruption and, 165b Prolactinoma, 163, 164t, 165b, 669, 670f Proliferation, tumor suppressor genes and, 224 Prolonged bleeding time, 324 Promyelocytic leukemia, 344 Propranolol, 9t–10t, 172, 172b, 378 Propylthiouracil (PTU), 178 Prospective cohort study, 584–585 Prostaglandins, 51b acetaminophen and, 539, 550 ductus arteriosus and, 596, 599b peptic ulcer disease and, 116 in renal blood flow, 76 synthetic, 203 Prostate, anatomy of, 237 Prostate cancer, 237–238, 237b–238b Prostate-specific antigen (PSA), 237, 237b–238b Protease inhibitors (PIs), 367, 368t cytochrome P-450 enzymes and, 118t Protein C, deficiency in, and deep venous thrombosis, 327

Protein kinase A (PKA), 5 Protein S, deficiency in, and deep venous thrombosis, 327 Proteins, 106, 106t in urine, 357 Proteinuria, systemic lupus erythematosus and, 464 Proteus mirabilis, struvite stones and, 72–73 Prothrombin time (PT), 136t, 139t, 317, 317f, 651–652 Proton pump inhibitors, peptic ulcer disease and, 117b Proto-oncogenes, 224 Protozoa, 522, 523t Proximal tubule, 52, 79, 79f, 79b Prussian blue stains iron, 651 Psammoma bodies, 646, 647f Pseudocholinesterase deficiency, 552–553 Pseudogout, 453, 453b, 472t–473t Pseudohypertrophy of calf muscles, 467, 468f Pseudomembranous colitis, 502 Pseudomonas aeruginosa, 489t, 500t–501t “Pseudopalisading” pattern, 242, 242f, 433 Pseudoparkinsonism, 409 PSGN. see Poststreptococcal glomerulonephritis (PSGN) Psoriasis, 459, 459f, 459b, 629, 630f Psoriatic arthritis, 458b, 459–460, 459f Psychiatry, 376b case studies alcohol abuse, 390b–391b anorexia nervosa, 395, 395b–396b bipolar disorder, 380, 380b, 382b childhood disorders, 387b–389b delirium and dementia, 393b depression, 382, 382b drug intoxication, 391b, 392t, 393b ego defenses, 385b–386b, 387t panic disorder, 394, 394b personality disorder, 396b–399b posttraumatic stress disorder, 394, 394b–395b schizophrenia, 376, 376b–377b, 380b Psychogenic polydipsia, ADH levels in, 84, 84t Psychology, 390b case studies, psychosexual development, 389b Psychosexual development, stages of, 389 Psychosis, 377 Psychotic defenses, 386 Psychotic disorders, schizophrenia, 380b Psychotropic medications, in SIADH, 82 PTSD. see Posttraumatic stress disorder (PTSD) Puberty, delayed, differential diagnosis for, 221 Pulmonary compliance, 24 Pulmonary edema, 14 Pulmonary embolism (PE), 98, 98b deep venous thrombosis and, 327 Pulmonary fibrosis, causes of, 40 Pulmonary hypertension, 3t, 46 Pulmonary infiltrate, 32 Pulmonary leukotrienes, 29 Pulmonary lobes, anatomy of, 621f Pulmonology, 23–47, 23b case study acute respiratory distress syndrome, 43, 43b–44b allergic bronchopulmonary aspergillosis, 32b anemia, 40, 40b–41b, 43b asbestosis, 38, 38b asthma, 28b–29b chronic bronchitis, 36, 36b–38b chronic obstructive pulmonary disease, 33b–34b, 34f hypersensitivity pneumonitis, 31b lobar (typical) pneumonia, 32b, 33, 33f pneumothorax, 46b–47b, 47 sarcoidosis, 45b–46b small cell lung carcinoma, 44, 44b–45b

INDEX  703 Pulmonology (Continued) gas exchange in, 27–41 mechanics of breathing, 23–26, 26b ventilation-perfusion matching in, 26 Pulseless disease, 475t “Punched-out” bone lesions, 642, 642f “Punched-out” lytic lesions, 336 Pupillary constriction, 440 Pupillary light reflex, 439, 440b Pupillary reactions, Edinger-Westphal nucleus and, 437 Pupillary response, left optic nerve and, 439, 440f Purified protein derivative skin (Mantoux) test, for tuberculosis, 514 Purine “salvage” pathway, 262, 262f causes hyperuricemia, 263 Purpura, palpable nonblanching, 474 PV. see Polycythemia vera (PV) PVD. see Peripheral vascular disease (PVD) Pyelonephritis, acute, 73b–74b, 74 Pygmalion effect, overview of, 578t–580t Pyloric stenosis, hypertrophic, 123b–124b, 124, 124f Pyramidal cells, 406 Pyrazinamide, 494t–497t Pyridostigmine, 412 Pyridoxine, 513–514 Pyrogenic exotoxins, 483 Q Quellung reaction, 484, 484b Question format, 294b R RA. see Rheumatoid arthritis (RA) Rabeprazole, for gastroesophageal reflux disease, 112t Radial nerve, 593, 594f, 594t, 596b Radiation exposure, and cancer risk, 584t, 584b Radiolucent substances, 153b Random error, defined, 577 Randomized controlled trials, 586, 586b Randomized study, 577t Ranibizumab, age-related macular degeneration and, 443 Ranitidine, for gastroesophageal reflux disease, 112t Rapamycin, 372t Rapid plasma reagin (RPR), 672 tests, 509 Rapidly progressive glomerulonephritis (RPGN), 69, 659 Rathke’s pouch, 160 Raynaud phenomenon, 342b, 460, 460f, 460b Reaction formation, 387t Reactivation tuberculosis, 514 Reactive arthritis, 374, 374b–375b, 458, 458b organisms associated with, 374 pathogenesis of, 374 Reactive bone formation, 446 Reactive leukocytosis, 332 Reactive psychosis, 377 Recall bias, 578, 578t–580t Receptive aphasia, 421 Receptor activator of nuclear transcription factor-kappa B ligand (RANKL), 642 β2-Receptor agonists, 207 α1-Receptor antagonists, 3 Receptor-linked kinases, 157, 158f, 158t, 158b Recovery phase, of acute tubular necrosis, 57 Recrudescent (secondary) tuberculosis, 513 Red blood cells (RBCs) cell surface markers of, 352t hypochromic, 297, 311, 312f sickled, 289–290 conditions of, 289–290 spherocytes, 305f

Red blood cells (RBCs) (Continued) splenomegaly and, 290 in urine, 357 5α-Reductase deficiency, 198 Reed-Sternberg cells, 332, 334b, 340b, 341, 341f, 644, 644f Reflex tachycardia, 4, 12 Regression, 386 Regulatory T cells (Tregs), 375 Reid index, 37 Reiter syndrome, 458, 516 Relative risk (RR), 572, 572t, 580, 585 Reliability, defined, 573 REM sleep, 561 Renal acid excretion, in acid-base balance, 91 Renal angiomyolipoma, 434 Renal artery stenosis, 81, 82b Renal biopsy, hypertensive nephrosclerosis and, 59–60 Renal blood flow (RBF) glomerular filtration rate and, 76, 76b, 78b nonsteroidal antiinflammatory drugs in, 76, 78b prostaglandins in, 76 regulation of, 51, 52f, 52t Renal cell carcinoma, 248 Renal clearance, 49, 50f–51f Renal compensation, for respiratory alkalosis, 98, 98b Renal cysts, 64 Renal dysfunction, Lesch-Nyhan syndrome and, 263 Renal erythropoietin synthesis, 60 Renal failure diffuse scleroderma and, 461 etiologic classifications of, 55–56 multiple myeloma and, 337 obstructive acute, 55b–56b Renal filtration, transport processes and, 75–78, 75f, 76b, 77f, 78b Renal ischemia, G6PD and, 309 Renal osteodystrophy, 62, 471, 471f Renal papillae, 49b Renal tubular acidosis (RTA), 91 classification of, 97t Renin-angiotensin-aldosterone system, 9f, 15, 80 in regulating blood pressure, 81 in renovascular hypertension, 81 Renovascular hypertension, 81, 81b Repaglinide, 186 Repression, 387t Reproductive system basic concepts of, 195–207, 195b, 198b, 207b case studies amenorrhea (secondary), 199b–200b, 201f androgen insensitivity syndrome, 222b breast cancer, 218b–219b, 230b–232b breast mass, benign, 217b–218b endometrial cancer, 216b endometriosis, 211, 211b Kallmann syndrome, 220b–221b pelvic inflammatory disease, 208b preeclampsia, eclampsia, HELLP syndrome, 206b sex chromosome disorders, 221b uterine fibroids, 212b, 213f female, 618f, 619b gestational trophoblastic diseases, 204 hydatidiform mole, 204 male, 606b male and female, 195–223 oral contraceptives, 199 pelvic inflammatory disease, 209b Residual lung volume, 29 Resistance, breathing and, 23f

704  INDEX Respiratory acidosis, 27–28, 104, 104b–105b compensation for, 105 exacerbation in, 105, 105b in pneumothorax, 47 Respiratory alkalosis, 27–28, 91f, 99, 99b compensation in, 98–99 in mechanical ventilation, 38 in pneumothorax, 47 Respiratory burst, 362, 362f–363f Respiratory compensation lungs in, 102 for metabolic acid-base disturbances, 90t Respiratory drive, primary determinants of, 27 Respiratory infection, cystic fibrosis and, 274 Respiratory suppression, 27 Rest pain, peripheral vascular disease and, 588 Resting tremor, 407 Restless legs syndrome, 568b–569b Restrictive cardiomyopathy, sarcoidosis and, 46 Restrictive lung disease, 28, 28t Reticulocytes corrected count, 285b definition of, 284–285 folate/vitamin B12 deficiency and, 301 G6PD and, 309 Retinal detachment, 444, 444b Retinoblastoma, 224 Retinoblastoma (Rb) protein, 225 Retrospective cohort study, 584–585 Retrospective studies, case-control vs. cohort, 578 Reverse transcriptase inhibitors, 367 Reye syndrome, 154, 154b–155b nonsteroidal antiinflammatory drugs and, 540b Rh antigen, pregnancy, during, 310 Rhabdomyolysis, 58, 58b Rheumatic fever, 374, 507b acute, 506t clinical signs of, 506–507 Rheumatoid arthritis (RA), 447, 447f, 449–450, 449f–450f, 449b–451b, 472t–473t, 663–664, 663f Rheumatoid factor, rheumatoid arthritis and, 450 Rheumatology, 445b basic concepts of, 445–468 case studies of carpal tunnel syndrome, 456–457, 456b–457b fibromyalgia, 454–455, 454f, 454b–455b gout, 451b–454b, 452–453, 452f–453f inflammatory myopathies, 465b–467b, 466f muscular dystrophies, 467b, 468f, 469–473, 469b osteoarthritis, 446, 446f–447f, 446b, 448t, 449b osteopetrosis, sickle cell avascular necrosis, osteogenesis imperfecta, 469b–471b, 473b polymyalgia rheumatica, 455, 455b–456b rheumatoid arthritis, 449–450, 449f–450f, 449b–451b scleroderma, 460, 460b, 461f, 462t, 462b seronegative spondyloarthropathies, 457b–459b, 458 systemic lupus erythematosus, 462f, 462b–465b, 463 overview of disorders, 472t–473t Rickets, pathophysiology of, 472t–473t Rickettsia rickettsii, 491t Riedel thyroiditis, 179 Rifampin, 199, 494t–497t for tuberculosis, 514 Right coronary artery (RCA), 619, 621b Right eye, pupillary response to light and, 439 Right hemianopia, 438f homonymous, 439 with macular sparing, 438f Right lower quandrantanopia, 438f Right upper quadrant, pain of, 144, 144b, 146b

Right upper quandrantanopia, 438f Right ventricular failure, sarcoidosis and, 46 Right-sided colon carcinomas, 245 Right-sided heart failure, 15 Risperidone, 380 Rituximab, for bullous pemphigoid, 628 RNA viruses, 520, 520b, 521f Robertsonian translocation, 279–280 Rods, gram negative, 489t Rofecoxib, peptic ulcer disease and, 116 Romberg sign, 417 Roseola infantum, 533t Rosiglitazone, 186 Rotator cuff, anatomy of, 595, 595f Rotavirus, 503t Roth’s spots, 505, 505t, 655–657 Rotor syndrome, 144, 145t Rouleaux formation, 337, 337f Round ligament, 617, 618f Roundworms, 523t RPGN. see Rapidly progressive glomerulonephritis (RPGN) RR. see Relative risk (RR) RTA. see Renal tubular acidosis (RTA) Ruptured esophageal varices, 135t, 137 S Saccular aneurysms, 423 Saliva, 106 Salmeterol, 30, 30t Salmonella spp., 488t, 503t Sampling bias, overview of, 578t–580t Sarco/endoplasmic reticulum calcium adenosine triphosphatase, 5 Sarcoidosis, 45b–46b Sarcomas, 228 Sausage digit, 459, 459f Scarlet fever, 478, 533t, 635, 636f SCC. see Squamous cell carcinoma (SCC) Schilling test, 121, 121b, 303 Schistocytes, 641, 642f in disseminated intravascular coagulation, 321, 321f Schistosoma haematobium, 523t Schistosoma japonicum, 523t Schistosoma mansoni, 523t Schizoaffective disorder, 377 Schizoid personality disorder, 398, 398b–399b Schizonts, 672–673, 673f Schizophrenia, 376, 376b–377b, 380b, 409 Schizotypal personality disorder, 398 Schwann cells, 419 Schwannoma, 428 SCID. see Severe combined immunodeficiency (SCID) Scleral icterus, 137, 233 Scleroderma, 460, 460b, 461f, 462t, 462b Screening tests newborn, 257–258 cystic fibrosis, 258 prenatal genetic, 259b Seborrheic keratoses, 638, 638f Secondary hypertension diagnosis of, 7 pulmonary, 46 Secretin, 108t Secretory diarrhea, 502 Seizures, 206, 429t. see also Epilepsy new-onset, 528 Selection bias, overview of, 578t–580t Selective serotonin reuptake inhibitors (SSRIs) advantages of, 383 for Alzheimer’s disease, 430 manic episodes and, 383

INDEX  705 Selective serotonin reuptake inhibitors (SSRIs) (Continued) MAOI and, combination of, 384 for panic disorders, 394 for posttraumatic stress disorder, 395 premature ejaculation and, 383 in SIADH, 82 Selegiline, 409 Self-proteins, alteration of, 375 Sella turcica, 166 Senile calcific aortic stenosis, 18 Sensitivity, 570, 570t, 570b, 584b case studies, 581b, 582–583 specificity vs., 570–571, 571f Sensory information, 438f Sensory-motor, stage of cognitive development, 389 Sepsis, 483 Septic arthritis, crystal arthritis and, 451–452 Serine kinase receptors, 157, 158f, 158t Serologic test, hepatitis B and, 148t Seronegative spondyloarthropathies, 457b–459b, 458 Serosa, anatomy of, 106, 107f Serotonin, 383 fibromyalgia and, 455 Serotonin agonist, 542 Serotonin syndrome, 384 Sertoli cells, 197, 222 Serum, hormone level in, 160 Serum albumin, liver disease and, 137 Serum amyloid A, 659 Serum cancer marker, 233–234 Serum electrolytes, diuretics in, 87 Serum sickness, 356, 356b–357b pathogenesis of, 356 Severe combined immunodeficiency (SCID), 359, 359b, 360t–361t, 361b Sex chromosome disorders, 222b case studies, 221b Sexual dysfunction, 565, 565b Sexual health, in elderly, 565b Sexual history, 239 Shaken baby syndrome, 420 Sheehan syndrome, 160, 160b, 163b Shigella spp., 488t diarrhea and, 503t Shingles/Herpes zoster, 635, 636f SIADH. see Syndrome of inappropriate secretion of antidiuretic hormone (SIADH) Sicca complex, 464 Sick euthyroid syndrome, 180 Sickle cell anemia, 289, 640 autosomal recessive pattern, 289 cause of, 289 parents of children with, 290b parvovirus and, 290 spleen, autoinfarction, 290 summary, 292b β-thalassemia major and, 294 vaccination, 290 Sickle cell avascular necrosis, 473b Sickle cell trait, 291 Sideroblastic anemias, 270–271 Sideroblasts, ringed, 330–331, 331f Signet ring cells, 229 Sildenafil, 220, 606 Simple seizures, complex seizures and, 429, 429t Simvastatin, mechanism of action of, 21 Sinoatrial (SA) node, 619, 621b Sinus venosus defect, 598 Sirolimus, 372t Sitagliptin, 186

Sjögren syndrome, 464 gastroesophageal reflux disease and, 112 SJS. see Stevens-Johnson syndrome (SJS) Skewness, 575, 576f, 576b Skin anatomy of, 622, 622f, 623t blistering, disorders, 627t–628t bullous pemphigoid, 627, 627f, 627t–628t, 627b, 629b dermatitis herpetiformis, 627t–628t, 638, 638f erythema multiforme (EM), 627t–628t, 637b pemphigus vulgaris, 627, 627t–628t, 628b–629b, 629f Stevens-Johnson syndrome (SJS), 627t–628t, 635–637, 636f graft-versus-host disease and, 373 Skin cancer, 226 basal cell carcinoma, 624, 624f, 624b, 626b squamous cell carcinoma, 625, 626b Skin lesions, 623t–624t Skin sensitivity tests, 32 Slapped cheek appearance, 290 “Slapped cheek” rash, 674f, 675 Sleep age and, 562 electroencephalographic findings during, 561 stages of, 561, 561t Sleep apnea, 560b case study on, 559b–560b Sleep hygiene, 562 Sliding hiatal hernia, 111, 111f, 113b Small bowel obstruction, 132b Small cell carcinoma, 235, 236f Small cell lung cancer (SCLC), 44–45, 44b–45b, 646, 646f case study of, 237b Smoke inhalation injury, 42, 42b Smoking bupropion and, 384–385 chronic bronchitis and, 37 chronic obstructive pulmonary disease and, 34 emphysema and, 660 lung cancer and, 235 small cell carcinoma of lung and, 646 Smooth muscle hypertrophy, 30 SNOUT mnemonic, 571b Sodium (Na+), 92 Sodium balance, 80, 82b Sodium reabsorption, under normal conditions, 85, 85t Somatic motor pathways, examples of, 401f Southern blotting, 254 Specificity, 570, 570t, 584b case studies, 581b, 582–583 sensitivity vs., 570–571, 571f Spectrin, 643–644 Spermatic cord, 605 Spermatic fascia, 605 Spherocytes, 643–644, 643f Spherocytosis, hereditary, 643–644, 643f etiology, 305 spherocytes, 305f summary, 306b Sphincter of Oddi, cholecystokinin and, 108t Sphingolipidosis, 260 SPIN mnemonic, 571b Spina bifida cystica, 602 Spina bifida occulta, 602, 603b Spinal cord, 402f–403f, 414, 414f Spinal trigeminal nucleus, 435 Spinothalamic system, 413 Spinothalamic tract, 401, 402b, 435 Spiral arteries, preeclampsia and, 206 Spirochetes, 490t Spironolactone, 9t–10t, 86–87

706  INDEX Spleen, 109, 110b Splenectomy, spherocytosis and, 306 Splenomegaly hereditary spherocytosis and, 642–644 RBCs and, 290 spherocytosis and, 306 thrombocytopenia and, 326 Splinter hemorrhages, 480–481, 505t, 655–657, 656f Splitting, 387t Sporothrix schenckii, 525t Spread, measures of, 575–577 Squamous cell carcinoma (SCC), 235, 236f, 625, 626b, 645, 645f cervical, 239 diagnosis of, 236–237 esophageal, 112 SRY gene, 197 Stable angina, 11 Stable plaque, 660–661 Standard deviation (σ), 576, 576f, 581b Stapedius muscle, 428 Staphylococcus aureus, 487t, 632, 655–657 pneumonia and, 500t–501t Staphylococcus epidermidis, 487t Staphylococcus saprophyticus, 487t Statins, 21 Statins, for peripheral vascular disease, 591t Statistical power, 574, 574t Steatorrhea, 153 ST-elevation myocardial infarction (STEMI), 13, 662–663, 662f STEMI. see ST-elevation myocardial infarction (STEMI) Steroid hormone, 156, 157f, 157t Steroid receptor, classes of, 135t Stevens-Johnson syndrome (SJS), 382, 627t–628t, 635–637, 636f “Stocking-glove” distribution, 186 Stomach, 107, 108t Stratum basalis, 200 Streptococcal pharyngitis, 69 Streptococcus agalactiae, 487t Streptococcus bovis, 487t Streptococcus pneumoniae, 487t pneumonia and, 500t–501t Streptococcus pyogenes, 632 Streptococcus viridans, 655–657 Streptomycin, 494t–497t Stress fractures, 395 Strictures, Crohn’s disease with, 657–658 Stroke, 420 Stroke volume, 1 Stromal tumors, 251, 251f Struma ovarii, 174t, 179, 215t Struvite stones, 71–73, 71f Study designs, overview of, 577–586, 577t Subacute bacterial endocarditis, 655–657 Subacute thyroiditis, 174t Subarachnoid hemorrhage, 420, 423, 423b, 424f Subchondral cysts, 447 Subdural hematoma, 426, 427f, 427b, 665, 666f Subdural hemorrhage, 420, 666f Sublimation, 385 Submucosa, anatomy of, 106, 107f Submucosal plexus, 106, 107f Substance P, fibromyalgia and, 455 Substantia nigra, 408–409, 408f Substituted judgment, 563, 563b β-subunit, hemoglobin, homozygous mutation, 292 Succinylcholine, 552–553 Sudden cardiac death, 14 Suicide, schizophrenia and, 377 Sulbactam, 486 Sulfamethoxazole, 492, 492f

Sulfasalazine, inflammatory bowel disease and, 129 Sulfonamides, cytochrome P-450 enzymes and, 118t Sulfonylureas, 186 Superego, 389 Superior vena cava syndrome, 45, 236 Superoxide dismutase (SOD), 362 Suppression, 387t Surface antigen (HBsAg), 526 Surfactant, 207 in alveolar surface tension, 25, 25f Surgical adhesions, small bowel obstruction and, 132, 132b “Suspended-dissociated” loss, sensory loss, 414, 414f Suspensory ligament, 617, 618f “Swan-neck” deformation, of fingers, 451, 663–664, 663f Swinging-flashlight test, 439b Sympathetic nervous system in effective circulating volume, 80–81 heart rate increase and, 6–7 increased activity of, 16 response of, to blood pressure reduction, 3, 4f Sympathoadrenal activation, 16 Sympathomimetics, 354t Symptomatology, 669 Synarthrodial joints, 445 Syncope, 19, 668 Syndesmophytes, 457, 458f Syndrome of inappropriate secretion of antidiuretic hormone (SIADH), 82, 82b, 248 ADH levels in, 84, 84t Synovitis, 663–664 Syphilis, 510b, 672, 673f case study on, 507b, 508f, 509b diagnosis of, 508–509, 509t diffuse maculopapular rash in, 508–509, 508b penicillin and, 510 secondary, 508 stages of, 508, 509t tertiary, 509 treatment of, 510 Syringobulbia, 413 Syringomyelia, 407, 413, 414f, 415b Systematic error, 577 Systemic lupus erythematosus, 462f, 462b–465b, 463 Systemic sclerosis, 460 Systolic dysfunction, 14 Systolic heart failure, 3t, 15–16 Systolic pressure, primary determinants of, 1 T T-cell tolerance, 375 T cells, 345, 352–353 cell surface markers of, 352t T lymphocytes, 328, 663–664 Tabes dorsalis, 417 Tachypnea, 96, 96b Tacrolimus, 370f, 372t Tadalafil, 606 Taenia saginatum, 523t Taenia solium, 523t in neurocysticercosis, 529 Taeniae coli, 106 Taeniasis, 528 Takayasu arteritis, 475t, 481b–482b, 482 Tamm-Horsfall protein, 337 Tamoxifen, 219, 232 Tamponade, cardiac, 236, 664 Tarasoff decision, 564 Tardive dyskinesia, 378, 410 Tartrate-resistant acid phosphatase (TRAP), 328f, 328b Taste, Bell’s palsy and, 428

INDEX  707 Tau protein, 653 Tau protein aggregates, 430 Tay-Sachs disease, 259, 261t pathogenesis of, 260 summary, 262b Tazobactam, 486 TB. see Tuberculosis (TB) T-cell deficiencies, 360t–361t T-cell receptors, 346 Teichoic acid, 483 Temperature sensation, 415 Temporal (giant cell) arteritis, 474, 475b–477b, 475t, 476 polymyalgia rheumatica and, 455 Temporal lobe, 437, 439 Tensilon test, 412 Teratogens, 201, 201t Terazosin, 9t–10t Testicular artery, 605 Testicular cancer, 605, 605b Testicular feminization syndrome, 222 Testis (testes), 197 anatomy of, 606f Testosterone, 222–223 Tetracyclines, 492, 494t–497t Tetrahydrocannabinol (THC), 542–558, 543b Tetralogy of Fallot, 598–599, 599f, 599b Textbook cases, 294b Thalamus, 403b Huntington disease and, 410 Thalassemia, 675 α-thalassemia, 294 summary, 295b β-thalassemia major, 292 bone fractures and, 293 hemochromatosis and, 293 pathogenesis of, 292 sickle cell anemia and, 294 summary, 295b Theophylline, 31 Therapeutic index, 537 Therapeutic window, 537 Thiamine deficiency, 431 Thiazide diuretics, 10b, 86 gout and, 452 for hypoglycemia, 8–9 Thiazolidinediones, 186 Thick ascending loop of Henle, 53 Thiopental, 547–548 Thioridazine, 378 Thromboangiitis obliterans, 475t Thrombocytopenic purpura, immune, 325, 325f, 325b–326b Thrombocytosis, essential, presentation of, 343 Thrombolytic therapy, 14, 14b Thrombotic thrombocytopenic purpura (TTP), 322t, 323b disseminated intravascular coagulation and, 322 pentad of, and hemolytic uremic syndrome, 322 Thromboxane A2 (TXA2), 11, 314 Thymic education, 365–366 Thymic hyperplasia, 412 Thymoma, 412 Thymus, 364–365 Thyroglossal duct, 616 Thyroid cancer, 243, 243t–244t, 245b Thyroid gland, 182, 616 Thyroid hormone, 156, 157f, 175, 175t, 176f replacement, 181 Thyroid panel, 205 Thyroid storm, 178 Thyroidectomy, injury to recurrent laryngeal nerve during, 613, 613b Thyroiditis, 174t, 178f

Thyroid-stimulating hormone (TSH), 161f, 163t–164t Thyrotoxicosis factitia, 174t Thyrotropin-releasing hormone (TRH), 161f Thyroxine (T4), 175t, 176f, 206b Tight blood pressure control, 8 Tinel sign, 456, 456b Tinkling bowel sounds, 132, 132b Tirofiban, 325t Tissue factor, 314 Tissue plasminogen activator (tPA), 318, 318b, 319t for ischemic strokes, 665–666 Tissue resistance, 23f, 24 Tocolytics, 207, 207b Tolbutamide, 186 “Tombstone” appearance, of basal cells, 652–653 Tongue, 182, 428b Tonic-clonic seizure, 428–429 Tophaceous gout, 453f Tophi, 452 Torphyrema whipplei, 127 Total iron-binding capacity (TIBC), 639 Total oxygen content, formula for, 40 Total peripheral resistance (TPR), 1, 3 Total resistance (RT), 24 Total iron-binding capacity, anemia of chronic disease and, 299 Toxic adenoma, 174t Toxic megacolon, 128 Toxic multinodular goiter, 174t Toxoplasma gondii, 523t Tracheoesophageal fistula, 123, 124b Transdermal patch, contraceptive and, 199 Transferrin saturation, anemia of chronic disease and, 299 Transformation zone of cervix, 239 Transfusions, anaphylaxis after, 360, 360b Transmural inflammation, 657–658 Transplant rejection, 370, 370b–372b, 372t types of, 371 Transport processes, renal filtration and, 75–78, 75f, 76b, 77f, 78b Transthyretin, 659 Transudates, 138–139, 139t, 139b Transudative pleural effusion, 667f, 668 Trastuzumab, 232 Traveler’s diarrhea, 501, 502t, 504b Trazodone, 384, 606 Trematodes, 523, 523t Tremor, 407 Trendelenburg sign, 612, 612f Treponema pallidum, 490t spirochete, 672 syphilis and, 508 Triamterene, 86 Trichinosis, inflammatory myopathies and, 467 Trichomonas vaginalis, 523t Trichophyton, 525t Tricyclic antidepressants, 383, 430 Trigeminal nerve, 415f, 416 Trigeminal neuralgia, 415, 416b Triglycerides, 21 Triiodothyronine (T3), 175t, 176f, 206b Trimethoprim, 492, 492f Trimethoprim-sulfamethoxazole (TMP-SMX), 492f, 494t–497t Trinucleotide repeat disorders, 469 Trinucleotide repeat expansion diseases, 278b–279b Trinucleotide repeats, 411 Triptans, 203 Trisomy 13, 280 Trisomy 18, 280 Trisomy 21. see Down syndrome Trisomy X, 222 Trophoblastic tumors, 174t

708  INDEX Trophozoites, 672–673, 673f Tropical sprue, celiac disease and, 127 Troponin I (TnI), 13, 13f, 662–663 Trousseau, Armand, 234b Trousseau syndrome, 234b True negative, defined, 571t True positive, 571t, 582 Truncus arteriosus, tetralogy of Fallot and, 598–599 Trypanosoma brucei, 523t Trypanosoma cruzi, 523t achalasia and, 115 Trypanosomiasis, American, 115 Trypsin, 108 Trypsinogen, 108 TTP. see Thrombotic thrombocytopenic purpura (TTP) Tuberculin skin test (PPD), 671 Tuberculosis (TB), 512, 515b acid-fast stain for, 513 case study on, 512b first-line drugs for, 513 pathogenesis and clinical course of, 513f SIADH and, 82 standard treatment of, 514 Tuberoinfundibular pathway, 379 Tuberoinfundibular tract, 165, 165b Tuberous sclerosis, 63, 64t, 64b, 434 Tubular necrosis, acute, 57, 57b–58b G6PD and, 308 Tumor lysis syndrome, 339, 453 Tumor markers, 250t Tumor necrosis factor alpha (TNF-α), 483 Tumor suppressor genes, 224, 246b–247b Tunnel vision, 437–438 Turcot syndrome, 649 Turner syndrome, 221–222, 222b “Two-hit hypothesis,” 224 Type grouping, 406 Type I (α) error, 574, 574t Type I hypersensitivity reactions, 352–353, 353t Type II (β) error, 574, 574t Type II hypersensitivity reaction, 353t, 355, 371 Type III hypersensitivity reactions, 353t, 356, 464 Type IV hypersensitivity responses, 353t, 357–358 Tyramine, 384 Tyrosine kinase receptor, 157, 158f–159f, 158t U UC. see Ulcerative colitis (UC) Ulcerative colitis (UC), 128, 128t, 128b, 130b, 247, 657–658 Ulcers, gastric, 116–117, 117f, 118b–119b Ulnar deviation, 450f, 451 Ulnar nerve, 592, 594f, 594t, 596b Uncal herniation, 242 Undescended (cryptorchid) testes, 223 Unicellular yeast, 524 Unprovoked seizures, 430 Unstable angina, 11 Upper esophageal sphincter (UES), 107 Upper extremity (UE) injuries, 591–593, 595b–596b Upper motor neuron, 400, 400b, 401f, 404–405, 419 Urea, 49, 51f Urea breath test, peptic ulcer disease and, 117, 118b–119b Urea cycle, 283f Urease, 117 Uremia, 62, 324 Uremic pericarditis, 62 Uric acid, 263 gout and, 452 overproduction of, 658–659 Uric acid calculi, 452

Uric acid stones, 71, 72f, 73 Uridine diphosphate (UDP) glucuronyl transferase, 144b Urinary obstruction, prostate cancer and, 238 Urinary retention case study, 55, 55b low-potency typical antipsychotics and, 378 Urinary tract infections, in men, 74 Urine anion gap, 97 Urobilinogen, 134 Urticaria (hives), 631, 631f anaphylaxis and, 353 Uterine fibroids, 213 Uterus, 196, 617, 618f, 619b UV damage, as risk for melanoma, 625 V Vaginal discharge, infectious causes of, 210t Vaginal pouch, 223 Vaginosis, bacterial, 672, 672f Valacyclovir, for herpes zoster, 635 Validity, defined, 574 Valproic acid, 381, 429 Valsartan, 9t–10t Vancomycin, 491, 494t–497t Vardenafil, 606 Variant angina, 11 Varicella-zoster (VZV) virus, 635 Varicocele, 606b–608b, 607, 607f Vas deferens, 605 Vasa recta, 53 Vascular dementia, 430 Vascular disease, peripheral, 587, 591t, 591b Vasculitic syndrome, 474 Vasculitides basic concepts of, 474–482, 474b case studies for defined, 475b Kawasaki disease, 477f, 477b–479b, 478 polyarteritis nodosa, 480, 480b–481b, 481f Takayasu arteritis, 481b–482b, 482 temporal (giant cell) arteritis, 475b–477b, 476 Wegener granulomatosis, 479, 479b–480b Vasectomy, production of ejaculate after, 605, 606f Vaso-occlusive crisis, 290–291 Vasospastic angina, 11 Velocity, of enzyme, 538 Vena cava, 607, 607f, 608b Venereal Disease Research Laboratory (VDRL) test, 464, 464b, 509 Venous return curves, shift of, 4, 5f Venous thrombosis, deep, 234b Ventilation-perfusion matching, basic concepts of, 26 Ventilation/perfusion ratio, 26 Ventricular fibrillation, 4, 5f Ventricular myocyte action potential, 5, 6f Ventricular relaxation, 5 Ventricular rupture, 14 Ventricular septal defect (VSD), 597, 599b Verapamil, 6, 9t–10t, 544, 545t Vertigo, 435 Very low-density lipoproteins (VLDLs), 21 Vestibular neuritis, 435 Vibrio cholerae, 488t diarrhea and, 502, 503t Vioxx, 448 Viral diseases, 520–534, 520b basic concepts in, 520–522 Viral hepatitis, 527 Virchow’s node, 116–117 Virchow’s triad, and deep venous thrombosis, 327 Viridans streptococci, 487t

INDEX  709 Virology, 520–522 Viruses clinical manifestations of infection with, 522t structural components of, 520 Visual cortex, visual fields and, 438f Visual fields, 437, 437f Visual information, 437 Visual pathways, lesions and, 438f Vitamin A toxicity, 137, 282 symptoms, 282 Vitamin B12, 301, 418b absorption, 301–303 biochemical role of, 302f role of, 302f treatment for pernicious anemia, 303 Vitamin B12 deficiency, 301b, 304t, 640f, 641 case study, 416b–417b causes of, 303 chronic gastritis and, 121b Crohn’s disease and, 657–658 folate deficiency and, 301 Hashimoto thyroiditis and, 180–181 hypersegmented neutrophil, marker for, 300f, 301b macrocytic anemia and, 120 pathogenesis of, 301 peripheral blood smear results, 300–301 signs and symptoms, 301 summary, 304b Vitamin D activation of, 192f osteomalacia and, 471, 471f parathyroid hormone and, 61, 62b renal failure and, 60 Vitamin K, 318 V(D)J rearrangements, 348, 348b Volume depletion, pathophysiology of, 100 Vomiting, in hyponatremia, 83–84 von Gierke disease. see Glycogen storage disease; type I von Hippel-Lindau syndrome, 63–64, 64t, 64b von Willebrand disease (vWD), 323, 323b–324b hemophilia A and, 323, 323b pathogenesis of, 323 von Willebrand factor (vWF), 314–315, 323 vWD. see von Willebrand disease (vWD) vWF. see von Willebrand factor (vWF)

Waterhouse-Friderichsen syndrome, 171, 171b Watershed areas, 422 Webbed neck, 221 Wegener granulomatosis, 474, 475t, 479, 479b–480b Wernicke-Korsakoff syndrome, 431 Wernicke’s aphasia, 421, 422f Wernicke’s encephalopathy, 431, 432f Western blotting, 255 Wheezing, 29 anaphylaxis and, 353 Whipple disease, celiac disease and, 127, 127b Whipple triad, 189 White blood cells, cell surface markers of, 352t Wilms tumor, 648, 649f Wilson’s disease, 140, 140f, 140b–141b, 411, 411b Window period, 147 Wiskott-Aldrich syndrome, 360t–361t Withdrawal from alcohol abuse, 390, 432 from cocaine use, 392 drug intoxications and, 392t Wolff-Chaikoff effect, 178 World Health Organization grading system, 242 Wrist, anatomy of, 593, 593f WT1 gene, 648

W WAGR complex, 648 Waiter’s tip position, 592f Waldenström macroglobulinemia, 334t–335t, 337 Wallenberg syndrome, 436 Warfarin, 557 mechanism of action of, bleeding disorders and, 318, 318b, 319f, 319t Warm autoimmune hemolytic anemia, 355

Z Zero-order kinetics, 391, 537 ZES. see Zollinger-Ellison syndrome (ZES) Zileuton, 30t Ziprasidone, 380 Zollinger-Ellison syndrome (ZES), 117, 118b–119b, 136–137, 190, 248t “Zosins,” 3 Zymogens, 108

X X chromosome inactivation, 254f Xerophthalmia, 464 Xerostomia, 464 X-linked Bruton agammaglobulinemia, 359 X-linked enzymopathies in females, 254 in males, 253 X-linked hyper-IgM syndrome, 360t–361t X-linked hypogammaglobulinemia, 360t–361t X-linked recessive diseases, 467b X-linked severe combined immunodeficiency, 359 XYY individuals, 222b XYY phenotype, 222 Y Yersinia enterocolitica, 489t diarrhea and, 503t Yersinia pestis, 489t

This page intentionally left blank